Carlyn Cpa Encode

   EMBED

Share

  • Rating

  • Date

    December 1969
  • Size

    1.1MB
  • Views

    12,390
  • Categories

Preview only show first 6 pages with water mark for full document please download

Transcript

A1 PASSERS TRAINING, RESEARCH, REVIEW & DEVELOPMENT COMPANY 2nd Floor Sommerset Bldg., Lopez Jaena St. Jaro, Iloilo City Tel. No.: (033) 320-2728; 09106547262 Email Address: [email protected] BOARD OF CERTIFIED PUBLIC ACCOUNTANT CERTIFIED PUBLIC ACCOUNTANT Licensure Examination MANAGEMENT ADVISORY SERVICES GENERAL INSTRUCTIONS: 1. This test booklet contains 49 test questions. 2. Read INSTRUCTIONS TO EXAMINEES printed on your answer sheet. 3. Shade only one (1) box for each question on your answer sheets. Two or more boxes shaded will invalidate your answer. 4. AVOID ERASURES. INSTRUCTIONS: 1. Detach one (1) answer sheet from the bottom of your Examinee ID/Answer Sheet Set. 2. Write the subject title “MANAGEMENT ADVISORY SERVICES” on the box provided. 3. Shade Set Box “A” on your answer sheet if your test booklet is Set A; Set Box “B” if your test booklet is Set B. __________________________________________________________________________________________ 1. The following statements characterized MAS Practice, except: A. The risk associated to multifarious ongoing changes in business, environment and conditions greatly contribute in making MAS Practice visibly important. B. CPA advisors must always maintain Independence and integrity despite so many situations that may negate their practical applicability. C. The practice of advisory services is too broad to include those outside the traditional professional services of CPA’s and allow him to deviate from the general code of professional involves from problem-solving affecting the future operations of the client. 2. What is one primarily benefit of an enterprise resource planning (ERP) system? A. It reduces inventory levels. B. It permits companies to be more streamlined in production. C. It replaces research and development in a company. D. It requires an increased emphasis on product quality. 3. Which one of the following is classified as direct labor? A. Flour in a bakery C. Bottles of cola in a bottling company B. Wages of factory-janitors D. Coy machine cost of a copy shop E. 4. What is value chain management best defined as? A. A large chain that keeps the machines from falling on the production floor B. Management decisions that affect how quickly the production run occurs C. The increment value of costs associated with hiring a new production floor leader D. All activities associated with providing a product or service F. 5. For what reason is a production cost primarily tracked? A. To keep with current cost trends to being products to market B. To effectively allocate the cost of products to measure profitability C. To determine what amounts are necessary for allocating overhead D. To keep management’s bonuses increasing every year G. 6. The following activity and cost data were provided by: Hammer Corp., which would like help in estimating its future maintenance costs: H. Units – Maintenance Cost I. 3 P450 J. 7 P530 K. 11 P540 L. 15 P700 M. Using the least-squares regression method to estimate the cost formula, the expected total cost for an activity level of 10 units would be closest to; A. P612.50 C. P595.84 B. P581.82 D. P601.50 A1 PASSERS REVIEW CENTER///1 E. 7. Selected information about Buehler Corporation’s operations at high and at low levels of activity follow. F. G. Level of Activity Low High H. Number of units produced 25,000 30,000 I. Total manufacturing costs P575,000 P680,000 J. Direct material cost per unit P5 P5 K. Direct labor cost per unit P6 P6 L. Using the high-low method, what is the cost formula for manufacturing overhead? A. P50,000 per period plus P10 per unit C. P50,000 per period plus P22 per unit B. P50,000 per period plus P21 per unit D. P347,000 per period plus P0.10 per unit E. 8. Expected sales for next year for the Jona Division are 120,000 units. Nestor Carey the manager of Jona Division, is under pressure to improve the performance of the Division. As he plans for next year, he has decide whether to produce 120,000 units or 140,000 units. The Jona Division will have higher profit, if Nestor Carey decides to A. Produce 140,000 units if income is measured under absorption costing. B. Produce 140,000 units if income is measured under variable costing. C. Produce 120,000 units if income is measured under absorption costing. D. Produce 120,000 units if income is measured under variable costing. F. 9. Which is not true concerning sales mix? A. Sales mix is the relative percentage in which each product is sold when a company sells more than one product. B. Sales mix is important to managers because different products often save. C. Sales mix does not affect breakeven analysts D. The computation of weighted average unit contribution margin is useful in sales mix analysis. G. 10. During the last year, Ruth Company had profit under absorption costing that was P5.500 lower than its income under variable costing. The company sold 9,000 units during the year, and its variable costs were P10 per unit, of which Pt was variable setting expense. If fixed production cost is P5 per unit under absorption costing every year, then how many units did the company produce during the year? A. 7,525 units. C. 10,100 units. B. 8,450 units. D. 7,900 units. E. 11. Last year, Francis Company had 20,000 units in its ending inventory. During the year, Francis variable production costs were P12 per unit. The fixed manufacturing overhead cost was P8 per unit in the beginning inventory. The company profit for the year was P9,600 higher under variable costing than it was under absorption costing. Given these facts, the number of units of product in the beginning inventory last year must have been: A. 21,200. C. 18,800. B. 19,200. D. 19,520 E. F. Questions 12 through 13 are based on the following information: G. Jason Company’s variable costing income statement for August appears below: H. Jason Company I. Income Statement J. For the month ended August: 31 K. Sales (P15 per unit) P600,000 L. Less variable costs: M. Variable cost of goods sold: N. Beginning inventory P72,000 O. Add variable CGM 315,000 P. Goods available for sale 387,000 Q. Less ending Inventory 27,000 R. Variables cost of goods sold 360,000 S. Total variable costs 80,000 T. Contribution margin 440,000 U. Fixed costs: 160,000 V. Fixed manufacturing 105,000 W. Fixed selling and administrative 35,000 X. Total fixed costs 140,000 Y. Profit P20,000 Z. The company produces 35,000 units each month. Variable production costs per unit and total fixed costs have remained constant over the past several months. AA.12. The peso value of the company’s inventory on August 31 under the absorption costing method would be: A. P27,000. B. P42,000. A1 PASSERS REVIEW CENTER///2 C. P36,000. D. P47,000. E. 13. Under absorption costing for the month ended August 31, the company woul report a : A. P20,000. C. P35,000 profit. B. P5,000. D. P5,000. E. 14. ABC had P400,000 budgeted fixed costs and based its standard and normal activity of 40,000 units. Actual fixed overhead costs were P430,000, actual production was 36,000 units, and sales were 30,000 units. The volume variance was A. P30,000. C. P70,000. B. P40,000. D. P77,777. E. 15. Myla Inc. makes a single product that sells for P50. The standard variable manufacturing cost is P32.50 and the standard fixed manufacturing cost is P7.50 based on producing 20,000 units. During the year Myla produce 22,000 units and sold 21,000 units. Actual fixed manufacturing costs were P157,000; actual variable manufacturing costs were P735,000. Selling and administrative expenses, all fixed, were P75,000. There were no beginning inventories. F. Which costing model, absorption or variable, has a higher profit and how much? A. Absorption, P7,500. C. Absorption, P25,000. B. Variable, P7,500. D. Variable, P15,000. E. 16. Which one of the following is true about graph of a mixed cost? A. The portion of the cost that is fixed cannot be determined from the graph. B. There are three lines on the graph for this cost-total variable cost total fixed costs, revenue. C. The fixed cost portion of the graph is the same amount at all levels of activity. D. The variable cost portion of the graph is rectangular shape. F. 17. Company A and Company B sell their product for exactly the same sales price. Both have the same annual fixed costs, Company’s A variable and fixed costs at break even total P60,000 and P30,000 respectively. Company B’s variable and fixed costs at break even total P30,000 and P60,000 respectively. Both companies have the same profit. If both companies experience as increase in sales, which company will have the higher profit? A. Company A B. Company B C. Both companies will report the same profit since total cost are the same. D. More information is needed to determine the answers. G. 18. Jonathan Corporation sells its product for P40. The variable are P18 per unit. Fixed costs are P16,000. The company is considering the purchase of an automated machine that will result in a 2 reduction in unit variable and an increase of P5,000. In fixed costs. Which of the following is true about the break-even point in units? A. It will remain unchanged. D. It cannot be determined from the information B. It will decrease. provided. C. It will increase. E. 19. Which of the following statements is not true> A. Operating leverage refers to the extent to which a company’s profit reacts to a given change in sales. B. Companies that have higher fixed costs relative to variable costs have higher operating leverage. C. When a company’s sales revenue is increasing highly operating leverage is a good thing because it means that profits will increase rapidly. D. When a company’s sales revenue is decreasing high operating leverage is a good thing because it means that profits will decrease at a slower pace than revenues decrease. F. 20. Pinoy Buhay-Buhay is a nonprofit organization that captures stray deer from residential communities. Fixed costs are P10,000. The variable cost of capturing each deer is p10.00 each. How many deers can Pinoy Buhay-Buhay capture during 2012? A. 2,200. C. 4,200. B. 3,200. D. 2,000. E. 21. Ruth Company’s degree of operating leverage is 2.0. Myla Corporation’s degree of operating leverage is 0.0. If the sales of the companies are expected to increase by 10%, the increase (decrease) in the profit before interest and tax of Ruth as compared to Myla would be: A. 30% higher. C. 3 times higher. B. 2 times higher. D. 33% lower. E. Questions 22 and 23 are based on the following information. F. Below are income statements that apply to ABC Company’s three product lines: G. Product A Product B Product C H. Sales P100 P100 P100 I. Variable costs (10) (20) (30) J. Contribution margin P90 P80 P70 K. Fixed costs (30) (20) (10) L. Profit before taxes P60 P60 P60 M. A1 PASSERS REVIEW CENTER///3 N. 22. Within the relevant range. If sales go up by P1 for each firm which product line will experience the greatest increase in profit? A. Company A C. Company C B. Company B D. Can’t be determined from the information given E. 23. At sales of P100, what is the difference in the margin of safety rate between the two product lines having the highest profitability performance? A. 87.71% C. 66.67% B. 75.00% D. 10.71% E. F. Vinzon Company produces and sells three products that, after with their relative quality and price structure as follows: G. H. De Luxe I. Premium J. Economy K. Unit sales L. M. N. price P400 P200 P100 O. Unit variable P. Q. R. rate 150 120 75 S. Budget T. U. V. sales in 1,000 3,000 6,000 units W.Direct fixed X. P2.5 Y. P4.0 Z. P1.2 cost million million million AA. AB.In the next counter, the following changes are being considered. AC. AD. De Luxe AE.Premium AF. Economy AG. Unit sales AH. Increase AI. AJ. Decreased price d by 10% by 10% AK.Unit AL. AM. Decreas AN. Increased variable ed by by 10% rate 15% AO. Budget AP.Increase AQ. AR. sales in d by 20% units AS.Direct fixed AT. AU. Increase AV. Decreased cost d by 40% by 10% AW. AX. 24. After embedding the changes, what is the difference break even point its between the two least profitable products in terms of pesos? A. 57,143 C. 92,857 B. 150,000 D. 8,621 E. 25. Which one of the following is a critical factor in budgeting for a service company? A. Coordinating professional staff needs with and operated services. B. Determining how to allocate the disproportionate costs. C. Budgeting expenditures before anticipated receipts. D. Determining client needs. F. 26. The performance report of the Bohol Division of South Inc. showed a difference between the budget and the actual results for the year. Management determines this difference was controllable by the manager in charge. Should the division manager be held responsible? A. No, since budget difference over time. B. Yes, because managers are responsible for controllable cost for their department. C. Only if the difference is favorable. D. Yes, manages are responsible for all cost in their division. G. 27. Which of the following includes people who make up the budget committee? A. Sales manager, company president treasurer. C. Sales manager, controller, investors. B. Company treasurer, creditors, controller. D. External auditors, controller, treasurer E. 28. How does a sales forecast from states budget? A. A sales forecast includes the company, while a sales budget includes the industry. B. A sales forecast includes the company and the industry, while a sales budget includes only the industry. C. A sales forecast includes the company and the industry, while a states budget includes only the company. D. They are both the same. F. 29. Burnham Park Company makes and sells umbrella. The company is in the process of preparing its selling and administrative Expense Budget for the last half of the year. The following budget data are based on 20,000 units: A1 PASSERS REVIEW CENTER///4 G. Item H. Variable Cost I. Unit Sold P0.60 P1.30 P0.30 J. Monthly Fixed K. Cost P3,000 L. Sales commissions M. Shipping N. Advertising O. Depreciation on office Equipment P4,000 P. Other Operating expenses P0.35 P34,000 Q. R. Expenses are paid in the month incurred. If the company has budgets to spend P90,900 for the expenses, how many unit does the company expect to sell in the last half of the year? A. 20,000 C. 37,102 B. 22,000 D. 20,367 E. F. G. 30. The production budget shows expected unit sales are 1,800. The required production units are 1,700. Which of the following represents possible inventory balance? H. Beginning Units Ending Units A. 200 100 B. 100 200 C. 200 200 D. 0 100 I. 31. Bayantel Company showed the following on its direct materials budget for June: J. Units to be produced 50,000 K. Total pounds needed for production 4,000 L. Total pounds of materials to be purchased 5,000 M. Increase in materials inventory 400 N. O. The materials cost P2 per pound. How much is the cost of direct per unit? A. P 0.16 C. P20 B. P25 D. P 0.20 E. 32. Bajada Company reported the following information for 2012; F. G. Septem H. October I. Novemb J. Dece K. Janu ber er mbe ary r L. Budgeted Sales M. P240,0 N. P310,000 O. P250,00 P. P360, Q. P20 00 0 000 0,0 00 R. Budgeted S. P90,00 T. P120,000 U. P128,00 V. P144, W.P88, purchases 0 0 000 000 X.  Cost of goods sold is 35% of sales.  Bajada purchase and pays for merchandise 60% in the month of acquisitions and 40% in the following month.  Account payable is used only for inventory acquisitions. Y. How much is the budgeted balance for Accounts Payable at October 31, 2012? A. P48,000 C. P102,000 B. P72,000 D. P51,200 E. 33. All-Purpose Equipment, Inc. is planning to sell 200 buckets and produce 190 buckets during March. Each buckets requires 500 grams of plastic and one half hour of direct labor. Plastic costs P10 per 500 grams and employees of the company are paid P15.00 per hour. Manufacturing overhead is applied at a rate of 10% of direct labor costs. All-Purpose Equipment has 300 kilos of plastic in beginning inventory and wants to have 200 kilos in ending inventory. How much is the total amount of budgeted labor for March. A. P1,500. C. P1,425. B. P3,000. D. P2,850. E. 34. If a company is concerned with the potential negative effects standards, they should. A. Set loose standards that are easy to fulfil. C. Not employ any standards. B. Offer wage incentives to those meeting D. Set tight standards in order to motivate people. standards. E. 35. Which of the following statements is false? A. The overhead volume variance whether plant facilities were used efficiently during the period. B. The costs that cause the overhead volume variance are usuDurian controllable cost. C. The overhead volume variance relates solely to fixed cost. A1 PASSERS REVIEW CENTER///5 D. The overhead volume variance is favorable if standard hours allowed to output is greater than the standard hours at normal capacity. F. 36. An unfavorable labor quantity variance may be caused by. A. Paying workers higher wages than expected. C. Worker fatigue or carelessness. B. Paying workers a bonus at a year end. D. Higher pay rates mandated by union contacts. E. 37. The standard number of hours that should have been worked for the output attained is 8,000 direct labor hours and the actual number of direct labor hours worked was 8,400. If the direct labor price variance was P8,400 unfavorable, and the standard race of pay was P16 per direct labor hour, what was the actual rate of ay for direct labor? A. P17.00 per direct labor hour C. P15.00 per direct labor hour B. P17.00 per direct labor hour D. P18.00 per direct labor hour E. Questions 38 and 37 are based on the following information. The controller for Oriental Gravity is reviewing the production cost report for July. Ari analyst of direct materials costs reflects an unfavourable flexible budget variance of P25,000. The plant manager believes this is an excellent performance on a flexible budget for 5,000 units of direct materials. However, the production supervisor is not pleased with this result because he claims to have saved P1,200 in materials cost on actual production using 4,900 units of direct materials. The standard materials costs is P12.00 per unit. Actual materials used for the month amounted o P60,025. F. 38. The actual average cost per unit of materials was A. P12.00 C. P12.24 B. P12.01 D. P12.25 E. F. G. 39. If the direct materials variance is investigated further. It will reflect a price variance of: A. Zero. C. P1,225 unfavorable. B. P1,176 favorable. D. P1,250 unfavorable. E. 40. The Dongyi Company has a standard cost system. In July the company purchased and used 22,500 pounds of direct material at an actual cost of 53,000; the materials quantity variance was P1,875 UF; and the standard quantity of materials allowed for July production was 21,750 pounds. The budgeted materials for the period are 23,000 pounds for purchases and 24,000 for usage. F. G. Also during the period, the company incurred 12,540 direct labor hours and paid 5,300,000 for plant payroll. The materials price variance for July was 24,000. A. P2,725 F. C. P3,250 F. B. P2,725 U. D. P3,250 U. E. 41. The Reedy Company uses a standard costing system. The following data are available for F. November: G. Actual direct labor hours worked 5,800 hours H. Standard direct labor rate P9 per hour I. LAbor rate variance P1,150 favorable J. Standard pounds per hour K. Standard variable rate per hour P10 L. Normal Capacity in hours 6,000 M. The actual direct labor rate for November is: A. P8.80. C. P9.00. B. P8.90. D. P9.20. E. 42. Borden Enterprises uses standard costing. For the month of April, the company reported the following data:  Standard direct labor rate: P10 per hour  Standard hours allowed for actual production: 8,000.  Actual direct labor rate: P9.50 per hour  Labor efficiency variance: P4,800 F F. The labor rate variance for April is: A. P3,760 U. C. P2,850 F. B. P3,760 F. D. P2,850 U. E. Questions 43 and 44 are based on the following information: F. The Lon Company makes and sells a single product called a Grix and employs a standard costing system. The following standards have been established for one unit of Grix. G. Standard Quantity of Hours Standard Cost per Grix H. Direct Materials 6 board feet P7.00 I. Direct labor 0.8 hours P9.60 J. There were no inventories of any kind on August. During August, the following events occurred:  Purchased 15,000 board feet at the total cost of P24,000. A1 PASSERS REVIEW CENTER///6  Used 12,000 board feet to produce 2,100 Grix.  Used 1,700 hours of direct labor time at a total cost of P20,060. K. 43. To record the purchase of direct materials, the general ledger would include what entry to the Materials Price Variance Account? A. P1,500 credit. C. P6,000 credit. B. P1,500 debit. D. P6,000 debit. E. 44. To record the incurrence of direct labour cost and its use in production, the general ledger would include what entry to the Labor Efficiently Variance account? A. P480 credit. C. P1,200 debit. B. P240 debit. D. P1,200 credit. E. F. Questions 45 and 46 are based on the following information: G. The following standards for variable manufacturing overhead have been establish for a company that makes only one product: H. Standard hours per unit of output 1.5 hours I. Standard variable overhead rate P11.55 per hour J. The following data pertain to operations for the last month: K. Actual hours 4.900 hours L. Actual total variable overhead cost P58,310 M. Actual output 3,000 units N. 45. What is the variable overhead spending variance for the month? A. P2,870 U. C. P1,715 U. B. P2,870 F. D. P1,715 F. E. F. 46. What is the variable overhead efficiency variance for the month? A. P1,680 F. C. P1,155 U. B. P1,190 U. D. P1,190 F. E. F. Questions 47 through 49 are based on the following information: G. Durian Corporation produces a model plastic casing, TD4 for desktop computers. H. Summary data from its 2012 income statements are as follows: I. Revenues P 5,000,000 J. Variable cost 3,000,000 K. Fixed cost 2,160,000 L. Operating income P (160,000) M. Aro N. Cillo, Durians president, is very concerned about Durian Corporation’s poor profitability. She asks Cove Yellow, production manager, and T.D Puyat, Controller, to see if there are ways to reduce costs. N. After two weeks, Cove returns with a proposal to reduce variable costs to 52% of revenues by reducing the costs. Durian currently incurs for safe disposal of wasted plastic. T.D Puyat concerned that this would expose the company to potential environment cost and include them in our analysis. “You can’t do that,” Cove replies. “We are not violating any laws. There is some possibility that we have to incur environmental costs in the future, but if we bring it up now this proposal will not go through because our senior management always assumes these costs to be larger than they turn out to be. The market is very tough and we are in danger of shutting down the company. We don’t want all our colleagues to lose their jobs. The only reason our competitors are making money is because they are doing exactly what I am proposing. O. 47. What is Durian Corporation’s breakeven revenues for 2012? A. P5,000,000. C. P3,500,000. B. P4,900,000. D. P5,400,000. E. 48. What is Durian Corporation’s breakeven revenues if variable costs are 32% of revenues? A. P4,500,000. C. P5,000,000. B. P5,500,000. D. P3,900,000. E. 49. What is Durian Corporation’s operating income for 2012 if variable costs had been 52% of revenues? A. P240,000. B. P200,000. C. P180,000. D. P260,000. F. G. H. I. A1 PASSERS REVIEW CENTER///7 J. K. L. M. N. O. P. Q. R. S. T. U. V. W. X. Y. Z. AA. AB. AC. AD. AE. AF. AG. AH. AI. AJ. AK. ANSWER KEY 1. C 2. A 3. C 4. C 5. B 6. C 7. B 8. A 9. C 10. D 11. C 12. C 13. A 14. B 15. A 16. C 17. A 18. C 19. D 20. A 21. A 22. A AL. Management Advisory AM. 23. C 24. C 25. B 26. B 27. A 28. C 29. B 30. 31. A 32. A 33. C 52. 34. D 35. D 36. C 37. C 38. D 39. D 40. D 41. A 42. B 43. D 44. A 45. C 46. A 47. D 48. A 49. A 50. 51. 53. 54. 55. 56. 57. 58. 59. 60. 61. 62. 63. 64. 65. 66. A1 PASSERS REVIEW CENTER///8 67. 68. 69. 70. 71. 72. 73. 74. 75. 76. 77. 78. 79. 80. 81. 82. 83. 84. 85. 86. 87. A1 PASSERS TRAINING, RESEARCH, REVIEW & DEVELOPMENT COMPANY 88. 2nd Floor Sommerset Bldg., Lopez Jaena St. Jaro, Iloilo City 89. Tel. No.: (033) 320-2728; 09106547262 90. Email Address: [email protected] 91. 92. BOARD OF CERTIFIED PUBLIC ACCOUNTANT 93. 94. CERTIFIED PUBLIC ACCOUNTANT Licensure Examination 95. 96. AUDITING THEORY 97. 98. 1. Which of the following defines the specific requirements of the Securities and Exchange Commission (SEC) regarding financial reporting of entitles registered with the SEC? A. SRC Rule B. SEC Circulars C. SRC Rule 68 D. SEC Bulletins 99. 2. SRC Rule 68, as amended, is effective for audited financial statements covering periods ended 100. and onwards, and for Interim financial statements starting the 101. , and thereafter. 102. 3. Part I of SRC Rule 68, as amended, deals with the general financial reporting requirements of the SEC to the following entities: (For each covered entity, indicate the required threshold must be met.) 103. 104. Covered Entity Thresholds a. Stock corporations b. Non-stock corporations c. Branch offices of stock 105. foreign corporations d. Brand offices of non-stock 106. corporations e. Regional operating headquarters 107. of foreign corporations A1 PASSERS REVIEW CENTER///9 108. 4. In addition to the financial reporting requirements under Part I of Rule 68, as amended, additional requirements are set forth under Part II of the Rule. The following issuers of securities to the public must comply with the requirements under this part of the Rule: a. Issuer which has sold a class of their securities pursuant to a registration under Section 12 of the Securities Regulation Code (SRC). b. Issuer with a class of securities listed for trading on an Exchange; and c. Issuer with assets of at least P50 million and has 200 or more holders each holding at least 100 shares of a class of its equity securities as of the first day of the Issuer’s fiscal year. 109. 5. Rule 68, as amended, deleted the term “Generally Accepted Accounting principles (or GAAP) and replace it with . 110. 111. 112. 6. Large and/or publicity accountable entities shall use as their financial reporting framework the 113. a. 114. b. 115. 116. 7. Small and medium-sized entitles (SMEs) shall use as their Financial Reporting Framework 117. the . 118. 119. 8. If an SME that uses the PFRS for SMEs In a current year the floor or ceiling of the size criteria at the end of that current year, and the event that caused the change is considered “significant and continuing”, the entity shall transition to the applicable financial reporting framework in the next accounting period. 120. The assessment of “significant and continuing” shall be based on management’s judgement taking into consideration relevant qualitative and quantitative factors. 121. As a general, rule, would be considered significant. 122. 123. 9. Rule 68, as amended, prescribed the wording of the Statement of Management’s Responsibility (SMR) for Financial Statements that shall be attached to the financial statements filed with the SEC. All of the following shall sign the SMR: a. b. c. 124. 10. The failure of any of the prescribed signatories to sign the SMR constitutes a . 125. 126. 11. In the case of branch offices or regional operating headquarters of foreign corporations, the SMR shall be signed by its who is in charge of its operations within the Philippines. 127. 128. 12. The SMR of the entities covered under Part II of Rule 68, as amended, shall be signed 129. . 130. 131. 13. A regulated agency shall report to the SEC its action on a report of its independent auditor pertaining to the following: a. Any material findings involving fraud or error. b. Losses or potential losses the aggregate of which amounts to at least 10% of the consolidated total assets of the entity. c. Any finding to the effect that the consolidated assets of the entity, on a going concern basis, are no longer adequate to cover the total claims of creditors; and d. Material internal control weaknesses which may lead to financial reporting problems. 132. 14. A regulated agency shall submit the required report to the SEC within from the date the report is submitted by the independent auditor. 133. 134. In the case regulated agency falls to submit the required report, the Independent auditor shall, within 135. from the submission of his findings to the entity, file a report to the SEC. 136. 137. 15. Report of Independent Auditor when the Company Incurred Capital Deficiency. 138. The external auditor of a company which has incurred a capital deficiency shall provide in the audit report an Indicating the following Information: a. The fact that the company has incurred a capital deficiency that raises an issue on its going concern status. A1 PASSERS REVIEW CENTER///10 b. A brief discussion of a concrete plan of the company to address the capital deficiency and reference to the note to financial statements that provides a complete disclosure of the said plan; c. A statement that the auditor conducted sufficient audit procedures for verify the validity of the aforementioned plan. 139. 140. In case the company falls to present to the external auditor a concrete plan or sufficient supporting documents to address the capital deficiency, the auditor shall provide an Indicating that the company is no longer a going concern and should use liquidation basis in the preparation of its financial statements. 141. 142. 16. It is considered as a violation of Rule 68, as amended, If the audited financial statements covered by Part II of the Rule with an auditors opinion other than unqualified arising from the following: a. b. 143. 17. For , a qualified opinion from the Independent auditors shall not be considered a non-compliance with Rule 68, as amended, if the qualification pertains to a deviation from the financial reporting framework adopted by the Bangko Sentral ng Pilipinas as part of its prudential reporting requirements. 144. 145. 18. The financial statements that may be found by the SEC to be deficient and/or misstated, the Commission shall make a determination whether such misstatement or Incompleteness is significantly material that would necessitate the of the financial statements. 146. 147. 19. Corporation covered by PART II of Rule 68, as amended, shall not reissue their financial statements without . 148. 149. 20. Under Rule 68, as amended, error means an act by one or more individual among management, employees, or third parties that results in a misrepresentation of financial statements which reduces or increases the consolidated total assets, total liabilities or income of the company by . 150. 151. 21. Under Rule 68, as amended, error means an unintentional mistake in the financial statements which reduces or increases the consolidated total assets, total liabilities or income of the company 152. by _____________________ . 153. 154. 155. 22. The financial statement of companies not covered by SRC Rule 68 should be accompanied 156. by a . 157. Such financial statements should have at least a: a. Statement of Financial Position Fund Balance. b. Income Statement (or a Statement of Receipts and Disbursements) and c. Applicable explanatory notes. 158. 159. 23. Companies with no operation but are covered under Rule 68: a) If no operation only for 1 year, a must be submitted by the company despite its nonoperation. b) If no operation for the last 2 years, the need not be included in the audited financial statements. 160. 161. 24. It is the responsibility of the entity’s management to provide all schedules, reports, computations/projection, reconciliations, reports, analyses, and other financial Information collectively referred to as in PAPS 1000ph. 162. 163. 25. Which of the following procedure is Inappropriate when the entity’s management is unable to provide the audit Information requirements? A. The auditor shall prepare the audit Information requirements. B. The auditor shall consider assisting the client, for example by preparing a pro forma recondition statement or schedule format for the client to complete or use as a guide. C. The auditor shall discuss with the management and/or those changed with governance the possibility of outsourcing the preparation of audit Information requirements to third party service providers. D. The auditor shall evaluate the reasons why management is unable to prepare the audit Information requirements. 164. A1 PASSERS REVIEW CENTER///11 165. 26. In situations where the client provided Incomplete, and Insufficient audit Information requirements, the auditor shall consider the following except. A. Communication in writing the issues and concerns on the Incomplete and Insufficient audit Information requirements with management and/or those changed with governance. B. Discuss if the audit information requirements can be revised to meet adequacy the needs of the auditor. C. Express a qualified or an adverse opinion. D. Discuss with management and/or those charged with governance the possibility of outsourcing the completion of the information to third party service providers. 166. 167. 27. The term “ “ is generally used at present to refer to a set of incomplete or unaudited financial statements accompanied by a report of an external auditor who has not yet completed his audit of such financial statements. 168. 169. 28. The BIR accepts incomplete or unaudited financial statements to accompany tentative ITRs. (True or False) 170. 171. 29. To avoid any association with incomplete or unaudited financial statements, no report (even a report containing a disclaimer of opinion) shall be issued by the auditor on such incomplete or unaudited financial statements. (True or False) 172. 173. 30. PAPS 1001Ph (Guidance in Dealing with Tentative Financial Statements) shall be applied to situations where the auditor has completed an audit and has concluded that there are scope limitations that affected the audit giving rise to either a qualified opinion or disclaimer of opinion. (True or False) 174. 175. 31. The purpose of PAPS 1002Ph is to provide additional guidance on the application of the existing PSA 700, specifically on the description of the applicable Philippine financial reporting framework when this is other than Philippine Financial Reporting Standards. (True or False) 176. 177. 178. 32. Which of the following types of engagements may be undertaken by an auditor in connection with a corporation’s application for an increase in its authorized capital stock? I. An engagement to perform agreed-upon procedures regarding-financial information. II. An engagement to report on supplementary information in connection with an audit of financial statements. III. An engagement to report on a specific element, account or item of a financial statement. A. I and II only. C. II and III only. B. I and III only. D. I, II, and III. 179. 33. An agreed-upon procedures engagement Is to be performed if the application for increase in authorized capital stock is filed long after (e.g., more than 120 days) the date of the most recent audited financial statements submitted to the SEC and even if the application is closed to the most recent audited financial statements submitted, the corresponding information in such financial statements is considered by the SEC to be insufficient. (True or False) 180. 34. The auditor may issue a report on supplementary information if I. The application for increase in authorized capital shock is made on a date close (e.g., 120 days or less) to the date of the most recent audited financial statements submitted to the SEC. II. The applicable information in the audited financial statements is considered sufficient by the SEC. A. I only. C. II only. B. Neither I nor II. D. Both I and II. E. 35. Which of the following statements concerning the form and content of Inter-bank confirmations is Incorrect? A. The form and content of a confirmation request letter will depend on the purpose for which it is required, on local practices and on the requesting bank’s account. B. Not all information for which confirmation is usually sought will be required at the same time. C. The confirmation request should be prepared in a clear and concise manner to ensure ready comprehension by the confirming bank. D. Whenever possible, the confirmation request should be prepared in the language of the requesting bank or in the language normally used for business purposes. F. 36. Which of the following are required to be reported by the external auditor to the BSP? I. Any material finding during the audit involving fraud or dishonesty which will reduce capital funds by at least one percent (1%). II. Adjustments or potential losses amounting to at least one percent (1%) of capital funds of the bank. A1 PASSERS REVIEW CENTER///12 III. Any finding to the effect that the total bank assets, on a going concern basis, are no longer adequate to cover the total claims of creditors. A. I and II only. B. I and III only. C. II and III only. D. I,II, and III. G. 37. The most significant of banking risks, in terms of historical loss experience, is the risk that a customer or counterparty will not settle an obligation for full value. This risk is sometimes referred to as A. Market risk B. Liquidity and funding risk C. Credit risk D. Operational risk H. 38. An external auditor monitors the present and future viability of banks and uses their financial statements in assessing their condition and performance. (True or False) I. The BSP is primarily concerned with reporting on the bank’s financial statements ordinarily either to the bank’s shareholders or board of directors (True or False) J. 39. As defined in PAPS 1005, a is any entity in which: a. There is concentration of ownership and management in a small number of individuals (often a single individual); and b. One or more of the following are also found:  Few sources of income;  Unsophisticated record-keeping;  Limited internal controls together with the potential for management override of controls. K. 40. Identify the following BANKING RISKS: a. The risk of the foreign customers and counterparties falling to settle their obligations because of economic, political, and social factors of counterparty’s home country and external to the customer or counterparty. Counter Risk b. The risk of loss arising from future movements in the exchange rates applicable to foreign currency assets, liabilities, rights and obligations. Currency risk c. The risk of loss arising from factors such as failure to maintain safe custody or negligence in the management of assets on behalf of other parties. Fiduciary risk d. The risk that a movement in interest rates would have an adverse effect on the value of assets and liabilities or would affect interest cash flows. Interest rate risk e. The rate that contacts are documented incorrectly or are not legally enforceable in the relevant jurisdiction in which the contracts are to be enforced or where the counterparties operate. Legal and documentary risk f. The risk of loss arising from the changes in the bank’s ability to sell or dispose of an asset. Liquidity risk g. The risk associated with the imperfections and subjectivity of valuation models used to determine the values of assets or liabilities. Modelling risk h. The risk of direct or Indirect loss resulting from inadequate or failed internal process, people and systems or from external events. Operational risk i. The risk of losses arising from adverse changes in market prices, including interest rates, foreign exchange rates, equity and commodity prices from the movements in the market prices of investments. Price risk j. The risk of loss arising from failure to comply with regulatory or legal requirements in the relevant jurisdiction in which the bank operates. Regulatory risk k. The risk of failure of a customer or counterparty to perform the terms of a contact. This failure creates the need to replace the failed transaction with another at the current market price. It is sometimes called performance risk. Replacement risk l. The risk of losing business because of negative public opinion and consequential damage to the bank’s reputation arising from failure to properly manage some of the above risk, or from involvement in improper or illegal activities by the bank or its senior management, such as money laundering or attempts to cover up losses. Reputation risk m. The risk that one side of a transaction will be settled without value being received from the customer or counterparty. This will generally result in loss to the bank of the full principal amount. Settlement risk n. The risk of loss of arising from the possibility of the bank not having sufficient funds to meet his obligations, or from the bank’s inability to access capital markets to raise required funds. Solvency risk o. The risk of loss arising when a counterparty’s obligations is not denominated in the counterparty’s home currency. The counterparty may not obtain the currency of the obligation irrespective of the counterparty’s particular financial condition. Transfer risk A1 PASSERS REVIEW CENTER///13 L. 41. In the exercise of its power to conduct an oversight into the quality of audits, the BOA organized the M. Department of the Philippine Institute of Certified Public Accountant (PICPA) to conduct an independent study, appraisal, or review of the quality of audit of financial statements. A. Quality Assurance Review C. Independent Quality Assurance Review B. Quality Control Review D. Independent Quality Control Review E. 42. The operation of the Quality Assurance Review Department (QARD) of the PICPA shall be supervised by a /an A. Executive Committee C. Oversight Committee B. Supervisory Board D. Accountability Board E. 43. Which of the following is an Incorrect statement concerning of office of the Executive Committee members? A. The term of office of the Chairman and members shall be co-terminus with their respective term in the BOA or PICPA unless replaced earlier by the Board or ICA B. The terms of office of the chairman and members in no case shall be more than 3 years. C. The chairman and members may continue to be in active practice of public accountancy during their term subject to certain restrictions imposed by the Board. D. The chairman of the Executive Committee shall be from the Board’s appointees. F. 44. The head of the QARD who will be appointed by the Executive Committee is the A. Chief Inspector C. Chief Auditor B. Chief of Administration D. Chief Quality Assurance Reviewer E. 45. Who is responsible for the non-technical aspect of the QAR program? A. Chief Inspector C. Assistant to the Chief Inspector B. Chief of Administration D. Chairman of the Executive Committee E. 46. The QAR program covers all CPAs in A. Public Accounting C. Commerce and Industry B. Government D. Education/Academic E. 47. CPAs in public practice required to register with the QAR program under the prescribed category. Those whose clients are public-interest entities (listed and not-listed but with public accountability) should register under A. Category A C. Category C B. Category B D. Category D E. F. G. Section 19 provides that CPA practitioners should be registered in accordance with the following categories:  Category A – registration for CPA practitioners handling clients that use the full IFRS or their Philippine equivalents. These would cover CPA practitioners auditing public-interest entities (listed and not-listed but with public accountability)  Category B - registration for CPA practitioners handling clients that use the full IFRS or their Philippine equivalents for Small and Medium-sized enterprises. H. 48. What is the effectivity date of the BOA Resolution No. 88 (Series of 2008) prescribing the rules and regulations for the accreditation of accounting teachers? A. June 4, 2008 C. December 30, 2008 B. June 20, 2008 D. June 30, 2008 E. 49. Which of the following requirements for the accreditation of accounting teachers shall not apply to those CPAs already engaged in teaching as of the effectivity date of the Rules and Regulations? I. Possession of relevant Master’s degree. II. Completion of 12 units of relevant education subjects from the CHED recognized schools. III. A total of 3 years meaningful experience in actual accounting work either in Public Practice, Commerce & Industry or Government sector. IV. Proof that the CPA has undergone Continuing Professional Education (CPE). A. I and II only C. I and IV only B. II and III only D. II only 50. The following statements relate to the complete 12 units of relevant education subjects for the accreditation of accounting teachers. Which is correct? A. For purposes of compliance, the 12 units may be earned from the undergraduate education program or from a graduate degree program of any Higher Education Institution (HEI) duly recognized by CHED. B. The 12 units may be earned from In-service or In-house trainings on relevant education subjects offered by schools or training centers. C. The 12 units may be a combination of In-service trainings and units earned in an undergraduate or graduate education programs. A1 PASSERS REVIEW CENTER///14 D. CPAs who have passed the Teachers Board Exams and are licensed Professional Teachers should earn the 12 units through in-service or in-house service. 51. Which of the following shall be considered proof of compliance with the CPE requirement for accreditation of accounting teachers? I. Certification of CPE units from accredited CPE providers issued by the PICPA. II. Certificate of Attendance or other proofs of meaningful participation in other CPE programs as approved by the PRC/CPE Council upon recommendation of the PICPA of the individual CPA of a minimum of sixty (60) credit units earned for the past immediate 3 years. A. I only. C. Either I or II. B. II only. D. Neither I nor II. 52. The Certification of Accreditation issued by the PRC to an accounting teacher shall A. Be valid for 2 years and renewable every 2 years. B. Be valid initially for 3 years and renewable annually. C. Remain in full force and effect unless revoked, cancelled or withdrawn. D. Be valid for 3 years and renewable every 3 years. 53. According to the Transitory Provisions of the BOA Resolution No. 88, any tenured/full time/full load faculty member who doest not meet the accreditation requirements as of the effectivity date of the Rules and Regulations may be issued a Provisional Accreditation. Choose the correct statement. I. The Provisional Accreditation will be valid for a period not exceeding 3 years unless earlier withdrawn, revoked, or cancelled for cause by the BOA. II. The Provisional Accreditation may be Issued only once and is not renewable. III. The transitory provision shall also apply to returning teachers who have not been teaching for the last 5 years. A. I only. C. II and III only. B. I and II only. D. I, II, and III 54. Issuance of exposure drafts on proposed Philippine Standards or Practice Statements requires the approval by A. A majority of the AASC members. C. At least 5 members of the AASC. B. At least 10 members of the AASC. D. All members of the AASC. 55. The time allowed for each exposure draft to be considered by the organizations and persons to whom the exposure draft is sent for comment is generally not shorter than A. 60 days B. 90 days C. 30 days D. 120 days 56. The AASC considers the comments and suggestions received as a result of the exposure of Philippine Standards and Practice Statements and revises the exposure draft when deemed appropriate. Such exposure draft is then issued as a final Philippine Standard or Practice Statement after meeting the approval of A. A majority of the AASC members. C. At least 5 members of the AASC. B. At least 10 members of the AASC. D. All the AASC members. 57. In case of an initial public offering (IPO) of securities by a company, the interim financial statements to accompany the Registration Statement (SEC Form 12-1, as amended) shall be audited by an external auditor accredited by the A. Board of Accountancy (BOA) B. Professional Regulation Commission (PRC) C. BOA/PRC D. BOA/PRC and SEC (Securities and Exchange Commission) 58. The COA chairman and the two (2) commissioners shall be appointed by the president of the Philippines with consent of the Commission on Appointments for a term of A. 5 years C. 7 years B. 6 years D. 9 years 59. The Code states that when a financial statements audit client becomes a listed entity, the length of time the engagement partner or the individual responsible for the engagement quality control review has served the audit client in that capacity should be considered in determining when the individual should be rotated. Before rotating off the engagement, the person is allowed to continue to serve as the engagement partner or as the individual responsible for the engagement quality control review for A.3 additional years C.2 additional years B.1 additional years D.4 additional years 60. The established of an overall audit strategy involves I. Determining the characteristics of the engagement that define its scope. II. Ascertaining the reporting objectives of the engagement to plan the timing of the audit and the nature of the communications required. A1 PASSERS REVIEW CENTER///15 III. Considering the important factors that will determine the focus of the engagement team’s effort. A. I and II only. C. I and III only. B. II and III only. D. I, II, and III. 61. In an initial public offering of securities, a written communication from the independent accountant to be underwriter about procedures the accountant performed on the unaudited financial statements and schedules accompanying a client’s registration statement . 62. Refers to a company’s maintaining a steady rate of earnings growth over a series of years, thereby providing no reason for the financial markets to impute unexpected risk in the company or in expected stock prices. 63. All the components whose financial information is included in the group financial statements . 64. Financial statements that include the financial information of more than one component. It also refers to combined financial statements aggregating the financial information prepared by components that have no parent but are under common control. 65. Historical financial information that is derived form financial statements but that contains less detail than the financial statements, while still providing a structured representation consistent with that provided by the financial statements of the entity’s economic resources or obligations at a point in time or the changes therein for a period of time. 66. A third-party organization (or segment of a third-party organization) that provides services to user entities that are part of those entities’ information systems relevant to financial reporting. 67. An auditor established a P180,000 tolerable misstatement for an asset with an account balance of P3,000,000. The auditor selected a sample of every 20th item from the population that represented the asset account balance and discovered a net overstatement of P10,500 (P11,100 overstatement minus P600 understatements). Under these circumstances, the auditor most likely would conclude that A. The asset account is fairly states because the tolerable misstatement exceeds the net of projected actual overstatement and understatement. B. The assets account is fairly stated because the total projected misstatement is less than the tolerable misstatement. C. There is unacceptably high risk that the actual misstatements in the population exceed the tolerable misstatement because the total projected misstatement exceeds the tolerable misstatement. D. There is an unacceptable high risk that the tolerance misstatement is more than the sum of actual overstatements and understatements. 68. 69. 70. A purpose of a management representation letter is to reduce A. The possibility of a misunderstanding concerning management’s responsibility for the financial statements. B. The scope of an auditor’s procedures concerning related party transactions and subsequent events. C. Audit risk to an aggregate level of misstatement that could be considered material. D. An auditor’s responsibility to detect material misstatements only to the extent that the letter is relied on. 71. To which of the following matters would materiality limits not apply in obtaining written management representations? A. Reductions of obsolete inventory to net realized value. B. The disclosure of compensating balance arrangements involving related parties. C. Losses from purchase commitments at prices in excess of market value. D. The availability of minutes of stockholders’ and directors meetings. 72. An entity’s management is responsible for the preparation and fair presentation of the financial statements. Its responsibility includes the following except A.Designing, implementing, and maintaining internal control relevant to the preparation and presentation of financial statements. B.Making accounting estimates that are reasonable in the circumstances. C.Selecting and applying appropriate accounting policies. D.Assessing the risks of materials misstatement of the financial statements. 73. Sam, CPA was engaged to audit the financial statements of Mantha Corp. after its fiscal year had ended. The timing of Sam’s appointment as auditor and the start of field work made confirmation of accounts receivable by direct communication with the debtors ineffective. However, Sam applied other procedures and was satisfied as to the reasonableness of the account balances. Sam’s auditors most likely contained a/an A. Qualified opinion because of a scope limitation. B. Qualified opinion because of a departure from GAAS. C. Unmodified opinion. D. Unmodified opinion with an emphasis of matter paragraph. 74. Under which of the following circumstances would a disclaimer of opinion not be appropriate? A1 PASSERS REVIEW CENTER///16 A. The financial statements fall to contain adequate disclosure concerning party transactions. B. The auditor is engaged after fiscal year-end is unable to observe the physical inventories or apply alternative procedures to verify their balances. C. The auditor is unable to determine the amounts associated with fraud committed by the clients management. D. The client refuses to permit its attorney to furnish-information requested in a letter of audit inquiry. 75. Comparative financial statements include the prior year’s statements that were audited by a predecessor auditor whose report is not presented. If the predecessor’s report was unqualified, the incoming auditor should A. Indicate in the auditor’s report that the predecessor auditor expressed an unqualified opinion. B. Express an opinion on the currents year’s statement alone and make no reference to the prior year’s statements. C. Obtain a letter of presentations from the predecessor auditor concerning any matters that might affect the incoming auditor’s opinion. D. Request the predecessor auditor to reissue the prior year’s report. 76. An auditor expressed a qualified opinion on the prior year’s financial statements because of a lack of adequate disclosure. The financial statements are properly restated in the current year and presented in comparative from with the current year’s financial statements. The auditor’s updated report on the prior year’s financial statements should. A. Make no reference to the type of opinion expressed on the prior year’s financial statements. B. Express an unmodified opinion on the restated financial statements of the prior year. C. Be accompanied by the auditor’s original report on the prior year’s financial statements. D. Continue to express a qualified opinion on the prior year’s financial statements. 77. PSA 800 applies to A. Review engagements D. Audit of financial statements prepared in B. Agreed-upon procedures engagements accordance with a special purpose C. Compilation engagements framework. 78. In an audit of special financial statements, the auditor shall obtain an understanding of I. The purpose for which the financial statements are prepared. II. The interior users. III. The steps taken by management to determine that the applicable financial reporting framework is acceptable in the circumstances. A. I and II only. C. I,II, and III. B. II and III only. D. I and II only. 79. Whenever a report, filed on a printed designed by authorities, calls upon the independent auditor to make an assertion that the auditor believes is not justified, the auditor should A. Withdraw from the engagement. B. Submit the form with questionable items clearly omitted. C. Reword the form or attach a separate report. D. Submit a standard report with explanations. 80. An auditor may express an opinion on an entity’s accounts receivable balance event if the auditor has disclaimed an opinion on the financial statements taken as a whole provided the A. Report on the accounts receivable is presented separately from the disclaimer of opinion on the financial statements B. Auditor also reports on the current asset portion of the entity’s balance sheet C. Use of the report on the accounts receivable is restricted D. Report on the accounts receivable discloses the reason for the disclaimer of opinion on the financial statement 81. An auditor may accept an engagement to report on summary financial statements in accordance with PSA 810 on when A. The auditor has been engaged to conduct an audit in accordance with PSAs of the financial statements from which the summary financial statements are derived B. Summary financial statements are distributed only to management and the board of directors C. Auditor describes the additional review procedures performed on the summary financial statements D. Summary financial statements are presented in comparative from with the prior year’s summary financial statement 82. In the auditor’s report on summary financial statements that are derived from an entity’s audited financial statements, a CPA should indicate that the A. CPA has audited and expresses and opinion on the complete financial statements B. CPA expresses limited assurance that the financial statements conform with another comprehensive basis of accounting C. Summary financial statements are not fairly presented in all material respects D. Summary financial statements are prepared in conformity with another comprehensive basis of accounting 83. Which of the following procedures should a practitioner perform during an engagement to review an entity’s financial statements? A1 PASSERS REVIEW CENTER///17 A. Examining cash disbursements in the subsequent period for unrecorded liabilities B. Sending bank confirmation letter to the entity’s financial institutions C. Obtaining a client representation letter from members of management D. Communicating material internal control weakness during the assessment of control risk 84. During an engagement to review the financial statements of an entity, a practitioner becomes aware of a material departure from GAAP. If the practitioner decides to modify the review report because management will not revise the financial statements the practitioner should A. Express negative assurance on accounting principles not conforming with Philippine Financial Reporting Standards. B. Express positive assurance on accounting principle conforming with Philippine Financial Reporting Standard C. Express a qualified opinion D. Express a qualification of the negative assurance provided or give an adverse statement that the financial statements are not presented fairly in all material respects in accordance with Philippine Reporting Standard 85. If there has been a significant limitation on the practitioner’s review of an entity’s fincial statements the practitioner should describe the limitation and I. Express a qualification of the negative assurance II. Provide no assurance A. I only C. Either I or II B. II only D. Neither I nor II 86. An agreed-upon procedures engagement may involve the accountant in performing certain procedures concerning I. Individual items of financial data II. A financial statement III. A complete set of financial statements A. I and II only C. I and III only B. II and III only D. I, II and III 87. In performing a compilation of financial statements of an entity, the accountant decides that modification of the report is not adequate to indicate deficiencies in the financial statements taken as a whole and the client is not willing to concert the deficiencies. The accountant should therefore A. Express an adverse audit opinion B. Express a qualification of the negative assurance C. Withdraw from the engagement D. Perform a review of the financial statements 88. 89. 90. A1 PASSERS TRAINING, RESEARCH, REVIEW & DEVELOPMENT COMPANY 91. 2nd Floor Sommerset Bldg., Lopez Jaena St. Jaro, Iloilo City 92. Tel. No.: (033) 320-2728; 09106547262 93. Email Address: [email protected] 94. 95. BOARD OF CERTIFIED PUBLIC ACCOUNTANT 96. 97. CERTIFIED PUBLIC ACCOUNTANT Licensure Examination 98. 99. BUSINESS LAW 100. 1. In an obligation to give a determinate thing, what is rights are available to the creditor? 101. First Answer- To compel specific performance 102. Second Answer- To record damages in case of breach of the obligation 103. Third Answer- To ask that the obligation be complied with at the expenses of the labior. a. All answers are correct c. Only the first answer is correct b. Only the first and second answers are d. All answers are wrong correct 2. The following contacts, except one, are void an initio. Which is the exception? a. That whose object is outside the commerce of men. b. That whose object did not exist at the time of the transaction. c. That which contemplates an impossible service. d. That which is undertaken in fraud of creditors. 3. S, minor, owns a specific property valued at P50,000. B capacitated, by means of fraud induced S to sell his property to him for P30,000, which S did so. The contact is in writing. Which is correct? a. the contract is valid and binding from the start b. the contract remains unenforceable because it falls under the Statute of Frauds. A1 PASSERS REVIEW CENTER///18 4. 5. 6. 7. 8. c. The contract is rescissible because the ward suffered lesion by more than ¼ of the value. d. The contract is voidable and B can ask for annulment within the period prescribed by law. Legal compensation is allowed when one of the debts a. Arises from a sale of real property made by a minor to a capacitated person b. Arises from the obligation of a depositary c. Consist in civil liability arising from a penal offense d. Arises from a claim for support by gratuitous title This is not a characteristics of a void or inexistent contract. a. The right to raise a defense of illegality cannot be waived b. The action or defense for declaration of the nullity or inexistent of the contract does not prescribed c. Not subject to ratification d. Defense of illegality of the contract is available to third persons whose interest are not directly affected Reformation is not the proper remedy if a. The mutual mistake of the parties causes the failure of the instrument to disclose their real agreement b. One party was mistaken and the other acted fraudulently or inequitably in such a way that the instrument does not show their true intention. c. There was mistake, fraud, inequitable conduct or accident which prevented the meeting of the minds of the contracting parties d. The true intention of the contracting parties in not expressed in the instrument purporting to embody the agreement by reason of mistake, fraud, inequitable conduct or accident The following contracts are defective. Which one is unenforceable? a. Contract of sale by which a guardian acquires the property of his ward without the approval of the court b. Contract of sale by which a city mayor acquires a portion of the city part pursuant to a city ordinance enacted by the City Council c. Contracts whose object is outside the commerce of man d. Those were both parties are incapable of giving consent to a contract Which of the following is not a requisite of fraud? a. There must be a misrepresentation or concealment b. The fraud must be serious c. A party is induced by fraud to enter into contract d. The fraud is employed by both parties upon each other 9. 10. 11. I agreed to paid his debt and in case of non-payment, to render free service as a servant. Is the obligation valid. a. No, the obligation to pay and to render service as a servant is contrary to law and morals b. Yes, if the service will be rendered in satisfaction of the debt and in case of non-performance, the proper remedy is specific performance c. No, the nullity of the condition carries with the nullity of the principal obligation d. Yes, in so far as the obligation to pay but not as undertaking to render domestic services for free. 12. E, a manufacturer of locks, hired a for 2 years, on the condition that for 5 years, A should not engage in competitive locks business. After one year, A left for reasons of health. Shortly afterwards, after regaining his health, A competed with E, who now seeks to restrain him for such competition, will the action prosper? a. No, the restriction is void, because it is an unreasonable restrain of trade. b. Yes, it is a reasonable restrain, considering that it was only 5 years. c. No, the contract of employment for 2 years was not completed because of a justifiable reason d. Yes, if E will allow a to complete the 2 years services agreed upon. 13. Which of the following is not correct in rescission? a. Cannot be availed when the party who has been damage has other legal remedy b. The person who seeks rescission must be able to return what he ought to return c. The object of the contract in not in the possession of the third persons who have acquired it in good faith d. Can be demanded by any of the contracting parties 14. A owes B p10,000 with 12% interest B owes C P1,200. It was agreed between A and B that A would give the interest of P1,200 to C and C communicated his acceptance of the stipulation between A nd B. there is a. Stipulation pour atrui c. Compensation b. Pactum commissorium d. Assignment 15. -A donated to B a real property, the donation is made orally 16. -A sold to B a real property, the sale is made orally. a. Both contracts are valid c. Only the first is valid b. Both contracts are void d. Only the second is valid 17. If an obligation is with a penal clause 18. Statement A- Proof of actual damages suffered by the creditor is not necessary in order that the penalty may be demanded. A1 PASSERS REVIEW CENTER///19 19. Statement B- When the debtor is guilty of fraud in the fulfilment of the obligation, the creditor can demand payment of damages and interest in addition to the penalty. a. True, True c. False, False b. False, true d. True, False 20. A, B, and C solidarity owe D and E P30,000. D remitted the entire obligation in favor of A without the consent of E. the effect is a. A cannot recover from B and C because remission in her favor extends to the benefit of B and C b. A can recover from B and C their respective share of the debt c. The obligation is not extinguish until a collects from B and C d. The obligation is not yet extinguished until E is paid by D of her share of the credit. 21. In quasi-delict, the liability of a person who is held liable for the fault or negligence of another person who is under his care or responsibility shall case if. a. There was contributory or responsibility shall cease if. b. He was not present at the time of the act, hence it was not possible to him to prevent the damage. c. He observed ordinary diligence to prevent the damage. d. He observed the diligence of a father of a good family. 22. What is the effect of the loss of the prestations in alternative obligation where the right of choice belongs to the creditor and the loss is due to the fault of the debtor? e. First Answer; If only one remains the obligation still subsist, but it ceases to be alternative as it becomes a simple obligation. f. Second answer; If 2 more objects remain, the obligation still subsist and the choice is limited to the remaining objects. g. Third Answer; If none remains, the obligation is extinguished because the debtor chose not to give anything. a. All answers are correct c. Only one of the answers is correct b. Only the first and second answers are d. All answers are wrong correct 23. A owes b P20,000 which became due on December 20,2011. On that date, a offered P10,000, the only money he then had, but B refused to accept the payment. Thereafter A met, C, B’s 23 year old daughter, to whom she gave the P10,000 with the request that she turn the money over to her mother. The money was stolen while in C’s possession. How much may B still recover from A? a. P20,000 c. P15,000 b. P10,000 d. P0 24. A wife, of B and daughter of C, while employed in a jewelry store owned by d embezzled P100,000 belonging to said jewelry store. In order to prevent her criminal prosecution for estafa, B and C signed a document obligating themselves jointly and severally to pay D the amount embezzled plus interest. Because of their failure to comply with their promise, d brought an action against B and C. will the action prosper? a. Yes, both knowingly and voluntarily agreed to assume the obligation of A. b. No, C should not have been included because A is no longer under his authority. c. Yes, but only in so far as B is concerned because husband and wife are required by law to support each other. d. No, it is considered contrary to public policy to allow parties to make an agreement designed to prevent prosecution for crimes. 25. A contract in which a person literally contracts with himself is a. Adhesion contract c. Accessory contract b. Auto contract d. Unilateral contract 26. The statement “Contracts shall be obligatory in whenever from they have been entered into provided all the requisites for their validity are present” refers to a. Real contract c. Formal contracts b. Consensual contract d. Solemn contracts 27. The pre-nuptial agreement of H and W, husband and wife, provides for complete separation of property. Later, H, with violence and intimidation forced W to sell to him (H), W’s pieces of jewelry. The sale is a. Rescissible, if W suffered a lesion of more than ¼ of the value of the property b. Voidable, because the consent of W is vitiated c. Unenforceable, if the value is at least P500 d. Void, because husband and wife are not allowed to sell property to each other. 28. This obligation is dencandable at once a. With a suspensive condition d. When it depends on the happening of a b. With a period in diem specified event c. When my means permit me to do so 29. This contract is without effect unless ratified a. Marriage between first degree cousins b. Contract of sale between two insane persons A1 PASSERS REVIEW CENTER///20 c. Contract of sale between a guardian and his d. Donation between husband and wife event 30. One of the following is void a. Policitation c. Stipulation pour atrui b. Pactum commissorium d. Walver intentionada 31. To be valid and enforceable, the following contracts should be in writing, except: a. Contract to pay interest on loan b. Contract of donation of real property c. Contract of giving authority to an agent to sell a piece of land d. Contract made in consideration of marriage. 32. A condition which if imposed on an obligation will be disregarded and will therefore make the obligation immediately demandable a. If Lala kills Lele c. If Lolo commits suicide b. If Lili passes the CPA board exams d. If Lulu will not rise from the dead 33. If the law or contract does not state the diligence which is to be observed in the performance of an obligation, the obligor is expected to observed. a. Ordinary diligence c. Diligence of a father a good family b. Extraordinary diligence d. Utmost care 34. Facultative as distinguished from alternative obligation a. The right of choice id given only to the debtor b. Various things are due, but the giving of one is sufficient c. If one of the prestations is illegal, the other may be valid and the obligation remains. d. If it is impossible to give all except one, that last one must still be given 35. Within what period must recovery be made if the debtor did not know that payment was not yet due? a. Before maturity with regard to both was aid and the interest b. Even after maturity with regard to both the interest and what was paid c. Before maturity with regard to what was and even after maturity with regard to the interest d. Before maturity with regard to interest and even after the maturity with regard to what was paid 36. Which of the following is wrong in alternative obligations? a. The obligor shall completely perform one of them b. The oblige cannot be compelled to receive part of one and part of the other undertaking c. The right of choice belongs to the creditor, unless it has been expressly granted to the debtor d. The debtor shall have no right to choose those prestations which are impossible 37. A remedy in equity by means of which a written instrument is made or construed so as to express or conform to the real intention of the parties when some error or mistake has been committed a. Rescission c. Reformation b. Annulment d. Consideration 38. Payment by cession as distinguished from dation in payment a. The debtor is not necessarily in a state of financial difficulty b. The effect is to release the debtor for the net proceeds of the things ceded or assigned. c. The property is alienated by the debtor to the creditor in satisfaction of a debt in money. d. What is delivered by the debtor is merely a thing to be considered as the equivalent of the performance of the obligation 39. If a third person pays an obligation. What are the rights, which are available to him if he pays the obligation which the knowledge and consent of the debtor? 40. First Answer- He can recover from the debtor the entire amount, which he has paid. 41. Second Answer- He is subrogated to all of the rights of the creditor a. True, True c. False, True b. True, False d. False, True 42. I. The loss or determination of the thing intended as a substitute through the negligence of the obligor, does not render him liable 43. II. A person alternatively bound by different prestations shall completely perform one of them a. True, true c. False, True b. True, False d. False, True 44. There being no express stipulation and if the undertaking is to deliver a determinate thing, the payment shall be made a. At the domicile of the debtor b. At the domicile of the creditor c. Wherever the thing might be at the moment the obligation was constituted d. Wherever the thing might be at the moment the obligation is to be fulfilled 45. Which of the following statements is not correct? a. The validity and compliance of a contract cannot be left to the will of one of the parties A1 PASSERS REVIEW CENTER///21 b. In case of foreclosure and the price of the sale is less than the amount due the pledge cannot recover any deficiency c. Persons who are prohibited by law to enter into construct of donation cannot from universal partnership d. Actions for fraud cannot be waived 46. The right of the creditor to exercise all the rights of his debtor to satisfy his claims, except rights which are inherent and personal on the part of the debtor a. Actor subrogatoria c. Action pauliana b. Action redhibitoria d. Action quanti minoris 47. An assignor of credit warrants a. Solvency of the debtor c. Assurance of payment b. Collectibility d. Existence and legality of credit 48. A, B, and c executed a promissory note binding themselves to pay P9,000 to X, and Y and Z. the note is now duo and demandable. Can the creditors proceed against A alone fro the payment of the entire debt? a. No, each creditors can collect only P3,000 from A. b. Yes, either X, y or z can collect P9,000 from A. c. No, each creditor can collect only P1,000 from A. d. Yes, since the promissory note is silent with respect to the right of the creditors, the oblibation is resumed to be solidary. 49. Using the preceding number, suppose that C is insolvent, can A and B be held liable for C’s share in the obligation? a. Yes, the debt shall be presumed to be divided into as many equal share as there are debtors. b. Yes, but A and B will be liable proportionately c. No, the debts are considered distinct from one another. d. No, only either a and B but not both will be liable 50. Using No.40, suppose that the obligation was about to prescribed, but X wrote a letter to A demanding for payment of the entire debt. Will this have the effect of interrupting the running of the period of prescription? a. Yes, because the demand by X covers the entire debt and will therefore it was to the benefit of the other creditors b. Yes, insofar as A is concerned nut not with regard to B and C. c. No, because the demand should have been made to all the debtors. d. No, all the creditors should have made the demand. 51. Using the preceding number, and prescription sets in, how much can y collect from A? a. P9,000 c. P1,000 b. P3,000 d. P0 52. Using the preceding number, how much can X collect from A? a. P9,000 c. P1,000 b. P3,000 d. P0 53. Indivisibility as distinguished from solidarity. a. Plurality of subject is indivisible. b. When the obligation is converted into one of indemnity for damages because of breach, the character of the obligation remains c. Refers to the legal or vinculum d. Refers to the prestation which constitute the object of the obligation 54. I. When one of the parties has brought an action to enforce the instrument, he cannot subsequently ask for its reformation 55. II. The injured party may seek rescission, even after he has chosen fulfilment if the latter should become impossible. a. True, true c. False, true b. True, false d. False, false 56. Culpa aquiliana as distinguished from culpa contractual a. Proof of due diligence in the selection and supervision of employees is not available as a defense b. Proof of the contract and of its breach is sufficient prima facle to warrant recovery. c. The negligence of the defendant is merely an incident in the performance of the obligation d. The source of liability is the defendant’s negligent act or omission itself 57. A assenger on a bus was hurt, but in a criminal case against the driver, said driver was acquitted. The victim now sues the owner of the truck for culpa contractual. May the suit still prosper? a. No, this will constitute double jeopardy b. No, the acquittal means that the guilt of the accused was proven by proof beyond reasonable doubt c. Yes, it is sufficient fro him to prove the existence of the contract of carriage and the injuries suffered. d. Yes, provided he can prove the negligence of the driver. 58. A owes B P11,000 due on July 2, 2011. B owes A P6,000 due on July 3, 2011 and P4,000 due on July 10, 2011. B owes C P11,000 due on July 3, 2011. On July 3, 2011 B cannot pay C so B assigns to C her credit A1 PASSERS REVIEW CENTER///22 P11,000 against A, without the knowledge of A. on July 10, 2011, C tries to collect from A the P11,000. How much can C compel A to ay? a. P11,000 c. P5,000 b. P9,000 d. P1,000 e. f. Questions 50-52: A, B, C ad D owe W,Y and Z P9,000. On maturity, how much can W collect from A? 59. If debtors are joint and creditors are joint a. P2,250 c. P9,000 b. P3,000 d. P750 60. If debtors are joint and creditors are solidary a. P9,000 c. P3,000 b. P750 d. P2,250 61. If debtors are joint solidary and creditors are joint a. P750 c. P9,000 b. P2,250 d. P3,000 62. Using the same data in No.28, but the share of debtors A, B, C and D in the indebtedness is 1:2:3:4 and creditors W, Y and Z is 2:3:5. How much can Y collect from b if debtors are joint and creditors are joint? a. P1,800 c. P675 b. P2,700 d. P540 63. How much can Z collect from d, if debtors are solidary and creditors are joint? a. P1,800 c. P3,600 b. P1,125 d. P4,500 64. How much should A pay W, if debtor are joint and creditors are solidary? a. P180 c. P2,250 b. P9,000 d. P900 65. A lost P100,000 in a card game called pusoy, but he had no more cash to apply in full the winner at the time session ended. A promised to pay B, the winner, two weeks thereafter. A failed to pay despite the lapse, of two months, so B filed in court a suit to collect the unpaid amount. Will the collection suit against A prosper? a. Yes, if A made a negotiable promissory note was made by A. b. Yes, whether or not a negotiable promissory note was made by A. c. No, if A has no property to pay the obligation d. No, because pusoy is fundamentally game of chance. 66. A filed a suit for ejectment against B for non-payment of condominium rentals amounting to P150,000. During the pendency of the case, B offered and A accepted the full amount due as rentals from B who then filed a motion to dismiss the suit. Is b correct? a. Yes, the acceptance of the payment constitutes a waiver of the ejectment case b. Yes, because there is a novation c. No, the payment should first result to benefit to A. d. No, the payment by B of the rentals in arrears is not an abandonment of the ejectment case. 67. A was employed as professional driver B Transit Bus owned by C. in the course of his work, a hit a pedestrian who was seriously injured and later died in the hospital as a result of the accident. The victim’s heirs sued the driver and the owner of the bus for damages. Which of the following statements is not correct? a. There is a presumption of negligence on the art of the employer C if a passenger is injured b. The conviction of A in a criminal case makes C liable for the damages arising from the criminal act c. The presumption that C is negligent is rebuttable by proof of observance of ordinary diligence d. The liability of C shall cease when he proves that he observed ordinary diligence to prevent damage to passengers and pedestrians. 68. N<, r and J solidarily bound themselves to deliver to S a Honda motorcycle valued at P60,000. The obligation was not fulfilled through the fault of J. thereupon, S filed an action in court against N and the court awarded P72,000 to S representing the value of the motorcycle plus damages. Which of the following situations is valid? a. If N pays s the P72,000, N can collect from R and J P24,000 each. b. S has to collect P24,000 cash from N, R and J to satisfy the court’s award of P72,000. c. N can refuse to pay the penalty because it should be charged against J, the guilty party. d. If S succeeds in collecting the P72,000 from N, n in turn can collect from R P20,000 and from J P32,000. 69. In which of the following debts is legal compensation proper? When one of the debts a. Arises from the obligation of a depository b. Arises from a voidable contract c. Arises from a claim for support by gratuitous title d. Consists in civil liability arising from penal offense A1 PASSERS REVIEW CENTER///23 70. A borrowed P10,000 from B on Dec. 25, 201. The debt is evidenced by a promissory note executed by A wherein she promised to ay as soon as she has the money or as soon as possible. B made several demands upon A for payment, the first, an oral demand was made on May 2, 2011 and the last written demand was dated September 25, 2011 but up to now, there is no payment of the debt. B sued A, will the case prosper? a. Yes, A is in default since May 2, 2011 b. Yes, a is in default since September 25, 2011 c. No, the obligation is subject to a suspensive condition d. No, the obligation is one with a period and the period has not arrived yet 71. On September 25, 2011, A sent a telegram to B in Cebu, offering to sell to b his house and lot for P2M, cash. On the same date, B sent to A a telegram offering to buy A’s same house and lot for P2M, Is there a perfected contract? a. No, because both telegram are mere offers b. Yes, a promise to buy and sell a determinate thing for a price certain is reciprocally demandable. c. No, because B did not offer to buy the house and lot for cash. d. Yes, being a consensual contract, it is perfected by, mere consent. 72. Which of the following statement is not correct? a. The vendor is bound to deliver the thing sold and its accessions and accessories in the condition in which they were upon the perfection of the contract. b. All the fruits of the thing sold shall pertain to the vendor from the day on which the contract was perfected. c. The vendor shall not be bound to deliver the thing sold, if the vendor has not paid him the price, or if no period for the payment has been fixed in the contract. d. The vendor is bound to transfer the ownership of and deliver, as well as warrant the thing which is the object of the sale. 73. As a rule, which of the following contract of sale is void? a. Between two insane persons c. Between husband and wife b. Between brother and sister d. Between pupil and teacher 74. A and B are co-owners of a one hectare rural land. A sold his ½ share to Y, C, an adjoining land owner is interested in buying the share which A sold to y. which of the following is correct? a. B can redeem what A sold to Y if Y already owns a rural land b. B can redeem what A sold to Y even if Y does not own any rural land c. As adjoining land-owner C has a superior right to redeem what A sold to Y. d. C can redeem what A sold to Y whether or not Y already owns a rural land 75. A sells to B her car for P300,000 on a “sale or return within 10 days” after delivery. On the 5 th day after delivery, the car was lost through fortuitous event. Who bears the loss? a. A under the principle of “res perit domino” b. B and he must pay the purchase price c. Both A and B jointly d. No one because the loss was due to a fortuitous event 76. P1 and P2 are co-owners of a pieces of land and they named and authorized. A to sell their land. Who will be liable to A for the payment of his commission? a. Both P1 and P2 jointly b. Both P1 and P2 solidarily only if stipulated c. Both P1 and P2solidarily even without stipulation d. 50% from P1 and 50% P2 77. J is the sole owner of one hectare of land. In need of money, she sold ½ of the land without specifying which portion she is selling to H. in this case, the sale is a. Void, because co-ownership is discourage by law b. Void, because the ½ part is not determinate c. Valid, only if H has paid the purchase price d. Valid as the sole owner of a thing may sell an undivided interest therein 78. E owns a piece of land and sells it to S with a right of repurchase within 4 years from the date of sale. If S sells property to T, which of the following is not correct? a. The sale is valid because things subject to a resolutory condition maybe the object of a contract of sale b. E can still exercise her right of redemption against T c. The sale is void because it is a conditional sale d. T acquires the property but subject to the right of conventional redemption 79. In which of the following cases will delivery transfers ownership the thing sold, a. In case of express reservation by the seller until certain conditions have been fulfilled, particularly the full payment of the purchase price. b. In case of implied reservation of title as when goods are deliverable to the order of the seller or his agent. A1 PASSERS REVIEW CENTER///24 c. In sale or approval, or on trial or on satisfaction d. In sale or return within seven days 80. A owns a piece of land and slls it to B with a right of purchase within one year from the date of sale, can b sell the land purchased from A, to C, a third party? a. Yes, provided the sale is with the consent of A. b. No, B is not yet the absolute owner of the land. c. Yes, but A can still redeem the land from C. d. No, third parties that acquire real property are bound by prior contracts affecting such property even if the third person is not a party thereto. 81. When goods are delivered to the buyer on “sale or return” for a period of seven days, ownership of the goods passes to the buyer. a. Upon perfection of the contract d. Upon acceptance by the buyer of the offer of b. Upon delivery of the goods the seller c. Upon expiration of seven days 82. In case of foreclosure of the personal property mortgaged, where such things was previously sold to the buyer on an instalment basis and the proceeds of the sale at the public auction is less than the principal obligation can the seller recover the deficiency from the buyer? 83. First Answer- No, the seller is not entitled to recover the deficiency from the buyer. 84. Second Answer- Yes, if there is a stipulation to that effect in the contract of sale with mortgage. a. True, true c. False, true b. True, false d. False, false 85. Statement 1- If the same thing should have been sold to different vendees, the ownership shall be transferred to the person who may have first taken possession thereof in good faith. e. Statement 2- The vendor is responsible to the vendee for any hidden faults or defects in the thing sold, even though he was not aware thereof a. True, true c. False, true b. True, false d. False, false 86. A, B and C are co-owner of a parcel of land, A sold his 1/3 share on Sept 1, 2011. B his 1/3 share on Sept 12, 2011 and C his 1/3 share on Sept 25,2011, all to D and with the right of repurchase. Which of the following is correct? A may redeem a. The entire property from D if he is required by D to do so. b. The entire property even if D allows him to redeem 1/3 share only. c. His 1/3 share even if d requires him to redeem the whole property. d. His 1/3 share if D allows him to do so. 87. A owns a parcel of land, which he sells to B with 3 year redemption period. After the second year, A dies, leaving his children C, D and E as his heirs. Which of the following is not correct? a. As A can purchase the whole thing, so any among C, D and E may purchase the whole thing. b. C can redeem his 1/3 share, D his 1/3 share and E his 1/3 share if B does not require all of them or any one of them to redeem the whole party. c. B may demand that all the co-heirs come to an agreement upon the repurchase of the whole thing sold. d. E cannot be compelled to consent to a partial redemption. 88. A bought a car from B, a minor for P100,000. One week later, A discovered that B was a minor at the time of sale so he filed a complaint in court to annul the sale. Will the action prosper? a. Yes, B being a minor is incapacitated to enter into contract b. No, the right to annul the sale is given to B c. Yes, B cannot file the action to annul the sale because he is a minor d. No, unless there is lesion of more than ¼ of the value of the property 89. Which of the following may not be object of a contract of sale? a. Thing having a potential existence c. Future inheritance b. The sale of hope or expectancy d. Things subject to a resolutory condition 90. A, a minor sold his ring to B for P4,000. Later, B borrowed P6,000 form C and as security pledged the ring to C. B failed to pay C and the latter foreclosed the pledge, sold it at public auction for P5,000 to X. As a result, a. The title of B is not valid, therefore the pledge of the ring to C is also not valid because in pledge the pledgor must be the owner b. The deficiency of P1,000 cannot be recover by C from B c. The deficiency of P1,000 can be recovered if there is a stipulation to that effect d. If X is in bad faith (X has knowledge that A, the original owner is a minor) ownership will not pass to B 91. This shall take place when the vendor reserves the right to repurchase the thing sold a. Policitacion c. Equitable mortgage b. Conventional redemption d. Legal redemption 92. A sold his piano to B for P200,000 payable in instalment. A chattel mortgage was constituted on the piano. B defaulted in two instalment payments. A demand payment of the unpaid obligation amounting to A1 PASSERS REVIEW CENTER///25 P120,000 and a writ of attachment was issued and the piano was sold for P100,000. Can A still recover the deficiency? 93. First Answer- No, the foreclosure of the piano extinguished B’s obligation 94. Second Answer- Yes, only if it is stipulated that in case of foreclosure the buyer will pay any deficiency a. True, true c. False, true b. True, false d. False, false 95. A and B entered into a “contract to sell” in private writing involving a specific parcel of land for P2M. B paid 50% of the purchase price, the balance payable in 3 years. A delivered the land o B. What is the effect of the delivery of the land to B? a. B is the owner because there was a delivery already. b. B’s ownership before it is converted into real ownership must compel A to execute a deed of sale in a public instrument. c. The partial payment made B the owner of the land. d. A is still the owner because the price is not yet totally paid. 96. In 2007, at age 15, A sold his land to B, of legal age for P4M at P1M in 2007, P1M in 2008, P1M in 2009 and P1M in 2010. In 2012, A wants annul the contract on the ground of minority. Will his action prosper? a. No, A is allowed to ask for annulment of the contract within 4 years form the perfection of the contract. b. Yes, A has 4 years counted form the time to time he becomes of legal age to ask for annulment of the contract. c. No, the acceptance of the instalment payments amounted to ratification of the sale. d. Yes, provided A was then acting in good faith he sold the property. 97. Where it is stipulated that the repurchase of the property sold could be made at any time, the repurchase shall be exercised a. Within four years, from the date of contract c. After ten years after the date of the contract b. Within ten years from the date of the contract d. Within six years from the date of the contract 98. A, seller sold to a buyer a brand new car at a price of P500,000. At the time of sale, the buyer has only P200,000 cash and his sold car with a fair market value of P300,000 which he offered as payment of the purchase price which was accepted by the seller. The nature of the contract is a. Barter c. Sales b. Exchange d. Partly sales and partly barter 99. In 2010, a, 17 years old, sold his land orally to B a. The sale is unenforceable c. A cannot ask for annulment b. The sale is voidable d. B can ask for annulment 100. If the thing sold had any hidden fault or defect at the time of the sale and should thereafter be lost by a fortuitous event but the seller is not aware of the defect, how much can be the vendee recover from the vendor, if the selling price on date of sale is P100,000 and the value on date of loss is P60,000. a. P100,000 c. P40,000 b. P60,000 d. P0 101. Using the preceding number, except that the loss is through the fault of the vendee, how much can the vendee recover from the vendor? a. P100,000 c. P40,000 b. P60,000 d. P0 102. Which of the following contracts is void? a. Lease of piece of land made by a minor without a written authority from his parents (the owners) b. Oral sale of a piece of land made by the owner to insane person. c. Written sale of gold necklace made without authority from the owner d. Written sale of a piece of land with oral authority from the owner. 103. Which of the following contracts is valid? a. Contract of sale between an insane person and a deaf-mute who does not know how to write and read. b. Contract of donation between a husband and his paramour. c. Contract of sale between husband and wife who are under the conjugal partnership of gains. d. Contract of donation between a wife as donee and her former boyfriend as donor. 104. As an agent , A was given a guarantee, commission, in addition to his regular commission, after he sold 20 units of refrigerators to a customer. The customer, however failed to pay. A’s principal, P demanded payment from A for the customer’s accountability. Which is correct? a. A can refuse to pay on the ground that his job was only to sell and not to collect payment b. P should demand payment from the customer c. A is not liable if he was authorized to sell on credit d. A bears the risk of collection and should pay P the proceeds of the sale. 105. The following are the obligations of the agent, except: a. In the execution of the agency, the agent shall act in accordance with the instructions of the principal. b. Shall be bound in advance the necessary funds, except when the principal is insolvent c. Shall finish the business already begun on the death of the principal, should delay entail any danger. A1 PASSERS REVIEW CENTER///26 d. In case a person declines an agency, he is bound to observe the diligence of a good father of a family. In the custody and preservation of the goods forwarded to him by the owner until the latter should appoint an agent. 106. Which of the following statements is false? a. P in writing appoints A as bis agent to sell his specific parcel of land for P40,000. A sold it orally to X. the contract of A and X is valid. b. B wrote A, bis alster, to sell his parcel of land. The land was purchased by X, but A did not forward the money to B. B now wants to recover the parcel of land. B can recover because the authority of S is not in special power of attorney. c. P orally appoints as his agent to sell his land for P40,000. A sold it to X in writing. The sale by A to X is not valid. d. P orally appointed A to sell a house on a parcel of land belonging to X. a sold it to B orally. The contract by P and A are valid. 107. P sends a letter on A in Cavite, authorizing A to sell his specific car for P90,000. If no reply is made by A, is there a prescription of implied agency between the two of them? a. None, because as between persons who are absent, acceptance of the agency cannot be resumed from the silence of the agent. b. Yes, because no reply was send, there is an implied acceptance or implied consent c. Yes, unless A is habitually engaged in the business of selling and buying cars. d. None, acceptance must always be expressed 108. M appointed R and Z as her agents to sell her specific her specific property for P100,000 on cash basis. Solidarity has been agreed upon. Can M hold Z liable if R sell the property for P80,000? a. Yes, because the obligation is solidary b. No, because the appointment of 2 or more agents in one and the same obligation is joint and any stipulation to the contrary is void c. Yes but not only for Z’s share that is P10,000 d. No, because R acted beyond the scope of his authority. 109. The following may be objects of contract of pledge, except a. Shares of stocks c. An agricultural land b. Pieces of jewelry d. A negotiable bill of exchange 110. Which of the following is not correct? a. A movable or immovable project may be the object of commodatum b. The bailor in commodatum need not be the owner of the thing loaned c. A stipulation that the bailee make use of the fruits of the thing loaned is valid. d. The bailor is oblihed to pay for the ordinary expenses for the use and preservation of the thing loaned 111. D mortgaged his land to C as security for a loan. Fearing foreclosure of the mortgage due to his inability to pay the loan, D sold the land to B without the consent of C. which is correct? a. The sale is void unless D can give another security. b. B cannot acquire ownership over the land even if D delivers the land to him. c. D cannot sell the land if there is an agreement prohibiting the mortgagor from alienating the land. d. D can sell the land even without the consent of C. 112. The loan of movable or immovable things but not the fruits. a. Mutuum c. Expromission b. Delegacion d. Commodatum 113. Agreement giving a person the right to enjoy the thing as well as its fruits a. Quantum meruit c. Usufruct b. Quantum valebant d. Pacto de retro sale e. v. f. w. g. x. h. y. i. z. j. aa. k. ab. l. ac. m. ad. n. ae. o. af. p. ag. q. ah. r. ai. s. aj. t. ak. u. al. A1 PASSERS REVIEW CENTER///27 am. an. ao. ap. aq. ar. as. at. au. av. aw. ax. ay. az. ba. bb. bc. bd. be. bf. bg. bh. bi. bj. bk. bl. bm. bn. bo. bp. bq. br. bs. bt. bu. bv. bw. bx. by. bz. ca. cb. cc. cd. ce. cf. cg. ch. ci. cj. ck. cl. cm. cn. co. cp. cq. cr. cs. ct. cu. cv. cw. cx. cy. cz. da. db. dd. de. 1. B 2. D 3. A 4. A 5. D 6. C 7. D 8. D 9. D 10. B 11. D 12. A 13. D 14. A ANSWER KEY dc. BUSINESS LAW df. 15. 16. 17. 18. 19. 20. 21. 22. 23. 24. 25. 26. 27. 28. A C D A D C B B B B B B A A 29. 30. 31. 32. 33. 34. 35. 36. 37. 38. 39. 40. 41. 42. A C C C B A C C D A D D D B A1 PASSERS REVIEW CENTER///28 43. 44. 45. 46. 47. 48. 49. 50. 51. 52. 53. 54. 55. 56. 57. 58. 59. 60. 61. 62. A D D A D C D D D D D D D D D D D B D A 63. 64. 65. 66. 67. 68. 69. 70. 71. 72. 73. 74. 75. 76. 77. 78. 79. 80. 81. 82. B C B B C D C D C B B C C A B C B B D D 83. C 84. B 85. C 86. C 87. C 88. C 89. C 90. A 91. D 92. B 93. B 94. A 95. D 96. C 97. D 98. D 99. D dg. 100. C dh. di. dj. dk. dl. dm. A1 PASSERS TRAINING, RESEARCH, REVIEW & DEVELOPMENT COMPANY dn. 2nd Floor Sommerset Bldg., Lopez Jaena St. Jaro, Iloilo City do. Tel. No.: (033) 320-2728; 09106547262 dp. Email Address: [email protected] dq. dr. BOARD OF CERTIFIED PUBLIC ACCOUNTANT ds. dt. CERTIFIED PUBLIC ACCOUNTANT Licensure Examination du. dv. TAXATION dw. 1. Spouses A and B sold their family home a capital asset for P5,000,000. It was in 1980 at P2,000,000. The fair market value as by the BIR is P6,000,000 but the fair market value as shown in the schedule of values of the City Assessor is P5,500,000. Later, the spouses utilized P4,000,000 for the acquisition of their new family home. The capital gains tax due is a. P72,000 b. P60,000 c. P300,000 d. P360,000 2. Using the above data the cost basis of the new family home is a. P4,000,000 b. P1,600,000 c. P1,000,000 d. P400,000 3. A corporation was merged with B Corporation, G, a stockholder of A Corporation was asked to surrender his shares of stock of A Corporation which C acquired for P200,000 and in exchanged he received shares of stock of B Corporation with a fair market value of P220,000 plus cash of P30,000. After the merger, C later sold his B Corporation shares first P240,000. C will recognize a gain of: 4. Merger Sale a. P30,000 P40,000 b. P50,000 P20,000 c. P30,000 P30,000 d. P50,000 P40,000 A1 PASSERS REVIEW CENTER///29 5. A transferred his commercial land which he acquired for P10M to B Corporation and as consideration he received shares of stock of B Corporation with a fair market value of P10.2M and cash of P300,000. As a result, A acquired control of B Corporation. Two months later, A sold the B Corporation shares for P10.5M. the gain to be recognized on the transfer and on the sale is 6. Transfer Sale a. P300,000 P500,000 b. P500,000 P300,000 c. P300,000 P300,000 d. P500,000 P500,000 7. A transferred his land in Manila which he acquired for P2M to B corporation and as consideration received shares of stock of B Corporation with fair market value of P1.8M and as a result A gained control of B Corporation. Later, A transferred his land in Makati which he acquired for P3M to C Corporation and as consideration he received shares of stock of C Corporation with fair market value of P2.7M. even before the transfer, A already owns 51% of C Corporation outstanding shares. The loss to be recognized by A is 8. B Cor. C Corp. a. P0 P300,000 b. P0 P0 c. P200,000 P300,000 d. P200,000 P0 9. Use the following information for question 6 and 7. 10. A, sold 3 parcels of land held as capital assets, as follows: 11. Land1 Land2 Land3 12. Selling Price P160,000 P160,000 P160,000 13. Cost 100,000 100,000 100,000 14. 15. Installment Payment: 16. Year 1 (year of sale) 40,000 40,000 40,000 17. 2 120,000 60,000 60,000 18. 19. Assumption of remortgaged 0 60,000 110,000 20. The capital gains tax to be paid in year 1 is: a. P28,000 c. P19,840 b. P15,840 d. P12,640 21. The capital gains tax to be paid in year 2 is: a. P12,960 c. P16,160 b. P8,960 d. P19,200 e. Use the following information for questions 8 and 9. f. A and B are stockholders in corporations which are being consolidated into a new corporation, has the following data on their shares of stock: g. A B h. Consideration received: P75,000 P81,000 i. Fair Market value of shares 5,000 5,000 j. Fair Market value of property 10,000 4,000 k. Cash 80,000 80,000 l. Cost of shares surrendered m. 22. The gain or (loss) to be recognized by A is a. P10,000 c. P5,000 b. P15,000 d. 0 23. The gain or (loss) to be recognized by B is a. P9,000 c. P1,000 b. P10,000 d. P0 24. 25. In 1998, Cadena de Amor Corporation gave the following fringe benefits to its employees 26. To managerial employees -----------------------------------------------P1,320,000 27. To rank and file employees----------------------------------------------5,000,000 28. 29. The allowable deduction from the gross income of the corporation for the benefits given to employees is: a. P2,000,000 c. P6,320,000 b. P2,148,000 d. P7,000,000 30. 31. A vat taxpayer has the following data for a particular quarter during the current year: e. Sale of shares of stock: P1,000,000 A1 PASSERS REVIEW CENTER///30 a. P0Held as inventory f. 500,000 c. P2,500 b. P1,500 g. Held as investment d. P7,500 e. h. Cost of the shares sold: 300,000 32. An invoice i. is issued for theHeld saleas ofinventory vat taxable goods shows the following: 600,000 f. j. Held as investment (thru local stock exchange) g. k. Total The stock invoice transaction amount tax is: P50,400 h. Less: trade discount of 5% 2,520 i. Net P47,880 j. k. If the vat taxpayer enjoys a partial exemption of 40% including VAT and the amount is VAT inclusive, how much is the output tax? a. P3,078 c. P3,447 b. P3,216 d. P3,628 33. A, a vat taxpayer, made the following purchases of capital goods from vat registered sellers for use in his business (amounts are net of vat) for the 3rd quarter 34. 35. Year 2001 Estimated Life Cost 36. July 10-Machine1 2 years P200,000 37. 16-Machine2 6 years 900,000 38. Aug.8- Machine3 2 years 400,000 39. 20- Machine4 6 years 500,000 40. Sept.14- Machine5 7 years 2,000,000 41. 42. Machine 1 was retired on September 30, 2011. The input tax in July is: a. P2,800 c. P132,000 b. P108,000 d. P2,200 43. 44. The input tax in August is: a. P108,000 c. P5,800 b. P3,000 d. P110,800 45. The input tax for September is a. P4,000 c. P26,000 b. P6,800 d. P27,800 46. The power to tax is exclusively ledged with the legislature but the following exceptions are provided by the contribution, except: a. Power of local government units to levy taxes, fees and charges b. Authority of the President to fix tariff rates, import and export quotes, tonnage wharfage dues and other duties and imposes. c. Delegation to administrative agencies of the implementation and collection of taxes. d. Delegation to private entities the levy and enforcement of tax laws. 47. Statement 1- The prizes received by Manny Pacquiao and Ricky Hatton in their last fight are subject to Philippine income tax. 48. Statement 2- If the fight between Pacquiao and Cotto is a world championship fight promoted by the Arum, the gross received will be exempt from percentage tax. a. True, true c. False, false b. True, false d. False, true 49. This is an inherent limitation on the power of taxation. a. The rule on taxation shall be uniform and equitable. b. No law impairing the obligations of contracts shall be enacted. c. Charitable institutions, churches, personages or convents appurtenants thereto, mosque and non-profits cemeteries and all kinds of lands, buildings and improvements actually, directly and exclusively used for religious or charitable purposes shall be exempt from taxation. d. The tax laws cannot apply to the property of foreign governments. 50. This is a constitutional limitations on the power of taxation a. The power of taxation cannot be delegated. b. The power of taxation is limited to the territorial jurisdiction of the taxing state. c. No person shall be imprisoned for debt or non-payment of a poll tax. d. Exemption of government instrumentalities and agencies through which the government exercised sovereign powers from taxation. 51. Taxation as distinguished from police power and eminent domain. a. May be exercised by private individuals b. May be exercised only by the government c. Limited to cover the cost to regulate A1 PASSERS REVIEW CENTER///31 d. Generally no limit as to the amount that may be imposed. 52. Statement 1- The levymg, imposition and collection of tax are legislative in character 53. Statement 2- The aspects of taxation are shared by the legislative and executive branches of the government a. True, true d. False, b. True, false e. false c. False, true 54. Which of the following statements is not correct? a. Taxes are neither political nor penal in nature. b. Taxes should be prospective and should not be given retroactive effect because they are burdens. c. The power to collect taxes is not lost even if foreign invaders are in control of our country. d. The application of a constitutional limitation may disregard an inherent limitation 55. Police power as distinguished from power of taxation a. It exist independent of the constitution being an inherent power of the state. b. The compensation received in the protection given by the state. c. It involves the taking of property of individual affected by the exercised of the power. d. The amount collected is limited to the cost of regulation. 56. The following are the characteristics of special assessment, except. a. It is based on the government’s need of money to support its legitimate objectives. b. It is levied only on land. c. It is based solely on the benefits derived by the owner of the land. d. Did not result to personal liability of the person assessed. 57. As regards a revenue bill, which of the following is not correct? a. The Senate may propose amendments if the bill originates from the House of Representatives. b. The House of Representatives may propose amendments if the bill originates from the Senate. c. The President may recommend a revenue bill to Congress d. A House of Representatives version and a Senate version approved separately and then consolidated with both approving the consolidated version. 58. In 2004, X Corporation incurred the following expenses in connection with the conduct of its business f. A) Plane tickets and hotel bills of its employees g. Who were sent to business seminars.................................... P100,000 h. B) Transportation allowances of its officers from i. home to office and vice versa as part of their j. employment contract net of P64,000.................................... 136,000 k. C) Transportation expenses of messengers from l. Office to several clients.......................................................... 80,000 m. n. The allowable expenses that could be claimed by X Corp. Is a. P180,000 c.P316,000 b. P244,000 d. P380,000 59. Mr. Araki, a non-resident alien stockholder, received a dividend income of P300,000 in 2010 from a foreign corporation doing a business in the Philippines. The gross income of the foreign corporation from within and without the Philippines for three years preceding 2010 is as follows: 60. Source of income 2007 2008 2009 61. From within the Philippines P20,000,000 P12,000,000 P20,000,000 62. From without the Philippines 18,000,000 14,000,000 16,000,000 63. 64. How much of the dividend income received by Mr. Araki is considered income from sources within the Philippines? a. Zero c. P144,000 b. P156,000 d. P300,000 65. A, married to M had the following during the taxable year: 66. Gross Income 67. From the practice of profession-------------------P 700,000 68. Rental income in their conjugal property-------300,000 69. Allowable deductions 70. For the practice of profession----------------------520,000 71. For the property rented to tenants---------------140,000 72. 73. The taxable income before exemption of A is a. P340,000 c.P260,000 b. P180,000 d. P170,000 A1 PASSERS REVIEW CENTER///32 74. K sold for P10M her Baguio rest house with a FMV of P12M to buy a new principal residence. If K utilized P8M of the proceeds of the sale in acquiring a new principal residence, the capital gains tax payable is a. P720,000 c.P144,000 b. P600,000 d. P120,000 75. On August 12, 2010 A sold a land held as capital assets for P2M with a FMV of P1.8M. a acquired the land for P1M, and at the time of sale, the property was subject to a mortgage of P1.3M. payments shall be; P100,000 on the date of sale and the balance shall be paid in equal monthly instalments beginning Jan 1, 2011. The capital gains tax for 2010 is a. P120,000 c. P36,000 b. P24,000 d. P48,000 76. A took a life insurance policy for P5M where the annual premium is P20,000. The proceeds will be paid to A after 25 years or to A’s estate should A dies before completing 25 years of payment. If A outlived the policy, which of the following is correct? a. The proceeds will be part of A’s taxable income. b. The proceeds will be part of A’s gross estate. c. The proceeds will be partly taxable income and partly exempt. d. The proceeds will be partly taxable estate and partly exempt 77. Using the preceding number, but after paying the annual premiums for 10 years, A transferred the policy to B for P250,000 and B paid the annual premiums as they mature. After 10 years, A died. Which of the following is correct? a. The amount received by A from B is part of A’s taxable income. b. The proceeds received by B is part of his taxable income. c. The amount received by A from B is partly taxable income and partly exempt. d. The proceeds will be part of A’s gross estate. 78. The following forms of escape from taxation result to loss of revenue to the government, except a. Exemption c. Avoidance b. Shifting d. Evasion 79. 80. 81. The following are the objects of taxation, except a. Citizens c. Intangible property b. Corporations d. Municipal halls 82. A scope of the power of taxation which means that taxation covers many things a. Plenary c. Supreme b. Comprehensive d. Unlimited 83. This is not a purpose of taxation a. To expropriate property for the promotion of the general welfare. b. To reduce inequalities of wealth c. As protective tariff on imported goods to protect local producers against foreign competition d. To encourage the growth of home industries through the proper use of tax incentives 84. One of the characteristics of a tax is that it is a. Superior to the non –impairment clause of the constitution b. Generally unlimited in amount c. Based on the tax payers ability to pay d. Legislative in implementation 85. Statement 1- A tax is generally unlimited because it is based on the needs of the state 86. Statement2- One of the essential characteristics of tax is it is unlimited in amount a. True, true c. False, true b. True, false d. False, false 87. In January 1970, A bought one hectare of agricultural land in Laguna for P100,000. This property has a current fair market value of P10M in view of the construction of a concrete road traversing, the property. A, agreed to exchange his agricultural lot in Laguna for a ½ hectare residential property located in Batangas, with a fair market value of P10M, owned by B, a domestic corporation engaged in the purchase and sale of real property. B acquired the property in 2007 for P9M. Which of the following statements is correct? a. Both real properties are ordinary assets c. The agricultural land is an ordinary asset b. Both real properties are capital assets d. The residential land is an ordinary asset 88. In our jurisdiction, which of the following statements may be erroneous? a. Taxes are pecuniary in nature. b. Taxes are enforced charges and contributions. c. Taxes are imposed on persons and property within the territorial jurisdiction of a state. d. Taxes are levied by the executive branch of the government. 89. Incidence of taxation means a. Shifting of tax b. Refunds of tax A1 PASSERS REVIEW CENTER///33 c. Payment of tax d. Imposition of tax 90. This rule is not applicable on the construction of tax laws a. If the law is repeated, taxes assessed before repeal of the law may no longer be collected. b. If the intent of the tax is not clear as to whether the taxpayer is covered by the tax obligation, the law shall be construed against government. c. Where the intent to tax is clear and the taxpayer claims he is exempt from the tax obligation, the tax shall be construed against the taxpayer. d. Provisions intended for the security of the taxpayer or to insure equality or uniformity of taxation are mandatory. 91. Income derived by a depository bank under the expanded foreign currency deposit system from foreign currency transaction with local commercial bank, including branches of foreign banks that maybe authorized by the Bangko Sentral ng Pilipinas, shall be subject to a final tax a on such income at a. 7.5% c. 15% b. 10% d. 20% 92. A privileged granted a taxpayer to deduct or set-off against Phil. Income tax, the income, war profits and excess profits taxes that he has paid or has accrued to a foreign country. a. Tax exemption c. Tax consolidation b. Tax deduction d. Tax credit 93. Tariff may generally be defined as a. Amounts which are levied on goods leaving the imposing country. b. Amounts imposed on goods entering the imposing country. c. Amounts, imposed on goods passing through the imposing country, but destined ultimately to another country. d. List of commodities with corresponding duties collectible therefrom. 94. Any amount subsequently received on account of a bad debt previously charged off and allowed as a deduction from gross income for prior years must be included in gross income for the taxable year in which received. This is a. End-result doctrine c. Severance theory b. Destination of income test d. Equitable doctrine of tax benefit 95. Taxation, just like other fields of human learning is dynamic, not static, keeps on changing so we must-all study, otherwise we might be facing the battles of today with the antiquated weapons of yesterday. Regardless of economic and business conditions, a sound tax system should provide for the collection of sufficient revenue to run the government. a. True, true c. False, false b. True, false d. False, true 96. A progressive system of taxation means a tax structure a. Where the tax rate increases as the tax base increases. b. Where persons who desire to avail of government services or benefits are required to pay for their cost in the form of taxes c. Where persons who possess more in wealth or income are required to bear the cost of government corresponding to such capacity. d. Where the tax rate increases as the tax base decrease 97. Will not raise money for the government a. Power of taxation c. Police power b. License fee d. Eminent domain 98. Similarities of police power, power of taxation and power of eminent domain, except: a. There is compensation c. There is taking of property b. Sovereign powers of the government d. Superior to the non-impairment clause 99. Which of the following has no power to impose taxes? a. Provinces c. Barangays b. Cities d. President 100. Which statement is wrong? a. A tax is a demand of sovereignty c. A special assessment is a tax b. A toll is a demand of ownership d. Customs duty is a tax 101. Which of the following is not an element of double taxation? a. Two taxes c. Same year b. Same subject matter d. Same amount 102. One of the following is the least principle of sound tax system. a. Fiscal adequacy c. Theoretical justice b. Administrative feasibility d. Economic consistency 103. Basis of Taxation a. Lifeblood theory c. Benefit-protection theory b. Necessity theory d. Reciprocal theory A1 PASSERS REVIEW CENTER///34 104. Tax as distinguished from license free a. For regulation purpose c. For revenue purposes b. Amount imposed is limited d. Exercise of police power 105. The following are example of non-taxable compensation for injuries, except. a. Actual damages for injuries suffered. b. Exemplary damages c. Compensatory damages for unrealized profits d. Moral damages for grief, anxiety and physical sufferings 106. How much is the allowable deduction from business income of a domestic corporation which granted and paid P99,000 fringe benefits to its key officers in 1998? a. P150,000 c. P99,000 b. P100,000 d. P65,000 107. What would be the allowable deduction for P8,000 contribution made by a resident citizen to a religious organization from his P70,000 net income after contribution? a. P8,000 c. P7,800 b. P7,000 d. P3,500 108. A bought a condominium unit under instalment basis, to be used as his office in the practice of his profession and paying P10,000 monthly. For income tax purposes, the P10,000 monthly payment shall be a. Treated as business rental, hence deductible. b. Treated as capital expenditure, hence not deductible. c. Treated as depreciation expense, hence deductible. d. Treated as ordinary business expense. 109. The following are not taxable, except: a. Refund of tringe benefits tax c. Refund of estate of donor’s tax b. Refund of Philippine income tax d. Refund of special assessment 110. Which of the following does not represent compensation income? a. Honorarium as a guest speaker c. Vacation and sick leave pay b. Emergency lave pay d. Gratuitous condonation of obligation 111. 112. One of the following is not subject to final tax a. Interest on savings deposit c. Prizes amounting to more than P10,000 b. Royalties d. Professional fees paid to individuals 113. Gain on sale of domestic shares of stock in New York is a. Income within the Phils c. Income party within and without b. Income without the Phils d. Exempt from income tax 114. A was selected as the most outstanding teacher in her region. Her name was submitted by the school principal without her knowledge. She received a trophy and a cash award of P15,000. a. Taxable income c. Exempt from income tax b. Subject to final tax d. Partly taxable, partly exempt 115. This will not result to a taxable gain or loss a. The sale by a corporation of its shares of stock from the unissued stock over its par or stated value. b. The sale by a corporation of its treasury stock over its cost or other basis of acquisition. c. The purchase and retirement by a corporation of its bonds at a price less than the issue price or face value. d. The issuance by a corporation of its bond at a premium. 116. For ordinary contributions, an individual may be allowed to claim as deductions an amount not to exceed. a. 5% of the net income before contributions c. 5% of the net income after contribution b. 10% of the net income before contributions d. 10% of the net income after contribution 117. One of the following is a taxable income a. Compensation for injuries and sickness c. Proceeds of life insurance b. Income derived from gifts, bequest and d. Amounts received as returns of premium. devices 118. A domestic corporation made a borrowing from ABC bank thereby incurring a business connected interest expense of P60,000 for taxable year 1999. During the same year, the corporation earned an interest income subject to final tax in the amount of P100, 000. The deductible interest is: a. P19,000 c. P60,000 b. P21,000 d. P0 119. The share in the profits of a partner in a general professional partnership is regarded as received by him end thus taxable although not yet distributed. The principle is known as a. Actual receipt of income c. Accrual method of accounting b. Advance reporting of income d. Constructive receipt of income 120. Which of the following statements on tax exemptions is not correct? A1 PASSERS REVIEW CENTER///35 a. When an electric light and power franchise holder is exempt under its franchise property tax on its poles, wires and transformers, is exemption does not extend to the VAT of its importation of said articles. b. Where a taxpayer receives as payment for the land expropriated by the government tax exempt bonds, such tax exempt bonds should be included in the total price to determine correct taxable profit there from c. The salaries of the CIR and the four deputy CIR are exempt from income tax d. Exemptions granted to cooperatives does not extends to be members thereof in the sale of their products 121. A donated a parcel of land to B, his 15 year old son on account of B’s graduation. A did not pay the gift tax on the property donated. Then, B took possession of the property and received the rental derived from it. In 2007, an assessment on the income derived from the property was issued against B, which is correct? a. The assessment against B is correct. b. The income should be included in A’s income tax return. c. The assessment against B should be deferred. d. A validly effected a transfer of the property to B by virtue of delivery. 122. A feature of ordinary gains as distinguished from the capital gains: a. Gains from sale of assets not stock in trade c. Sources are capital assets b. May or may not be taxable in full d. No holding period 123. On capital gains tax on real property, which of the following statements is not correct? a. The tax should be paid, if in one lump sum, within 30 days from the date of the sale b. The instalment payment of the tax should be made within 30 days from receipt of each instalment payment on the selling price c. The tax may be paid in instalment if the initial payment does not exceed 25% of the contract price. d. The initial payment maybe more than down payment 124. A operates a retail store and owns the following properties. Which of the following is capital assets in the hand of A a. Building which houses the retail store b. Fixtures used in the retail store c. Inventory on hand at the end of the year d. Trade accounts receivable 125. Amount receivable by the estate of h deceased, his executor or administrator as an insurance under policy taken by the decedent upon his own life is a. Part of gross income whether the beneficiary is revocable or irrevocable b. Part of gross income if the beneficiary is irrevocable c. Part of gross income if the beneficiary is revocable d. Excluded from gross income 126. In 2007, ABC Corp. Paid total premiums of P1, 000 for the life insurance policy of the vice president, where the beneficiary in the corporation. At the end of the year, ABC received dividend of P100 because of the policy. In 2007, the corporation should indicate a claim for a deduction for life insurance premium of a. P1,000 c. P1,100 b. P900 d. Zero 127. If a friend inquires whether or not the cost of educational assistance to the employee and/or his dependents which are borne by the employer be taxable. What will your answer be? 128. 129. First Answer- A scholarship grant to the employee by the employer shall not be treated as taxable fringe benefit if the education or study involved is directly connected with the employer’s trade, business or profession and there is a written contract between them that the employee is under obligation to remain in the employ of the employer foe the period of time that they have mutually agreed upon. 130. 131. Second Answer- The cost of educational assistance extended by an employer to the of an employee shall be treated as taxable fringe benefits of the employee unless the assistance was provided through a competitive scheme under the scholarship program of the company. 132. a. True, true c. False, true b. True, false d. False, false 133. All of the following taxpayers are not foreign tax credit, except a. Resident citizen with income only from the Phils b. Resident citizen with income only from the abroad c. Resident citizen with income only from within and without the Phils d. Non-resident citizen with income only from and without the Phils 134. 1st Statement- Monetized unused vacation leave credits not exceeding 10 days is an exempt de minimis benefit. A1 PASSERS REVIEW CENTER///36 135. 136. 2nd Statement- Daily meal allowance for overtime work not exceeding twenty five (25%) percent of the basic minimum wage is exempt fringe benefit a. True, true c. True, false b. False, true d. False, false 137. 1st Statement- Laundry allowance not exceeding P300 per month is exempt de minimis benefit. 138. 139. 2nd Statement- Medical cash allowance to dependents of employees not exceeding P750 per employee per semester, or one hundred twenty five pesos (125) per month is exempt de minimis benefit. a. True, true c.True, false b. False, true d. False, false 140. 1st Statement- Flowers, fruits and books or other similar token items given to employees under certain circumstances are exempt de minimis benefits. 141. 142. 2nd Statement- gift given during Christmas and major anniversary celebrations not exceeding P5,000 per employee per annum is an exempt de minimis benefit. a. True, true c. True, false b. False, true d. False, false 143. 1st Statement- Rise subsidy of one thousand five hundred pesos or one sack of 50 kg. Rice per month amounting to not more than one thousand five hundred pesos is an exempt de minimis benefit. 144. 145. 2nd Statement- Employee achievement awards, e.g., for length of service, or safety achievement, 146. 147. which must be in the form of a tangible personal property other than cash or gift certificate, with an 148. annual monetary value not exceeding ten thousand pesos received by an employee under an established written plan which does not discriminate in favour of highly paid employees is an exempt de minimis benefit. a. True, true c. True, false b. False, true d. False, false 149. 150. 151. 152. 1st Statement- uniforms and clothing allowance not exceeding four thousand pesos per annum is an exempt de minimis benefit. 153. 2nd Statement- Actual medical benefits not exceeding P10,000 per annum is an exempt de minimis benefit. a. True, true c. True, false b. False, true d. False, false 154. Which of the following is included in gross income? a. Amounts received by insured as return of premium b. Life insurance proceeds c. Compensation for injuries or sickness d. Share in the net income of a general professional partnership 155. One of the following does not form part of gross income? a. Interest c. Annuities b. Royalties d. Gifts, bequest and device 156. In computing allowable deduction for purposes of income taxation. e. f. 1st Statement- Beginning the year 2005 interest expense in connection with taxpayer’s business shall be reduced by an amount equal to forty two percent (42%) of interest income subjected to final tax g. h. 2nd Statement- Interest incurred on money used to acquire property to be used in trade shall only be allowed as a capital expenditure. i. a. True, true k. c. True, false j. b. False, true l. d. False, false 157. Which of the following is not correct? a. An individual citizen of the Phils. Who is working and deriving income from abroad as an overseas contract worker is taxable only on income from sources within the Phils. b. A seaman who is a citizen of the Phils. And who receives compensation for services rendered abroad as a member of the complement of a vessel engaged exclusively in international trade shall be treated as an overseas contract worker. c. An alien individual is taxable only on income derived from sources within the Phils. d. A citizen of the Phils. Is taxable on income derived from sources within and without the Phils. A1 PASSERS REVIEW CENTER///37 158. A work for a manufacturing firm but due to business reverses, the firm offered a voluntary redundancy program in order to reduce overhead expenses. Under the program, an employee who offered to resign would be given separation pay equivalent to his 3 months basis salary for every year of service. A accepted the offer and received P800,000 as separation pay under the program. After all the employees who accepted the offer were paid, the firm found its overhead still excessive. Hence, it adopted another program, where various unprofitable departments were closed. As a result, B was separated from the services, B also received P800,000 as separation pay. At the time of separation both A and B have rendered at least 10 years of service but A was 55 years old while B was 45 years old. As a result, a. Both amounts are exempt from income tax. c. Only Mr. A is subject to income tax. b. Both amounts are subject to income tax. d. Only Mr. B is subject to income tax. 159. Which of the following expenses is deductible from gross income? a. Contribution to a newspaper fund for needy families when such newspaper organizes a drive solely for charitable purpose. b. Premiums paid by the self-employer for the insurance of his employees. c. Contribution to the construction of a chapel of a university dividends to its stockholders. d. Donation of prizes and awards to athletes in local and international competitions and sanctioned by their respective sport associations. 160. Cash dividends received by a non-resident foreign corporation from a domestic corporation is a. Exempt from income tax c. Part of taxable income b. Subject to final tax d. Partly exempt, partly taxable 161. Cash dividends received by a domestic corporation from a domestic corporation is a. Exempt from income tax c. Part of taxable income b. Subject to final tax d. d.Partly exempt, partly taxable 162. Which of the following statements is not correct? a. When a compulsory heir is given by will less than his legitime, the provisions of the will should be modified in such a way that he will receive his legitime b. The CIR may examine the bank deposit of a decedent for the purpose of determining his gross estate even if the estate did not request for a compromise on the ground of financial incapacity c. The P500,000 standard deduction for medical expenses for estate tax purpose is a legal mechanism to further exempt the less privileged estate and heirs from tax burden d. The sharing of heirs in testamentary succession must satisfy the rules on legitime 163. In a transfer in contemplation of death, revocable transfer and transfer under a general power of appointment, there are rules to observe to determine what amount to include in the gross estate, which is not a rule to observe? a. If the transfer was in the nature of a bona fide sale for an adequate and full consideration in money or money’s worth, no value shall be included in the gross estate. b. If there was no consideration received on the transfer as in donation intervivos, the value to include in the gross estate shall be the fair market value of the property at the time of transfer. c. If the consideration received on the transfer was less than adequate and full, the value to include in the gross estate shall be the excess of the fair market value of the property at the time of decedent’s death over the consideration received. d. If there was no consideration received on the transfer as in donation mortis cause, the value to include in the gross estate shall be the fair market value of the property at the time of the decedent’s death. 164. Which of the following is not directly reducing the inheritance? a. Standard deduction c. Unpaid mortgage b. Claims against the estate d. Funeral expense 165. Which of the following may reduce the taxable estate but not the inheritance? a. Funeral expense c. Judicial expense b. Losses d. Family home 166. Donor’s tax is a (an) a. Progressive tax c. Property tax b. Proportional tax d. Excise tax 167. Which of the following is a stranger donor? If the donee is the a. Grandfather of the donor’s great grandfather b. Granddaughter of the sister of the donor’s mother c. Grandson of the donor’s half-sister d. Brother of the donor’s grandfather 168. J, a Filipino citizen, married to M, died in a vehicular in NLEX on April 10, 2013. The spouses owned, among others a 100-hectare agricultural land in Sta. Rosa, Laguna with current fair market value of P20M, which was the subject matter of a Joint Venture Agreement about to be implemented with Star Land Corporation, a well known real estate development company. He bought the said real property for P2M fifty years ago. On January 5, 2013, the administrator of the estate and SLC jointly announce their big plans to start conversion and development of the agricultural lands in Sta. Rosa Laguna, into first class residential and commercial centers. As a result, the prices of real properties in the locality have doubled. The A1 PASSERS REVIEW CENTER///38 administrator of the estate of J filed the estate tax return on January 9, 2013 by including in the gross estate the real property at P2M. after 9 months, the BIR issued estate tax assessment by valuing the real property at P40M. As a result, the real property should be valued a. P2M b. P10M c. P20M d. P40M 169. While driving his car to Baguio P, together with his wife, A, and only son J, met an accident that caused the instantaneous death of J. The following day, A also died in the hospital. The spouses and their son had the following assets and liabilities at the time of death. 170. 171. (Exclusive) Conjugal J (Exclusive) 172. Cash P10,000,000 P1,200,000 173. Cars P2,000,000 500,000 174. Land 5,000,000 2,000,000 175. Residential House 4,000,000 176. Mortgage payable 2,500,000 177. Funeral Expenses 300,000 178. Which of the following statement is correct? a. The estate of J will pay corresponding estate tax. b. The administrator of J’s estate will claim vanishing deduction. c. The administrator of both estates will file one consolidated estate tax return. d. A CPA certificate as regards the valuation of A’s estate is required. 179. 180. 181. 182. 183. 184. 185. 186. 187. 188. 189.ANSWER KEY 190. TAXATION 191. 1. 2. 3. 4. 5. 6. 7. 8. 9. 10. 11. 12. 13. 14. 15. 16. 17. A B A A A D C A A D C B A D D D C 18. 19. 20. 21. 22. 23. 24. 25. 26. 27. 28. 29. 30. 31. 32. 33. 34. C C D C B D A B D B C A D C C C D 35. 36. 37. 38. 39. 40. 41. 42. 43. 44. 45. 46. 47. 48. 49. 50. 51. B S C B D D D A B D B D A A D D D A1 PASSERS REVIEW CENTER///39 52. 53. 54. 55. 56. 57. 58. 59. 60. 61. 62. 63. 64. 65. 66. 67. 68. C D D D C C A C B A D D A B A B B 69. 70. 71. 72. 73. 74. 75. 76. 77. 78. 79. 80. 81. 82. 83. 84. 85. B D C B D C D D D A B A A A A A B 86. 87. 88. 89. 90. 91. 92. 93. 94. 95. 96. 97. 98. 99. 100. D D D C D B A C B A C D B C D 192. 193. 194. 195. 196. A1 PASSERS TRAINING, RESEARCH, REVIEW & DEVELOPMENT COMPANY 197. 2nd Floor Sommerset Bldg., Lopez Jaena St. Jaro, Iloilo City 198. Tel. No.: (033) 320-2728; 09106547262 199. Email Address: [email protected] 200. BOARD OF CERTIFIED PUBLIC ACCOUNTANT 202. CERTIFIED PUBLIC ACCOUNTANT Licensure Examination 201. 203. 204. 205. THEORY OF ACCOUNTS 206. 1. Under PAS 32, which of the following assets is not financial asset? a. Cash b. An equity instrument of another entity c. A contract that may or will be settled in the entity’s own instrument and is not classified as an equity instrument of the entity d. Prepaid expense 2. Which of the following liabilities is a financial liability a. Deferred avenue. b. A warranty obligation. c. A constructive obligation. d. An obligation to deliver own shares worth a fixed amount of cash. 3. Which of the following instrument would not be classified as a financial liability? a. A preference share that will be redeemed by the issuer for cash on a future date. b. A contract for the delivery as many of the entity’s ordinary shares as are equal in value to P100,000 on a future date. c. A written call option that gives the holder the right to purchase a fixed number of the entity’s ordinary shares in return for a fixed price. d. An issued perpetual debt instrument. 4. What is the principal of accounting for a compound instrument? A1 PASSERS REVIEW CENTER///40 a. The issuer shall classify a compound instrument as either a liability or equity based on evaluation of the predominant characteristics of the contractual arrangement. b. The issuer shall classify the liability and equity components of a compound instrument separately as financial liability or equity instrument. c. The issuer shall classify a compound instrument as a liability in its entirely, until converted into equity, unless the equity components is detachable and separately transferable, in which case the liability and equity components shall be presented separately. d. The issuer shall classify a compound instrument as a liability in its entirely, until converted equity. 5. What is the accounting for treasury share transaction? a. On repurchase of treasury shares, a gain or loss is recognized equal to the difference between the amount at which the shares were issued and the repurchase price for the shares. b. On reissuance of treasury shares, a gain or loss is recognized equal to the difference between the previous repurchase price and the reissuance price. c. On repurchase or reissuance of previously repurchased own shares, no gain or loss is recognized. d. Treasury shares are accounted for as financial assets. 6. How are the proceeds from issuing a compound instrument allocated between the liability and equity components? a. First, the liability component is measured at fair value, and then the remainder of the proceeds is allocated to the equity component. b. First, the liability component is measured at fair value, and then the remainder of the proceeds is allocated to the liability component. c. First, the fair values of both the equity components and the liability component are estimated. Then the proceeds are allocated to the liability and equity components based on the relation between the estimated fair value. d. The equity component is measured at its intrinsic value. The liability component is measured at the par amount less the intrinsic value of the equity component. 7. What are the conditions for offsetting of financial assets and financial liabilities? a. A legal right of set-off. b. A legal right of set-off and an intention to settle net simultaneously. c. The existence of clearing mechanism or other market mechanism for net settlement and an expectation of net settlement. d. A netting agreement and an expectation of net settlement. 8. For what items is fair value required to be disclosed? a. All financial instruments. b. All financial instruments, except unquoted equity instruments and derivatives linked thereto. c. All financial assets and financial liabilities, except for investment in unquoted instruments and derivatives linked thereto. d. All financial assets, except for investment in unquoted instruments and derivatives linked thereto. 9. Transaction costs that are directly attributable to the ”issuance of new shares” should be a. Expensed immediately. b. Changed to retained earnings. c. Deducted from equity. d. Deducted from equity, net of any related income tax benefit. 10. Cost of public offering of shares or costs that relate to the “stock market listing of shares” should be a. Expensed immediately. b. Considered a component of other comprehensive income. c. Deducted from equity. d. Deducted from equity, net of any related income tax benefit 11. Transaction costs directly attributable to the issuance of new shares include all of the following, except a. Documentary stamp tax and other c. SEC registration fee for new shares percentage tax d. Stock listing fee b. Underwriting fee 12. Costs of public offering or listing of shares include I. Road show presentation II. Public relations consultant fee III. Newspaper publication fee directly relating to the shares issue. a. I only c. II and III only b. I and II only d. I,II and III 13. What is the treatment of “joint costs” that relate jointly to the concurrent listing and issuance of new shares, and listing of old existing shares? a. The joint costs should be expensed immediately. b. The joint costs should be deducted from equity, net of tax benefit. c. The joint costs should deducted from equity, plus tax benefit. A1 PASSERS REVIEW CENTER///41 d. The joint costs should be allocated between the newly issued and listed shares and the newly listed old existing shares prorate based on the number of shares outstanding. 14. Joint costs related to the concurrent listing and issuance of new shares and listing of old existing shares include all of the following, except a. Fairness opinion and valuation report. b. Tax opinion and opinion of counsel. c. Audit and other professional advice relating to prospectus design and printing. d. Documentary stamp tax. 15. Under PAS 33, contingent ordinary shares are treated as outstanding and included in the computation of both basic and diluted earnings per share if the conditions are satisfied. Which of the following statements is true? I. Contingent ordinary shares are included in the calculation of basic earnings per share from the date the condition is satisfied. II. Contingent ordinary shares are included in the calculation of diluted earnings per share from the beginning of the period or from the date of contingent agreement, if later. a. I only c. Both I and II b. II only d. Neither I nor II 16. Entity A has an ordinary “A” class, nonvoting share, which is entitled to a fixed dividend of 6% per annum. The “A” class ordinary share will a. Be included in the “per share” calculation after adjusting for the fixed dividend. b. Be included in the “per share” calculation for EPS without adjustment for the fixed dividend. c. Not be included in the “per share” calculation for EPS. d. Be included in the calculation of diluted EPS. 17. Earnings per share are calculated before accounting for which of the following items? a. Preference dividend for the period c. Taxation b. Ordinary dividend d. Minority interest 18. Ordinary shares issued as part of a business combination are included in the EPS from a. The beginning of the accounting period c. The end of the accounting period b. The date of acquisition d. The midpoint of the accounting year 19. 20. 21. 22. 23. Which statement is true concerning EPS calculation? I. Potential ordinary shares issued by a subsidiary should be included in diluted EPS as they could potentially have an impact on the net profit for the period and the number of shares to be included in the calculation. II. An entity should disclose diluted EPS only if it differs from basic EPS by a material amount. a. I only c. Both I and II b. II only d. Neither I nor II 24. Under PAS 34, interim financial reports shall include as a minimum a. A complete set of financial statements. b. A condensed set of financial statements and selected notes. c. A statement of financial position and an income statement only. d. A condensed statement of financial position income statement and statement of cash flows only. 25. If an entity does not prepare interim financial reports. a. The year-end financial statements are deemed not to comply with PERS. b. The year-end financial statements compliance with PFRS is not affected. c. The year-end financial statements will not be acceptable under local legislation. d. Interim financial reports should be included in the year-end financial statements. 26. PAS 36 applies to which of the following assets? a. Inventories c. Assets held for sale b. Financial assets d. Property, plant, and equipment 27. The internal sources of information indicating possible impairment include all of the following, except a. Evidence of obsolescence or physical damage of an asset. b. Significant change in the manner or extent in which the asset is used with an adverse effect on the entity. c. Evidence that the economic performance of an asset will be worse than expected. d. Significant decrease or decline in the market value of the asset. 28. The external sources of information including possible impairment include all of the following, except a. Significant change in the technological, market, legal or economic environment of the business in which the asset is employed. b. An increase in the interest rate or market rate of return on investment which will likely affect the discount rate used in calculating value in use. c. The carrying amount of the net assets of the entity is more than its market capitalization. A1 PASSERS REVIEW CENTER///42 d. Significant decline in budgeted net cash flows or significant increase in budgeted loss flowing from the asset. 29. When deciding on the discount rate to be used in calculating value in use, which factor should not be taken into account? a. The time value of the money. b. Risk specific to the asset for which future cash flows estimate have not been adjusted. c. Risk specific to the asset for which future cash flows estimate have been adjusted. d. Pretax rate 30. Which of the following impairment losses should never be reversed? a. Loss on property, plant, and equipment c. Loss on a business segment b. Loss on goodwill d. Loss on inventory 31. Where part of a cash generating unit is disposed of the goodwill associated with the element disposed of a. Shall not be written off to the income statement entirely. b. Shall not be included in the calculation of gain or loss. c. Shall be included in the calculation of gain or loss. d. Shall be written off against retained earnings. 32. When allocating an impairment loss, such a loss should reduce the carrying amount of which asset first? a. Property, plant, and equipment c. Goodwill b. Intangible assets d. Current assets 33. Under PAS 38, which of the following disclosures is not required with respect to intangible assets? a. Useful lives of the intangible assets. b. Reconciliation of carrying amount at the beginning and the end of the year. c. Contractual commitments for the acquisition of intangible assets. d. Fair value of similar intangible assets used by its competitors. 34. A new dot-com entity has recently completed one of its highly publicized research and development projects. Which of the following statement is true? a. Cost incurred during the “research phase” can be capitalized. b. Cost incurred during the “development phase” can be capitalized if criteria such as technical feasibility of the project being established are met. c. Training costs of technicians used in research can be capitalized. d. Designing of the jigs and tools would qualify as research activities. 35. 36. 37. 38. Under PAS 37, a “provision” id recognized I. When there is a legal obligation arising from a past obligating event, the probability of the outflow of resources is more than remote but less than probable, and a reliable estimate can be made of the amount of the obligation. II. When there is a constructive obligation as a result of a past obligating event, the outflow of resources is probable, and a reliable estimate can be made of the amount of the obligation. a. I only c. Both I and II b. II only d. Neither I nor II 39. Which of the following is within the scope of PAS 37? a. Financial instruments carried at fair value c. Future payments on vacant leasehold b. Future payments under employment contracts premises d. An insurance entity’s policy liability 40. A competitor has sued an entity for unauthorized use of its patented technology. The amount that the entity may be required to pay to the competitor succeeds in the lawsuits is determinable with reliability, and according to the legal counsel it is less than probable but more than remote that an outflow of the resources would be needed to meet the obligation. The entity that was sued should at year-end a. Recognize a provision for this possible obligation. b. Make a disclosure of the possible obligation in the notes to financial statement. c. Make no provision or disclosure and wait until the lawsuit its finally decided and then expense the amount paid on settlement. d. Set aside, as an appropriation, a contingency reserve, an amount based on the best estimate of the possible liability. 41. Under PAS 39, which of the following is not a category of financial assets? a. Financial assets at fair value through profit or c. Held for sale investments loss d. Loans and receivables b. Available for sale financial assets 42. All of the following are characteristics of financial assets classified as held-to-maturity investments, except. a. They have fixed or determinable payments and a fixed maturity. b. The holder can recover substantially all of its investment unless has been credit deterioration. c. They are quoted in an active market. d. The holder has a demonstrated positive intention and ability to hold them to maturity. 43. All of the following are characteristics of financial assets classified as loans and receivables, except. a. They have fixed or determinable payments. A1 PASSERS REVIEW CENTER///43 b. The holder can recover substantially all of its investment unless has been credit deterioration. c. They are quoted in an active market. d. The holder has a demonstrated positive intention and ability to hold them to maturity. 44. What is the principle for recognition of a financial asset? a. A financial asset is recognized when it is probable that future economic benefits will flow to the entity and the cost or value of the instrument can be measured reliably. b. A financial assets id recognized when the entity obtain control of the instrument and has the ability to dispose of the financial asset independent of the actions of others. c. A financial asset is recognized when the entity obtains the risks and rewards of ownership of the financial asset and has the ability to dispose the financial asset. d. A financial asset is recognized when the entity becomes a partly to the contractual provisions of the instrument. 45. At what amount is a financial asset or financial liability measured on initial recognition? a. The consideration paid or received for the c. Fair value financial asset or financial liability. d. zero b. Acquisition cost 46. In which of the following circumstances is derecognition of a financial asset not appropriate? a. The contractual rights to the cash flows of the financial assets have expired. b. The financial asset has been transferred and substantially all the risk and rewards of ownership of the transferred asset have also been transferred. c. The financial asset has been transferred and the entity has retained substantially all the risks and rewards of ownership of the transferred asset. d. The financial asset has been transferred and the entity has neither retained nor transferred substantially all the risks and rewards of ownership of the transferred asset but the entity has lost control of the transferred asset. 47. Which of the following transfers of financial asset would qualify for derecognition? a. A sale of financial asset where the entity retains an option to buy the asset back at its current fair value on the repurchase date. b. A sale of financial asset where the entity agrees to repurchase the asset in one year for a fixed price plus interest. c. A sale of portfolio of short-term accounts receivable where the entity guarantees to compensate the buyer for any losses in the portfolio. d. A loan of a security to another entity. 48. Which of the following is not a relevant consideration when evaluating whether to derecognize a financial liability? a. Whether the obligation has been discharged. b. Whether the obligation has been cancelled. c. Whether the obligation has expired. d. Whether substantially all the risks and rewards of the obligation have been transferred. 49. What is the evidence of the fair value of a financial instrument? a. Its cost, including transaction costs directly attributable to the purchase origination or issuance of the financial instrument. b. Its estimated value determined using discounted cash flows techniques, option pricing models or other valuation techniques. c. Its quoted price, if an active market exist for the financial instrument. d. The present value of the contractual cash flows less impairment. 50. Which of the following is not objective evidence of impairment of a financial asset? a. Significant financial difficulty of the issuer or obligor. b. A decline in the fair value of the asset below its previous carrying amount. c. A breach of contract, such as a default or delinquency in interest or principal payments. d. Observable data indicating that there is a measurable decrease in the estimated future cash flows from a group of financial assets although the decrease cannot yet be associated with any individual financial asset. 51. All of the following are characteristics of a derivative, except a. It is acquired or incurred by the entity for the purpose of generating a profit from short-term fluctuations in market factors. b. Its value changes in response to the change in a specified underlying. c. It requires no initial investment or an initial net investment. d. It is settled at a future date. 52. Which embedded derivatives should not be accounted for separately? a. An investment in a convertible bond that is classified as available for sale. b. A investment in a bond whose interest payments are linked to the price of gold and the bond is classified as available for sale. A1 PASSERS REVIEW CENTER///44 c. An investment in a bond whose interest payments are linked to the price of silver and the bond is classified as at fair value through profit or loss. d. A call option in an investment in an equity instrument that allows the issuer to repurchase the instrument. 53. This is defined as a “purchase or sale of a financial asset under a contract whose terms require delivery of the asset within the time frame established generally by regulation or convention in the market place concerned”. a. Regular way contract c. Firm commitment b. Forward contact d. Forecast transaction 54. What is the meaning of “trade date accounting” in relation to regular way purchase or sale of financial asset? I. The recognition of the asset to be received and the liability to be paid on the date on which the entity commits itself to purchase or sell an asset. II. The recognition of the asset on the date received by the entity or derecognition of the asset on the date delivered by an entity. a. I only c. Either I or II b. II only d. Neither I nor II 55. Under PAS 40, an investment property should be measured initially at a. Cost b. Cost less accumulated depreciation and impairment losses. c. Depreciable cost less accumulated impairment losses. d. Fair value less accumulated impairment losses. 56. A gain arising from a change in the fair value of an investment property for which an entity has opted to use the fair value model is recognized in a. Net profit or loss for the year. b. General reserve in the shareholder’s equity. c. Valuation reserve in the shareholder’s equity. d. Retained earnings. 57. Which of the following is not deal with by PAS 41? a. The accounting for biological assets. b. The initial measurement of agricultural produce harvested from the entity’s biological assets. c. The processing of agricultural produce after harvesting. d. The accounting treatment of government grant received in respect of biological assets. 58. Generally speaking, biological assets relating to agricultural activity should be measured using a. Historical cost b. Historical cost less depreciation impairment c. A fair value approach d. Net realized value 59. An entity had a o forest that is likely to be harvested and sold in thirty years. The income should be accounted for in which of the following way? a. No income should reported annually until first harvest and sale in thirty years. b. Income should be measured annually and reported using a fair value approach that recognizes and measures biological growth. c. The eventual sale proceeds should be estimated and matched to the profit and loss account over the 30-yearperiod. d. The plantation forest should be measures every five years and the increase in value should be shown in the statement of recognized gains and losses. 60. Which of the following information shall be disclosed in relation to biological assets and agricultural produce? a. Separate disclosure of the gain or loss relating to biological assets and agricultural produce. b. The aggregate gain or loss arising on the initial recognition of biological assets and agricultural produce and from the change in fair value less cost to sell of biological assets. c. The total gain or loss from biological assets, agricultural produce, and from changes in fair value less cost to sell of biological assets. d. There is no requirement in the standard to disclose separately any gain or losses. 61. Where there is a production cycle of more than one year for a biological asset separate disclosure is encourage for a. Physical change only c. Total change in value b. Price change only d. Physical change and price change 62. An unconditional government grant related to a biological asset that has been measured at fair value less cost to sell should be recognized as a. Income when the grant becomes receivable b. A deferred credit when the grant becomes receivable c. Income when the grant application has been submitted A1 PASSERS REVIEW CENTER///45 d. A deferred credit when the grant has been approved 63. If a government grant related to a biological asset is conditional on certain events, the grant should be recognized as a. Income when the conditions attaching to the grant are met. b. Income when the grant has been approved c. A deferred credit when the conditions attached to the government grant are met. d. A deferred credit when the grant is approved. 64. This is defined in PFRS 1 as the “first annual financial statements in which an entity adopts Philippine Financial Reporting Standards (PFRS) by an explicit and unreserved statement of compliance with PFRS”. a. PFRS financial statements b. First PFRS financial statements c. Opening PFRS statement of financial opinion d. First audited PFRS financial statements 65. An entity that presents in first PFRS financial statements is known as a. An originating entity c. A first-time adopter b. A provisional presenter d. A initial reporter 66. An entity’s statement of financial position, published or unpublished, at the date of transition to PFRS is best described as the a. Provisional PFRS statement of financial position b. Closing GAAP statement of financial position c. Opening PFRS statement of financial position d. Originating PFRS statements of financial position 67. Which of the following statements best describes the “date of transition to PFRS”? a. The beginning of the latest period presented in the entity’s most recent annual financial statements under previous GAAP. b. The end of the latest period presented in the entity’s most recent annual financial statements under previous GAAP. c. The beginning of the earliest period for which an entity presents full comparative information under PFRS in its PFRS financial statements. d. The end of the earliest period for which an entity presents full comparative information under PFRS in its first PFRS financial statements. 68. Which of the following transactions involving the issuance of shares does not come within the definition of a “share-based” payment under PFRS 2? a. Employee share purchase plans. c.Share-based payment relating to an b. Employee share option plans. acquisition of a subsidiary. d. Share appreciation rights. 69. Which of the following is true regarding the requirements of PFRS 2? a. Private entities are exempt. b. “Small” entities are exempt. c. Subsidiaries using their parent entity’s shares as consideration for goods and services are exempt. d. There are no exemptions from PFRS 2. 70. It is an arrangement that provides for automatic grant share whenever the option holder exercises previously granted options using the entity’s share, rather than cash, to satisfy the exercise price a. Reload feature c. Share option b. Reload option d. Equity option 71. Under PFRS 3, which of the following examples is unlikely to meet the definition of an intangible asset? a. Marketing related such as trademarks and internet domain names. b. Customer related such as customer lists and contracts. c.Technology based, such as computer software and databases. d. Pure research based, such as general expenditure on research. 72. Which of the following would not contribute to the creation of negative goodwill? a. Errors in measuring the fair value of the acquirce’s net identifiable assets or the cost of the business combination. b. A bargain purchase. c. A requirement in a PFRS to measure net assets acquired at a value other than fair value. d. Making acquisitions at the top of a “bull” market for shares. 73. PFRS 4 defines a reinsurance contract as a contract issued by one insurer, the reinsurer, to compensate another insurer for losses one or more contracts issued by the cedant. What is the meaning of “cedant” in reinsurance contract? a. Policyholder under a reinsurance contract. c.Insurer under an insurance contract. b. Policyholder under an insurance contract. d. Reinsurer under a reinsurance contract. 74. The recognition of unrealized gain or loss on the measurement of the financial assets and insurance liabilities as a component of other comprehensive income is described in insurance parlance as A1 PASSERS REVIEW CENTER///46 a. Fair value accounting c. Hedge accounting b. Current value accounting d. Shadow accounting 75. This is defined as “the payments to which policyholder has an unconditional right that is not subject to the contractual discretion of the insurer”. a. Guaranteed benefits c. Proceeds policy b. Unconditional benefits d. Executor benefits 76. It is the insurer’s net contractual right under an insurance contract. a. Insurance asset c. Reinsurance asset b. Insurance liability d. Reinsurance liability 77. Which of the following accounting practices has been outlawed in relation to insurance contracts? a. Shadows accounting b. Catastrophe provisions c. A test for the adequacy of recognized insurance liabilities d. An impairment test for reinsurance assets 78. All of the following are requirements for insurance contracts, except a. Nonoffsetting of reinsurance assets against related insurance liabilities b. An annual assessment of the adequacy of recognized insurance liabilities c. An impairment test for reinsurance assets d. Recognition of provisions for future claims relating to a catastrophe 79. Which of the following types of contracts would probably not be covered by PFRS 4? a. Motor insurance c. Medical insurance b. Life insurance d. Pension plan 80. Under PFRS 5, how should the assets and liabilities of a disposal group classifies as held for sale be shown in the statement of financial position? a. The assets and liabilities should be offset and presented as a single amount. b. The assets of disposal group should be shown separately from the other assets and the liabilities of the disposal group should be shown separately from other liabilities. c. The assets and liabilities should be presented as a single amount and as a deduction from equity. d. There should be no separate disclosure of assets and liabilities that from part of a disposal group. e. 81. Equity acquires a subsidiary exclusively with a view to selling it. The subsidiary meets the criteria to be classified as held for sale. At the end of reporting period, the subsidiary has not yet been sold, and six months have passed since its acquisition. How will the subsidiary be valued at the date of the first financial statements after acquisition? a. At fair value c. At carrying amount b. At the lower of its cost and fair value less d. In accordance with applicable PFRS cost to sell. 82. Equity has an asset that was classified as held for sale. However, the criteria for it to remain as held for sale no longer apply. The entity should a. Leave the noncurrent asset in the financial statements at its current carrying amount. b. Remeasure the noncurrent asset at fair value. c. Measure the noncurrent asset at the lower of its carrying amount before the asset was classified as held for sale adjusted for subsequent depreciation, amortization or revaluation, and its recoverable amount at the date of the decision not to sell. d. Recognize the noncurrent asset at its carrying amount prior to its classification as held for sale adjusted for subsequent depreciation, amortization or revaluation. 83. How should the income from discontinued operation be presented in the income statement? a. The entity should disclose a single amount on the face of the income statement wit analysis in the notes or a section of the income statement separate from continuing operations. b. The amounts from discontinued operations should be broken down over each category of revenue and expense. c. Discontinued operation should be shown as a movement on retained earnings. d. Discontinued operation should be shown as a line item after gross profit with the taxation being shown as part of income tax expense. 84. Which of the following criteria does not have to be met in order for an operation to be classified as discontinued? a. The operation should represent a separate major line of business or geographical area. b. The operation is part of single plan to dispose of a separate major line of business or geographical area. c. The operation is a subsidiary acquired exclusively with a view to resale. d. The operation must be sold within three months of the year-end. 85. Does PFRS 6 acquire an entity to recognize exploration and evaluation expenditure as asset? a. Yes, but only to the extent such expenditure is recoverable in future periods. A1 PASSERS REVIEW CENTER///47 b. Yes, but only to the extent the technical feasibility and commercial viability of extracting the associated mineral resource have been demonstrated. c. Yes, but only to the extent required by the entity’s accounting policy for recognizing exploration and evaluation asset. d. No, such expenditure is always expensed in profit or loss as incurred. 86. An entity is required to consider which of the following in developing accounting policies for exploration and evaluation activities? a. The requirements and guidance in Standards and Interpretations dealing with similar and related issues. b. The definitions, recognition criteria, and measurement concepts for assets, liabilities, income, and expenses in the Conceptual Framework c. Recent pronouncements of standard-setting bodies, accounting literature and accepted industry practices. d. Whether the accounting policy results in information that is relevant and reliable. 87. Which of the following expenditures would never qualify as an exploration and evaluation asset? a. Expenditure for acquisition of rights to explore. b. Expenditure for exploratory drilling. c. Expenditures related to the development of mineral resources. d. Expenditures for activities in relation to evaluating the technical feasibility and commercial viability of extracting a mineral resources. 88. Which of the following is not a disclosure required in relation to exploration and evaluation expenditures? a. Information about commercial reserve quantities. b. Accounting policies for exploration and evaluation expenditures, including the recognition of exploration and evaluation assets. c. The amounts of assets, liabilities, income and expense, and operating and investing cash flows arising from the exploration and evaluation of mineral resources. d. Information that identifies and explain the amounts recognized in the financial statements arising from the exploration and evaluation of mineral resources. 89. 90. 91. 92. 93. Which is not a characteristic of the “full cost” method of accounting in the oil and gas industry/ a. All costs incurred in acquiring, exploring and developing within a defined cost center are capitalized and amortized. b. Costs are capitalized even if a specific project in a cost center was a failure. c. Costs of unsuccessful acquisition and exploration activities are charged to expense. d. Exploration and evaluation asset is classified either as tangible asset or an intangible asset according to the nature of the asset. 94. Under PFRS 7, the risk arising from financial instruments that are required to be disclosed include all of the following, except a. Qualitative and quantitative information about credit risk. b. Qualitative and quantitative information about liquidity risk. c. Qualitative and quantitative information about market risk. d. Qualitative and quantitative information about operation risk. 95. This is defined as “the risk that one party to a financial instrument with cause a financial loss to the other arty by failing to discharge an obligation”. a. Credit risk c. Market risk b. Liquidity risk d. Operational risk 96. Which of the following best describes “the risk that an entity will encounter if it has difficulty in meeting obligations associated with its financial liabilities”? a. Liquidity risk c. Financial risk b. Credit risk d. Payment risk 97. This is defined as “the risk that the fair value or future cash flows of a financial instrument will fluctuate because of changes in market prices”. a. Credit risk c. Market risk b. Liquidity risk d. Operational risk 98. The components of market risks are a. Credit risk and liquidity risk c. Interest rate risk and currency risk b. Currency risk and credit risk d. Liquidity risk and currency risk 99. Disclosure of information about significant concentrations of credit risk is required for a. All financial instruments b. Financial instruments with off statement of financial position credit risk only c. Financial instruments with off statement of financial position market risk only A1 PASSERS REVIEW CENTER///48 d. Financial instruments with off statement of financial position risk of accounting loss only 100. Under PFRS 8, an operating segment is reportable when I. The segment external and internal revenue is 10% or more of the combined external and internal revenue of all operating segments. II. The segment profit or los is 10% more of the greater between the combined profit of all operating segments that reported profit and the combined loss of all operating segments that reported a loss. III. The assets of the segment are 10% or more of the total assets of all operating segments. a. I and II only c. II and III only b. I and III only d. I, II and III 101. Which of the following statements is true about major customer disclosure? I. A major customer is defined as one providing revenue which amounts to 10% or more of combined internal and external revenue of all operating segments. II. The identities of major customer must be disclosed. a. I only c. Both I and II b. II only d. Neither I nor II 102. Under SIC 15, what is the treatment of operating lease incentives (upfront cash, reimbursement of costs incurred by lessee and rent-free period granted by the lessor to the lessee? I. The lessor shall recognize the aggregate cost of the incentives as a reduction of rent income over the lease term on a straight line basis. II. The lessee shall recognize the aggregate benefit of the incentives as reduction of rent expense over the lease term. a. I only c. Both I and II b. II only d. Neither I nor II 103. Under IFRIC 2, members’ shares in cooperatives may give the holder the right to request redemption for cash or other financial asset. Such members’ shares shall be accounted for as a. Equity c. Either as equity or liability b. Liability d. Partly equity and partly liability 104. 105. 106. 107. 108. Member’s shares in cooperatives shall be classified as equity I. If the entity has unconditional right to refuse redemption of members’ share II. If the redemption of members’ shares is unconditionally prohibited by law. a. I only c. Either I or II b. II only d. Neither I nor II 109. Fair value of an asset should be based upon a. The replacement cost of an asset. b. The price that would be received to sell the asset at the measurement date. c. The original cost of the asset plus an adjustment for obsolescence. d. The rice that would be paid to acquire the asset. 110. Which of the following describes a principal market for establishing fair value of an asset? a. The market that has the greatest volume and level of activity for the asset. b. Any broker or dealer market that buys or sells the asset. c. The most observable market in which the price of the asset is minimized. d. The market in which the amount received would be maximized. 111. Which of the following would meet the qualifications as market participants? a. A liquidation market in which sellers are compelled to sell. b. A subsidiary of the reporting until interested in purchasing assets similar to those being valued. c. An independent entity that is knowledge about the asset. d. A broker or dealer that wishes to establish new market for the asset. 112. Which of the following in assumption used in fair value measurements? a. The asset must be in-use. b. The asset must be considered in-exchange. c. The most conservative estimate must be used. d. The asset in its highest and best use. 113. Which of the following is not a valuation technique used in fair value estimates? a. Income approach c. Market approach b. Residual value approach d. Cost approach 114. Valuation techniques for fair value that include the Black-Scholes formula, a binomial model, or discounted cash flows are example of which valuation technique? a. Income approach c. Cost approach b. Market approach d. Exit value approach A1 PASSERS REVIEW CENTER///49 100. a. b. c. d. The market approach valuation technique for measuring fair value requires which of the following? Present value of future cash flows. Prices and other related information of transactions from identical or comparable assets. The price to replace the service capacity of the asset. The weighted average of the present value of future cash flows. e. f. g. h. i. j. k. l. m. n. o. p. q. r. s. t. u. v. w. x. y. z. aa. ab. ac. ad. ae. ANSWER KEY af. THEORY OF ACCOUNTS ag. ah. 1. D 2. D 3. C 4. B 5. C 6. A 7. B 8. C 9. D 10. D 11. B 12. D 13. D 14. D 15. C 16. C 17. 18. 19. 20. 21. 22. 23. 24. 25. 26. 27. 28. 29. 30. 31. 32. B B A B B D D D C B C C D B B C 33. 34. 35. 36. 37. 38. 39. 40. 41. 42. 43. 44. 45. 46. 47. 48. B C B D D C C A D C B A C A A A A1 PASSERS REVIEW CENTER///50 49. 50. 51. 52. 53. 54. 55. 56. 57. 58. 59. 60. 61. 62. 63. 64. 65. 66. A C C B B D A A B C C C C D A D D A 67. 68. 69. 70. 71. 72. 73. 74. 75. 76. 77. 78. 79. 80. 81. 82. 83. 84. D A A B D D B B C A D C D C A C D A 85. A 86. C 87. C 88. A 89. D 90. D 91. C 92. C 93. C 94. B 95. A 96. C 97. D 98. B 99. A ai. 100. B aj. ak. al. am. A1 PASSERS TRAINING, RESEARCH, REVIEW & DEVELOPMENT COMPANY an. 2nd Floor Sommerset Bldg., Lopez Jaena St. Jaro, Iloilo City ao. Tel. No.: (033) 320-2728; 09106547262 ap. Email Address: [email protected] aq. ar. BOARD OF CERTIFIED PUBLIC ACCOUNTANT as. at. CERTIFIED PUBLIC ACCOUNTANT Licensure Examination SET A au. av. 1. 2. 3. 1. 2. AUDITING THEORY aw. ax. INSTRUCTIONS: Detach one (1) answer sheet from the bottom of your Examinee ID/Answer Sheet Set. Write the subject title “AUDITHING THEORY” on the box provided. Shade Set Box “A” on your answer sheet if your test booklet is Set A; Set Box “B” if your test booklet is Set B. ay. az. MULTIPLE CHOICE: ba. Which of the following statements is correct concerning the concept of materiality? a. Materiality depends on the nature of an item rather than the peso amount. b. Materiality is matter of professional judgment. c. Materiality is determined by reference to guidelines established by the PICPA. d. Materiality depends only on the peso amount of an item relative to other items in the financial statements. When considering internal control, an auditor must be aware of the concept of reasonable assurance which recognizes that the a. Employment of competent personnel assurance that managements control objectives will be achieved. b. Cost of internal control should not exceed the benefits expected to be derived there from. c. Separation of incompatible functions is necessary to ascertain that the internal control is effective. A1 PASSERS REVIEW CENTER///51 d. Establishment and maintenance of internal control is an important responsibility of the management. 3. The ultimate purpose of assessing control risk is to contribute to the auditor’s evaluation of the a. Possibility that the nature and extent of substantive test may be reduced. b. Risk that material mis-statements exist in the financial statements. c. Operating effectiveness of internal controls. d. Factors that raise doubts about the audit ability of the financial statements. 4. The auditor looks for an indication on duplicate sales invoices to see if the invoices have been verified. This is an example of a. A test of controls b. A substantive test of transaction c. A test of details of balances d. Both a test of control and substantive test of transaction 5. Which of the following statement about tests of controls is most accurate? a. Tests of controls ordinarily should be performed as of the balance sheet date during the period subsequent to the date. b. Tests of control include observation of the proper segregation of duties that ordinarily may be limited to the normal audit period. c. Auditing procedures cannot currently provide both evidence of the effectiveness of internal control procedures and evidence required for substantive tests. d. Tests controls should be based upon proper application of an appropriate statistical sampling plan. 6. The two phases of the auditor’s involvement with internal control are sometimes referred to as “understanding and assessment” tests of controls. In the tests of controls phase, the auditor attempts to: a. Obtain sufficient, appropriate evidential matter to afford a reasonable basis for the auditor’s opinion. b. Obtain a reasonable degree of assurance that the client’s controls are in use and are operating as planned. c. Obtain assurances that informative disclosures in the financial statements are reasonably adequate d. Obtain knowledge and understanding, the client’s prescribed procedures and methods. 7. The sequence of steps in gathering evidences as the basis of the auditor’s opinion is: a. Assessment of control risk, substantive tests, and test of controls. b. Tests controls, assessment of control risk, and substantive tests. c. Assessment of control risk, tests of controls, and substantive tests. d. Substantive tests, assessment of control risks, and tests of controls. bb. 8. Which of the following procedure would an auditor least likely perform in planning a financial statement audit? a. Coordinating the assistance of entity personnel in data preparation. b. Selecting a sample of vendor’s invoices for comparison to receiving reports. c. Reading the current year interim financial statements. d. Discussing matters that may affect the audit with the firm personnel responsible for non-audit services to the entity. 9. ROI Manufacturing Co. received a substantial sales return on December 30, 2010, but the credit memorandum for the return was not prepared and recorded until March 4, 2011. The returned merchandise was included in the year-end physical inventory taken on December 2010. The most effective procedure for preventing this type of error is to: a. Prepare an aged schedule of accounts receivable monthly. b. Pre-number and accounts for all credit memorandums. c. Reconcile the subsidiary accounts receivable ledger with the general ledger control account monthly. d. Prepare and numerically control receiving reports for all materials received. 10. Which of the following best describes why an independent auditor is asked to express an opinion on the fair presentation of financial statements? a. It is difficult to prepare financial statements that fairly resent a company’s financial position, financial performance, and cash flows without the expertise of an independent auditor. b. It is customarily courtesy that all share holders of a company receive an independent report on management’s stewardship of the affairs of the business. c. The opinion of an independent party is needed because a company may not be objective with respect to its own financial statements. d. It is customary courtesy that all shareholders of a company receive an independent report on management’s stewardship of the affairs of the business. 11. An independent auditor has the responsibility to design the audit to provide reasonable assurance of detecting errors and fraud that might have a material effect on the financial statements. Which of the following. If material would be a fraud as defined in PSA 240 (The Auditor’s Responsible Relating to Fraud in an Audit of Financial Statements)? a. Misappropriation of an asset or groups of assets. b. Clerical mistakes in the accounting data underlying the financial statements. c. Mistakes in the application of accounting principles. d. Misinterpretation of facts that existed when the financial statements were prepared. A1 PASSERS REVIEW CENTER///52 12. The date of the CPA’s opinion on the financial statements of his or her client should be the date of the a. Closing of the client’s books. c. Completion of all important audit procedures. b. Receipt of the client’s letter of representation. d. Submission of the report to the client. 13. During an audit engagement, pertinent data are complied and included in the audit working papers. The working papers primarily are considered to be a. A client-owned record of conclusions reached by auditors who performed the engagement. b. Evidence supporting financial statements. c. Support for the auditor’s representation as to compliance with relevant PSAs. d. A record to be used as a basis for the following year’s engagement. 14. Significant unexpected fluctuations indentified by analytical procedures will usually necessitate a (n) a. Audit report modification. c. Explanation in the representation letter. b. Understanding of the client’s internal control. d. Audit investigation. 15. Sandy requested permission to communicate with the predecessor auditor and review certain portion of the predecessor’s working papers. The prospective client’s refusal to permit this will bear directly on Sandy’s decision concerning the a. Adequacy of the preplanned audit program. b. Ability to establish consistency in application of accounting principles between years. c. Apparent scope limitation. d. Integrity of management. 16. Your analytic procedures and other test of the Yang Company reveal that the firm’s poor financial condition makes it unlikely that it will survive as a going concern. Assuming that the financial statements have otherwise been prepared in accordance with generally accepted accounting 17. principles, what disclosure should you make of the company’s precarious financial position? a. You should issue a qualified opinion and, in an emphasis of matter paragraph, direct the reader’s attention to the poor financial condition of the company. b. You should insist that a note to the financial statements clearly indicates that the company appears to be on the verge of bankruptcy. c. You need not insists on any specific disclosure, since the company’s poor financial condition is clearly indicated by the financial statements themselves. d. You should make sure there is adequate disclosure and appropriately modify your report because the company does not appear to have the ability to continue as a going concern. 18. An auditor is planning an audit engagement for a new client in a business with which he is unfamiliar. Which of the following would be the most useful source of information during the preliminary planning stage, when the auditor is trying to obtain a general understanding of audit problems that might be encountered? a. Client manuals of accounts and charts of accounts. b. Industry Audit Guides. c. Prior-year working papers of the predecessor auditor. d. Latest annual and interim financial statements issued by the client. 19. Which of the following best describes the element of inherent risk that underlies the application of generally accepted auditing standards? a. Cash audit work may have to be carried out in a more conclusive manner than inventory audit work. b. Intercompany transactions are usually subject to less detailed scrutiny than arm’s-length transactions with outside parties. c. Inventories may require more attention by the auditor on an engagement for a merchandising enterprise than on an engagement for a public utility. d. The scope of the audit need not be expanded if misstatements that arouse suspicion of fraud are of relatively insignificant amount. 20. Failure to detect material peso misstatements the financial statements is a risk that the auditor mitigates primarily by a. Performing substantive tests. c. Evaluation internal control. b. Performing tests of control. d. Obtaining a client representation letter. 21. Inherent risk and control risk differ from planned detection risk in that they a. Arise from the misapplication of auditing procedures. b. May be assessed in either quantitative or non-quantitative terms. c. Exist independently of the financial statement audit. d. Can be changed at the auditor’s discretion. 22. 23. For each of the following types of misstatements (Items 21-24) select the control that should have prevented the misstatements. 24. A manufacturing company received a substantial sales return in the last month of the year, but the credit memorandum for the return was not prepared until after the auditors had completed their filed work. The returned merchandise was included in the physical inventory. a. Credit memoranda are pre-numbered and all numbers are accounted for. A1 PASSERS REVIEW CENTER///53 b. Aged trial balance accounts receivable is prepared. c. A reconciliation of the trial balance of customer accounts with the general ledger control is prepared periodically. d. Receiving reports are prepared for all materials received accounts and such reports are accounted for on a regular basis. 25. The sales manager credited a salesman, Jun with sales that were actually “house account” sales. Later Jun divided his excess sales commission with the sales manager. a. Sales order are reviewed and approved by persons independent of the sales department. b. The summary sales entries are checked periodically by persons independent of sales. c. Sales orders are pre-numbered and all numbers are accounted for. d. The internal auditor compares the sales commission statements with the cash disbursements records. 26. Copies of sales invoices shows different unit prices for apparently identical items. a. All sales invoices are checked as to all details after their preparation. b. Differences reported by customers are satisfactorily investigated. c. Statistical sales data are complied and reconciled with recorded sales. d. All sales invoices are compared with the customers purchase orders. 27. A sales invoice for P52,000 was computed correctly but, by mistake, was key entered as P25,000 to the sales journal and to the accounts receivable master file. The customer remitted only P25,000 the amount on his monthly statement. a. Unauthorized remittance deductions made by customers of other matters in dispute are investigated promptly by a person independent of the accounts receivable function. b. The customer’s monthly statements are verified and mailed by a responsible person other than the bookkeeper who prepared them. c. Sales invoice serial numbers, prices, discounts, extensions, and footings are independently checked. d. Pre-listings and predetermined totals are used to control postings. 28. Auditors sometimes use comparison of ratios as audit evidence. For example, an unexplained decrease in the ratio of gross profit to sales may suggest which of the following possibilities? a.Unrecorded sales b.Unrecorded acquisitions c.Fictitious sales d.Merchandise acquisition being charged to selling and general expenses 29. An auditor is performing substantive tests of transactions for sales. One step is to trace a sample of debit from the accounts receivable master file back to supporting duplicate sales invoices. What would the auditor intended to establish by this step? a.All sales have been recorded. b.Sales invoices represent existing sales. c.All sales invoices have been properly posted to customer accounts. d.Debit entities in the accounts receivable files are properly supported by sales invoices. 30. In a random sample of 1,000 records, an auditor determines that the sample deviation rate is 2 percent. The auditor can state that the deviation rate in the population is: a. Not less than 2 percent. c. Probably about 2 percent. b. Not more than 3 percent. d. Not less than 1 percent. 31. As a result of analytical procedures, the independent auditor determines that the gross profit percentage has declined from 30 percent in the preceding year to 20 percent in the current year. The auditor should: a. Evaluate management’s performance in causing this decline. b. Require note disclosure. c. Consider the possibility of a misstatement in the financial statements. d. Express a qualified opinion due to inability of the client company to continue as a going concern. 32. Which of the following is true of generalized audit software programs? a. They can be used on any computer without modification. b. They can be used only in auditing on-line computer systems. c. They enable the auditor to perform all manual tests of controls procedures less expensively. d. They each have their own characteristics that the auditor must carefully consider before using in a given audit situation. 33. The negative form of accounts receivable confirmation request is useful except when? a. A large number of small balances are involved. b. Internal control surrounding accounts receivable is considered to be effective. c. Individual account balances are relatively large. d. The auditor has reason to believe the persons receiving the request are likely to give them consideration. 34. Which of the following is the best argument against the use of negative confirmations of accounts receivable? a. There is no way of knowing if the intended recipients received them. b. They each have their own characteristics that the auditor must carefully consider before using in a given audit situations. c. The inference drawn from receiving no reply may not be correct. d. The cost per response is excessively high. A1 PASSERS REVIEW CENTER///54 35. A factory foreman at Stable Corporation discharged an hourly worker but did not notify the payroll department. The foreman then forged the worker’s signature on time cards and work tickets and, when giving out checks, diverted the payroll checks drawn from discharge worker to his own use. The most effective procedure for preventing this activity is to: a. From time to time, rotate persons distributing the payroll. b. Require written authorization for all employees added to or removed from the payroll. c. Have someone other than persons who prepare or distribute the payroll obtain custody of unclaimed payroll checks. d. Have a paymaster who has no other payroll responsibility distribute the payroll checks. 36. A common audit procedure in the audit procedure in the audit of payroll transactions involves tracing selected items from the payroll journal to employee time cards that have been approved by 7 supervisory personnel. This procedure is designed to provide evidence in support of the audit proposition that: a. Jobs on which employees worked were changes with the appropriate labor cost. b. Only proper employees worked and their pay was properly computed. c. All employees worked the number of hours for which their pay was computed. d. Internal controls over payroll disbursements are operating effectively. 37. Buddy, the purchasing agent of Lander Hardware Wholesalers, has a relative who owns a retail hardware store. Buddy arranged for hardware to be delivered by manufacturers to the retail store on COD basis, thereby enabling his relative to buy at Lander’s wholesale prices. buddy was probably able to accomplish this because of Lander’s poor internal control over. a. Cash receipt c. Purchase requisition. b. Perpetual inventory records. d. Purchase orders. 38. Which of the following is an internal control that would prevent paid cash disbursement documents from being presented for payment as second time? a. Cash disbursement documents should be approved by at least two responsible managements officials. e. b. The date on cash disbursement documents should be within a few days of the date that the document is presented for payment. c. Unsigned checks should be prepared by individuals who are responsible for singing checks. d. The officials signing the check should compare the check with the documents and should deface the documents. 39. With respect to an internal control measure that will assure accountability for fixed asset requirements management should implement controls that include. a. Continuous analysis of miscellaneous revenue to locate any cash proceeds from sale of plant assets. b. Periodic inquiry of plant executives by internal auditors as to whether any plant assets have been retired. c. Periodic observation of plant assets by the internal auditors. d. Continuous use of serially numbered retirement work orders. 40. Which of the following comparison would be most useful to an auditor in evaluating the results of an entity’s operations? a. Prior year accounts payable to current year accounts payable. b. Prior year payroll expense to budgeted current year payroll expense. c. Current year revenue to budgeted current year revenue. d. Current year warranty expense to current year contingent liabilities. 41. In testing for unrecorded retirements of equipment, an auditor most likely would. a. Select items of equipment from the accounting records and then locate them during the plant tour. b. Compare the depreciation journal entries with similar prior-year entries in search of fully depreciated equipment. c. Inspect items of equipment observed during the plant tour and then trace them to the equipment master life. d. Scan the general journal for unusual equipment additions and excessive debits to repairs and maintenance expense. 42. Which of the following best describes the independent auditor’s approach to obtaining satisfaction concerning depreciation expense in the income statement? a. Establish the basis for depreciable assets and verify the depreciation expense b. Verify the mathematical accuracy of the amounts charged to income as a result of depreciation expense c. Determine the method .for computing depreciation expense and ascertain that it is in accordance with generally accepted accounting principles. d. Reconcile the amount of depreciation expense to those amounts credited to accumulated depreciation accounts. 43. For control purposes, the quantities of materials ordered may be omitted from the copy of the purchase order that is: a. Forwarded to the receiving department b. Returned to the requisitioner A1 PASSERS REVIEW CENTER///55 c. Forwarded to the accounting department d. Retained in the purchasing department files 44. When evaluating inventory controls with respect to segregation of duties, a CPA would be least likely to: a. Make inquiries c. Consider and procedure manuals b. Inspect documents d. Observeprocedures 45. A CPA auditing inventory may appropriately apply attributes sampling in order to estimate the: a. Peso value of inventory c. Physical quantity of inventory items b. Average price of inventory items d. Percentage of slow-moving inventory item 46. Assuming cash receipts from credit sales have been misappropriated. Which of the following is likely to conceal the misappropriated and unlikely to be detected? a. Overstating the accounts receivable control account b. Overstating the accounts receivable subsidiary ledger c. Understanding the sales of journal d. Overstating the cash receipts journal 47. The audit step most likely to reveal the existence of contingent liabilities is: a. Mortgage note confirmation b. Accounts payable confirmation c. An inquiry directed to legal counsel d. A review of vouchers paid during the month following the year-end 48. A principal purpose of a letter of representation from management is to: a. Discharge the auditor from legal liability for the audit. b. Remind management of its primary responsibility for financial statements. c. Confirm in writing managements approval of limitations on the scope of the audit. d. Serve as an introduction to company personnel & authorization to examine the records. 49. Management’s refusal to turnish a written representation on a matter that the auditor considers essential constitutes. a. Prima facie evidence that the financial statements are not presented fairly. b. A scope limitation sufficient to prelude an unmodified opinion. c. A violation of the Securities Regular Code. d. An uncertainly sufficient to prelude an unmodified opinion. 50. A major customer of an audit client suffers a fire just prior to completion of year-end field work. The audit client believes that this event could have a significant direct effect on the financial statements. The auditor should: a. Advise management to adjust the financial statement. b. Disclose the event in the auditor’s report. c. Advise the management to disclose the event in notes to the financial statements. d. With hold submission of the auditor’s report until the extent of the direct effect on the financial statements is known. 51. Which of the following fundamental ethical principles prohibits association of professional accountants with reports, returns, communications or other information that is believed to contain a materially or misleading statement? a. Professional competence due care c. Objectivity b. Integrity d. confidentiality 52. The auditor’s risk assessment procedures. a. Are designed to detect material misstatements at the assertion level for classes of transactions, account balances and disclosures. b. Are designed to test the effectiveness of the entity’s controls. c. Should not consider information obtained from the auditors previous experience with the entity. d. By themselves, do not provide sufficient appropriate audit evidence on which to base the audit opinion. 53. An entity’s internal control system contains manual elements and often contains automated elements. Manual elements in internal control may be less reliable than automated elements because: a. Manual elements include reliance on system or programs that are inaccurately processing data, processing inaccurate data or both. b. Consistency of application of manual control elements can always be assumed. c. Manual control elements facilitate the additional analysis information. d. Manual control elements can be more easily by passed, ignored, or overridden and they are also more prone to simple errors and mistake 54. When an auditor believes that an understanding with the client has not been establish, he or she should ordinarily: a. Assess control risk at the maximum level and perform a primarily substantive audit. b. Decline to accept or perform the audit. c. Perform the audit with increased professional scepticism. d. Modify the scope of the audit to reflect an increase of material misstatement due to fraud. 55. Which of the following comparison would an auditor most likely make in evaluating an entity’s and expenses? A1 PASSERS REVIEW CENTER///56 a. The budgeted current year’s sales with the prior’s sales b. The current years payroll expense with the prior years payroll expense. c. The current year’s accounts receivable with the prior year’s accounts receivable. d. The budgeted current year’s warranty expense with the current year’s contingent liabilities. 56. The following statements are ordinarily included in a management representation letter except: a. The financial statements are free of material misstatement, including omission. b. There have been no irregularities involving management or employees who significant role in internal control or that could have a material effect on the financial statement. c. Sufficient appropriate audit evidence has been made available to permit the expression of an unmodified opinion. d. The completeness and availability of minutes of stockholders and directors meetings. 57. Which of the following is the greatest advantage of a database system? a. Conversion to a database system is inexpensive and can be accomplished quickly. b. Multiple occurrences of data items are useful for consistency checking. c. Backup and recovery procedures are minimized. d. Data redundancy can be reduced. 58. The auditor’s risk assesstment procedures should always include the following except: a. Substantive test procedures and tests of controls b. Inquiries of management and others within the entity c. Observation and inspection d. Analytical procedures 59. Controls activities relate to the following except: a. An internal audit c. Performance reviews b. Segregation of duties d. Authorization 60. In connection with the planning phase of an audit engagement, which of the following statements is always correct? a. An engagement should not be accepted after the clients financial year-end. b. Final staffing decisions must be made prior to completion of the planning state. c. Observation of inventory count should be performed year-end. d. A portion of the audit of a continuing audit client can be performed at interim dates. 61. Which of the following audit procedures would allow an auditor to test whether receiving department procedures are applied properly? a. Interview receiving department personnel b. Test a sample of receiving documents c. Observed receiving procedures on a surprise basis d. Review procedures manual 62. When assessing application controls, which one of the following input controls or edit checks is most likely to be used to detect a data input error in customer account number field? a. Validity check c. Limit check b. Control total d. Hash total 63. Which of the following audit procedures is best for identifying unrecorded trade accounts payable? a. Investigating payables recorded just prior to and just subsequent to the balance sheet date to determine whether they are supported by receiving reports. b. Reviewing cash disbursements recorded subsequent to the balance sheet date to determine whether the related payables apply to the prior period. c. Reconciling vendor’s statements to the file of receiving reports to identify items received just prior to the balance sheet dates. d. Examining unusual relationships between monthly accounts payable balances and recorded cash payments. 64. An audit client’s description that its financial statements are prepared in accordance with a particular applicable financial reporting framework is appropriate only if: a. The terminology used in the financial statements, including the title of each financial statement, is appropriate. b. The financial statements adequately disclose the significant accounting policies selected and applied. c. The financial statements are in substantial compliance with the framework. d. The financial statements comply with all the requirements of that framework that are effective during the period covered by the financial statements. 65. PSA 620 (Revised and Redrafted), Using the Work of an Auditor’s Expert, deals with: a. The auditor’s responsibilities regarding the use of an individual or organization’s work in a field of expertise other than accounting or auditing, when that work is used to assist the auditor in obtaining sufficient appropriate audit evidence. b. Situation where the engagement team includes a member with expertise in a specialized area of accounting or auditing. A1 PASSERS REVIEW CENTER///57 c. How the auditor’s responsibility for the opinion on the entity’s financial statements will be divided between the auditor and the auditor’s expert. d. The auditor’s use of the work of a management’s expert. 66. PSA 620 considers the following individuals as “experts”, except: a. An individual with expertise in complex modelling for the purpose of valuing financial instruments. b. An individual with expertise in the valuation of assets acquired and liabilities assumed in business combinations. c. An individual with expertise in applying methods of accounting for deferred income tax. d. An expert in taxation law. 67. Which statement is incorrect regarding external confirmation? a. The auditor may use positive or negative external confirmation requests or a combination of both. b. External confirmation of an account receivable provides strong evidence regarding the valuation of the account as at a certain date. c. The auditor should tailor external confirmation requests to the specific audit objective. d. External confirmation is the process of obtaining and evaluating audit evidence through a direct communication form a third party in response to a request for information. 68. The form and content of working papers are affected by matters such as the: I. Nature and condition of entity’s accounting and internal control system II. Nature and complexity of the business III. Nature of the engagement IV. Forms of the auditor’s report V. Needs in particular circumstances for direction, supervision, and review of work performed by assistants. VI. Specific audit methodology and technology used in the course of the audit. a. All of the above c. I,II and III only b. All except V and VI d. All except VI e. f. g. h. i. j. 69. Observation: a. Consists of seeking information of knowledge persons, both financial and nonfinancial, throughout the entity or outside the entity. b. Consists of looking at a process or procedure being performed by others. c. Is the auditor’s independent execution of procedures of controls that were originally performed as part of the entity’s internal control. d. Is the process of obtaining a representation of information or of an existing condition directly from a third party. 70. An entity’s management is responsible for the preparation and fair presentation of the financial statements. Its responsibility includes the following except: a. Making accounting estimates that are reasonable in the circumstances. b. Selecting and applying appropriately accounting policies. c. Assessing the risks of material misstatement of the financial statements. d. Designing, implementing and maintaining internal control relevant to the preparation and presentation of financial statements. 71. Which of the following statements is correct regarding internal control system? a. A strong control environment, ensures the effectiveness of the internal control system. b. The internal control system is confined to those matter which relate directly to the functions to the accounting system. c. In the audit of financial statements, the auditor is only concerned with those policies and procedures within the accounting and internal control systems that are relevant to the financial statements. d. Internal control systems refers to all policies and procedures adopted by the auditor to assist in achieving managements objectives. 72. Which of the following conditions and events may most likely indicate the existence of risks of material misstatements? a. Significant amount of routine or systematic transaction b. Accounting measurements that involve simple processes c. Having personnel with appropriate accounting and financial reporting skill d. Constraints on the availability of capital and credit 73. The assessment of the risks of material misstatement at the financial statement level is affected by the auditor’s understanding of the control environment. Weaknesses in the control environment ordinarily will lead the auditor to: A1 PASSERS REVIEW CENTER///58 a. Conduct some audit procedures at an interim dates rather than at period of time b. Decrease the number of locations to be included in the audit scope c. Modify the nature of audit procedure to obtain more persuasive audit evidence d. Have more confidence in internal control, reliability of audit evidence within the entity 74. To reduce the risks associated with accepting e-mail responses to request for confirmation of accounts receivable, an auditor most likely would: a. Examine subsequent cash receipts for the accounts in question b. Mail second request to the e-mail respondents c. Request the senders to mail the original forms to the auditor d. Consider the e-mail responses to the confirmations to be exceptions 75. What type of assurance engagement is involved when the practitioner expresses negative form of conclusion? a. Limited assurance engagement c. Negative assurance engagement b. Reasonable assurance engagement d. Assertion-based assurance engagement 76. If adequate disclosure is not made by the entity regarding significant doubt about its ability to continue as a going concern, the auditor should include in his report specific reference to the significant doubts as to the ability to the entity to continue as a going concern and should be express: a. Either an “except for” qualified opinion or an adverse opinion b. Unmodified opinion with Emphasis of Matter paragraph c. A disclaimer of opinion d. A “subject to” qualified or adverse opinion 77. Who has the power to suspend or remove any member of the Board of Accountancy? a. The Chairman of the PRC c. The President of the Philippines b. The Chairman of the FRSC d. The Chairman of the AASC 78. If the auditor is unable to agree to a charge of the engagement and is not permitted to continue the original engagement, the auditor should: a. Express a qualified opinion d. Insist on continuing engagement, the auditor b. Express an adverse opinion should c. Withdraw from the engagement 79. 80. Which of the following statements concerning detection risk incorrect? a. Detection risk arises partly from uncertainties that exist when the auditor does not examine 100 percent of the population. b. Detection risk is a function of the effectiveness of an auditing procedure and its application. c. Detection risk arises partly because of other uncertainties that exist even if the auditor were to examine 100 percent of the population. d. Detection risk exists independently of the audit of the financial statements. 81. The auditors understanding of the entity and its environment consist of an understanding of the following aspects: I. Industry, regulatory and other external factors, including the applicable financial reporting framework II. Nature of the entity, including the entity’s selection and application of accounting policies. III. Objectives and strategies and related business risks that may result in a material misstatement of the financial statements. IV. Measurement and review of the entity’s financial performance V. Internal control a. All of the above b. I,II,III, & IV c. I,II,&III d. I,II,III,& V 82. The following matters are generally included in an auditor’s engagement letter, except: a. The fact that because of the test nature and other inherent limitations of an audit, together with the inherent limitations of internal control, there is an unavoidable risks that even some material misstatements may remain undiscovered. b. The scope of the audit. c. Management’s responsibility for the financial statements. d. The factors to be considered in setting preliminary judgements about materiality. 83. Which of the following would not be necessarily be a related partly transaction? a. A purchase from another corporation that is controlled by corporation’s chief shareholder. b. A sale to another corporation with a similar name. c. Sale of land to the corporation by the spouse of a director. d. A loan from the corporation to a major shareholder. 84. What type of assurance engagement is involved when the practitioner expresses a negative form of conclusion? a. Limited assurance engagement b. Reasonable assurance engagement c. Negative assurance engagement A1 PASSERS REVIEW CENTER///59 d. Assertion-based assurance engagement 85. 86. 87. 88. 89. 90. 91. 92. 93. 94. 95. 96. 97. 98. 99. 100. 101. 102. 103. 104. 105. 106. 107. 108. 109. 110. 111. 112. 113. 114. A1 PASSERS TRAINING, RESEARCH, REVIEW & DEVELOPMENT COMPANY 115. 2nd Floor Sommerset Bldg., Lopez Jaena St. Jaro, Iloilo City 116. Tel. No.: (033) 320-2728; 09106547262 117. Email Address: [email protected] 118. BOARD OF CERTIFIED PUBLIC ACCOUNTANT 120. CERTIFIED PUBLIC ACCOUNTANT Licensure Examination SET B 119. 121. 122. 123.BUSINESS LAW AND TAXATION 124. INSTRUCTIONS: 1. Detach one (1) answer sheet form the bottom of your Examinee ID/Answer Sheet Set. 2. Write the subject title “BUSINESS LAW AND TAXATION” on the box provided. 3. Shade Set Box “A” on your answer sheet if your test booklet is Set A; Set Box “B” if your test booklet is Set B. 125. 126. MULTIPLE CHOICE: 127. 1. This income of a resident is subject to the basic tax (scheduler) a. Compensation for personal injuries and sickness b. Interest income from a foreign currency deposit in PNB c. Winnings from wagering transactions d. Royalty income with suits in the Philippines 2. This income of a resident alien is subject to the basic tax(scheduler) a. Fringe benefits given to managerial employees b. Interest income from Philippine currency deposit in PNB c. Winnings from PCSO and Philippine Lotto d. Prizes from SM (P10,000), Rustan’s (P100,000) and Robinsons (P10,000) or a total of P30,000 3. The improperly accumulated earnings tax imposed on corporations is a. Based on the net income per books after tax b. Based on the net income per books before tax c. Calculated to encourage corporations to pay dividends A1 PASSERS REVIEW CENTER///60 d. 10% of excess retained earnings over the paid in capital 4. This income is subject to final tax a. Fringe benefits received by rank and file employees b. Cash dividend received by a non-resident alien from a domestic corporation c. Marriage fees, baptismal offerings received by a clergyman, evangelist or religious worker for services rendered d. Share of a partner form the undistributed net income of a general professional partnership 5. No additional exemptions can be claimed if the dependent is a. Recognized natural con who celebrated his 21st birthday during the taxable year b. Legitimate daughter who got married on January 1 of the taxable year, one day after she turned 21 c. Illegitimate son, minor who died January 1 of the taxable income d. Legally adopted daughter, 22 years old but incapable of self support due to mental defect 6. Amounts receivable by the estate of the decreased, his executor or administrator as an insurance under a policy taken by the decedent upon his own life is a. Excluded from gross income whether the beneficiary is revocable or irrevocable b. Part of gross income whether the beneficiary is revocable or irrevocable c. Excluded from gross income if the beneficiary is irrevocable d. Part of gross income if the beneficiary is revocable 7. Acting on the information given by A, the BIR seized and confiscated smuggled goods with a FMV of P15M. A’s reward will be subjected to a final tax of a. P1.5M c. P150,000 b. P1M d. P100,000 8. The deduction allowed for the payment on health and hospitalization insurance of a qualified resident citizen during the taxable year amounting to P300,000 a month for the months from April to December is a. P2,700 c. P1,800 b. P2,400 d. P900 9. In 2010, NBC Corporation paid for the annual rental of a residential house used by its general manager amounting to P136,000. The deductible amount for the rental and tax paid is a. P100,000 c. P168,000 b. P132,000 d. P200,000 10. 11. A substantial under declaration of taxable sales, receipts or income, or of substantial overstatement of deduction shall constitute a presumption of false or fraudulent return. This presumption is a. Conclusive c. Prima-facie b. Absolute d. irrevocable 12. 13. a created an irrevocable in favor of his 2 minor children. The trust provides that 50% of the net income should be distributed yearly to the children, share alike, the balance to accumulate for eventual. The taxable income of the trust is a. P480,000 c. P230,000 b. P500,000 d. P0 14. The following expenses maybe deducted from gross income, except a. Employer’s contribution to the Christmas party of his employees b. Contribution to the construction of a chapel of a university that declares dividends to its stockholders c. Premiums paid by the employer for the life insurance of his employees where the beneficiary is the estate of the employee d. Interest in taxes 15. Optional standard deduction a. Is equal to 40% of the gross income from business or practice b. Cannot be used as deduction of a chapel of a university that declares dividends to its stockholders c. May be availed by all individual taxpayers d. May be availed of by the taxpayer whether or not he signifies his desire to elect optional standard 16. M married to K, left Philippines in the middle of the year on July 1, 2009 to go to USA and work there as a contract worker for two years, the following income provided as of December 31, 2009. 17. 18. Philippines USA a. January 1 to June 30 P300,000 $10,000 b. July 1 to December 31 P100,000 $20,000 19. 20. His net taxable income is (1$=P56) a. P910,000 b. P350,000 c. P810,000 d. P250,000 21. A1 PASSERS REVIEW CENTER///61 22. ABC has an authorized capital stock of P5,000,000 divided into 50,000 shares with a par value of P100 per share. Of the 25,000 shares subscribed, 13,000 will belong to A, because of additional subscription of P9,500 shares, and to pay his additional subscription accounting to P950,000. A transferred to the corporation a parcel of land that he owns. The investigation conducted by the BIR revealed that A acquired the property for only P500,000. A’s taxable gain is a. P450,000 c. P500,000 b. P950,000 d. P0 23. 24. A, a resident citizen, single had the following during the taxable year: 1. Contributions during the year: a. To the Dept. Of Education- for priority activity in education Php 100,000 b. To religious and charitable organizations 20,000 c. To victims of typhoon and earthquake 40,000 d. To the government for public purpose 20,000 25. 2. Provision for bad debts 30,000 26. 3. Accumulated depreciation on machines: a. Beginning 80,000 b. Ending 100,000 27. 4. Allowance for bad depts: a. Beginning 75,000 b. Ending 85,000 28. 5. During the year, A sold a machine acquired three (3) years earlier. The machine had an accumulated depreciation of P30,000 29. 30. The net income of A before considering nos. 1 to % inclusive, amounts to Php300,000 31. The taxable income of A is a. P30,000 b. P40,000 c. P50,000 d. P57,000 e. 32. Recovery of bad debt written off by a taxpayer: f. No.1: P20,000 from accounts written off in a year which had a net income of P200,000 before write-off (write-off for the year was P20,000) g. No.2: P5,000 from accounts written off in a year which had a net loss before write off P36,000 (write-off for the year was P5,000) h. No.3: P10,000 from accounts written off in the year which had a net income of P8,000 before a write-off (write-off for the year was 12,000) i. j. The net income from the bad debt recovery is: a. P35,000 c. P26,000 b. P20,000 d. P30,000 33. Taxation is an inherent power because: a. It is lifeblood of the government b. It is about the protection and benefit theory c. It co-exist with the existence of the state d. It is exercised for the general welfare of the people 34. A tax is invalid in all of the following, EXCEPT: a. Theory of taxation and its purpose are disregarded. b. Basis of taxation is not recognized. c. Inherent and constitutional limitations are not observed. d. It results to double taxation. 35. The government may not do all EXCEPT TO: a. Tax itself b. Delegate its power to tax to private agencies c. Impose tax arbitrarily d. Disregard uniformly in taxation 36. The theory which most justifies the necessity of taxation: a. Protection and benefits theory c. Lifeblood theory b. Revenue purpose d. Ability to pay theory 37. 1st statement: Tax imposed outside the situs of taxation is void. a. True, false c. True, true b. False, true d. False, false A1 PASSERS REVIEW CENTER///62 38. The President of the Philippines and the Prime Minister of Malaysia entered into an executive agreement in respect of a loan facility to the Philippines form Malaysia whereby it was stipulated that interest on loans granted by private Malaysian financial institutions in the Philippines shall not be subject to Philippine income taxes. What basic, characteristic of taxation has been violated by this agreement? a. Inherent limitation c. Legislative in character b. Theoretical Justice d. Administrative feasibility 39. Which of the following statements constitutes tax avoidance? a. Deliberate failure of a taxpayer to pay the taxes due to the government. b. Cannotes fraud through the use of pretenses and forbidden devices to lessen or defeat taxes. c. Punishable by law d. Maybe contrary to the intent of the legislature but nevertheless does not violate by law. 40. In which of the following instances may a taxpayer include his income as part of gross income subject to basic tax at his point? a. When it involves a gain from sale of shares of stock held as a capital assets, and is not traded into the stock exchange and the gain does not exceed P100,000. b. When a real property held as capital asset is sold to the government or any of its political subdivision. c. When it involves a gain from sale of shares of stock held as capital assets and is traded in the stock exchange. d. When the sale involves a personal property held as capital asset. 41. Which of the following statement is NOT CORRECT? a. Income from a general professional partnership is constructively received by a partner when his share in the net income is credited to his capital account. b. Income from a taxable partnership is constructively received by a partner in the same taxable year that the partnership determined its net income after tax. c. Any distribution of dividend made to individual shareholders of a corporation shall be considered dividend and income of the taxpayer subject to final tax in the year received. d. Any distribution of dividend made to corporate shareholders of a corporation shall be deemed to have been made from the most recently accumulated earnings or surplus exempt from income tax. 42. Gross income from within the Philippines of a resident alien subject to final withholding tax at source. a. Dividend income from a resident foreign corporation. b. Share from the net income after tax of a domestic general partnership in trade. c. Interest income from investments in bonds. d. Gain from sale of capital assets. 43. 1st statement- the power of taxation must first be expressly granted, either by law or by the Constitution, before the state may valid exercise it. 44. 2nd statement- The Philippine government may subject the land where embassies of foreign governments are located to real property taxes. a. Both statements are false c. 1st statement is true while 2nd is false b. 1st statement is false while 2nd is true d. Both statements are true 45. A, a rank and file employee, receiving the statutory minimum wage and supporting an illegitimate child had the following during the taxable year. a. Salary, gross P120,000 b. Mid-year 10,000 th c. 13 month pay 10,000 d. Christmas bonus 10,000 e. Overtime pay 10,000 f. Fringe benefits (paid in cash) 1. Rice subsidy 18,000 2. Uniform and clothing allowance 4,000 3. Actual medical benefits 10,000 4. Christmas cash gift 5,000 5. Achievement award (10 years service) 10,000 6. Monetized unused leave credits (10 days) 5,000 g. Deduction made by the employer: 1. SSS premiums contributions 2,000 2. Philhealth premiums contributions 1,800 3. Pag-ibig premiums contributions 1,200 4. SSS loans 3,600 5. Pag-ibig loans 2,400 6. premiums on health and hospitalization insurance 2,000 46. 47. The net taxable income of A is a. P0 c. P48,000 b. P38,000 d. P58,000 48. Why tax credit is allowed to be availed of by certain taxpayer? A1 PASSERS REVIEW CENTER///63 a. To provide incentive thus inducing these taxpayers to report income derived from abroad b. To provide relief from the burden of taxation c. In compliance with commitments to international agreement d. For citizen doing business outside the Philippines to become competitive 49. One of the following compensation income of an individual taxpayer, is NOT an exclusion from gross income a. Monetized vacation leave not exceeding 10 days a year b. Separation pay of an employee who resigned from his employment c. Retirements benefits of an employee who has worked for at least 10 years, who at the time of retirements is not less than 50 years of age, and who avails of the retirement for the first time d. De minimis benefits 50. 32-34. ABC Corporation engaged in freight and trucking business received the following in 2010 51. Share in net income of the joint venture with XYZ Corp. P900,000 52. Dividend received from KLM Corp., a Domestic Corp. 100,000 53. Dividend received from NOP Corp., a Non-resident Corp. 10,000 54. Own net income 1,200,000 55. 56. Thenet taxable income of ABC Corp. is a. P1,210,000 c. P2,210,000 b. P2,110,000 d. P1,200,000 57. The total amount of income of ABC Corporation subject to final tax a. P1, 020,000 c. P1,018,000 b. P1,000,000 d. P0 58. The amount of final tax paid (due) on ABC’s income is a. P102,000 c. P2,000 b. P102,000 d. P0 59. The following are true, except, a. Fringe benefit tax shall be treated as a final income tax on the employee benefit withheld and paid by the employer. b. The gross up monetary value of the fringe benefit is the actual amount received by the employee. c. The grossed-up monetary value of the fringe benefit shall be determined by dividing the monetary value of the fringe benefit by the gross monetary value factor. d. The person liable for fringe benefit tax is the employer, whether he is an individual, professional partnership or a corporation regardless of whether the corporation is taxable or not. 60. The law requires that the object of sale must not only be licit but that the vendor must have a right to transfer the ownership thereof: a. At the time of perfection of the contract of c. At the time of payment or delivery. sale. d. At any time after perfection. b. At the time the thing is delivered. 61. Annulment and not reformation is the remedy if the ground is a. Accident c. Intimidation b. Fraud d. mistake 62. annulment or reformation is the remedy if the ground is (are) a. fraud and mistake c. fraud and inequitable conduct b. violence and undue influence d. intimidation and mistake 63. In 2010, A agreed to sell his land and B agreed to pay if C will pass the 2011 CPA Board Exam, C passes. As a result: 64. Answer I- The fruits of the land for the year period will remain A 65. Answer II- B will keep the legal interest on his money a. Both answers are true c. Only answer II is true b. Only answer I is true d. Both answers are false 66. Reformation and not annulment is the remedy if the ground is a. Inequitable conduct c. Fraud b. Undue influence d. mistake 67. the defect contacts arranged according to their decreasing validity a. a. Void, voidable, rescissible, unenforceable c. c. Rescissible, voidable, unenforceable, void b. b. Unenforceable, rescissible, voidable, void d. d. Voidable, unenforceable, rescissible, void 68. A, minor solid the propertry of B his guardian without B;s authority. The property has a FLAV of PIM but the selling price was P600,000. The contract is a. Rescissible c. Unenforceable b. Voidable d. Void 69. Using the preceding number, the remedy is a. A can ask for annulment c. The buyer can ask for annulment b. The guardian can ask for annulment d. The guardian can just ignore the contract 70. The following, except one are the requisites of cause or consideration in a contract. What is ijt? A1 PASSERS REVIEW CENTER///64 a. It must be mentioned in the contract c. It must exist b. It must not be contrary to law d. It should not be false 71. A owes B P20,000. Thereafter B assigns the credit to C. on due date, can C. collect from B if A cannot pay? 72. Answer No. 1 – No, if the obligation arose from a gambling debt 73. Answer No. 2 – Yes, if at time of assignment, the debt has prescribed a. True, true c. False, true b. True, false d. False, false 74. If an obligation is modified by changing its object and also changing the principal condition a. There is mixed novation c. There is real novation b. The original obligation remains d. There is personal novation 75. P appointed A as his agent orally to sell his parcel of land for P200,000. Ten days after, A sold the same property for P300,000 to B through a public instrument executed between A and B. what is the effect and status of the sale between A and B? a. The sale is valid because A was authorized and it was executed in a public instrument. b. The sale is unenforceable, because the agent went beyond the scope of his authority for selling the land or price higher than the agreed price. c. The sale is void and cannot be ratified by P. d. The sale can be ratified although the appointment of A was done orally, because the sale between A and B was in a public instrument. 76. A sold his car to B payable in ten equal monthly instalments and with a mortgage constituted on the same property. For B’s failure to pay a month’s instalment, which statement is correct? a. A may foreclose the mortgage on B’s car but he no longer has the right to recover the balance should it be sold for an amount lower that what be claims from B b. A may seek the cancellation of the sale made to B c. A may seek fulfilment of the obligation of B to pay the entire purchase price d. A may seek the cancellation of the sale and later of foreclose the mortgage should he find it impossible to collect from B 77. A was obliged to pay B PIM on December 31, 2011, A paid B on December 31, 2010 believing that the obligation was already due and demandable. How much many A recover from B on March 31, 2012? a. Nothing c. PIM plus legal interest for one year b. Legal interest for one year and three months d. Legal interest for one year 78. One of the following is considered fraudulent a. Failure to disclose facts when there is duty to reveal them b. The usual exaggeration in trade, when the other party had the opportunity to know the facts c. Misinterpretation made not in good faith d. “Caveat Emptor” or let the buyer beware 79. A owes X P500,000 payable on July 31, 2011. S who is not a party to the contract and without the consent of A paid X the P50,000 on April 1, 2011 when the prevailing rate of interest was 12% per annum. As a result, a. S can ask imbursement from A in the amount of P50,000 plus 12% interest from April 1 to July 30, 2011 b. S can ask reimbursement from A in the amount of P50,000 c. S cannot ask reimbursement from A because the payment by S is without the consent and against the will of A d. S can ask refund from X because the payment of S was without the consent of A 80. When one of the parties to a contract is complied to give his consent by a reasonable and well-grounded fear of an imminent and grave evil upon his person or property, or upon the person or property of his spouse, descendants or ascendants there is a. Violence c. Undue influence b. Intimidation d. Mistake 81. The right of a co-owner to redeem the share of his co-owner that has been sold to a third person is an instance of: a. Conventional redemption c. Legal redemption b. Venta co pacto de retro d. Pactum commissiorium 82. Which of the following statements is legally correct? a. Specific performance of the obligation is proper only where the obligation of the debtor is to give b. Under certain conditions, the performance of an obligation to do may be required by the creditor c. An obligation to do is otherwise referred to as a real obligation if it really exist d. Even is an obligation not to do, the debtor may incur in delay 83. Which of the following is not a ground for annulment of contract? a. Incapacity to give consent c. Lesion b. Vitiated consent d. Minority A1 PASSERS REVIEW CENTER///65 84. A stimulation whereby the creditor automatically becomes the owner of the thing pledged or mortgage upon the debtor’s default in the payment of his obligation is called a. Conventional redemption c. Pactum commissorium b. Acceleration clause d. Legal redemption 85. Which of the following statements is correct? a. The debtor’s consent is necessary for the validity of the assignment of the credit made by the creditor to another person b. In case of doubt, a contract purporting to be a sale with a right to repurchase shall be construed as a sale on instalment c. The sale of animal suffering from contagious disease is void d. The sale of a future thing or hope is valid even if the thing or hope not come into existence 86. When the sale of piece of land or any interest therein through agent, the authority of the agents must be a. In a public instrument c. Either in a public or private instrument b. In a private instrument d. Duty notarized 87. An agency cannot be revoked in the following cases, except when a. A bilateral contract depends upon it b. The agency is for a fixed term c. It is the means of fulfilling an obligation already contracted d. The authority is granted to a managing partner appointed through the contract of partnership 88. An agency couched in general terms comprises only a. Acts of strict dominion c. Acts of appropriation b. Acts of administration d. Acts of alienation 89. The vendor-a-recto shall be entitle to redeem the object of sale within a. Four (4) years if no period is agreed upon b. Thirty (30) days from notice of intention to repurchase c. Ten (10) years if there is a period stipulated d. The period agreed upon 90. Which one of the following is a common requisite of the remedies of possessory lien and stoppage in transit of an unpaid seller? a. Unreasonable delay in payment of the price b. Goods are in transit c. Object must be in vendor’s possession d. Insolvency of the buyer 91. A bought a residential house and lot from B Realty for P2M giving a down payment of P200,000n and promising to pay the balance of P1.8M in 15 years in monthly instalments of P10,000. After paying 72 instalments A defaulted in the payment of subsequent instalments. Despite the grace period given, he was not able to make any further payments. Accordingly, B Realty cancelled the sale. How much cash surrender value is A entitled to receive? a. P552,000 c. P462,000 b. P396,000 d. P506,000 92. A stole the goods of Y and deposits the goods to W a warehouseman. W issued an order negotiable warehouse receipt for the goods. Thereafter, A indorsed the note to B and B to C an innocent purchaser for value. Which of the following statements is correct? a. C cannot require W to deliver the goods to him b. W, A and B are liable to C c.C should first demand delivery from W and if W fails, C must give notice of dishonor to A and B to make them liable d. Only A is liable to C 93. If the thing sold had any hidden fault or defect at the time of the sale and should thereafter be lost by a fortuitous event but the seller is not aware of the defect , how much can the vendee recover form the vendor, if the selling price on April 19, 2011 is P100,000 and the value when lost or June 29, 2011 is P60,000? a. P100,000 c. P40,000 b. P60,000 d. P0 94. Using the preceding number, except that the loss is through the fault of the vendee, how much can the vendee recover from the vendor? a. P100,000 c.P40,000 b. P60,000 d. P0 95. X, Y and Z are joint creditors of A, B and D joint debtors in the amount of P60,000 . How much can X and Y collect from A, B and C? a. X and Y can collect P30,000 from A, B and C b. X and Y can collect P40,000 from A, B and C c.X and Y can collect P45,000 from A, B and C A1 PASSERS REVIEW CENTER///66 d. X and Y can collect P30,000 from anyone of A, B and C 96. A and B are co-owners of a one hectare rural land. A sold his ½ share to Y. C an adjoining land owner is interested in buying the share which A sold to Y. Which of the following is correct? a. B can redeem what A sold to Y if Y already owns a rural land b. B can redeem what A sold to Y even if Y does not own any rural land c. As adjoining land-owner C has a superior right to redeem what A sold to Y d. C can redeem what A sold to Y whether or not Y already owns rural land 97. A obtained an interest free loan of P50,000 from E evidenced by a promissory note payable in six months after date. At maturity, A called B by phone to ask for an extension of one month and offer to pay 20% interest on the loan. Enticed by the 20% interest, B agreed to the extension of the maturity. As a result a. The interest is a demandable interest by virtue of the agreement b. The interest is unenforceable c.The loan is valid but the interest is void d. The loan and interest are both demandable 98. Venus, Bea and Madonna are declared winner in a university-wide beauty7 pageant. Before their coronation, Sadam divested them of their precious belongings by requiring them to sign a deed of sale at the point of a gun. After the coronation, the three campus beauties went after Sadam for the payment of the price. The contract is a. Defective due to vitiated consent b. Defective due to intimidation c. Defective due to duress d. Valid due to ratification 99. 100. 101. 102. 103. 104. 105. 106. 107. 108. 109. 110.ANSWER KEY 111.BUSINES LAW AND TAXATION 112. 1. C 2. D 3. C 4. B 5. B 6. A 7. D 8. C 9. C 10. C 11. C 12. B 13. B 14. A 15. D 16. D 17. A 18. C 19. D 20. A 21. C 22. C 23. C 24. D 25. B 26. D 27. B 28. A 29. D 30. B 31. B 32. A 33. D A1 PASSERS REVIEW CENTER///67 34. A 35. B 36. B 37. C 38. A 39. A 40. A 41. C 42. C 43. D 44. A 45. C 46. C 47. C 48. C 49. D 50. A 51. B 52. B 53. C 54. B 55. C 56. C 57. C 58. C 59. B 60. B 61. A 62. D 63. D 64. A 65. C 66. C 67. A 68. B 69. C 70. D 113. 114. 115. 116. 117. 118. 119. A1 PASSERS TRAINING, RESEARCH, REVIEW & DEVELOPMENT COMPANY 120. 2nd Floor Sommerset Bldg., Lopez Jaena St. Jaro, Iloilo City 121. Tel. No.: (033) 320-2728; 09106547262 122. Email Address: [email protected] 123. 124. BOARD OF CERTIFIED PUBLIC ACCOUNTANT 125. 126. CERTIFIED PUBLIC ACCOUNTANT Licensure Examination 127. 128. MANAGEMENT ADVISORY SERVICES 129. 130. 1. 2. 3. INSTRUCTIONS: Detach one (1) answer sheet from the bottom of your Examinee ID/Answer Sheet Set. Write the subject title “MANAGEMENT ADVISORY SERVICES” on the box provided. Shade Set Box “A”on your answer sheet if your test booklet is Set A; Set Box “B” if your test booklet is set B. 131. 132. MULTIPLE CHOICES: 133. 134. THE FOLLOWING INFORMATION APPLIES TO QUESTIONS 1 TO 5: 135. A company produces a product with the following standard costs: 136. Materials – 2 plieces @ P5 per piece P10 137. Labor - 4 hours@ P8 per hour 32 138. Variable overhead – 4 hours @ P6 per hour 24 139. Fixed overhead* - 4 hour @ P4 per hour 16 140. Total standard manufacturing cost per unit P82 141. *based on a capacity level of 5,000 units A1 PASSERS REVIEW CENTER///68 1. If a flexible budget for 4,500 units, 5,000 units, and 5,500 units is prepared for a certain month the budgeted costs are 142. 4,500 units 5,000 units 5,500 units 143. a.P369,000 P410,000 P451,000 144. b. 297,000 330,000 363,000 145. c. 377,000 410,000 443,000 146. d.0 410,000 0 2. Assume that X is the number of units to be produced and TBC is the total budgeted cost, the flexible budget formula that the company may use to compute total budgeted cost for any value of X within the relevant range is a. TBC = 82x. c. TBC = 80,00/ b. TBC = 86x. d. TBC = 66x + 80,000. 3. Assume that during the month the company actually produced 4,800 units and incurred actual total manufacturing costs of P400,000, how much is the flexible budget for the actual production? a. P400,000 c. P393,000 b. P396,000 d. P316,000 4. How much is the total standard cost that should have been incurred for the actual production of 4,800 units? a. P396,000 c. P393,000 b. P400,000 d. P316,000 5. How much is the total standard cost variance? a. P3,200 unfavorable c. P10,000 favorable b. P6,400 unfavorable d. P83,200 unfavorable 6. Ethical challenges for management accountants include: a. Whether to accept gift from supplier, knowing it is an effort to indirectly influence decision b. Whether to report unfavourable department information that may result in unfavourable consequences for a friend c. Whether to file a tax return this year d. Both a and b are correct 7. Which of the following does NOTY affect the direct/indirect classification of cost? a. The level of budgeted profit for the next year b. The materiality of the cost in question c. Available technology to gather information about the cost d. The design of the operation 8. 9. Rover Company sells only two product, Product A and Product B. 10. 11. Product A Product B Total 12. Selling price P40 P50 13. Variable cost per unit P24 P40 14. Total fixed costs P840,000 15. 16. Rover sells two units of Product A for each unit it sells of Product B. rover faces a tax rate of 30%. Rover desires a net after-tax income of P73,500. The breakeven point in units would be: a. 21,750 units of Product A and 43,500 units of Product B b. 22,500 units of Product A and 45,000 units of Product B c. 43,500 units of Product A and 21,750 units of Product B d. 45,000 units of Product A and 22,500 units of Product B 17. Cost behavior refers to: a. How costs react to a change in the level of activity b. Whether a cost is incurred in a manufacturing, merchandising, or service company c. Classifying costs as either inventoriable or period costs d. Whether a particular expense has been ethically incurred 18. Management accounting is considered successful when it: a. Helps creditors evaluate the company’s performance b. Helps managers improve their decision c. Is accurate d. Is relevant and reported annually 19. When 10,000 units are produced fixed cost are P14 per unit. Therefore, when 20,000 units are produced fixed costs will: a. Increase to P28 per unit c. Decrease to P7 per unit b. Remain at P14 per unit d. Total P280,000 20. BLOF Manufacturing produces a unique valve and has the capacity to produce 50,000 valves annually. Currently BLOF produces 40,000 valves and is thinking about increasing production to 45,000 valves next A1 PASSERS REVIEW CENTER///69 year. What is the most likely behavior of total manufacturing costs and unit manufacturing costs given this change? a. Total manufacturing costs will increase and unit manufacturing cost will stay the same. b. Total manufacturing cost will increase and unit manufacturing costs will decrease. c. Total manufacturing cost will stay the same and unit manufacturing costs will stay the same. d. Total manufacturing cost will stay the same and unit manufacturing costs will decrease. 21. Management accounting : a. Focuses on estimating future revenues, costs and other measures to forecast activities and their results b. Provides information about the company as a whole c. Reports information that has occurred in the past that is verifiable and reliable d. Provides information that is generally available only on a quarterly or annual basis 22. In the cost classification system used by manufacturing firms, total manufacturing costs would include all of the following EXCEPT: a. Direct materials costs and conversion costs b. Direct materials cost, direct manufacturing labor cost and manufacturing overhead cost c. Indirect materials costs, indirect manufacturing labor costs and manufacturing overhead costs d. Prime costs and manufacturing overhead costs 23. Which of the following items is NOT an assumption of CVP analysis? a. Total costs can be divided into a fixed component that is variable with respect to the level of output. b. When graphed total costs curve upward. c. The unit-selling price is known and constant d. All revenues and costs can be added and compared without taking into account the time value of money. 24. Matulunging Kamay is a nonprofit organization that supplies electric fans during the summer for individuals in need/ fixed costs are P200,000. The fans cost P20.00 each. The organization has a budgeted appropriation of P480,000. How many people can receive a fan during the summer? a. 12,000 people c. 24,000 people b. 14,000 people d. 34,000 people 25. Which of the following cost(s) are inventoried when using variable costing? a. Direct manufacturing costs b. Variable marketing-costs c. Fixed manufacturing costs d. Both a and b are correct 26. THE FOLLOWING INFORMATION APPLIES TO QUESTIONS 18 THROUGH 21: 27. 28. Metro University is planning to hold a fundraising banquet at one of the local country clubs. It has two options for the 29. 30. Banquet: 31. OPTION 1: Quezon City Country Club 32. a. fixed rental cost of P1,000 33. b. P12 per person for food 34. 35. OPTION 2: Makati country Club 36. a. fixed rental cost of P3,000 37. b. a caterer who charges P8,000 person for good 38. 39. Metro University has budgeted P1,800 for administrative and marketing expenses. It plans to hire a band which will cost another P800. Tickets are expected to be P30 per person, local business supporters will donate any other items required for the event. 40. 41. Which option provides the least amount of risk? a. Option one b. Option two c. Both options provide the same breakeven amount of risk d. Neither option has risks 42. Which option has the lowest breakeven point? a. Option one b. Option two c. Both options have the same breakeven point d. The lowest breakeven point cannot be determined 43. Which option provides the greatest degree of operating leverage if 600 people attend? A1 PASSERS REVIEW CENTER///70 a. Option one b. Option two c. Both options provide equal degrees of operating leverage d. Operating leverage is indeterminable 44. How many persons should attend to make Metro University indifferent between options One and Two? a. 600 c. 500 b. 200 d. 255 45. Spikey Company currently sells banana cue. During a typical month, the stand reports a profit of P9,000 with sales of P59,000 fixed costs of P21,000 and variable costs of P0.64 per banana cue. 46. Next year, the company plans to start selling kamote cue for P3 per unit. Kamote cue will have a variable cost of P0.72 and new equipment and personnel to produce kamote cue will increase monthly fixed costs by P8,808. Initial sales of kamote cue should total 5,000 uits. Most of the kamote cue sales are anticipated to co me from current banana cue purchasers, therefore, monthly sales of banana cue are expected to decline to P20,000. 47. After the first year of kamote cue sales the company president believes that banan cue sales will increase to P33,750 a month and kamote cue sales will increase to 7,500 units a month. 48. The monthly breakeven sales during the first year of kamote cue sales, assuming a constant sales mix of 1 hotdog and 2 units of kamote cue, are: a. 5,400 banana cue and 10,800 units of kamote cue b. 16,200 banana cue and 21,600 units of kamote cue c. 31,250 banana cue and 5,000units of kamote cue d. 21,875 banana cue and 3,863 units of kamote cue 49. Activity-based costing (ABC) can eliminate cost distortion because ABC: a. Develops cost drivers that have a cause-and-effect relationship with the activities performed b. Establishes multiple cost pools c. Eliminates product variation d. Recognizes interactions between different departments in assigning support costs 50. When evaluating a make-or-buy decision, which of the following does NOT need to be considered? a. Alternative uses of the production capacity b. The original cost of the production equipment c. The quality of the supplier’s product d. The reliability of the supplier’s delivery schedule 51. 52. 53. THE FOLLOWING INFORMATION APPLIES TO QUESTIONS 25 AND 26: 54. Shirt Company produces two product lines: T-shirt s and Sweatshirts. Product profitability is analyzed as follows: 55. T-SHIRTS SWEATSHIRT 56. Production and sales volume 60,000 units 35,000 units 57. Selling price P16.00 P29.00 58. Direct material P 2.00 P5.00 59. Direct labor P4.50 P7.20 60. Manufacturing overhead P2.00 P3.00 61. Gross profit P7.50 P13.80 62. Selling and administrative P4.00 P7.00 63. Operating profit P3.50 P6.80 64. 65. Shirt Company’s manager have decide to revise their current assignment of overhead costs to reflect the following ABC cost information: 66. 67. Activity Activity cost Activity-cost-driver 68. Supervision P100,920 Direct labor hours (DLH) 69. Inspection P124,000 Inspections 70. 71. Activities demanded 72. T-SHIRTS SWEATSHIRTS 73. 0.75 DLH/unit 1.2 DLH/unit 74. 45,000 DLHs 42,000 DLHs 75. 60,000 inspections 17,500 inspections 76. 77. Under the revised ABC system, total overhead costs allocated to Sweatshirts will be: a. P 48,720 c. P224,920 b. P76,720 d. None of these answers are correct A1 PASSERS REVIEW CENTER///71 78. Using an ABC system, next year’s estimates show manufacturing overhead costs will total P228,300 for 52,000 T-shirt. If all other T-shirt costs and sales prices remain the same, the profitability that can be expected is a. P5.41 per t-shirt c. P1.11 per t-shirt b. P4.39 per t-shirt d. (P0.81) per t-shirt 79. If breakeven point is 100 units, each unit sells for P30, and fixed costs are P1,000, then on a graph the: a. Total revenue line and the total cost line will intersect at P3,000 of revenue b. Total cost line will be zero units sold c. Revenue line will start at P1,000 d. All of these answers are correct 80. Put the following ABC implementation steps in order: A. Compute the allocation rates. B. Compute the total cost of the product. C. Identify the products that are the cost object. D. Select the cost allocation bases 81. a. DACB c. BADC b. DBCA d. CDAB 82. Within the relevant range, if there is a change in the level of the cost driver, then a. Total fixed costs and total variable costs will change b. Total fixed costs and total variable costs will remain the same c. Total fixed costs will remain the same and total variable costs will change d. Total fixed costs will change and total variable costs will remain the same 83. Absorption costing: a. Expenses marketing costs as cost of goods sold b. Treats direct manufacturing costs as a period cost c. Includes fixed manufacturing overhead as an inventoriable cost d. Is required for internal reports to manager 84. If the unit level of inventory increases during an accounting period, then: a. Less operating income will be reported under absorption costing than variable costing b. More operating income will be reported under absorption costing than variable costing c. Operating income will be the same under absorption costing and variable costing d. The extract effect on operating income cannot be determined 85. 86. Price inelasticity means a. The quantity of goods demanded increases when price increases. b. The quantity of goods demanded decreases when price decreases. c. The quantity of goods demanded increases when price decreases. d. The quantity of goods demanded does not change much with price changes 87. One possible means of determining the difference between operating incomes for absorption costing and variable costing is try: a. Subtracting sales of the previous period from sales of this period b. Subtracting fixed manufacturing overhead in beginning inventory from fixed manufacturing overhead in ending inventory c. Multiplying the number of units produced by the budgeted fixed manufacturing cost rate d. Adding fixed manufacturing costs to the production volume variance 88. Which of the following costs are never relevant in the decision-making process? a. Fixed costs c. Relevant costs b. Historical costs d. Variable costs e. f. THE FOLLOWING INFORMATION APPLIES TO QUESTIONS 35 AND 36: g. Iking Corporation planned to be in operation for three years.  During the first year, 20X1, it had no sales but incurred P120,000 in variable manufacturing expenses and P40,000 in fixed manufacturing expenses.  In 20X2, it sold half of the finished goods inventory from 20X1 for P100,000 but it had no manufacturing costs.  In 20X3, it sold the remainder of the inventory for P120,000, had no manufacturing expenses and went out of business.  Marketing and administrative expenses were fixed and totalled P20,000 each year. h. 89. Under absorption costing, income for the years 20X1, 20X2 and 20X3 would be i. A1 PASSERS REVIEW CENTER///72 j. 20X1 20X2 20X3 k. l. a. (P60,000) P20,000 P40,000 m. b. (P20,000) 0 P20,000 n. c. 0 P20,000 P40,000 o. d. 0 P40,000 P60,000 p. 90. Under variable costing, income for the years 20X1, 20X2 and 20X3 would be q. r. 20X1 20X2 20X3 s. a. (P60,000) P20,000 P40,000 t. b. (P20,000) 0 P20,000 u. c. 0 P20,000 P40,000 v. d. 0 P40,000 P60,000 w. 91. Ajipe Company manufactures circuit boards for other firms. Management is attempting to search for ways to reduce manufacturing labor costs and has received a proposal from a consulting company to rearrange the production floor next year. Using the information below regarding current operations and the new proposal, which of the following decisions should management accept? x. y. Currently Proposed z. Required machine operators 5 4.5 aa. Material-handling workers 1.25 1.25 ab. Employee average pay P8 per hour P9 per hour ac. Hours worker per employee 2,100 2,000 ad. a. Do not change the production floor. b. Rearrange the production floor. c. Either, because it makes no difference to the employees. d. It doesn’t matter because the costs incurred will remain the same. ae. af. ag. ah. ai. 92. Bong Manufacturing is approached by a European customer to fulfil a one-time-only special order for a product similar to one offered to domestic customers. Bong Manufacturing has excess capability. The following per unit data apply for sales to regular customers: aj. ak. Variable costs: al. Direct materials P40 am. Direct labor 20 an. Manufacturing support 35 ao. Marketing costs 15 ap. Fixed costs: aq. Manufacturing support 45 ar. Marketing costs 15 as. Total cost 170 at. Markup (50) 85 au. Targeted selling price P255 av. aw. What is the change in operating profits if the one-time-only special order for 1,000 units is acceptance for P180 a unit by Welch? a. P70,000 increase in operating profits c. P10,000 decrease in operating profits b. P10,000 increase in operating profits d. P75,000 decrease in operating profits 93. A variance shows a deviation of actual results from standard or budgeted results. In deciding whether to investigate a variance or not, management may consider the following factors, except a. The amount of the variance and the cost of investigation b. Whether the variance is favourable or unfavourable c. The possibility that investigation will eliminate future occurrences of the variance d. The trend of the variances over overtime. 94. Tricolor Company has the following information for the current year: 95. A1 PASSERS REVIEW CENTER///73 96. Beginning fixed manufacturing overhead in inventory P95,000 97. Fixed manufacturing overhead in production 375,000 98. Ending fixed manufacturing overhead in inventory 25,000 99. 100. Beginning variable manufacturing overhead in inventory P10,000 101. Fixed manufacturing overhead in production 50,000 102. Ending variable manufacturing overhead in inventory 15,000 103. 104. What is the difference between operating incomes under absorption costing and variable costing? a. P70,000 c. P40,000 b. P50,000 d. P5,000 e. f. THE FOLLOWING INFORMATION APPLIES TO QUESTION 41 AND 42: g. Tagumpay Engine Company manufactures part TE456 used in several of its engine models. Monthly production costs for 1,000 units are as follows: h. Direct materials P 40,000 i. Direct labor 10,000 j. Variable overhead costs 30,000 k. Fixed overhead costs 20,000 l. Total costs P100,000 m. n. It is estimated that 10% of the fixed overhead costs assigned to TES456 will no longer be incurred if the company purchases TES456 from the outside supplier.tagumpay Engine Company has the option of purchasing the part from an outside supplier at P85 per unit. 105. If Tagumpay Engine Company purchases 1,000 TES456 parts from the outside supplier per month then its monthly operating income will: a. Increase by P2,000 c. Decrease by P3,000 b. Increase by P80,000 d. Decrease by P85,000 106. The maximum price that Tagumapy Engine Company should be willing to pay the outside supplier is: a. P80 per TES456 part c. P98 per TES456 part b. P82 per TES456 part d. P100 per TES456 part 107. Opportunity cost(s): a. of a resource with excess capacity is zero b. should be maximized by organizations c. are recorded an expense in the accounting records d. are most important to financial accountants 108. Maligaya Company is considering eliminating Model AE2 from its camera line because of losses over the past quarter. The past three months of information for Model AE2 are Summarized below: 109. 110. Sales (1000 units) P300,000 111. Manufacturing costs: 112. Direct materials 150,000 113. Direct labor (P15 per hour) 60,000 114. Overhead 100,000 115. Operating loss P10,000 116. Overhead costs are 70% variable and the remaining 30% is depreciation of special equipment from 117. model AE2 that has no resale value. 118. 119. If Model AE2 is dropped from the product line, operating income will: a. Increase by P10,000 c. Increase by P30,000 b. Decrease by P20,000 d. Decrease by P10,000 120. It is a sequence of economic activity in a nation’s economy that is typically characterized by four phasesrecession, recovery, growth and decline that repeat themselves over time. a. Business cycle c. Accounting cycle b. Operating cycle d. Life cycle 121. In a standard costing system, actual costs are compared with standard costs. The difference or variance determined, and responsibility for such variance is assigned or identified to a particular person or department, in order to a. Determine who is at fact and render the appropriate punishment b. Be able to set the correct selling price of the product c. Use the knowledge about the variance to promote learning and continuous improvement in the manufacturing operations d. Trace the variances to the proper inventory accounts so that they may be valued at actual costs. A1 PASSERS REVIEW CENTER///74 122. For January, the cost components of a picture frame include P0.35 for the glass, P.65 for the wooden frame, and P0.80 for assembly. The assembly desk and tools cost P400, 1,000 frames are expected to be produced in the coming year. What cost function best represents these costs? a. Y= 1.80 + 400X c. Y= 2.20 + 1,000X b. Y= 400 + 1.80X d. Y= 1.00 + 400X 123. An Enterprise Resource Planning System can best be described as: a. A collection of programs that use a variety of unconnected databases. b. A single database that collects data and feeds it into applications that support each of the company’s business activities, such as purchases, production, distribution, and sales c. A database that is primarily used by a purchasing department to department to determine the correct amount of a particular supply item to purchase d. A sophisticated means of linking two or more companies to failure their planning processes 124. In economics, it refers to a rise in the general level of price of goods And services in an economy over a period of time. a. Inflation c. Deflation b. Recession d. Depression 125. SML Company manufactures three sizes of kitchen appliances: small, medium, and large. 126. Product information is provided below: 127. 128. Small Medium Large 129. Unit selling price P150 P250 P500 130. Unit costs: 131. Variable manufacturing (60) (120) (200) 132. Fixed manufacturing (40) (50) (120) 133. Variable selling and administrative (30) (30) (30) 134. Unit profit P20 P50 P150 135. Demand in units 100 120 100 136. Machine-hour per unit 20 40 100 137. 138. The maximum machine-hours available are 6,000 per week. 139. How many of each product should be produced per month using the short-run profit maximizing strategy? 140. Small Medium Large 141. a. 0 120 12 142. b. 100 0 40 143. c. 100 100 0 144. d. 100 20 40 145. 146. ANSWER KEY 147. MANAGEMENT ADVISORY SERVICES 148. 1. C 2. D 3. B 4. C 5. B 6. D 7. C 8. D 9. C 10. B 11. C 12. B 13. A 14. C 15. B A1 PASSERS REVIEW CENTER///75 16. C 17. A 18. B 19. 20. B 21. C 22. D 23. B 24. B 25. D 26. D 27. C 28. D 29. C 30. C 31. B 32. D 33. B 34. B 35. B 36. A 37. B 38. A 39. A 40. A 41. C 42. B 43. A 44. B 45. A 46. C 47. B 48. B 49. A 50. B 51. 52. 53. 54. A1 PASSERS TRAINING, RESEARCH, REVIEW & DEVELOPMENT COMPANY 55. 2nd Floor Sommerset Bldg., Lopez Jaena St. Jaro, Iloilo City 56. Tel. No.: (033) 320-2728; 09106547262 57. Email Address: [email protected] 58. 59. BOARD OF CERTIFIED PUBLIC ACCOUNTANT 60. 61. CERTIFIED PUBLIC ACCOUNTANT Licensure Examination 62. 63. AUDITING PROBLEMS 64. INSTRUCTIONS: 1. Detach one (1) answer sheet form the bottom of your Examinee ID/Answer Sheet Set. 2. Write the subject title “BUSINESS LAW AND TAXATION” on the box provided. 3. Shade Set Box “A” on your answer sheet if your test booklet is Set A; Set Box “B” if your test booklet is Set B. 65. ____________________________________________________________________________________________ 66. MULTIPLE CHOICES: 67. 68. PROBLEM NO. 1 A1 PASSERS REVIEW CENTER///76 69. During 2010 and 2011, SUMAN CORPORATION experienced transactions involving property, plant, and equipment (PPE). A number of errors were made in recording some of these transactions. For each item listed below, determine the effect of the error (if any) by using the following codes: 70. 71. O = Overstates; U = understate; NE = No Effect or if errors was made 72. 73. 2010 2011 74. Net Book Net Book 75. Value of 2010 Value of 2011 76. PPE at Net PPE at Net 77. Transaction 12/31/10 Income 12/31/11 Income 78. 1. The cost of installing a new computer system in 2010 was not recorded in 2010. It was charged to expense in 2011. 79. A. U U U U 80. B. U O U U 81. C. U O U O 82. D. U U O U 2. In clerical workers were trained to use the new computer system at a cost of P30,000, which was, erroneously capitalized. The cost is to be written off over the expected life of the new computer system. 83. A. NE NE O O 84. B. NE NE U U 85. C. NE NE O U 86. D. NE NE U O 87. 88. 2010 2011 89. Net Book Net Book 90. Value of 2010 Value of 2011 91. PPE at Net PPE at Net 92. Transaction 12/31/10 Income 12/31/11 Income 93. 3. Interest cost qualifying for capitalization in2010 was charged to interest expense in 2010. 94. A. NE U NE O 95. B. U U U U 96. C. U U U O 97. D. U O U U 4. In 2010 land was bought for an employee parking lot. The P2, 000 title search fee was charged to expense in 2010. 98. A. U U U O 99. B. U O U O 100. C. U NE U NE 101. D. U U U NE 5. The cost of moving several manufacturing facilities from metropolitan locations to suburban areas in 2010 was capitalized. The cost was written off over a 10-year period beginning in 2010. 102. A. O U O U 103. B. U O U O 104. C. O O O U 105. D. NE NE NE NE 106. 107. PROBLEM NO. 2 108. The shareholders’ equity section of BENSON CORPORATION’s balance sheet as of December 31, 109. 2010 is a follows: 110. 111. Common stock, P5 par value; authorized, 2,000,000 shares; 112. Issued, 400,000 shares P2,000,000 113. Paid-in capital in excess of par 850,000 114. Retained earnings 3,000,000 115. P5,850,000 116. 117. The following events occurred during 2011: 118. 1. Jan. 5 10,000 shares of authorized and unissued common stock were sold for P8 per A1 PASSERS REVIEW CENTER///77 119. 120. 2. Jan. 16 121. 122. 3. Feb. 10 123. 124. 4. March 1 share. Declared a cash dividend of 20 centavos per share payable February 15 to shareholders of record o February 5. 20,000 shares of shares of authorized and unissued common stock were sold for P12 per share. a 30% stock dividend was declared and issued. Market value per share is currently P15. 125. 5. April 1 A two-for-one split was carried out. The par value of the stock was to be reduced to P2.50 per share. Market value on March 31 was P18 per share. 126. 6. July 1 A 15% stock dividend was declared and issued. Market value is currently P10 per share. 127. 7. Aug. 1` A cash dividend of 20 centavos per share was declared, payable September 1 to shareholders of record on august 21. 128. 129. Based on the preceding information, determine the balances of the following at December 31, 2011. 130. 6. Number of common shares issued and outstanding A. 642.850 C. 1,285,700 B. 1,228,000 D. 989,000 7. Common stock A. P2,000,000 C. P6,428,500 B. P3,214,250 D. P2,795,000 8. Paid in capital in excess of par A. P2,277,750 C. P3,567,750 B. P1,020,000 D. P2,310,000 9. Retained earnings A. P596,000 C. P338,860 B. P1,853,750 D. P306,610 10. A feature common to both stock splits and stock dividends is A. A transfer to earned capital of a corporation. B. That there is no effect on total stockholders’equity. C. An increase in total liabilities of a corporation. D. A reduction in the contributed capital of a corporation. 11. 12. 13. 14. 15. PROBLEM N0. 3 16. 17. LYKA COMPANY’S net incomes for the past three years are presented below: 18. 19. 2012 2011 2010 20. P480,00 P450,00 P360,00 21. 22. During the 2012 year-end the following items come to your attention: 23. 1. Lyka bought at truck on January 1, 2009 for P196,000 with a P16,000 estimated salvage value and a six-year life. the company debited an expense account and credited cash on the purchase date for the entire cost of the asset. (Straight-line method) 2. During 2012, Lyka changed from the straight-line method of depredating its cement plant to the double-declining balance method. The following computations present depreciation on both bases: 24. 2012 2011 2010 25. Straight-line 36,000 36,000 36,000 26. Double-declining 46,080 57,600 72,000 27. A1 PASSERS REVIEW CENTER///78 28. The net income for 2012 was computed using the double-declining balance method, on the January 1,2012 book value, over the useful life life remaining at that time. The depreciation recorded in 2012 was P72,000. 3. Lyka, in reviewing its provision for uncollectibles during 2012, has determined that 1% is the appropriate amount of bad debt expense to be charged to operations. the company had used ½ of 1% as its rate in 2011 and 2012 when the expense had been P18,000 and P12,000, respectively. The company recorded bad debt expense under the new rate for 2012. The company would have recorded P6,000 less of bad debt expense on December 31, 2012 under the old rate. 29. 30. What is the entry necessary to correct the blocks for the transaction in part 1 of the problem, assuming that the books for the current year are still open? 31. A. Equipment 196,000 32. Depreciation expense 30,000 33. Accumulated depreciation 120,000 34. Retained earnings 106,000 35. B. Equipment 196,000 36. Accumulated depreciation 90,000 37. Retained earnings 106,000 38. C. Equipment 166,000 39. Accumulated depreciation 30,000 40. Retained earnings 136,000 41. D. Equipment 196,000 42. Depreciation expense 30,000 43. Accumulated depreciation 60,000 44. Retained earnings 166,000 45. What is the correct net income to be reported in 2010? A. P330,000 C. P360,000 B. P336,000 D. P294,000 46. What is the correct net income to be reported in 2011? A. P393,400 C. P441,600 B. P420,000 D. P480,000 47. What is the correct net income to be reported in 2012? A. P510,000 C. P444,600 B. P428,400 D. P450,000 48. Which of the following is (are) the proper time period(s) to record the effects of a change in accounting estimate? A. Current period and prospectively B. Current period and retrospectively C. Retrospectively only D. Current period only 49. 50. 51. PROBLEM NO. 4 52. In making the first audit of the Delivery Equipment account of DELTA CORPORATION as 53. of December 31, 2011, you encounter the following facts. 54. 55. Delivery Equipment 56. Date Particulars Debit Credit 57. 1/1/09 Truck 1, 2, 3 and 4 P3,200,000 58. 59. 60. 61. 62. 3/15/10 7/1/10 7/10/10 Replacement of Truck 3 tires Truck 5 Reconditioning of Truck 4, which was 25,000 800,000 5,000 damaged In a collision 9/1/10 Insurance recovery on Truck 4 accident P33,000 10/1/10 Sale of Truck 2 600,000 4/1/11 Truck 6 1,000,000 150,000 A1 PASSERS REVIEW CENTER///79     63. 5/2/11 Repainting of Truck 4 64. 6/30/11Truck 7 27,000 720,000 65. 12/1/11Cash received on lease of Truck 7 22,000 66. 67. Accumulated Depreciation 68. Date Particulars Debit Credit 69. 12/31/09 Depreciation expense P300,000 70. 12/31/10 Depreciation expense 300,000 71. 12/31/11 Depreciation expense 300,000 72. 73. Additional Information: 1. On July 1, 2010, Truck 3 was traded in for a new truck, Truck 5, costing P850,000; the setting party allowed a P50,000 trade in value for the old truck. 2. On April 1, 2011, Truck 6 was purchased for P1,000,000; Truck 1 and cash of P850,000 being given for the new truck. 3. You are instructed y the senior in-charge of the audit to accept the depreciation rate of 20% by unit basis. 4. Unit cost of Trucks 1 to 4 is at P800,000 74. What is the correct cost of Truck No. 5? A. P850,000 C. P900,000 B. P800,000 D. P560,000 75. What is the correct depreciation expense for 2010? A. P725,000 C. P605,000 B. P305,000 D. P600,000 76. What is the correct depreciation expense for 2011? A. P552,000 C. P712,000 B. P592,000 D. P300,000 77. The entry to correct the depreciation charges for the years 2009 through 2011 should include a credit to Accumulated Depreciation of A. P645,000 C. P900,000 B. P937,000 D. P292,000 78. The balance of the Delivery Equipment account at December 31, 2011 should be A. P5,770,000 C. P4,170,000 B. P3,320,000 D. P3,370,00 E. F. PROBLEM NO. 5 G. In connection with your audit of the BRAHE CORPORATION’s financial statements for the year ended December 31, 2011, you noted the following items relative to the company’s intangible assets. A patent was purchased from Tycho Company for P6,000,000 on January 2, 2010. Brahe estimated that the remaining useful life of the patent to be 10 years. The patent was carried in Tycho’s accounting records at P4,000,000 when Tycho sold it to Brahe. During 2011, a franchise was purchased from Nova Company for P1,440,000. In addition, 5% of the revenue from the franchise must be paid to Nove. Revenue from the franchise for 2011 was P7,500,000. Brahe estimates the useful life of the franchise to be 10 years and takes full year’s amortization in the year of purchase. Brahe incurred research and development costs of P1,299,000 in 2011. Brahe estimates that these costs will be recouped by December 31, 2014. On January 1, 2011, Brahe, because of the recent events in the industry, estimates that the remaining life of the patent purchased on January 2, 2010, is only 5 years from January 1, 2011. H. I. Based on the above and the result of your audit, determine the following: J. 79. Amortization of patent for 2011 A. P1,080,000 C. P600,000 B. P1,200,000 D. P1,350,000 80. Carrying amount of patent as of December 31, 2011 A. P4,320,000 C. P3,600,000 B. P4,050,000 D. P4,800,000 A1 PASSERS REVIEW CENTER///80 81. A. Carrying P4,896,000 amount of intangible assets as of December 31, 2011 C. P5,616,000 B. P5,346,000 D. P6,915,000 82. Total amount that should be charged against income in 2011 A. P3,168,000 C. P3,018,000 B. P2,898,000 D. P1,599,000 83. The most effective means for the auditor to determine whether a recorded intangible asset possesses the characteristics of an asset is to A. Inquire as to the status of patent application B. Analyze research and development expenditures to determine that only those expenditures possessing future economic benefit have been capitalized. C. Evaluate the future revenue-producing capacity of the intangible asset. D. Vouch the purchase by reference to underlying documentation 84. 85. PROBLEM NO. 6 86. SUMPAK CORP. Began operations in 2006. On July 15, 2011, a fire broke out in the company’s 87. warehouse destroying all inventory and many accounting records. The following information 88. was assembled from the microfilmed records. All sales and purchases are on account. 89. 90. The following is a summary of prior year’s sales and gross profit on sales: 91. 2008 2009 2010 92. 93. Sales P1,252,000 P1,410,000 P1,360,000 94. Gross profit 375,600 366,600 462,400 95. 96. What is the company’s average gross profit ratio based on its prior years ‘sales? A. 26% C. 30% B. 34% D. 29% 97. What is the company's total sales for the period January 1 through July 15 of the current year? A. P1,504,200 C. P1,765,380 B. P1,511,000 D. 1,768,780 98. What is the company’s total purchases for the period January 1 through 15 of the current year? A. P905,580 C. P1,044,420 B. P912,170 D. P1,009,670 99. What is the company’s estimated inventory on July 15, 2011, before the fire? A. P186,605 C. P146,930 B. P244,430 D. P279,180 100. What is the inventory fire loss? A. P146,930 C. P132,250 B. P186,605 D. P112,180 E. F. PROBLEM NO. 7 G. H. On January 1, 201, ODAHU, INC. issued P100,000, 10 year bonds when the market rate of interest was 8%. Interest is payable on June 30 and December 31. The following financial information is available. I. Sale P300,000 J. Cost of sales 180,000 K. Gross income 120,000 L. Interest expense ? M. Depreciation expense (14,500) N. Other expense (82,000) O. Net Income ? P. Q. R. S. Dec.31, 2011 Jan.1.2011 T. Account receivable P55,000 P48,000 U. Inventory 87,000 93,000 V. Accounts payable 60,000 58,000 A1 PASSERS REVIEW CENTER///81 W. X. All purchases of inventory are on account. Other expenses are paid for in cash. 101. What is the carrying value of the bonds on December 31, 2011? A. P112,661 C. P100,000 B. P113,592 D. P112,233 102. What is the interest expense for 2010? A. P8,641 C. P9,069 B. P10,000 D. P4,544 103. How much was paid for inventory purchases? A. P172,000 C. P184,000 B. P186,000 D. P174,000 104. What is Idahu’s net income for 2010? A. P13,500 C. P23,000 B. P14,431 D. P14,859 105. How much was received from customers in 2010? A. P283,000 B. P245,000 C. P293,000 D. P307,000 106. 107. PROBLEM NO. 8 108. The inventory control account balance of SAMATHA CORP. At June 30, 2011 was P221,020 using the perpetual inventory system. A physical count conducted on that day found inventory on hand worth P220,200. Net realizable value for each inventory item held for sale exceeded cost. An investigation of the discrepancy revealed the following: a. Goods worth P6,600 held on consignment for Port Company had been included in the physical count. b. Goods costing P1,200 were purchased on credit from Rome, Inc. on June 27, 2011 on FOB shipping point terms. The goods were shipped on June 28, 2011 but, as they had not arrived by June 30, 2011, were not included in the physical count. The purchase invoice was received and processed on June 30, 2011. c. Goods costing P2,400 were sold on credit to Arriola Co. For P3,900 on June 28, 2011 on FOB destination terms. The goods were still in transit on June 30, 2011. The sale invoice was processed on June 29, 2011. d. Goods costing P2,730 were purchased on credit (FOB destination) from Sanmig Co. On June 28, 2011. The goods were received on June 29,2011 and included in the physical count. The purchase involve was received on July 2, 2011. e. On June 30, 2011, Samantha sold goods costing P6,300 on credit (FOB shipping point) to Pizza Co. For P9,600. The goods were dispatched from the warehouse on June 30, 2011 but the sales invoice had not been processed at that date. f. Damaged inventory items valued at P2,650 were discovered during the physical count. These items were still recorded on June 30, 2011 but were omitted from the physical count. 109. 110. Based on the preceding information, answer the following: 111. The adjusting entry for the consigned goods included in the physical count (Item a) should include a 112. A. Debt to inventory for 114. C. Credit to purchases for P6,600. P6,600. 113. B. Credit to inventory for 115. D. No adjusting entry is P6,600. necessary. 116. The error described in item e will 117. A. Overstate gross profit 119. C. Overstate sales by P3,300. revenue by P9,600. 118. B. Understate gross 120. D. Understate inventory profit by P3,300. by P6,300 121. The inventory control accounting balance should be increase be increased (decreased) by 122. A. (P3,820) 124. C. (P3,000) 123. B. P3,820 125. D. P3,000 126. If the perpetual inventory records show lower quantities of inventory than the physical count, an explanation of the difference might be unrecorded 127. A. Purchases. 129. C. Sales discounts. 128. B. Sales. 130. D. Purchase discounts. 131. A client maintains perpetual inventory record in both quantities and pesos. If the assessed level of control risk is high, an auditor will probably 132. A. Increase the extent of tests of controls relevant to the inventory cycle. 133. B. Request the client to schedule the physical inventory count at the end of the year. 134. C. Apply gross profit tests to ascertain the reasonableness of the physical counts. A1 PASSERS REVIEW CENTER///82 135. D. Insist that the client perform physical counts of inventory items several times during the year. 136. 137. PROBLEM NO. 9 138. CALACHUCHI CORP.’s accounts receivable subsidiary ledger shows the following information: 139. 140. ACCOUNT BALANCE INVOICE 141. COSTUMER DEC. 31,2011 DATE AMOUNT 142. Aruy, Inc. P35,180 12/06/11 P14,000 143. 11/29/11 21,180 144. 145. Naku Co. 20,920 09/27/11 12,000 146. 08/20/11 9,920 147. 148. Syak Corp. 30,600 12/08/11 20,000 149. 10/25/11 10,000 150. 151. Trip Co. 45,140 11/17/11 23,140 152. 10/09/11 22,000 153. 154. Uy Co. 31,600 12/12/11 19,200 155. 12/02/11 12,400 156. 157. Xak Corp. 17,400 09/12/11 17,400 158. 159. The estimated bad debt rates below are based or Calachuchi Corp.’s receivable collection 160. experience. 161. 162. Age of Account Rate 163. 0 – 30 days 1% 164. 31 – 60 days 1.5% 165. 61 – 90 days 3% 166. 91 – 120 days 10% 167. Over 120 days 50% 168. 169. The allowance for bad debts account had a debt balance of P5,500 on December 31, 2011, 170. before adjustment. 171. The company’s accounts receivable under “61-90 days” category should be 172. A. P32,600 174. C. P44,600 173. B. P44,320 175. D. P42,000 176. The company’s accounts receivable under “91-129 days” category should be 177. A. P38,320 179. C. P29,400 178. B. P40,000 180. D. P12,000 181. The allowance for bad debts to be reported on the statement of financial position at December 31, 2011, is 182. A. P9,699 184. C. P4,199 183. B. P15,199 185. D. 5,500 A1 PASSERS REVIEW CENTER///83 186. What account? 187. 188. 189. 190. 191. 192. entry should be made on December 31, 2011, to adjust the allowance for bad debts A. B. C. Bad debt expense Allowance for bad debt Bad debt expense Allowance for bad debt Allowance for bad debt Bad debt expense 15,199 15,199 4,199 4,199 5,500 5,500 193. D. Bad debt expense 9,699 194. Allowance for bad debt 9,699 195. 196. What is the net realizable value of accounts at December 31, 2011? 197. A. P165,641 199. C. P196,039 198. B. P171,141 200. D. P186,340 201. 202. PROBLEM NO. 10 203. TANYA COMPANY was organized on January 2, 2010, with authorized share capital of 50,000 204. shares of 10%, P200 par value preference and 200,000 shares of P10 par value ordinary. During 205. the company’s first two years of operations, the following equity transaction occurred. 206. 207. 2010 208. Jan. 2 Sold 10,000 ordinary shares at P16. 209. 2 Sold 3,000 ordinary shares at P216. 210. 211. Mar. 2 Sold ordinary shares as follows: 10,800 shares at P22; 2,700 shares at P25. 212. 213. July 10 Acquired a nearby piece of land, appraised at P400,000, for 600 preference 214. share and 27,000 ordinary shares. (Preference share capital was recorded at 215. P216, the balance being assigned to ordinary shares). 216. Dec. 16 Declared the regular preference share dividend and a P1.50 ordinary share 217. dividend. 218. 28 Paid the dividends declared on December 16. 219. 31 The income summary account showed a credit balance of P450,000 220. 221. 2011 222. Feb. 27 Reacquired 12,000 ordinary shares at P19. 223. June 17 Resold 10,000 treasury shares at P23. 224. July 31 Resold all of the remaining treasury shares at P18. 225. Sep. 30 Sold 11,000 additional ordinary shares at P21. 226. Dec. 16 Declared the regular preference share dividend and an P0.80 ordinary share 227. dividend. 228. 28 Paid the dividends declared on Dec.16. 229. 31 The income summary account showed a credit balance of P425,000. 230. 231. Based on the above information, determine the balance of the following: 232. 233. Preference share capital 234. A. P729,600 236. C. P720,000 235. B. P777,600 237. D. P548,000 238. Ordinary share capital 239. A. P615,400 240. B. P615,000 A1 PASSERS REVIEW CENTER///84 241. C. P577,000 242. D. P966,500 243. Total share premium 244. A. P95,600 246. C. P449,100 245. B. P447,100 247. D. 409,100 248. Inappropriate retained earnings 249. A. P875,000 251. C. P606,050 250. B. P416,000 252. D. P604,0 253. Total shareholders’ equity 254. A. 2,388,150 256. C. 2,376,630 255. B. 1,892,100 257. D. 2,498,150 258. 259. PROBLEM NO. 11 260. At the beginning of year 1, an entity grants to a senior executive 30,000 share options. The grant is conditional upon the executive remaining in the entity’s employ until the end of year 3. 261. The share options can be exercised if the entity’s share price increases from P20 at the beginning of year 1 to above P30 at the end of year 3. If the share price is above P30 at the end of year 3, the share options can be exercised at any time during the next five years, i.e., by the end of year 8. 262. The entity estimates the fair value of the share options on grant date to be P5 per option. This estimate takes into account the following market condition:  The possibility that the share price will exceed P30 at the end of year 3, i.e., the share options become exercisable; and  The possibility that the share price will not exceed P30 at the end of year 3, i.e., the share options will be forfeited. 263. The following actual events occurred in years 1 to 3: 264. YEAR 1  The share price has increased to P24  The entity’s estimate the fair value of the options is P4 at the end of year 1. This taxes into account whether the market condition will be satisfied by the end of year 3. 265. YEAR 2  The share price has decreased to P22. However, the entity remains optimistic that the share price target will be met by the end of year 3.  The estimated fair value of the share option is P3. Again, this estimate takes into account the market condition noted above. 266. YEAR 3  The share price only reaches P28 by the end of year 3.  The estimated value of the share option is zero, as the market condition has not been satisfied. 267. Based on the preceding information, determine the following: 268. Compensation expense for year 1 A. P 50, 000 P 60, 000 B. P 40, 000 P30, 000 E. 52. Compensation expense for year 2 A. P 50, 000 C. P 60, 000 B. P 40, 000 D. P 30, 000 E. 53. Compensation expense for year 3 A. P 50, 000 C. P 40, 000 B. P 0 D. P30, 000 E. 54. Share options outstanding at the end of year 2. A. P 70, 000 C. P 90, 000 B. P 80, 000 D. P 100, 000 E. 55. Cumulative compensation expense for the three year period. A. P 0 C. P 100, 000 B. P 150, 000 D. P 70, 000 E. F. PROBLEM NO. 12 G. MARUJA CO. Was organized on January 2, 2011. The following items are from the company’s trial balance on December 31, 2011. H. Ordinary share capital P 1,500, 000 I. Share premium 150, 000 J. Merchandise inventory 69, 000 K. Land 1,000, 000 L. Building 1,400, 000 M. Furniture and fixtures 367, 000 N. Accounts receivable 165, 400 O. Accounts payable 389, 650 A1 PASSERS REVIEW CENTER///85 P. Notes payable bank 500, 000 Q. Sales 6, 235, 200 R. Operating expenses (Including depreciation of 400,000) 1, 005, 150 S. T. Additional information as is follows: U. 1. Deposits in transits, December 31 P384, 660 V. 2. Service charge for December 2000 W.3. Outstanding checks, December 31 475, 000 X. 4. Bank balance, December 31 892, 000 Y. 5. MARUJA CO.’ S mark up on sales 30% Z. AA. Determine the following: AB. 56. Total collections from sales A. P 6,114,967 C. P 6,235,200 B. P 4,119,240 D. P 6,069,800 E. 57. Total payments for merchandise purchases A. P 3,905,990 C. P 4,043,990 B. P 4,649,140 D. P 5,914,550 E. 58. Total cash receipts per books A. P 7,819,800 C. P 8,219,800 B. P 8,169,800 D. P 8,069,800 E. 59. Total cash disbursements per books A. P 7,816,140 C. P 8,021,290 B. P 7,416,140 D. P 7,278,140 E. 60. Cash balance per books on December 31 A. P 653,660 B. P 803,660 C. P 1,203,660 D. P 707,060 F. G. H. I. J. ANSWER KEY K. AUDITING PROBLEMS L. 1.A 2.A 3.C 4.A 5.D 6.C 7.B 8.A 9.C 10. B 11. A 12. A 13. B 14. B 15. A 16. A 17. C 18. B 19. B 20. B 21. A 22. A 23. C 24. A 25. C 26. C 27. A 28. C 29. C 30. A A1 PASSERS REVIEW CENTER///86 31. A 32. C 33. A 34. B 35. C 36. B 37. A 38. A 39. C 40. A 41. A 42. C 43. A 44. B 45. B 46. C 47. B 48. B 49. C 50. A 51. A 52. A 53. A 54. D 55. B 56. D 57. C 58. C 59. A 60. B 61. 62. 63. 64. 65. 66. 67. 68. 69. A1 PASSERS TRAINING, RESEARCH, REVIEW & DEVELOPMENT COMPANY 70. 2nd Floor Sommerset Bldg., Lopez Jaena St. Jaro, Iloilo City 71. Tel. No.: (033) 320-2728; 09106547262 72. Email Address: [email protected] 73. 74. BOARD OF CERTIFIED PUBLIC ACCOUNTANT 75. 76. CERTIFIED PUBLIC ACCOUNTANT Licensure Examination 77. 78. PRACTICAL ACCOUNTING 1 79. 80. INSTRUCTIONS: 81. 82. 1. Detach one (1) answer sheet from the bottom of your Examinee ID/Answer Sheet Set. 83. 2. Write the subject title “PRACTICAL ACCOUNTING I “on the box provided 84. 3. Shade Set Box ‘’A” on your answer sheet if your test booklet is Set A; Set box “B” if your if your test booklet is Set B. 85. __________________________________________________________________________________ 86. MULITIPLE CHOICE: 87. 88. 1. On December 31, 2011, the current receivable of Marty Company consisted of the following: 89. 90. Trade accounts receivable 1,200,000 91. Allowance for uncollectible accounts (100,000) 92. Claim against shipper for goods lost in transit (November 2011) 50,000 93. Selling price of unsold goods sent by Marty on consignment at 125% 94. Of cost (not included on marty’s ending inventory) 300,000 95. Security deposit on lease of warehouse used for storing some inventories 200,000 96. 97. What is the correct total of current net receivables on December 31, 2011? A1 PASSERS REVIEW CENTER///87 a. 1,150,000 c.1,350,000 b. 1,390,000 d. 1,650,000 e. 2. Farmville Company had sales of P2, 500,000 during December 2011. Experience has shown that merchandise equalling 6% of sales will be returned within thirty days and an additional 4% will be returned within ninety days. Returned merchandise is readily resalable. In addition, merchandise equalling 15% of sales will be exchanged for merchandise of equal or greater value. What amount should Farmville will report for net sales in its statement of comprehensive income for the month of December 2011? a. 2,125,000 c. 2,250,000 b. 1,875,000 d. 2,025,000 e. 3. On December 1, 2011, Mandela Company gave Mosgov Company a P2, 000,000, 12% loan. Mosgov received proceeds of P1, 940,000 after the deduction of a P60, 000 non-refundable loan origination fee. Principal and interest are due sixty monthly instalments of P44, 500 beginning January 1, 2012. The repayments yield effective interest rate of 12% at a present value of P2, 000, 000 and 13.4% at a present value of P 1,940,000. What amount of accrued interest receivable should Mandela report on December 31, 2011? a. 20, 000 c. 22, 333 b. 21, 663 d. 19, 400 e. 4. Rogers Company factored P3, 000, 000 of accounts receivable to Musburger Company on August 1, 2011. Control was surrendered by Rogers. Musburger accepted the receivables subject to recourse for non-payment. Musburger assessed a fee of 2% and retain a holdback equal of 5% of the accounts receivable. In addition, Musburger charged 15% interest computed on a weighted average time to maturity of the receivable of forty days. The fair value of the recourse obligation is P300, 000. Assuming all receivables are collected. What is the cost of factoring the receivables? a. 110, 000 c. 259, 315 b. 260, 000 d. 109, 315 e. 5. Indira Company offers three-year warranty on its products. Indira previously estimated warranty costs to be 5% of sales. Due to a technological advancement in production at the beginning of 2011, Indira now believed 4% of sales to be a better estimate of warranty costs warranty costs of P300,000 and P400,000 were reported in 2009 and 2010, respectively. Sales for 2011 were P9,000,000. What should be reported as warranty expense for 2011? a. 360, 000 c. 220, 000 b. 500, 000 d. 450, 000 e. f. 6. Fidel Company began operations on January 1, 2010 and used the FIFO method of inventory costing. Management is contemplating a change to the average method to be consistent with Fidel’s parent company. The following information has been developed: g. h. 2010 2011 i. Ending inventory j. FIFO 150,000 400,000 k. Average 300,000 200,000 l. Income under FIFO 2,500,000 3,000,000 m. n. What is the income after a change to the average method? a. 3,200,000 c. 2,650,000 b. 2,950,000 d. 3,050,000 e. 7. Adolph Company provided the following information for 2011: f. g. Gross profit 500,000 h. Costs of goods manufactured 2,800,000 i. j. Beginning inventories k. l. Goods in process 350,000 m. Finished goods 520,000 n. o. Ending inventories p. q. Goods in process 230,000 r. Finished goods 150,000 s. t. How much was the amount of sales for 2011? A1 PASSERS REVIEW CENTER///88 a. 3,050,000 c. 3,790,000 b. 3,670,000 d. 2,810,000 e. 8. Marilyn Company regularly buys sweaters from Marcus Company and is allowed trade discount of 20% and 10% from the list price. Marilyn made a purchase on March 20 and received an invoice with a list price of P1, 200,000 under terms of 2/10, n/30, and a freight charge of 50,000. Marilyn uses the net method of recording purchases. What is the amount of inventory from this purchase assuming payment was made on April 1, 2011? a. 914,000 c. 896,720 b. 890,000 d. 873,200 e. 9. Ronald Company specializes in the sale of Apple accessories and software packages. Ronald had the following transactions with one of its supplies: f. g. Purchase of accessories 3,000,000 h. Purchase of software packages 2,500,000 i. Returns and allowances ` 200,000 j. Purchase discounts taken 160,000 k. l. Purchase were made throughout the year on terms 5/10, n/30. All returns and allowances took place within 5 days of purchase and prior to any payment on the account, what is the amount of “Discounts lost” for the purchases? a. 115,000 c. 120,000 b. 105,000 d. 100,000 e. 10. Edison Company began the year with 500 units of inventory. The sale price of each unit was varied throughout the month of January. During January, Edison completed the following inventory transactions: f. Units Unit Cost Unit Sale Price g. January 3 Purchase 200 P3, 000 h. 10 Sale 400 P4, 000 i. 14 Sale 100 4, 200 j. 20 Purchase 400 3200 k. 22 Sale 500 4500 l. 30 Purchase 200 3,300 m. n. o. Edison used the periodic average method of inventory costing. What is the total cost of the beginning inventory if the gross profit in the month of January was P 1,470,000? a. 1,100,000 c. 1,400,000 b. 1,210,000 d. 1,300,000 e. 11. The following information pertains to Lech Company for the current year: f. g. Cash sales 750,000 h. Cash collected on accounts receivable 4,800,000 i. Accounts receivable, January 1 400,000 j. Accounts receivable, December 31 850,000 k. Bad debts written off 100,000 l. Purchases 4,000,000 m. Inventory, December 31 620,000 n. Gross profit on sales 30% o. p. What was the cost of goods for the current year? a. 3,640,000 c. 4,200,000 b. 3,745,000 d. 4,270,000 e. 12. Evington Company sells new equipment with P880, 000 list price at a mark up of 20% on selling price. A dissatisfied customer returned a piece of equipment. Evington Company determined that the returned equipment can be resold if it is conditioned. The estimated sales price of the reconditioned equipment is P800, 000. The reconditioning cost is estimated to be P50, 000 and estimated cost to sell is 10% of the selling price. What is the equipment’s net realizable value? a. 742, 000 c. 510, 000 b. 670, 000 d. 566, 000 e. 13. The records for Thurgood’s Discount Store are summarized below for the month of July. f. g. Inventory, July 1 - at retail, P 450, 000; at cost, P300, 000 A1 PASSERS REVIEW CENTER///89 h. i. j. k. l. Purchase in July - at retail, P5, 400,000; at cost, P350, 000,000; Freight-in - P75, 000 Purchase returns – at retail, P225,000: at cost, P 125,000; Purchase allowance Net mark-ups –P 675, 000; net markdowns –P 600,000 Inventory losses due to normal breakage- at retail P 150,000, Employee discounts-P 100,000 Sales-P5,000,000; sales returns-P250,000;sales allowance-P 50,000; Sales discounts-P 25,000 m. n. Using the average retail inventory method, what is the estimated cost of sales? o. a. 3,250,500 q. c. 3,400, 000 p. b. 3,151,500 r. d. 3,300,000 s. t. 14. Information regarding Marshall Company’s portfolio of available for sale securities in as follows: u. Aggregate cost on December 31, 2011 1,500,000 v. Unrealized gains on December 31, 2011 50,000 w. Unrealized losses on December 31, 2011 200,000 x. Net realized gains during 2011 70,000 y. z. On December 31, 2010, Marshall reported an unrealized loss of P60,000in other comprehensive income to reduce this securities should be reported as other comprehensive income in the other shareholders equity on December 31, 2011? a. 200,000 c. 90,000 b. 150,000 d. 80,000 e. 15. Guevara Company‘s statement of comprehensive income for the year ended December 31, 2011 reported income before taxes of P3, 000,000. The auditor questioned the following amounts that had been included in income: f. g. Unrealized gain on available-for-sale investment 250,000 h. Equality in earnings of James Company 400,000 i. Dividends received from James 150,000 j. k. Guevara owns 30% of James preference share but has no interest in ordinary shares. What should be reported as income before taxes? a. 2,750,000 c. 2,350,000 b. 2,600,000 d. 2,050,000 e. 16. On January 1, 2011, Stallone Company purchased “held to maturity” bonds with face value of P5, 000,000 for p 4,682,600 including transaction cost of P82600. The bonds are purchased to yield 10% interest. The nominal interest rate on the bonds is 8% payable annually every December 31. Stallone appropriately used the effective interest method of amortization. The market value of the bonds on December 31, 2011 is 105. What is the carrying amount of the investment on December 31, 2011? a. 4,750,860 c. 5,250,000 b. 4,682,600 d. 4,614,340 e. 17. On December 31, 2011, Seifert Company purchased 12,000 ordinary shares of Alexander Company with P100 per value for P1, 650,000 to be held as available for sale. On March 1, 2012. Seifert received a 10% stock dividend from Alexander Company. On this date; Alexander’s ordinary share is selling for P140. On June 1, 2012, Seifert sold all the stock dividends that were received on March 1, 2012 at P150 per share. What is the total amount of income to be recorded by Seifert in 2012 from its investment in Alexander? a. 198,000 c. 30,000 b. 183,000 d. 15,000 e. 18. On July 1, 2011, Aguilar Company purchased bonds with face value of P5, 000,000 to be held as “available for sale”. The entity paid P4, 600,000 plus transaction cost of P142, 000 and accrued interest of P150, 000. The bonds pay 6% interest annually on December 31 of each year with an 8% effective yield. The transaction cost was approximately included in the carrying amount of the investment while payment for the interest was recorded as interest receivable. The bonds are quoted at 105 on December 31, 2011. What is the amount of interest that was received on December 31, 2011? a. 184,000 c. 300,000 b. 150,000 d. 189,680 e. 19. Winston company has seven operating segments, five of which are reportable. A reconciliation prepared by the chief financial officer with regard to the portable and noty reportable operating segments as is follows: f. g. Reportable operating segments h. Sales from external customers 25,000,000 i. Intersegment sales 5,000,000 j. Intersegment transfers 3,000,000 k. Total segment expenses 10,000,000 A1 PASSERS REVIEW CENTER///90 l. Total segment assets 75,000,000 m. Total segment liabilities 18,000,000 n. o. All other segments p. Sales from external customers 3,000,000 q. Intersegment sales 500,000 r. Intersegment transfers 200,000 s. Total segment expenses 700,000 t. Total segment assets 5,000,000 u. Total segment liabilities 1,200,000 v. w. What is the minimum revenue from a customer to qualify as a “major customer?” a. 2,500,000 c. 3,300,000 b. 2,800,00 d. 3,670,000 e. 20. Benazir Company prepares quarterly interim financial reports. The entity sells it products through agents who are paid a fixed monthly salary and a commission of 5% that is paid at year-end. Sales for the first quarter were P10, 000,000. However, in the second quarter, the employees union negotiated that agent’s commission should be increased to 8%and to be applied as of the beginning of the current year. Sales in the second quarter were P15, 000,000. What would be the sales commissio9n expense charged in the second quarters interim financial statements? a. 1,500,000 c. 1,700,000 b. 1,200,000 d. 2,000,000 e. 21. On January 1, 2011. Wallace Company acquired 100,000 ordinary share of Milestone Company for P5, 000,000. At the time of purchase, Milestone Company had 500,000 outstanding shares with a fair value and book value of P25 million. For the year ended De4cember 31, 2011, the following events took place:  Milestone reported net income of P 1,800,000 for the calendar year 2011.  Wallace received from Milestone a dividend of P2.50 per ordinary share.  Milestone recognized unrealized gains of P600,000 on its investment on available for sale securities as other comprehensive income.  The market value of Milestone Company’s shares had temporarily declined to P45 per share. f. Wallace does have significant influences over Milestone. What is the carrying amount of the investment on December 31, 2011? a. 4,500,000 c. 5,110,000 b. 5,230,000 d. 5,000,000 e. 22. The audit of Benoit Company for the year ended December 31, 2011 was completed on March 1, 2012. The financial statements were signed by the managing director on March 15, 2012 and approve by the shareholders on March 31, 2012. The following events have occurred before March 31, 2012:  On January 15, 2012, a customer owing P900, 000 to Benoit filed for bankruptcy. The financial statements include an allowance for doubt6ful debts pertaining top this customer of P100, 000.  An asset acquired in December 2011 was recorded at a cost of P600, 000. It was determined on February 15, 2012 that the total cost of this asset is P 1,600,000.  Benoit Company discovered errors on March 10, 2012 that showed that the correct cash balance should be P1, 200,000 instead of the previous balance of P1, 000,000.  A noncurrent asset with a carrying amount of P2, 000,000 was classified as held for sale on March 13, 2012. The recoverable amount of the noncurrent asset was determined to be P1, 500,000. f. What is the net increase or net decrease in the total assets to be re4ported by Benoit Company on December 31, 2011 as a result of “events after the reporting period”? a. 100,000 decrease c. 1,100,000 decrease b. 400,000 increase d. 300,000 decrease e. f. 23. On December 1, 2011, Landsman Company purchased for P3, 000,000 a tract of land as a factory site. Landsman razed an old building on the property and used the materials it salvaged from demolition for the new factory. Landsman incurred additional costs and realized salvage proceeds during De4cember 2011 as follows: g. h. Demolition of old building 300,000 i. Legal fees for purchase contract and recording ownership 20,000 j. Filling, grading and levelling 50,000 k. Title guarantee insurance 30,000 l. Salvaged materials 100,000 m. n. What is the total cost of the land? a. 3,300,000 c. 3,400,000 b. 3,250,000 d. 3,350,000 A1 PASSERS REVIEW CENTER///91 a. b. e. e. 24. Kennedy Company reported an impairment loss of P200, 000 in its statement of comprehensive income for the year ended December 31, 2010. This loss was related to noncurrent asset which Kennedy currently used in its operations. On the December 31, 2010, Kennedy reported that the non current asset after impairment at P1, 000,000 and estimated that the noncurrent asset would be still be used for another five years regardless whether the impairment was recognized or not, Kennedy used the straightline depreciation with no residual value. On December 31, 2011, Kennedy determined that the fair value of its impaired noncurrent asset had increased by P50, 000 over its fair value on December 31, 2010. In the 2011 statement of comprehensive income, what amount should be reported as gain on the reversal of the impairment loss? 250,000 c. 50,000 160,000 d. 0 25. Mikhail Company started construction of a building on January 1, 2011 and completed construction on December 31, 2012. Mikhail had two interest bearing notes outstanding during both years and all of the notes were outstanding for all 12months of each year. The following information is available: 8% note (specifically for the project) 2,000,000 10% long term note 8,000,000 2010 average accumulated expenditures 2,800,000 2010 ending balance in construction in progress Before capitalization of interest 3,600,000 2011 expenditures which were incurred evenly 2,000,000 f. g. h. i. j. k. l. m. What is the total cost of the building on December 31, 2011? a. 6,282,000 c. 5,920,000 b. 5,600,000 d. 7,520,000 e. 26. On January 1, 2011, Truman Company acquired equipment to be used in its manufacturing operations. The equipment has an estimate useful life of 8 years and an estimated residual value of P300, 000. The depreciation applicable for this equipment was P900, 000 for 2012 computed under the 200% double declining balance method. What was the acquisition cost of the equipment? a. 3,600,000 c. 4,800,000 b. 4,500,00 d. 5,100,000 e. 27. Luther Company has recently acquired a computer system for its central office in Makati. Luther provided the following information for the new computer: f. List price 1,000,000 g. Trade discount 10% h. Removal of old computer 20,000 i. Concrete slab poured as a base for the new computer 30,000 j. Insurance taken during delivery 20,000 k. Repairs occurred while in transit 10,000 l. Transportation cost 50,000 m. Purchase discount available on purchase (not taken) 5% n. Computer software that is an integral part of the system 95,000 o. p. What is the total amount to be capitalized as cost of the computer system? a. 1,050,000 c. 1,060,000 b. 1,070,000 d. 955,000 e. 29. on March 1, 2011, Daniella Company classified a noncurrent asset as held for sale that had a carrying amount of P1,500,000. On this date, the noncurrent asset has a fair value of P1, 400,000. Estimated disposal cost to be incurred for sale was P50, 000. By December 31, 2011, the asset had not been sold but the sale is still considered to be “highly probable” and management’s intention to sell has not changed. At the same time, Daniellas manager estimated that because of recent changes for the demand of the non current asset and price increases, the noncurrent asset was now expected to be selling for P1,600,000 with the related disposal cost unchanged at P50,000. Depreciation gain from March 1, 2011 to December 21, 2011 was computed at P80, 000. How much gain from the increase in fair value less cost to sell should be reported in 2011? a. 150,000 c. 230,000 b. 250,000 d. 200,000 e. 30. Carson Corporation purchased a new machine on January 1, 2012. A P500, 000 down payments was made and three quarterly instalments of P200, 000 each are to be made beginning on April 1, 2011. The cash price of the new machine would have been P1, 000,000 if paid in full. Carson paid in no installation charge under the quarterly instalment plan but a P20, 000 installation charge would have been incurred with a cash purchase. Delivery of P30, 000 was also incurred with the purchase. What is the amount top be capitalized as the cost of the new machine? a. 1,030,000 c. 1,050,000 b. 1,100,000 d. 1,150,000 A1 PASSERS REVIEW CENTER///92 e. 31. on January 1, 2011, Altidor Company was received a 4 year variable interest rate loan of P4, 000,000 with principal payment at maturity and annual interest payment at the end of each year. The interest rate for 2011 is 10% and the rate in each succeeding year is equal to market interest rate on January 1 of that year. In connection with the loan, Altidor Company entered into an interest rate swap agreements with another financial institution to the effect that Altidor will receive a swap payment if the interest rate on January 1 is more than 10%and will make a swap payment if the interest rate is less than 10%. The swap payments are made at the end of the year on December 31. On January 1, 2012, the market rate of interest is 12% and on January 1, 2013, the market rate of interest is 14%. f. g. What is the derivative asset or liability under the interest swap agreement on December 31, 2012? (Round off PV factor to two decimal places) a. 264,000 asset c. 192,000 asset b. 264,000 liability d. 192,000 liability e. 32. The total debits and credits in selected accounts of Condor Company after closing entries were posted on December 31, 2011 are given below: f. Debits Credits g. Materials 200,000 800,00 h. Goods in process 500,000 700,000 i. Materials purchases 2,000,000 2,000,000 j. Purchase discount 80,000 80,000 k. Transportation in 150,000 150,000 l. Direct labor 1,000,000 1,000,000 m. Manufacturing overhead 5900,000 500,000 n. Finished goods 250,000 700,000 o. p. What was the cost of goods sold for 2011? a. 4,820,000 c. 4,520,000 b. 4,750,000 d. 4,450,000 e. 33. The general ledger balance trial balance of Armani Company included the following accounts on December 31, 2011: f. Inventory including inventory expected in ordinary course of g. operations to be sold beyond 12 months amounting to 70,000 1,000,000 h. Trade receivables 1,200,000 i. Prepayments 50,000 j. Listed investments at “fair value through profit or loss” 200,000 k. Available for sale securities 800,000 l. Cash and cash equivalents 300,000 m. Deferred tax asset 150,000 n. Assets of a disposal group 250,000 o. p. What amount should be reported as total current assets on December 31, 2011? a. 3,000,000 c. 3,550,000 b. 2,750,000 d. 2,500,00 e. 34. Bankrupt Bank has a 5 year loan receivable with a face value of P2, 000,000 dated January 1, 2010 from a borrower that is due on December 31, 2014. Interest on the loan is payable at 10% every December 31. The borrower paid the interest that was due on December 31, 2010 but informed Bankrupt on December 31, 2011 that interest accrued in 2011 and the interest for 2012 will be paid at the maturity date. The borrower will also probably miss the last two years for interest payments because of financial difficulty. After that, the borrower is expected to pay the loan and accrued interest in 2011 and 2012 on December 31, 2011? (Round off present value factor to three decimal places)? a. 97,200 c. 597,600 b. 397,600 d. 197,600 e. 35. Blake Company used the composite method of depreciation based on a composite rate of 10%. At the beginning of 2011, the total cost of Blake’s depre4ciable assets was P2, 500,000 with a total residual value of P300, 000. Accumulated depreciation was p750, 000 on the same date. During 2011, Blake sold assets with an original cost of P500, 000 and a residual value of P60, 000 for P150, 000. No gain or loss was recognized on the sale assets costing P1, 000,000 with a residual value of P120, 000 were also acquired during 2011 to replace assets sold within the year. What is the balance of accumulated depreciation on December 31, 2011? a. 1,050,000 c. 700,000 b. 664,000 d. 850,000 e. 36. For the year ended December 31, 2011. Greenberg Company estimated its allowance for uncollectible accounts using the year-end aging of accounts receivable. The following data for 2011 are available; A1 PASSERS REVIEW CENTER///93 f. g. Allowance for uncollectible accounts. 1/1/11 100,000 h. Provision for uncollectible accounts during 2011(2% on credit sales i. Of P25, 000,000 500,000 j. Uncollectible accounts written off 200,000 k. Recovery of accounts previously written off 120,000 l. Estimated uncollectible accounts per aging 12/31/11 900,000 m. n. How much doubtful accounts expense should be reported for 2011? a. 880,000 c. 180,000 b. 380,000 d. 500,000 e. 37. Precious Company purchased 10%of an investee’s 100,000 outstanding ordinary shares on January 1, 2011 for P500, 000. On December 31, 2011. Precious purchased an additional 20,000 shares of the investee for P1, 500,000. There was no goodwill as a result of either acquisition, and the investee had not issued any additional shares during 2011. The investee reported earnings of P3, 000,000 for 2011. What is the carrying amount of the investment on December 31, 2011? a. 1,700,000 c. 2,300,000 b. 2,000,000 d. 2,900,000 e. 38. During the current year, Aura Company incurred the following costs: f. Research and development services performed by Amor Company for Aura 150,000 g. Design, construction and testing of preproduction prototypes and models 200,000 h. Testing in search for new product or process alternative 175,000 i. What should be reported as research and development expense? a. 150,000 b. 200,000 c. 350,000 d. 525,000 e. 39. Mariel Company has acquired a trademark relating to the introduction of a new manufacturing process. The costs incurred were as follows: f. Cost of trademark 3,500,000 g. Expenditure on promoting the new product 50,000 h. Employee benefits relating to the testing of the power i. Functioning of the new process 200,000 j. k. What is the total cost that should be capitalized as intangible noncurrent asset in respect of the new process? a. 3,750,000 b. 3,700,000 c. 3,500,00 d. 3,550,000 e. 40. In 2010, Vanna Company paid P1, 000,000 to purchase land containing total estimated 160,000 tons of extractable mineral deposits. The estimated value of the property after the mineral has been removed is P200, 000. Extraction activities began in 2011, and by the end of the year, 20,000 tons had been recovered and sold. In 2012, geological studies indicated that the total estimated tons of extractable mineral deposits had been underestimated by 60,000 tons. During 2012, 50,000 tons were extracted and 48,000 tons were sold. What is the depletion included in cost of sales for 2012? a. 175,000 b. 168,000 c. 200,000 d. 192,000 e. f. A1 PASSERS TRAINING, RESEARCH, REVIEW & DEVELOPMENT COMPANY g. 2nd Floor Sommerset Bldg., Lopez Jaena St. Jaro, Iloilo City h. Tel. No.: (033) 320-2728; 09106547262 i. Email Address: [email protected] j. k. BOARD OF CERTIFIED PUBLIC ACCOUNTANT l. m. CERTIFIED PUBLIC ACCOUNTANT Licensure Examination SET B n. o. p. PRACTICAL ACCOUNTING II q. r. s. t. u. INSTRUCTIONS: 1. Detach one (1) answer sheet from the bottom of your Examinee ID/Answer Sheet Set. 2. Write the subject title “PRACTICAL ACCOUNTING II “on the box provided 3. Shade Set Box ‘’A” on your answer sheet if your test booklet is Set A; Set box “B” if your if your test booklet is Set B. v. __________________________________________________________________________________ _____ A1 PASSERS REVIEW CENTER///94 w. MULTIPLE CHOICE: x. y. 1. The following amounts were taken from the statement of affairs for ABC Company: z. aa. Unsecured liabilities with priority P131, 250 ab. Stockholders equity P472, 500 ac. Estimated liquidation expenses that have not been entered ad. In the accounting records 59,060 ae. Unsecured liabilities without priority 1,181,250 af. Loss on realization of assets 590,625 ag. ah. How much is the total free assets? A. P1,004,080 C. 1,063,125 B. P1,021,685 D. 1,135,315 E. F. 2. J and P formed a joint venture to purchase and sell a special type of merchandise. The ventures agreed to contribute cash of P30, 000 each to be used in purchasing the merchandise, and to share profits and losses equally. They also agreed that each shall record his purchases, sales, and expenses in their own books. G. H. Upon termination of the joint venture, the following data are made available: I. J P J. Joint Venture P26,000 CR P23,400 CR K. Inventory Taken 800 2,500 L. Expenses paid from JV Cash 1,200 2,200 M. N. How much cash is to be received by J in the final settlement? A. P52,700 C. P56,350 B. P55,550 D. P55,750 E. F. 3. Partners A, B and C share profits and losses in the ratio of 5:3:2. At the end of a very unprofitable year, they decided to liquidate the firm. The partner’s capital account balances at this time are as follows: A. P616, 000 : B. P697,200 : C.P420,000. The liabilities accumulate to P840, 000, Including a loan of P280, 000 from A. The cash balance is P168, 000. All partners are personally solvent. The partners plan to sell the assets in instalment. G. H. If B received P100, 800 from the first distribution of cash, how much did C receive at that time? A. 56,000 C. 33,600 B. 22,400 D. 61,600 E. F. G. H. I. J. K. 4. Peace Inc. uses the percentage of completion method in recognizing program. In 2010, the company was engaged by World on a fixed price contract to build a 10 storey building. L. M. On January 1, 2011, a fire damaged the accounting records of Peace. The following data were taken from the salvage files: N. 12/31/2010 12/31/2011 O. Architect’s estimated cost of completion 1,875,000 2,000,000 P. Cost incurred 750,000 Q. Percentage of completion 60% R. Income recognized to data 125,000 300,000 S. T. What is the percentage of completion 2010? A. 40% C. 30% B. 25% D. 20% E. 10. During 2009, Canlubang Construction, Inc. started a construction job with a total contract [rice P600,000. The job was completed on December 20, 2010. Additional data are as follows: F. 2009 2010 A1 PASSERS REVIEW CENTER///95 G. Actual costs incurred to data P225, 000 P480, 000 H. Estimated remaining costs 225,000 I. Progress billings 240,000 300,000 J. Collections 200,000 400, 000 K. Under the percentage of completion method. What amount should Canlubang Construction recognizes as gross profit for 2010? A. P 0 C. P 80,000 B. P 45,000 D. P 120,000 E. 11. On July 1, 2010, Mary Ann, Inc. acquired most of the outstanding common stock of Pepito Company for cash. The incomplete working paper elimination entries on that date for the consolidated statement of financial position of Many Ann Inc. and its subsidiary are shown below: F. G. E(1) stockholders equity- Pepito 975,000 H. Investment in Pepito 633,750 I. Non-controlling 341,250 J. E(2) Inventories 25,000 K. Equipment 125,000 L. Patent 24,500 M. Goodwill ? N. Investment in Pepito 633,750 O. Non controlling interest P. Q. Assuming NCI is measured at fair value, what is the amount of goodwill to be reported in the consolidated statement of financial position on July 1, 2010 if there is a control premium of P27, 500? A. P71,654 C. 99,154 B. 128,769 D. 113,962 E. 12. On January 2, 2010, POKEMON sign an agreement to operate as a franchise of DIGIMON for an initial franchise fee of P 10,000,000 for 10 years. Of this amount p2, 000,000 was paid when the agreement was signed and the balance payable in four equally annual payments beginning on December 30, 2010.POKEMON signed a non interest bearing note for the balance. POKEMONS rating indicates that it can borrow money at 24% for a loan of this type. PV of annuity of 1 for 4 periods at 24% is 2.40. Assume that the substantial services amounting to 1,020,000 had already been rendered by DIGIMON and that additional indirect franchise cost of P272, 000 was also incurred. If the collection of the note is not reasonably assured, the realized gross profit for the year ended December 31, 2010 is: A. 5,780,000 C. 2,148,000 B. 2,420,800 D. 5,508,000 E. 13. On December 1, 2009, LOJ. Inc authorized Silber Company to operate as a franchise for an initial franchise fee of P150, 000. Of this amount, P60, 000 was received upon signing the agreement and the balance, represented by a note. Is due in three annually payments of P30,000 each beginning December 31,2010. The present value on December 31, 2009, for three annual payment appropriately discounted a P72, 000. According to the agreement, the non-refundable down payment represents a fair measure of the service already performed by LOJ and substantial future services are still to be rendered. However, collectability of the note reasonably certain. F. G. LOJ’s December 31,2009 balance should report unearned franchise foe from Cloe Company in the amount of A. 132,000 C. 60,000 B. 90,000 D. 72,000 E. 14. On January 1, 2011, Mr. CPA got a franchise of Jollibee. The franchise agreement provides a P500, 000 initial franchise fee, payable P100, 000 upon signing of the franchise contract and the balance in four annual instalments starting Dec 31, 2011. At present value using 12% as discount rate, the four instalments would approximate P199, 650. The fees once paid are not refundable. The franchise may cancelled subject to the provisions of the agreement. Should there be unpaid franchise fees attributed to the balance of the initial franchisor entitled to a 5% continuing fee on gross sales payable monthly within the first ten days of the following month. The first year of operations yielded gross sales of P9 million. The collectability of the note is not assured. F. G. On December 31, 2011. What is the earned franchise fee A. 550,000 C. 650,000 B. 749,650 D. 950,000 E. 15. on January 1, 2011, a acquired a 50% interest in B for P60 million. A already held a 20% interest which had been acquired for P20 million but which was valued at P24 million at January 1, 2011 was 30 million and the fair value of the identifiable net assets of B was P110 million. How much is the goodwill to be recognized as a result of the business combination? A1 PASSERS REVIEW CENTER///96 A. 3,000,000 C. 0 B. 7,000,000 D. 4,000,000 E. 16. On May 1, 2009, JAO builders obtained a contract to builds a building. The building was to built at a total cost of P10, 000,000 and is scheduled for completion on May 2011. The contract contains a penalty clause to the effect that the other part was to deduct of P20, 000 from the contract price for each week of daily. Completion was delayed five weeks. Below are data pertaining to the construction period. F. G. 2009 2010 2011 H. Cost incurred 1,000,000 3,680,000 620,000 I. Estimated cost to complete 4,000,000 520,000 J. Progress billings 800,000 8,700,000 2,400,000 K. L. Using the percentage of completion method, what is the realized gross profit (loss) for the year 2011? A. 650,000 C. (20,000) B. (70,000) D. 480,000 E. 17. Joy is trying to decide whether to accept a bonus of 25% of net income after salaries and a bonus of a salary of P243,750 plus a bonus of 10% of net income after salaries and bonus as a means of allocating profit among the partners. Salaries traceable to the other partners are estimated to be P450, 000. What amount of income would be necessary so that Joy would consider the choice to be equal? A. 2,750,000 C. 2,275,000 B. 3,156,250 D. 2,993,750 E. 18. The following statement of financial position for the peripheral of CC, DD and EE were taken from the books on October 1, 2010. F. G. Assets liabilities and Capital H. Cash 1,000,000 liabilities 2,000,000 I. Other assets 4,000,000 CC, Capital 1,200,000 J. DD, Capital 950,000 K. EE, Capital 850,000 L. Total assets 5,000,000 5,000,000 M. The partners agreed to distribute profits as follows: a. Annual salaries to CC and DD to P50,000 each] b. Annual interest of 5% on beginning capital c. Bonus of 15% to CC based on income after salaries interest and bonus d. Remaining profit: 25% to CC., 35% to DD and 40% to EE N. O. In addition, Company A incurred the following expenditure, costs to issue equity amounting to P900, 000. Professional fees paid to independent values: P 280,000. Professional fees paid to lawyers taking part in the negotiation process - P350, 000. Fees to accountant for processing SEC registration of shares- P60, 000. Fees to advisors and bankers - P230, 000. Since the parties cannot agree on the true value of the Acquired on the data of acquisition. They contractually agreed that additional consideration of P8, 000,000 will be paid in the event the net earnings of Company B. P. Q. The partnership began its operation on October 1, 2010 and net income for the year ended Dec. 31, 2010 is P695, 000. Which of the following is true? A. The bonus to CC is P58,040 B. Net income after salaries, interest and bonus is P 386,960. C. DD’s total share in the net income is P216,875 D. EE’s share on the profit after salaries, interest and bonus is P135,430 R. 19. Angels Corp reported of the following accounts for the year just ended 2011: S. Instalment receivable bag – sales of 2011 P 2,610,000 T. Instalment receivable bag – sales of 2010 1,500,000 U. Deferred gross profit end, 2010 380,000 V. Deferred gross profit end, 2011 840,000 W. Regular sales 1,400,000 X. Cost of regular sales 1, 050,000 Y. Z. The gross profit rate on instalment sales was 10% higher than regular sales. For 2011, the gross profit on instalment sales was 3% lower than in 2010. Total realized gross profit for 2011 is A. P252,700 C. P602,700 B. P286,300 D. P636,300 E. 20. A home office ships inventory to its branch at 125% of cost during 2010. The required balance of the unrealized intercompany profit account is P438, 750 for year 2009. During year 2010, the home office sent A1 PASSERS REVIEW CENTER///97 A. 856,250 merchandise to the branch costing 3,920,000. At the start of C.the 907,750 year 2011, the branch’s balance sheet B. 1,103,750 shows P1, 575,000 of inventory that was acquired from theD.home 1,025,000 office. E. 21. On January 2, 2010, F Corp. sold used equipment for P150, 000 resulting into gain of P45, 000. On that date, G paid P25, 000 cash and signed a P125, 000, 10% interest bearing note. And was payable in three annual instalments of P41, 667 beginning January 2, 2011. If appropriately accounted fort he sale under instalment method. G made a timely payment of the first instalment on January 2, 2011, of P54, 167 which included an interest of P12500. What amount of deferred gross profit should F report at December 31, 2011? A. 28,750 C. 30,000 B. 25,000 D. 37,500 E. 22. Ding, Dong and Dang decided to dissolve their partnership on May 31, 2010. On this date, their capital balances were as follows: F. Ding P87, 500 G. Dong 105,000 H. Dang 35,000 I. J. The following provision for sharing profits and losses is provided in the agreement: K. Income is distributed only as far as it is available. L. Available income is to be distributed in the following sequence: 1. Ding, who is the managing partner gets a salary of P180, 000 a year, the remaining partners gets a salary Of P72, 000 each. 2. Interest is imputed on the average capital balances at 12% per annum M. N. 23. The Ivonne Inc. opened an agency in Sampaloc, Manila in 2008. The following is a summary of the transaction of the agency: O. P. Sales Orders sent to home office 55,000 Q. Sales orders filled by Home Office in 2008 46,500 R. Freight on shipment to agency 1,100 S. Collection, net of 2% discount 39,690 T. Selling expenses paid from the agency working fund 2,820 U. Administration expenses charged to agency 5%sales V. Samples shipped to agency W.Cost 2,000 X. Inventory, Dec 31, 2008 1100 Y. Z. The company maintains its gross margin on agency sales at 30% excluding freight cost on shipment agency. The agency’s net income must be: A. 4,995 C. 5,995 B. 5,390 D. 6,390 E. 24. J, F and K formed a joint venture to purchase3 a piece of lot and to erect an apartment building for sale J is to manage the venture hence, he will receive a bonus of 10% of the venture’s gain before deducting the bonus as an expense. Any remaining gain or loss to be divided equally among the ventures. The venture is completed on august 31, 2010, on this date, the accounts of F and K show the following balances: F. F K G. Account with J 56,000Cr 56,000Cr H. Account with F 112,000Cr I. Account with K 63,000Dr. J. K. There are unused constructions supplies which J agreed to take over at it cost of 147,000. Final settlement with the ventures will require payments as follows: A. J pays K P39,200 and F pays K P49,000 C. J pays f P50,400 and k pays j P107,800 B. J pays K P89,600 and F P50,400 D. J pays F P124,600 and K pays J P50,400 E. 25. ANNA and JANNA are partners sharing profits and losses in the ratio of 6:4 respectively. On January 2, the partners decided to admit HANNAH as a new partner upon her investment of P96, 000. On this date, the interest in partnership of ANNA and JANNA are as follows: ANNA, P138, 000; JANNA, P111, 600. Assuming that the new partner is given as 1`/4 interest in the firm. The agreed capital of the partnership is P360, 000. The admission of a new partner will result to which of the following: A. Goodwill is P20,400 D. Capital balance of ANNA after admission is B. Bonus from JANNA is P2,400 P150, 240 C. Bonus to HANNAH is P6,000 E. 26. On June 1, 2011, the following payments were made by Company A to the former owners and officers of COMPANY B F. Cash paid, for settlement of pre-existing relationship P300, 000 A1 PASSERS REVIEW CENTER///98 G. PN issued, for remuneration for future employee services 1,200,000 H. Cash paid, for reimbursement for paying the I. acquirer’s acquisition costs 800,000 J. K. Also, Company A issued 300,000 shares with par value of P10 per share and market price of P45 per share in exchange for 60% ownership of the outstanding shares of the Company B. The net assets of Company B on this date is P24, 375,000. Company opted to use the partial goodwill method in this transaction. L. M. C. Branch collections of P75, 000 were deposited for the account of the home office on Dec 29, 2010; sixty percent was credited on the home office bank account as of Jan. 9, 2011; and the balance was returned to the deposited marked NSF. N. D. Merchandise shipments of P200, 000 made by the home office on Dec. 28, 2010 were received by the branch on Jan 4, 2011. However only ninety percent of this shipment was charged by the home office to Baguio branch; the balance was debited to other branch. O. P. Compute the unadjusted balance of the home office current account as of Dec. 31, 2010 A. 442,600 C. 242,600 B. 232,600 D. 212,600 E. 29. Prominence Inc. is a contractor for the fabrication of motor service centers. At the end of 2010, the following projects were in progress: F. Contract Price Cost Incurred Est’d Cost to Complete G. Project 1 P 3,800,000 P 2,070,000 P 1,380,000 H. Project 2 9,000,000 5,670,000 2,430,000 I. Project 3 13,250,000 1,800,000 10,200,000 J. During the year 2011, the following costs were incurred: K. Project 1 – P1, 030,000 (estimated cost to complete, P775, 000) L. Project 2 - P2, 580, 000 (project completed) M. Project 3 – P5,700,000 (est’d cost to complete, P5,000,000) Project 4 – P1,500,000 (contract price, P7,200,000; est’d cost to complete, P4,500,000) N. O. Using the zero profit method, the total gross profit to be reported in 2011 would be: A. P-0 C. P750,000 B. P675,000 D. 825,000 E. 30. XYZ Company is engaged in merchandising both at Home office in Manila and Branch in Davao, Selected accounts in the trial balance of the Home Office, and the branch at December 31, 2010 follow : F. Debits Home Office Branch G. Inventory, January 115,000 57,750 H. Branch Current 291,500 I. Purchases 950,000 J. Shipment from Home Office 525,000 K. Freight in from Home Office 27,500 L. Sundry expenses 250,000 125,000 M. N. Credits O. Home Office Current 266,500 P. Sales 775,000 700,000 Q. Shipments to branch 550,000 R. Allowance for overvaluation of branch inventory S. Jan 1 5000 T. U. Additional information: V. Davao receives all its merchandise from the home office. The Home Office bills the goods at cost 10% mark up. At December 31, 2010 a shipment with a billing price of P25, 000 was in transit to the branch. Freight on this shipment was P1, 250 which is to be treated as part of inventory. W. X. December 31, 2010 inventories, excluding the shipment in transit was: Y. Home office, at cost 150,000 Z. Davao branch, at billed value (excluding freight of P2, 600) 52,000 AA. AB.Refer to data above: the true net6 income of the branch: A. 52,350 C. 62,350 B. 57,350 D. 67,350 A1 PASSERS REVIEW CENTER///99 E. 31. Examination of the reciprocal accounts between Manila Home office and Cebu Branch shows the following: F. I. P10, 000 advertising expense of another branch was erroneously charged by the Home Office to Cebu Branch. G. II. Cebu recorded shipments of merchandise from Home Office amounting to P75, 000 twice. H. III. Home office recorded cash transfer of P65, 700 I. IV. Transfer of equipment from Home Office amounting to P53, 000 was not recorded by the branch. J. V. Cebu recorded a debit memo from Home Office of P5, 540 as P5, 450 K. L. How much is the net adjustments to Cebu Branch Current Account and to the Home Office Current Account? M. N. Cebu Branch Current Account Home Office Account A. P75,700 Dr P20,910 Dr B. P75,700 Cr P21,910 Dr C. P75,700 Dr P21,910 Cr D. P65,700 Cr P22,000 Cr O. 32. On April 1, 2011, the R&R Company paid P30, 000,000 to the former stockholders of S&S to acquire 75% ownership interest (representing 135,000 shares outstanding of S&S) in a transaction properly accounted for as acquisition. On this date, the assets and liabilities of S&S Company were as follow: P. Q. Cash 1,200,000 R. Inventories 7,500,000 S. Plant assets 31,000,000 T. Liabilities 13,500,000 U. V. Furthermore, it was determined that the merchandise inventory of S&S Company had a fair market value of P8,250,000 and the plant assets of P25,650,000. What should be the amount reflected as goodwill (gain) by R&R Company in its separate financial statements as a result of the business combination? W. X. The NCI is initially measured at fair value. Quoted price on the date of acquisition of S&S shares is P175,000 per share. A. P0 C. P13,800,000 B. P16,275,000 D. P18,400,000 E. 33. on January 1, 2010,m congratulations, Inc. issues 12,000 shares of its P10 per value stock to acquire the net assets of Successful, Inc. Underlying book value and fair value information for the balance sheet items of successful at the time of acquisition are as follows: F. Book Value Fair value G. Cash P80, 000 P80,000 H. Accounts receivable 120,000 120,000 I. Inventory 60,000 115,000 J. Land 50,000 70,000 K. Building and Equipment 400,000 350,000 L. Less Accumulated depreciation (120,000) M. Total assets 590,000 735,000 N. Accounts Payable P30, 000 P30, 000 O. Bonds Payable P200, 000 P180, 000 P. Common Stock (P5 per value) 150,000 Q. Additional paid-in Capital 70,000 R. Retained Earnings 140,000 S. Total Liabilities and Equities 590,000 T. U. Successful, Inc. share were selling at P18 and Congratulations, Inc. were selling at P50 just before the merger announcement. Additional cash payments made by Congratulations, Inc. in completing the acquisition were: V. W.Broker’s fee paid to firm located Successful P15, 000 X. Engagement fee on agreed upon procedures for share issuance 10,000 Y. Legal fees for the merger 12,000 Z. Cost of SEC registration of Congratulations shares 7,000 AA. AB.What is the amount of additional paid in capital by Congratulations related to the issuance of shares? A1 PASSERS REVIEW CENTER///100 A. P463,000 C. P448,000 B. P463,000 D. 473,000 E. F. 34. Cinco, a partner of Antonio and Danan, decided to withdraw from ACD partnership. Cinco’s share in the profits and losses was 25%, while that Antonio and Danan are 50% and 25%respectively. In the final settlement of his interest, he was paid 95,000, although the capital balance before his retirement was only 85,000. The 10,000 difference implied that an equipment of the partnership was undervalued. Prior to recording Cinco’s withdrawal, and adjustment was made by the partnership to bring the equipment to its fair value. G. H. The total of partners capitals before anyu adjustments and before Cinco’s withdrawal was 340,000. What would be the partnership’s net assets after the withdrawal of Cinco? A. 285,000 C. 295,000 B. 245,000 D. 325,000 E. F. 35. The following data were taken from the statement of realization and liquidation of ABC Corp. For the G. H. quarter ended June 30, 2010 I. Assets to be realized P160, 000 J. Supplementary credits 220,000 K. Liabilities to be liquidated 160,000 L. Supplementary charges 196,000 M. Liabilities liquidated 60,000 N. Assets acquired 200,000 O. Assets realized 80,000 P. Liabilities assumed 100,000 Q. Assets not realized 180,000 R. S. The ending capital balances of capital stock and retained earnings are P200, 000 and P36, 000, respectively. How much is the ending balance of cash? A. P116,000 B. P70,000 C. P116,000 D. P90,000 T. U. V. W. X. Y. Z. AA. AB. AC. AD. AE. AF. AG. AH. AI. AJ. AK. AL. AM. AN. ANSWER KEY AO. ACCOUNTING II AP. A1 PASSERS REVIEW CENTER///101 1. B 2. B 3. B 4. D 5. D 6. A 7. D 8. C 9. B 10. D 11. C 12. B 13. D 14. A 15. B 16. D 17. B 18. C 19. B 20. B 21. B 22. B 23. C 24. D 25. D 26. D 27. C 28. A 29. A 30. C AQ. AR. AS. AT. AU. AV. AW. AX. AY. AZ. BA. BB. BC. BD. BE. BF. BG. BH. BI. BL. BJ. BK. A1 PASSERS TRAINING, RESEARCH, REVIEW & DEVELOPMENT COMPANY BM. 2nd Floor Sommerset Bldg., Lopez Jaena St. Jaro, Iloilo City BN. Tel. No.: (033) 320-2728; 09106547262 BO. Email Address: [email protected] BP. A1 PASSERS REVIEW CENTER///102 BQ. BOARD OF CERTIFIED PUBLIC ACCOUNTANT BR. BS.CERTIFIED PUBLIC ACCOUNTANT Licensure Examination SET A BT. BU. BV. THEORY OF ACCOUNTANT BW. BX. INSTRUCTIONS: BY. BZ.1. Detach one (1) answer sheet from the bottom of your Examinee ID/Answer Sheet Set. CA. 2. Write the subject title “PRACTICAL ACCOUNTING “on the box provided CB. 3. Shade Set Box ‘’A” on your answer sheet if your test booklet is Set A; Set box “B” if your if your test booklet is Set B. CC. CD. MULTIPLE CHOICES: CE. CF.1. Which one of the following enhances the relevance of accounting information? a. Monetary unit c. Neutrality b. Understandibility d. Predictive e. 2. The principle of objectivity includes the concept of a. Classification c. Verifiability b. Conservation d. Summarization e. 3. Using the gross profit method for inventory rather than physical count at interim periods is a trade off between a. Neutrality and consistency c. Reliability and prudence b. Timeline and verifiability d. Objectivity and materiality e. 4. Which of the following is synonymous to realizable value? a. Past purchase exchange price c. Current purchase release exchange price b. Current sale exchange price d. Future exchange price e. 5. The recording of doubtful accounts expense is an example of what expense recognition principle? a. Systematic and rational location c. Cause and effect b. Immediate recognition d. Partial recognition e. 6. A user computes Times Interest Earned (TIE) Ratio based on an entity’s financial statement is most likely which of the following? a. Lender c. Investor b. Borrower d. Customer e. 7. When special journal are used, accounts written of shall be recorded in the a. Sales journal c. General journal b. Cash disbursements journal d. Purchase journal e. 8. Which of the following is an example of a nominal and contra account? a. Freight out c. Allowance for sales discount b. Freight in d. Purchase discount e. 9. The manufacturing summary account summarizes. a. All revenues, expenses, gains and losses b. All accounts that enter into computing cost of goods manufactured c. All accounts that enter into computing cost of goods sold d. All accounts that enter into computing total manufacturing costs f. 10. All of the following are classified as current assets, except a. Cash set aside for payroll b. Financial assets at fair value through profit or loss c. A deposit on merchandise ordered, to be delivered in 15 months d. Trade receivable due in 18 months g. 11. A quoted debt instrument that an issuer may buy back in the near term depending on changes in fair value is classifies as a a. Current asset c. Noncurrent asset b. Current liability d. Noncurrent liability e. f. g. h. 12. All of the following are in minimum line items for current liabilities, except a. Trade and other payables c. Current provisions b. Accounts payables d. Current tax liability’ A1 PASSERS REVIEW CENTER///103 e. a. b. e. a. b. c. d. f. a. b. c. d. g. a. b. c. d. h. a. b. c. d. i. a. b. e. a. b. c. d. f. a. b. c. d. g. a. b. c. d. h. a. b. c. d. i. a. b. e. a. b. c. d. f. 13. All of the following are considered nondistributable equity reserves, except Share premium reserve c. Foreign translation credit adjustment Revaluation surplus d. Unappropriated retained earnings 14. The summary of significant accounting policies shall disclose The composition of property, plant and equipment and the depreciation method used The composition of property, plant and equipment only The depreciation method used only None of the above 15. All of the following will require related party disclosure in the financial statements of an entity, except Sold goods to another entity owned by the son of the entity’s chief director Sold goods to the aunt of the entity’s chief director Sold goods to shareholders of the entity who owns 10% of the ordinary shares but is a member of the board of directors. Sold a fully depreciated car to an associate 16. Between the date of the financial statements and the date these statements were authorized for issue, the following events took place. All of the following are considered non adjusting events, except. Major ordinary share transactions Issuing guarantees Significant decline in the market value of available for sale securities Determination of bonus payment since entity has already the present obligation at the reporting date. 17. Between the date of the financial statements and the date these statements were authorized for issue. The following events took place. All of the following are considered adjusting events, except: Resolution of a court case Plan to discontinue an operation Sale of finished goods that may give evidence about the net realizable value at the reporting date. Determination of proceeds from asset sold before the reporting period. 18. This form of an income statement is also known as the “cost of sales method” Natural form c. Account form Functional form d. Report form 19. An entity’s profit or loss differs from its total comprehensive income. Which of the following may be the reasons? Dividends received Dividends paid Foreign currency translation adjustment Impairment loss on noncurrent asset held for sale 20. Which of the following would not result to reclassification adjustment in the statement of comprehensive income? Permanent decline in market value of available for sale securities Ineffective portion of a cash flow hedge instrument Realization of evaluation surplus Gain or loss from disposal of available for sale securities 21. A statement of changes in equity will show all of the following, except Total comprehensive income Changes in accounting policies treated retrospectively Dividends received Contributions by shareholders 22. How is impairment loss treated if it relates to a disposal group? It is allocated to the assets based on their carrying amounts The impairment loss is first allocated to goodwill, if any and any remainder is allocated to the other assets based on their carrying amounts. It is deferred until the group is actually disposed of. It is not recognized since it violates prudence. 23. An entity classified a noncurrent asset as held for sale in accordance with PFRS 5 on October 1, 2011. The year end is December 31, 2011. On July 1, 2012, some of the criteria as held for sale were not met, thus the asset can no longer be classified as held for sale. The entity will adjust for depreciation in 2012 for 7 months c. 9 months 8 months d. No adjustment for depreciation 24. According to PFRS 5, which of the following is true? An extension of the period required to complete a sale precludes an asset being classified as held for sale if the delay is caused by fortuitous events. Subsequent increase in fair value less cost to sell may be recognized in excess of the cumulative impairment loss that was previously recognized. Immediately before the initial classifications of the assets as held for sale, the carrying amount of the asset shall be measured in accordance with PFRS 5. An entity can classify a noncurrent asset that is to be abandoned as held for sale. A1 PASSERS REVIEW CENTER///104 g. h. i. 25. Which of the following is true regarding the discontinued operations? a. A component of an entity always represents the same concept as an operating segment used in reporting disaggregated information. b. Discontinued operation shall show gross income on the face of the income statement c. Impairment loss is recognized at year end if the disposal group is unsold and if fair value less cost to sell is below the carrying amount. d. The result of discontinued operation excludes income or loss from operating the discontinued operations. j. 26. Which of the following is not treated as change in accounting estimate? a. Doubtful accounts c. Inventory obsolescence b. Initial adoption to carry assets at revalue d. Warranty cost amount e. 27. Which of the following is not a justification for a change in depreciation method? a. Change in estimated useful life b. Change in pattern of receiving estimated future benefits from asset c. Change in manage intent d. Change in estimated future benefits from the asset f. 28. Which of the following accounting treatment is proper for a change in reporting entity? a. Restatement of all financial statements presented b. Restatement of current period financial statements c. Note disclosure and supplementary schedules d. Adjustment to retained earnings and note disclosure g. 29. Which of the following is correct? a. Correction of an inappropriate accounting policy is treated as a change in accounting policy b. If it is impracticable to apply a change in accounting policy retrospectively, the fact should not be disclosed in the notes to financial statements. c. A change in reporting entity is correction of an error. d. Disposition of a business unit is not a change in reporting entity h. 30. Which of the following cannot be allocated over the interim periods benefited? a. Depreciation c. Inventory write-down b. Property tax d. Insurance premium e. 31. An entity follows the fiscal year beginning July 1. It also prepares quarterly financial statements for intern reporting purposes. For the third quarter ended and for comparative purposes, the entity shall, among the others prepare a statement of cash flows for a. The 3 months ended March 31 and for the 9 months ended March 31 both for previous and current year. b. The immediately preceding year and for the 3 months ended March 31 of the current year. c. The immediately preceding year and for the 9 months ended March 31 of the current year. d. The 9 months ended March 31 both for previous and current fiscal year. f. 32. An entity is preparing its financial statement for the first half of its financial year ending June 30, 2011. One class of inventory has a cost per unit of P1000 and a net realizable value at June 30, 2011 of P1, 200 per unit. The net realizable value at year end is expected to be P900 per unit. The entity’s budget for the year scheduled a major refurbishment project from April to June 2011. For legal reasons, the contract was not signed until July 15, 2011on which date the work started. Which of the following statements is true? g. I. Inventory shall be carried at its net realizable value at June 30, 2011. h. II. The cost of the project shall be accrued on June 30, 2011. a. I only c. Both I and II b. II only d. Neither I nor II e. 33. Which of the following are true regarding operating segments? f. I. Corporate headquarters may be operating segments g. II. Postemployment benefit plans may be operating segments h. III. Start up operations may not be operating segments a. I and II c. III only b. I,II and III d. All statements are not true e. 34. What is the practical limit to the reportable segments? a. Six b. Ten c. No precise limit, but if the number exceed six, the entity shall consider if a practical limit has been reached. d. No precise limit, but if the number exceeds ten, the entity shall consider if a practical limit has been reached. f. 35. All of the following shall be disclosed as segment assets for each reportable segment, except a. Accounts receivable c. Deferred tax assets b. Inventory d. Trademark e. 36. Which of the following is true? a. Segment revenue and segment expense are defined in PFRS 8. b. A major customer’s revenue is 10% or more of the combined revenue of all segments. c. The chief operating decision maker is usually the chief executive officer. A1 PASSERS REVIEW CENTER///105 d. f. g. h. i. a. b. e. a. b. c. d. f. a. b. c. d. g. a. b. c. d. h. a. b. e. a. b. c. d. f. a. b. c. d. g. a. b. c. d. h. a. b. c. d. i. j. k. a. b. e. a. b. e. a. b. e. a. All reportable segments are operating segments. 37. All of the following are considered cash, except Bills and coins c. Stale check Bank draft d. Money order 38. Which of the following is true? The practice of opening books of accounts beyond the close of accounting period is called kiting. Misappropriation of collections from customers is called window dressing. Lapping is the transfer of cash from one bank to another bank to conceal a shortage. Sinking fund can be classified as a current asset under certain circumstances. 39. Which of the following will not require an adjusting entry on the depositor’s books? DAIF check from customer Collection from a customer amounting to P90, 000 recorded by depositor as P900,000. Deposit of another entity credited by the bank to the account of the depositor Loans proceeds credited to the depositor’s account 40. A Cash Over and Short account Is not generally accepted Is debited when the petty cash fund proves our over. Is debited when the petty cash fund proves our short. Is a contra cash account 41. Trade accounts receivable are subsequently measured at Present value c. Face value Amortized cost d. Future value 42. Which of the following is true? Origination fees received from the borrower will increase interest income. Direct origination cost incurred will increase interest income. Indirect origination cost incurred is included in the carrying amount of the loan receivable. Loan receivable is subsequently measured at amortized cost using straight line. 43. In computing in the present value of the loan receivable to test for impairment, the discount rate is the Prevailing market rate of similar loans Original effective rate of the loan New interest rate from agreement between the debtor and creditor Weighted average cost of capital 44. Equity in assigned accounts of an entity is the difference between Accounts receivable-Unassigned and accounts receivable-assigned Accounts receivable- assigned and outstanding principal of bank loan Accounts receivable- unassigned and outstanding of principal of bank loan Accounts receivable- assigned and accounts receivable pledged 45. Which of the following is true? Accounts receivable hypothecated will decrease total current assets Accounts receivable assigned will increase total cure4nt assets Notes receivable discounted with recourse will increase with total current assets. Factors holdback should be included in the seller’s total receivables 46. On November 1 of the current year, an entity received a one year note receivable bearing interest which is below the market rate. When the note receivable is recorded on November 1. Which of the following of the following will be debited? I. Interest receivable II. Discount on note receivable I only c. Both I and II II only d. Neither I nor II 47. After being held for 30 days, a 90-day 10% interest bearing note was discounted at a bank at 12%. Discount will based on. 30 days at 12% c. 30days at 10% 60 days at 12% d. 60 days at 10% 48. All of the following will be included in the entity’s inventory, except Goods in transit purchased free alongside c. Goods in transit sold ex-ship Goods in transit purchased CIF d. Goods held on consignment 49. Goods shipped FOB Buyer that are in transit at the end of the year should be included in the financial statements of the Buyer b. Seller A1 PASSERS REVIEW CENTER///106 c. e. a. b. c. d. f. g. h. i. j. a. b. c. d. k. a. b. e. a. b. e. a. b. c. d. f. a. b. e. a. b. c. d. f. a. b. c. d. g. Agent d. Common carrier 50. An entry debiting accounts payable and crediting inventory would be made when Merchandise is acquired and the periodic method is used Merchandise is returned and the periodic method is used Merchandise is acquired and the perpetual method is used Merchandise is returned and the perpetual method is used 51. Which of the following is true regarding the allowance method for recording inventory write-down? Inventory is measured at cost. If the allowance at thebe and of the year exceeds the allowance at the beginning of the year, the difference will decrease cost of sales. If the allowance at the beginning of the year exceeds the allowance at the end of the year, the difference will not be recognized. The presence of an allowance account indicates that cost exceeds net realizable value of inventory at year-end. 52. Which of the following would cause a decrease in the cost ratio used in the retail inventory method? Higher retail prices c. More employee discounts Lower net mark-ups d. Higher freight in costs 53. All of the following are products after harvest, except Leaf c. Lumber Tea d. Cured tobacco 54. Which of the following is not within the scope of PAS 41 agriculture? Accounting for a herd cattle Initial measurement of wool from sheep Ripening of mangoes for commercial sale Government grant received in respect of trees in plantation forest? 55. Which of the following should not be considered financial asset? Held to maturity securities c. Investment associate Leased assets d. Bank account 56. Which of the following is true? Interchange of management personnel is not an indicator of significance influence. The existence of convertible bonds may be considered whether significant influence is present. Under the equity method, the investor and investee are viewed as two separate legal units. Investment preference shares may have significant influence 57. An associate recognized revaluation surplus on its equipment on it’s in the current year. How will it affect the investment in associate balance? No effect It will increase the investment with a corresponding increase in investment income. It will increase the investment balance with a corresponding increase in the investor’s equity. It will decrease in investment balance. 58. An investor uses the equity method. On the date of purchase, the investee’s FIFO inventory and land fair values exceeded their carrying amounts. How will these excesses affect investor’s share in the investee’s net income? h. i. Inventory excess Land excess a. Decrease Decrease b. Decrease No effect c. Increase Increase d. Increase No effect j. 59. Bonds usually sell at a discount when investors are willing to invest in the bonds a. At the coupon interest rate c. At rate higher than the stated interest rate b. At rate lower than the stated interest rate d. When the need arises e. 60. Which of the following is not considered investment priority? a. Land to be leased out under the finance lease b. Building held for undetermined use c. Building being constructed for use as investment property d. Land held for capital gains f. 61. Which of the following is not a “notional” of a derivative contract? a. Principal amount of bank loan c. Total amount of foreign currency b. Number of kilos d. Commodity price A1 PASSERS REVIEW CENTER///107 e. a. b. e. 62. Uncertainty about the future market value of an asset is referred to as Price risk c. Interest rate risk Credit risk d. Exchange rate risk 63. An entity acquired a tract of land by issuing bonds by payable. Both the fair value of the land and the fair value of the bonds can be determined reliably. What will be the initial measurement of the land? a. Fair value of the land c. Par value of the bond b. Fair value of the bonds d. Assed value of the land e. f. g. h. i. j. 64. If an asset was acquired in the exchange and there is cash involved, how will the recipient of cash compute the cost of the new asset received? a. Fair value of the asset given plus cash received b. Fair value of the asset received plus cash received c. Fair value of the asset given minus cash received d. Carrying amount of the asset given minus cash received k. 65. Which of the following is considered government assistance? a. Improvement in communication network b. Imposition on trading constraints on competitors c. Free marketing advice d. Improvement in parks and other recreational areas l. 66. Which of the following is false concerning government grant? a. Nonmonetary government grant is recorded at fair value. b. Grants related to depreciable assets must recognize as an income over the useful life of the asset using the straight line. c. Grants that become repayable because of violations are treated as changes in accounting estimate. d. Grants in recognition of specific costs are recognized as income over the same period as the relevant expense. m. 67. Which of the following is not capitalized as part of the cost of a building? a. Architect fee b. Renovation costs prior use c. Property tax incurred after date of acquisition d. Cost of temporary housing for construction workers n. 68. If a qualifying asset is financed by both specific and general borrowing capitalizable borrowing cost include a. Actual borrowing cost on specific borrowing minus any investment income from temporary investment of proceeds from borrowing b. Average expenditures of the asset multiplied by the average capitalization rate c. Actual borrowing cost on specific borrowing d. Actual borrowing cost on general borrowing minus investment income from temporary investment of borrowing? o. 69. Revaluation surplus is based on p. I. fair value q. II. Replacement cost in the absence of fair value a. I only c. Both I and II b. II only d. Neither I nor II e. 70. Revaluation surplus that is realized because of the use of the asset or its disposal may be transferred to a. Retained earnings c. Other comprehensive income b. Share premium d. Profit or loss e. 71. A purchased patent with a remaining legal life of 15 years should be a. Amortized over 15 years b. Amortized over its useful life if shorter than 15 years c. Amortized over its useful life when longer than 15 years d. Not amortized but tested for impairment annually. f. 72. The term “betterment “refers to a. An expenditure made for new facilities which increase “capacity” b. An expenditure made to restore “capacity “after abandonment or retirement c. An expenditure made to improve existing facilities by increasing “capacity” d. An expenditure made to help insure continuity of service capacity. g. 73. Which of the following expenditure may properly be capitalized? A1 PASSERS REVIEW CENTER///108 a. Expenditure for massive advertising campaign. b. Insurance on plant during construction c. Research and development related to long term asset which is giving the entity a competitive market advantage. d. The search and other legal cost related to a piece of property which was not acquired. h. 74. In which of the following depreciation method is residual value not a factor in determining depreciation charge in the early years of the assets life?’ a. Straight line c. Productive output b. Service hours d. Declining balance e. 75. An addition that is an integral part of an older asset normally would be depreciated over a. The useful life of the addition b. The useful life of the addition or the original asset, whichever is shorter c. The useful life of original asset d. Either the useful life of addition or the original asset, as a matter of professional judgement f. g. h. i. 76. Information needs to compute a depletion charge per unit includes the a. Estimated of total amount of resources available for removal b. Amount of resources removed during this period c. Cumulative amount of resources removed d. Amount of resources sold during this period. j. 77. Which of the following expenditures would never qualify as an exploration and evaluation asset? a. Expenditure for acquisition of rights to explore b. Expenditure for exploratory drilling c. Expenditures related to the development of mineral resources d. Expenditures for activities in relation to evaluating the technical feasibility and commercial viability of extracting mineral resource. k. 78. The amortization method used for an intangible asset shall reflect the pattern in which the assets economic benefits are consumed by entity, if such pattern cannot be determined reliably. What is the amortization method used? a. Straight line b. Production method c. Diminishing balance method d. Ratio of current year’s sales to the total expected sales l. 79. Which is not a considered a research and development activity? a. Routine ongoing efforts to refine, enrich or improve quality of existing product. b. Laboratory research aimed at discovery of new knowledge. c. Conceptual formulation and design of possible product or processes. d. Design, construction and operation of a pilot plant. m. 80. At the beginning of the current year, an entity purchased equipment for use in developing a new product. The entity uses the straight line depreciation method. The equipment could provide benefits over a 10 year period. However, the new product developments expected to take five years, and the equipment can be used only for this project. The entity’s expense for the current year equals a. The total cost of the equipment. b. One - fifth of the cost of the equipment c. One - tenth of the cost of the equipment d. Zero n. o. p. q. r. s. t. u. v. w. x. y. z. aa. ab. A1 PASSERS REVIEW CENTER///109 ac. ad. ae. af. ag. ah. ai. aj. ak. al. am. an. ao. ap. aq. ar. as. at. au. av. A1 PASSERS TRAINING, RESEARCH, REVIEW & DEVELOPMENT COMPANY aw. 2nd Floor Sommerset Bldg., Lopez Jaena St. Jaro, Iloilo City ax. Tel. No.: (033) 320-2728; 09106547262 ay. Email Address: [email protected] az. ba. BOARD OF CERTIFIED PUBLIC ACCOUNTANT bb. bc. CERTIFIED PUBLIC ACCOUNTANT Licensure Examination SET A bd. be. THEORY OF ACCOUNTS bf. bg. INSTRUCTION: Select the correct answer for each of the following questions. Mark only one answer for each item by shading the box corresponding to the letter of your choice on the answer sheet provided. STRICTLY NO ERASURES ALLOWED. Use the pencil no. 1 only. bh. __________________________________________________________________________________ ______ bi. bj.MULTIPLE CHIOCE: bk. bl. 1. Special journals help most by A. Easing the preparation of the financial statements B. Limiting the number of transactions that have to be recorded C. Reducing the cost of operating this accounting system D. Improving accuracy in posting to subsidiary ledgers bm. 2. The key to good internal control of payroll is A. Using a payroll bank account C. Separating payroll duties B. Using time cards D. Using a payroll register E. 3. Achiever Corporation owns 50 of the outstanding capital stock of Door Farms, Inc. last year, the profits of Achiever from the transactions with Doer amounted to P140, 000. The elimination for intercompany profit is A. P14,000 C. P70,000 B. None D. P140,000 E. 4. When translating an amount for fixed assets shown on the statement of financial position of a foreign subsidiary, the appropriate rate of translation is the A. Average exchange rate for the current year B. Current exchange rate C. Average exchange rate over the life of each fixed asset D. Historical exchange rate F. 5. Which of the following is the appropriate basis for valuing fixed assets acquired in a business combination accounted for as a purchase carried out by exchanging cash for common stock? A. Cost plus any excess of purchase price over book value of asset acquired B. Historical cost A1 PASSERS REVIEW CENTER///110 C. Fair value D. Book value G. 6. Balancing the ledger at the end of the period is most closely related to A. Control C. Flexibility B. Favorable cost? benefit relationship D. compatibility E. 7. How about gain or loss from the disposal of a business enterprise be shown on the financial statements? A. A separate item before income taxes C. A separate item after continuing operations B. As extraordinary item D. A retained earning adjustment E. 8. It denotes the responsibility to others that one or more persons have for their actions and behaviour. A. Balanced budget C. Accountability B. All of these D. Obligations E. 9. Which of the following is the transaction between related parties that commonly occur in the normal course of business? A. Services received or furnished, such as accounting, management, engineering, and legal services B. Use of property and equipment by lease or otherwise, and etc. C. All of these D. Sales, purchases and transfer of realty and personal property F. 10. Financial accounting is said to be concerned with A. Analyzing the financial reports B. Summarizing and presenting financial information on reports C. All of these D. Recording financial transaction G. H. I. J. K. 11. The acquisition of some or all the stock held by minority stockholders of subsidiary ---whether acquired by the parent, the subsidiary itself, or another affiliate shall be accounted for by the A. Pooling of interest method B. None of these C. Purchase method D. Purchase method rather than by the pooling of interest method L. 12. Which of the following is a disadvantage of the corporate form of business organization? A. No mutual agency C. Limited liability of stockholders B. Transferability of ownership D. Government regulation E. 13. Accrual – basis accounting A. Omits adjusting at the end of the period B. Leads to the reporting of more complete information than does cash basis accounting. C. Is not acceptable under GAAP D. Results in higher income than cash basis accounting F. 14. Depletion is computed in same manner as which depreciation method? A. Unit of production C. Sum of years digit B. Double declining balance D. Straight line E. 15. Operating income using direct costing as compared to absorption costing would be higher A. Under no circumstances B. When the quantity of beginning inventory is more than the quality of ending inventory C. When the quantity of beginning inventory equals the quantity of ending inventory D. When the quantity of beginning inventory is less than the quantity of ending inventory F. 16 The ratio at which the currencies of two counties are exchanged at a particular time. A. Foreign currency C. Closing rate B. Foreign entity D. Exchange rate E. 17. The possession of direct and indirect power to direct or cause the direction of the management and policies of an enterprise through ownership by contract or otherwise. A. Principal owners C. Control B. Related parties D. Immediate family E. 18. Entries in the purchases journal are posted to the A. Accounts receivable ledger and the accounts payable ledger B. General ledger only C. General ledger and the accounts payable ledger D. General ledger and the accounts receivable ledger F. 19. Cost of goods manufactured is used to compute A. Manufacturing overhead applied C. Cost of gold sold B. Finished goods inventory A1 PASSERS REVIEW CENTER///111 D. 20. Net income is P17, 000, depreciation is P9, 000, and amortization is P3, 000. In addition, the sale of a plant asset generated a P4, 000 gain. Current assets other than cash increased P6, 000 and current liabilities increased by P8, 000. What was the amount of cash flow from operations? A. P27,000 C. P35,000 B. P31,000 D. P23,000 E. 21. Rearrange the following accounts in their logical sequence in the ledger F. Notes payable Cash G. Accounts receivable Jun Cruz, Capital H. Sales revenue Salary expense I. A. Cash, Accounts Receivable, Jun Cruz, Capital, Notes Payable Sales Revenue, Salary Expense B. Accounts Receivable, Notes Payable, Cash, Jun Cruz, Capital Sales Revenue, Salary Expense C. Notes Payable, Accounts Receivable, Cash, Sales Revenue Jun Cruz, Capital, Salary Expense D. Cash, Accounts Receivable, Notes Payable, Jun Cruz, Capital Sales Revenue and Salary Expense J. 22. Value engineering is used to reduce costs from A. Initial budgeted cost to allowable cost C. Life- cycle sale price to allowable cost B. Initial budgeted cost to target cost D. Target cost to allowable cost E. 23. The purpose of the statement of cash flows is A. All of these B. Determine the ability to pay dividends and interest C. Evaluate management decisions D. Predict future cash flows F. 24. Which is the most widely held class of stock? A. Par value preferred stock C. No par preferred stock B. No par common stock D. Par value common stock E. 25. Dividends paid to stockholders are reported on the statement of cash flows as a(an) A. Combination of the above C. Operating activity B. Financing activity D. Investing activity E. F. G. H. 26. Which account causes the main difference between merchandisers adjusting and closing process and that of as service business? A. Interest revenue C. Accounts receivable B. Cost of goods sold D. Advertising expense E. 27. Ten-year, 11 percents bonds payable of P500, 000 were issued for P532, 000. Assume that the straight – line amortization method is appropriate. The total annual interest expense on these bonds is A. P58,200 B. P51,800 C. P55,000 D. A different amount each year because the bonds book value decreases as the premium amortized. F. 28. Why does a pharmacy need a different kind of accounting system than a physician uses? A. They have different kinds of journals and ledgers B. They have different kinds of business transactions C. They have different kinds of employees D. They work different hours G. 29. A plan or program of activities together with costs of undertaking them to meet goals or targets which emphasize on expected results A. Obligations C. Performance budget B. Supplemental budget D. All of these E. 30. Owners of record of known beneficial owners of 10 percent or more of the voting interest of the enterprise A. None of these C. Principal owners B. Immediate family D. Related parties E. 31. Which of the following is the period cost? A. Selling expense C. Materials inventory B. Manufacturing overhead D. Direct labor E. 32. It is generally referred to as a financial plan for carrying out activities in the future A. Budget C. Government budget B. None of these D. Government accounting E. 33. For which of the following products is a process costing system most appropriate? A. Houses C. Breakfast cereal B. Automobiles D. Furniture E. 34. Which of the following items is a nonmonetary asset? A. Merchandise inventory C. Cash B. Sinking fund-unremitted reach D. Investment in bonds A1 PASSERS REVIEW CENTER///112 E. 35. Cash is P10, 000, net accounts receivable amount to P22,000 inventory is P55,000, prepaid expenses total P3, 000, and current liabilities are P40, 000. What is the acid test ratio? A. 2.25 C. 0.80 B. 0.25 D. 2.18 E. 36. In the absence of discontinued operations and changes in accounting principles, the following main captions should appear in the income statement, if extraordinary items are reported A. Income before extraordinary items less extraordinary items B. Income before extraordinary items F. Extraordinary items (less applicable income tax of P) (Note) G. C. None of these H. D. Income aft6er extra ordinary items less Applicable income taxes of P (Note) I. 37. Statements which present the assets, liabilities, stockholders equity, revenues, and expenses of a parent company and its subsidiaries as if the group were a single enterprise A. Balance sheet and income statement C. Balance sheet B. Consolidated financial statements D. Income statement E. 38. The amount credited for a receipt of cash on account is A. Accounts payable C. Cash B. Service revenue D. Accounts receivable E. 39. Sales total P440,000, cost of goods sold is P210,000, and operating expenses are P 160,000. How much is gross margin? A. P440,000 C. 70,000 B. P230,000 D. 210,000 E. 40. It is the systematic recording, classifying, summarizing, governmental transactions in terms of the money and other resources consistent with accounting and budgetary laws. A. Local government C. Government budgeting B. National government D. Government accounting E. 41. The credit balance in Allowance for Uncollectible is P14, 300 prior to the adjusting entries at the end of the period. The aging of accounts indicates that an allowance of P78, 900 is needed. The amount of expense to record is. A. P93,200 B. P14,300 C. P78,900 D. P64,600 F. G. H. I. J. 42. A partnership records a partner’s investment of assets in the business at A. The market values of the assets invested B. A special value set by the partners C. The partner’s book value of the assets invested D. Any of the above, depending upon the partnership agreement K. 43. The functions and activities necessary for the performance of major purpose of which a government entity is established A. Resources C. Appropriation B. Program D. Performance budget E. 44. The consistency principle has most direct impact on A. Whether to use the periodic of the perpetual system B. Whether to include or exclude an item in inventory C. Whether to write inventory down to a market value below cost D. Whether to change from one inventory method to another F. 45. Which of the following items is not part of the cash inflows from financing activities? A. Proceeds from issuing equity investments B. Proceed from issuing bonds, mortgages and notes C. Repayment of amount borrowed D. Proceeds from other short-term or long-term borrowing G. 46. Entries prepared, as a step in accounting process, to bring the books and accounts up to date, is known as: A. Closing entries C. Opening entries B. Adjusting entries D. Reversing entries E. 47. The information provided in the statement of cash flows, if used with related disclosures and information in the other financial statements, should help investors, creditors, and others to A. Assess the enterprise’s ability to generate positive future net cash flows B. All of these C. Assess the enterprises’ ability to pay its dividends, and its need for financing D. Assess the reasons for differences between net income and associated cash receipts and payments A1 PASSERS REVIEW CENTER///113 F. 48. A machine with a ten year useful life is being depreciated on a straight-line basis for financial statement purposes, and over five years for income tax purposes under the accelerated cost recovery system. Assuming that the company is profitable and that there are and have been no other timing differences, the related deferred income taxes would be reported on the balance sheet at the end pf the first year of the estimated useful life as a A. Current liability C. Current asset B. Noncurrent asset D. Noncurrent liability E. 49. A company paid P450,000 for a building and depreciated it by the straight line method over a 40 year life with estimated residual value of P50,000. After 10 years, it became evident that the building’s remaining useful life would be 40 years. Depreciation for the eleventh year is A. P7,500 C. P8,750 B. P12,500 D. P10,000 E. 50. Beginning inventory was P35,000, purchases were P`146,000, and sales totalled P240,000. With normal gross margin of 35 percent, how much is ending inventory. A. P 181,000 C. P 35,000 B. P 25,000 D. P 97,000 E. 51. Which of the following is a nonmonetary asset? A. Due from employees C. Cash B. Merchandise inventory D. Accounts receivable E. 52. An advantage of using the payback method of evaluating capital budgeting alternative is that payback is A. Insensitive to the life of the projected considered B. Precise in estimates of profitability C. Based on noncash flow data D. Easy to apply F. 53. The account number 211031 most likely refers to A. Accounts payable C. Liabilities B. An individual vendor D. Current liabilities E. 54. The work sheet is a A. Financial statement F. Convenient device for completing the accounting cycle G. B. Ledger H. C. Convenient device for completing the accounting cycle I. D. Journal J. 55. Adjusting entries K. A. Help to properly measure the period’s net income or net loss L. B. All of these M. C. Bring asset and liability accounts to correct balances N. D. assign revenues to the period in which they are earned O. P. Q. R. 56. Which of the following quantitative techniques is used to determine the fixed and variable elements of a semi variable cost? a. Queuing theory c. Linear programming b. Simple method d. Least squares e. 57. In a make or buy decision a. Only conversion costs are relevant b. Only variable costs are relevant c. Fixed costs that can be avoided in the future are relevant d. Fixed costs that will continue regardless of the decision are relevant f. 58. Which of the following transactions would increase a company’s positive current ratio? a. Use the equity method to reflect earnings of an investee b. Sell a temporary investment at a loss c. Repay the principal on a short term note d. Borrow money on a short term note g. 59. How much will an investor pay for P100, 000 bond priced at 101, plus brokerage commission of P1, 100? a. P101,975 c. P101,100 b. P101,875 d. P100,000 e. 60. The currency other than reporting currency of an enterprise is a. Foreign currency c. Foreign currency loan b. Foreign ratio d. Closing rate e. 61. You have purchased some unclaimed freight for P20, 000 and can sell it immediately for P30, 000. What accounting concept principle governs the amount at which to record the goods you purchased? a. Going concern concept c. Cost principle b. Entity concept d. Reliability principle e. 62. The income statement item that is likely to be most useful for predicting income from year to year is A1 PASSERS REVIEW CENTER///114 a. Net income c. Income from continuing operations b. Discontinued operations d. Extraordinary items e. 63. Financial plan for the general expenditure of government a. Special budget c. Supplemental budget b. Deficiency budget d. General budget e. 64. Discounting a note receivable is a way to a. All of these c. Update an account b. Collect on a note d. Increase interest e. 65. An understatement in reported net income may arise from the failure to record a. The amortization of discount in bonds payable c. An occurred liability b. A deferred income d. A prepaid insurance e. 66. Pocholo Corporation owns 30 percent of the voting stock of Maxie, Inc. Maxi reports a net income of P100, 000 and declares and pays cash dividends’ of P40, 000. Which method should Pocholo use to account for this investment? a. Cost c. Equity b. Market value d. Consolidation e. 67. The matching principle provides guidance in accounting for a. Owner’s equity c. Liabilities b. Expense d. Assets e. 68. The recording phase of financial accounting covers the following steps, except: a. Business documents are received and c. Transactions are posted to the ledger prepared d. Financial statements are prepared b. Transactions are journalized e. 69. A payment on account was recorded by debiting Inventory and crediting cash. This entry was posted. The correcting entry is f. a. Accounts payable x g. Inventory x h. b. Inventory x i. Accounts payable x j. c. Cash x k. Inventory x l. d. Cash x m. Accounts Payable x n. 70. Trading investments are reported at the o. A. Total equity value r. D. Lower of the total cost or total market value p. B. Total cost of the investment of the investment q. C. Total market value of the investment s. 71. A party that controls, is controlled by, or is under common control with an enterprise either directly through one or more intermediaries t. A. None of these v. C, Subsidiary u. B. Affiliate w. D. Parent company x. y. z. aa. 72. Which of the following items appears on the bank side of bank reconciliation? ab. A. Outstanding check ad. C. NSF Check ac. B. Interest revenue earned on bank balance ae. D. Book error af. 73. In considering in financial reporting, how did the Accounting Principles Board conclude that such reporting should be viewed? ag. A. As useful only if activity is evenly spread throughout the year so that estimates are unnecessary ah. B. as a special type of reporting that need not follow generally accepted accounting principles ai. C. As reporting for a basic accounting period aj. D. As reporting for an integral part of an annual period ak. 74. discounting a note receivable create a (an) al. A. Protest fee an. C. Contingent liability am. B. Interest expense ao. D. Cash disbursement ap. 75. Stockholders of one company give up their stock in exchange for the stock of the other company; they continue to be stockholders; but now in the expanded entity aq. A. None of these as. C. Acquisition method of recording a ar. B. Leverage or trading on equity combination at. D. Pooling of Interests au. 76. Expenditures in the government are broadly classified into av. A. Current operating ax. C. Obligations incurred aw. B. Obligation of Allotment ay. D. Capital outlays az. 77. Which of the following definitions fits subsidiary ba. A. An enterprise that is controlled by another enterprise A1 PASSERS REVIEW CENTER///115 bb. B. None of these bc. C. Ownership, directly or indirectly through subsidiaries, of more than half of the voting power of an enterprise bd. D. An enterprise that has one or more subsidiaries be. 78. Family members whom a principal owner or member of management might control or influence or by whom they might be controlled or influenced because of the family relationship bf. A. None of these bh. C. Related parties bg. B. Principal owners bi. D. Immediate family bj. 79. A financial plan to augment the general budget bk. A. Deficiency budget bm. C. Special budget bl. B. General budget bn. D. Supplemental budget bo. 80. In foreign exchange transactions, the ratio at which the currencies of two countries are exchanged at a particular time bp. A. Closing rate br. C. Exchange rate bq. B. Spot rate bs. D. Forward rate bt. 81. Under the revenue principle, revenue is recorded bu. A. at the end of accounting period bw. C. After it has been earned, but not before bv. B. At the latest acceptable time bx. D. At the earliest acceptable time by.82. Using direct materials in production and charging direct labor costs to a job in debits to bz. A. Finished Goods Inventory cc. D. Materials inventory and manufacturing ca. B. Work in Process Inventory wages cb. C. Direct materials and direct labor cd. 83. Which of the following items is revenue expenditure? ce. A. Legal fee paid to acquire land cf. B. Survey of fee paid during the acquisition of land cg. C. Building permit paid to construct a warehouse on the land ch. D. Property tax paid on the land one year after it is acquired ci. 84. How are financial ratios used in decision making? cj. A. They can help identify the reasons for success and failure in business but decision making requires information beyond the ratios. ck. B. They remove the uncertainty of the business environment. cl. C. They aren’t useful because decision making is too complex cm. D. They give clear signals about the appropriate action to take cn. 85. Cash outflows for financing activities co. A. Other principal payments to creditors who have extended long term credit cp. B. All of these cq. C. Payment of dividends cr. D. Repayment of amounts borrowed cs.86. The ratio of security to long term liabilities, which measures the protection against the presently outstanding debt and indicates the available sources of additional funds, is computed as follows: ct. A. Total liabilities over total assets cv. C. Long term liabilities over fixed assets cu. B. Stockholders equity over total liabilities cw. D. Fixed assets over long term liabilities cx.87. Net income was P240, 000 in 1074, P210, 000 in 1975, and 252, 000 in 1976. The change from 1975 to 1976 is a (an) cy. A. Decrease of 10 percent da. C. Decrease of 12:15 percent cz. B. Increase of 20 db. D. Increase of 5 percent dc. dd. de. df. dg. 88. When earnings per share is computed dividends on preferred stock are dh. A. added because they represent earnings to be preferred stockholders di. B. Reported separately on the income statement dj. cause they represent earnings to the preferred stockholders dk. D. Ignored because they do not pertain to the common stock dl. 89. Which of the following accounts would be included in the calculation of a company’s acid test ratio? dm. A. Accounts receivable: No Inventories: No dn. B. Accounts receivable: No Inventories: Yes do. C. Accounts receivable: Yes Inventories: Yes dp. D. Accounts receivable: Yes Inventories: No dq. 90. Which of the following methods of reporting attempts to eliminate the effect of the changing value of the peso? dr. A. Historical cost restated for changes in the general price levels ds. B. Replacement value dt. C. Discontinued net present value of future cash flows du. D. Retirement value dv.91. What are the rules on the use of government funds? A1 PASSERS REVIEW CENTER///116 dw. A. No obligation shall exceed allotment dy. C. All of these dx. B. No allotment shall exceed appropriation dz. D. No liquidation shall exceed obligation ea. 92. If the note payable in the preceding question had been the cash proceeds from issuance would have been eb. A. P9,100 ed. C. P9,700 ec. B. P9,625 ee. D. P10,000 ef. 93. A management consultant is scheduling a long term research and development project. The time table is very tight due to the rainy season and to in adequate research staff the requirements are extensive and complex, what is the appropriate approach for planning and controlling the government highway project. eg. A. Time series or trend regression analysis ej. D. Program evaluation review technique eh. B. Cost volume earnings analysis (PERT) ei. C. Queuing theory analysis ek. 94. Investment in activity –based costing systems justice only they lead to el. A. More accurate inventory values en. C. Better cost driver analysis em. B. Better management decisions eo. D. More accurate product costs ep. 95. A supportive argument for the pooling method of accounting or a business combination is that eq. A. A portion of the total cost is assigned to individual asset er. B. Goodwill is generally a part of acquisition es. C. The company is clearly the dominant and continuing et. D. It was developed within the boundaries of the history cost system and is compatible with it eu. 96. Posting is the process of transferring information from the ev. A. Ledger to the trial balance ex. C. Journal to the trial balance ew. B. Ledger to the financial statement ey. D. Journal to the ledger ez. 97. Truelove Corporation is a stable, going concern which each invest about P50,000 in new plant and equipment as an equip amount of older equipment is retired. It has been reporting income or both tax and financial statement purposes by using straight line depreciation, but it has not changed to accelerated depreciation for tax purposes. This difference in depreciation methods caused a deferred tax credit which over the years will build up. fa. A. Rapidly and then a slowly decline fc. C. And then remain relatively constant fb. B. Indefinitely fd. D. Rapidly and slowly increase fe. 98. Used by national government that provides uniform accounting for incurring and liquidating obligations. The books used in this phase of accounting are the general journal, the journal and analysis of obligations, the journal disbursements by disbursing officers, the journal of warrants issued, and the journal of checks issued. ff. A. Advice of allotment fh. C. All of these fg. B. Obligation of allotment fi. D. Obligation of accounting office fj. 99. A company has current ratio of 2 to 1. The ratio will decrease if the company fk. A. Receivable a 5% stock dividend on one of its marketable securities fl. B. Sells merchandise for more than cost and records the sale using the perpetual inventory method fm. C. Plays a large account payable which had been a current liability fn. D. Borrow cash on a six-month note fo. 100. In a capital lease, the lease records fp. A. A leased asset and lease liability fq. B. Depreciation on the leased asset fr. C. All of these fs. D. Interest on the leased liability ft. fu. fv. ***END*** fw. fx. fy. fz. ga. gb. gc. gd. ge.A1 PASSERS TRAINING, RESEARCH, REVIEW & DEVELOPMENT COMPANY gf. 2nd Floor Sommerset Bldg., Lopez Jaena St. Jaro, Iloilo City gg. Tel. No.: (033) 320-2728; 09106547262 gh. Email Address: [email protected] gi. gj. BOARD OF CERTIFIED PUBLIC ACCOUNTANT gk. gl. CERTIFIED PUBLIC ACCOUNTANT Licensure Examination A1 PASSERS REVIEW CENTER///117 gm. gn. AUDITING THEORY go. gp. INSTRUCTION: Select the correct answer for each of the following questions. Mark only one answer for each item by shading the box corresponding to the letter of your choice on the answer sheet provided. STICTLY NO ERASURES ALLOWED. Use pencil no. 1 only. gq. gr.MULTIPLE CHOICES: gs. 1. When an auditor qualifies an option because of inadequate disclosure, the auditor should describe the nature of the commission in a separate explanatory paragraph and modify the introductory and scope paragraphs. gt. gu. When an auditor qualifies an option because of in adequate disclosure, the auditor should describe the nature of the omission in a separate explanatory paragraph and modify the scope and opinion paragraphs. gv. gw. When an auditor qualifies an option because of inadequate disclosure, the auditor should describe the nature of the commission in a separate explanatory paragraph and modify the introductory paragraphs. gx. A. First statement is correct, the second and third statements are correct B. First and third statements are not correct, the second statement is correct C. All statements above are not correct D. First and second statements are not correct, the third statement is correct gy.2. To be competent, audit evidence should be either persuasive or relevant, but need not be both. gz. The difficulty and expense of obtaining audit evidence concerning an account balance is a valid basis for omitting the test. ha. Clients accounting data can be sufficient audit evidence to support the financial statements A. First and second statements are not correct, the third statement is correct B. All above statements are not correct C. First and third statements are not correct, the second statement is correct D. First statement is correct, the second and third statements are not correct hb. 3. An auditor would most likely be concerned with internal control policies and procedures that provide reasonable assurance about the entity’s ability to process and summarize financial data. hc. hd. An auditor would most likely be concerned with internal control policies and procedures that provide reasonable assurance about the efficiency of management’s decision making process. A. Both statements are correct B. The first statement is not correct, the second statement is correct C. Both statements are not correct D. The first statement is correct, the second statement is not correct. he. 4. Select the correct statement; A. An auditor’s analytical procedures most likely would be facilitated if the entity corrects material weaknesses in internal control before the beginning of audit. B. Auditor’s analytical procedures most likely would be facilitated if the entity’s develops its data from sources solely within the entity C. Auditor’s analytical procedures most likely would be facilitated if the entity segregates obsolete inventory before the physical inventory count D. An auditor’s analytical procedures most likely would be facilitated if the entity uses a standard cost system that produces variants reports. hf. 5. An auditor assesses control risk because it indicates where inherent risk may be the greatest. hg. An auditor assesses control risk because it determines whether sampling risk is sufficiently low. hh. An auditor assesses control risk because it includes the aspects of non sampling risk that are controllable. A. All above statements are correct B. First statement is correct; the second and third statements are not correct C. The second statement is correct; the first and third statements are not correct D. All above statements are not correct hi. hj. hk. hl. hm.6. The objective of a review of interim financial information is to provide the accountant with a basis for reporting whether material modifications should be made to conform with generally accepted accounting principles. hn. A1 PASSERS REVIEW CENTER///118 ho. The objective of a review of interim financial information is to provide the accountant with a basis for reporting whether the financial statements are presented fairly in accordance with generally accepted accounting principles. hp. hq. The objective of a review of interim financial information is to provide the accountant with a basis for reporting whether the financial statements are presented fairly in accordance with standards of interim reporting. hr. A. First and second statements are correct, third statement is not correct B. First statement is correct, second and third statements are not correct C. All statements are not correct D. First and third statements are correct, second statement is not correct hs. 7. The objective of the tolerable rate in sampling for the tests of controls of an internal control is to determine the probability of the auditor’s conclusion based upon reliance factors. ht. hu. The objective of the tolerable rate in sampling for the tests of controls of an internal control is to determine that financial statements taken as a whole are not materially in error. hv. hw. The objective of the tolerable rate in sampling for the tests of controls of an internal control is to estimate the reliability of substantive tests. A. First statement is correct ; the second and third statements are not correct B. First and second statements are not correct ; the third statements is correct C. First and third statements are not correct ; the second statements is correct D. All above statements are not correct hx. 8. A written engagement letter formalizing the level of the service to be rendered is a required documentation in an audit in accordance with generally accepted auditing standard A. True, False C. False; True B. Both statements are False D. Both statements are True E. 9. In the context of an audit of financial statements, substantive test are audit procedures that will increase proportionately with the auditor’s reliance on internal control. F. G.In the context of an audit of financial statements, substantive test are audit procedures that will increase proportionately with the auditor’s reliance on internal control. A. First statement is not correct; the second statement is correct B. Both statements are correct C. Both statements are not correct D. First statement is correct; the second statement is not correct H. 10. The various evidences obtained directly by the auditor through observation such as tests counts of inventory and those obtained directly from independent sources are considered relatively more persuasive compared to evidences secured solely from within the entity. I. J. Evidence secured from within the entity like prenumbered purchase order forms is considered least persuasive type of audit evidence compared to externally created documents such as bank statements received from the client. A. First statement is not correct, the second statement is correct B. Both statements are correct C. First statement is correct, the second statement is not correct D. Both statements are correct K. 11. In designing audit programs which of the following objectives should be given more weight: A. That the auditor can make constructive suggestions to management B. That the audit evidence gathered supports the auditor’s conclusions C. That inherent risk is assessed at a sufficiently low level D. The most of the required procedures can be performed as interim work L. 12. Analytical procedures used in planning an audit should focus on identifying possible scope limitations and gathering evidence in assessing control risk environmental factors. M. N. Analytical procedures used in planning an audit should focus on aggregating data at a low level and substantiating management’s assertions that are embodied in the financial statements. O. P. Analytical procedures used in planning an audit should focus on discovering material weaknesses in the internal control structure and reporting them to the entity’s management for corrective action. A. First and second statements are not correct; the third statement is correct B. All above statements are not correct C. First and second statements are not correct; the third statement is correct D. First and third statements are not correct; the second statement is correct Q. A1 PASSERS REVIEW CENTER///119 R. S. T. U. 13. Nonsampling risk includes only those aspects of audit-risk that are not due to sampling. V. W. Inherent risk is the susceptibility of an assertion to a material misstatement assuming that there are no related internal control structure, policies and procedures. X. Y. Sampling risk arises from the possibility that, when a test of controls or a substantive test is restricted to a sample, the auditor’s conclusions may be different from the conclusions he would reach if the test were applied on the same way to all items in the account balance or class of transactions. A. First and second statements are not correct; the third statement is correct B. All above statements are not correct C. First statements is correct; the second and third statements are not correct D. All above statements are correct Z. 14. Cut-off tests designed to detect credit sales made before the end of the year that have been recorded in the subsequent year provide assurance about management’s assertion of completeness. AA. AB. Cut-off tests designed to detect credit sales made before the end of the year that have been recorded in the subsequent year provide assurance about management’s assertion of rights. AC. A. First statement is not correct; the second statement is correct B. Both statements are not correct C. First statements is correct; the second is not correct D. Both statements are correct AD.15. Determine the audit test for which an auditor would most likely use attribute sampling; A. Inspecting employee time cards for proper approval by supervisors B. Examining invoices in support of the valuation of fixed asset additions C. Selecting accounts receivable by for confirmation of account balances D. Making an independent estimate of the amount of LIFO inventory AE. 16. Which of the following procedures would most likely detect a fraud involving misstatement of income statements due to recording of journal entries whit unusual combination of debits and credits to expense and revenue account? A. Investigating and reconciliations between controlling accounts and subsidiary record. B. Tracing a sample of journal entries to general ledger C. Evaluating the effectiveness of internal control D. Performing analytical procedures designed to disclose differences from expectations AF.17. In the process of obtaining an understanding of an entity’s internal control in a financial statement audit, the auditor is not obligated to determine whether the control procedure have been placed in operation. AG. AH.In the process of obtaining an understanding of an entity’s internal control in a financial statement audit, the auditor is not obligated to perform procedures to understand the design of the internal control policies. AI. AJ. In the process of obtaining an understanding of an entity’s internal control in a financial statement audit, the auditor is not obligated to search for significant deficiencies AK. In the operation of the internal control A. Second and third statement are not correct; the first statement is correct B. First and third statements are not correct; the second statement is correct C. All above statements are not correct D. First and second statements are not correct; the third statement is correct AL.18. The expected population deviation rate of client billing errors is 2%. The auditor has established a tolerable rate of 4%. In the review of client invoices the auditor should use A. Discovery sampling C. Stratified sampling B. Variable sampling D. Attribute sampling E. 19. Identify the correct combination of procedure which auditors most likely perform to obtain evidence about fixed asset additions: A. Recomputing calculations and obtaining written management representations B. Observing operating activities and comparing balances to prior period balances C. Confirming ownership corroborating transactions through inquiries of client personnel D. Inspecting documents and physically examining assets F. 20. When evaluating an entity’s accounting estimates, one of the auditor’s objectives is to determine whether the estimates are not subject to bias.1 G. When evaluating an entity’s accounting estimates, one of the auditor’s objectives is to determine whether the estimates are consistent with industry guidelines. H. When evaluating an entity’s accounting estimates, one of the auditor’s objectives is to determine whether the estimates are based on objective assumptions. A1 PASSERS REVIEW CENTER///120 A. First statement is correct; second and third statements are not correct. B. All above statements are not correct C. All statements are correct D. The third statement is correct; first and second statements are not correct I. J. K. L. 21. When there has been a change in accounting principle that materially affects the compatibility of the comparative financial statements presented and the auditor concurs with the change, the auditor should (select the best answer) A. Refer to the change in explanatory paragraph B. Refer to the change in explanatory paragraph and concur explicitly with the change C. Issue an “except for” qualified opinion D. Concur explicitly with the change M. 22. In statiscal sampling methods used in substantive testing, when would an editor most likely to stratify a population into meaningful groups? A. If the population has highly variable recorded amounts B. If probability proportion to size sampling is used C. If the auditor estimated tolerable mis statement is extremely small D. If the standard deviation of recorded amounts is relatively small N. 23. An auditor should design the written audit program so that all materials transactions will be selected for substantive testing. O. P. An auditor should design the written audit program so that substantive test prior to the balance sheet date will be minimized. Q. R. An auditor should design the written audit program so that each account balance will be tested under either test of controls or test of transactions. A. The first statement is correct; the second and third statement are not correct B. All above statements are not correct C. All above statements are correct D. The third statement is correct; first and second statements are not correct S. 24. Choose the element of the audit planning process most likely to be agreed upon with he client before implementation of the audit of strategy. A. The determination of the evidence to be gathered to provide a sufficient basis for the auditor’s opinion B. The determination of the timing of inventory observation procedures to be performed C. The determination of the pending legal matters to be included in the inquiry of the clients attorney D. The determination of the procedures to be undertaken to discover litigation, claims and assessments T. 25. The working papers serve to satisfy the auditor’s responsibilities concerning the code of professional of ethics. U. V. The working papers serve to monitor the effectiveness of the CPA firm’s quality control procedures.. W. X. The working papers serve to document the level of independence maintained by auditor. A. First and third statements are not correct, the second statement is correct B. Second and third statement are not correct; the first statement is correct C. All above statements are not correct D. First and second statements are not correct, the third statement is correct Y. 26. Identify reason why employers bond employees who handle cash receipts: A. Because fidelity bonds reduce the possibility of employing dishonest individuals and force employees in position of trust to take periodic vacation s vacation and rotate their assigned duties B. Because fidelity bonds reduce the possibility of employing dishonest individuals and facilitate an independent monitoring of the receiving and depositing of cash receipts C. Because fidelity bonds reduce the possibility of employing dishonest individuals and deter dishonesty making employee aware those insurance companies may investigate and prosecute dishonest acts. D. Because fidelity bonds reduce the possibility of employing dishonest individuals and protects employees who make unintentional errors from possible monetary damages resulting from their errors Z. 27. What should be the direction of the audit testing when determining the completeness of the recording of transaction? A. From the original source documents C. From the general ledger balances B. From general journal entries D. From the adjusted trial balance E. 28. Identify the correct statement A. As part of understanding the internal structure culture , an auditor is not required to consider factors that affect the risk of material misstatement B. As part of understanding the internal control structure, an auditor is not required to ascertain whether internal control structure policies and procedures have been placed in operation. A1 PASSERS REVIEW CENTER///121 A. B. C. D. C. As part of understanding the internal control structure, an auditor is not required to identify the types of potential misstatements that can occur D. As part of understanding the internal control structure, an auditor is not required to obtain knowledge about the operating effectiveness of the internal control structure F. G. H. I. J. K. L. M. 29. Choose the correct statement: A. An auditors best source of corroborative information of a clients plans to discontinue a line of business is a written client representation letter B. An auditors best source of corroborative information of a clients plans to terminate an employee pension plan is a written client representation letter C. An auditor’s best source of corroborative information of client’s plans to settle an outstanding lawsuit for an amount less than the accrued loss contingency is a written client representation letter. D. An auditor’s best source of corroborative information of client’s plans to make a public offering of its common stock is a written client representation letter. N. 30. In evaluating the reasonableness of an accounting estimate, an auditor most likely would concentrate in key factors and asu8mptuions that are consistent prior periods. O. P. In evaluating the reasonableness of an accounting estimate, an auditor most likely would concentrate in key factors and assumptions that are objective and not susceptible to bias. Q. R. In evaluating the reasonableness of an accounting estimate, an auditor most likely would concentrate in key factors and assumptions that are deviations from historical patterns. Third statement is correct; first and second statements are not correct. All above statements are correct First statement is correct; second and third statements are not correct Second statement is correct; first and third statements are not correct S. 31. Identify the required documentation in an audit in accordance with generally accepted auditing standards A. A written audit program describing the necessary procedure to be performed B. A written engagement letter formalizing the level of service to be rendered C. A memorandum setting forth the scope of audit D. A flowchart depicting the segregation of duties and authorization of transactions T. 32. Compared to systematic selection, block selection is least desirable for use by an auditor. U. Comparing stratified selection, with block selection the least desirable for use by an auditor is the block selection. V. Block selection the least desirable for use by an auditor for use by an auditor compared to to systematic selection, stratified selection and sequential selection. A. Only second statement is correct; all others are not correct statements are not correct. B. Only first statement is correct; all others are not correct statements are not correct C. All above statements are correct D. All above statements are not correct W.33. Select which of the following statements regarding the auditor’s primary objective in performing procedures to obtain an understanding of internal control structure is correct A. The primary objective is to provide the auditor with evidential matter to use in assessing inherent risk B. The primary objective is to provide the auditor with a basis for modifying tests of controls C. The primary objective is to provide the auditor with knowledge necessary for audit planning. D. The primary objective is to provide the auditor with an evaluation of the consistency of application of managements policies X. 34. Select the correct statement about the reliability of evidential matter: A. To be reliable, evidential matter should be convincing rather than persuasive B. Reliability of evidential matter refers to the amount of corroborative evidence obtained C. Information obtained indirectly from outside sources is the most reliable evidential matter D. An effective internal control structure provides more assurance about reliability of evidential matter Y. 35. Which of the following is considered the principal advantage of statistical method of attribute sampling over nonstatistical methods? A. The statistical method of attribute sampling provides a scientific basis for planning the tolerable rate. B. The statistical method of attribute sampling provides a scientific basis for planning the sample size C. The statistical method of attribute sampling provides a scientific basis for planning the risk of assessing control risk too low D. The statistical method of attribute sampling provides a scientific basis for planning the expected population deviation rate A1 PASSERS REVIEW CENTER///122 Z. 36. An accountant’s compilation report should be dated as of the date of completion of the compilation. AA. AB. An accountant’s compilation report should be dated as of the date of transmittal of the compilation report. A. Both statement are correct B. First statement is not correct; the second statement is correct C. First statement is correct, the second statement is not correct D. Both statements are not correct AC. AD. AE. AF. AG. AH. AI. AJ. AK. 37. Performing test of transactions to corroborate management financial statement assertion is included when analytical procedure is used to the overall review stage of an audit. Restating control of procedures that appeared to be ineffective during the assessment of control risk is included when analytical procedures are used in the overall review stage of an audit.. Considering unusual or unexpected account balance that were not previously identified is included when analytical procedure are used in the overall review stage of an audit A. All above statements are correct B. Second statement is correct; first and third statement are not correct C. First statement is correct; Second and Third statement are not correct D. Third statement is correct; First and Second statement are not correct AL. 38. Choose the statement which reflects auditors responsible for detecting errors and irregularities.; A. An auditor is responsible for detecting employee errors and simple frauds, but not for discovering irregularities involving employee collusion or management override. B. An auditor should plan the audit to detect errors and irregularities that are caused by departures from generally accepted accounting procedures. C. An auditor should design the audit to provide reasonable assurance of detecting errors and irregularities that are material to the financial statements. D. An auditor is not responsible for detecting errors and irregularities unless the application of generally accepted auditing standards would result in such detection. AM. 39. When an entity changes its method of accounting for inventories which has a material effect on comparability, the auditor should refer to the change in an explanatory paragraph added to the auditor’s report. The paragraph should identify the nature of the change and A. Explain why the change is justified under generally accepted accounted principles. B. Describe the cumulative effect of the change on the audited financial statements C. State the auditors explicit concurrence with or opposition to the change D. Refer to the finical statement note the discusses the change in detail AN.40. An auditor assesses control risk because it affects level of detection risk the auditor may accept. AO. An auditor assesses control risk because it indicates where inherent risk may be the greatest. AP. An auditor may access control risk because it determines whether sampling risk is sufficiently low. A. First statement is correct, the second and third statement are not correct B. The second statement is correct, the first and third statement are not correct C. All above statements are correct D. The third statement is correct, the first and second statements are not correct AQ. 41 Why would an auditor assess control risk? A. Because it indicates where inherent risk may be the greatest B. Because it determines whether sampling risk is sufficiently low C. Because it affects the level of detection risk the auditor may accept D. Because it includes the aspects of non sampling risk that are controllable AR.42. An advantage of statistical sampling over nonstatistical sampling is that statistical sampling helps an auditor to eliminate the nonstatistical sampling. AS. AT. An advantage of statistical sampling over nonstatistical sampling is that statistical sampling helps an auditor to reduce the level of audit risk and materiality to a relatively low amount . AU. AV. An advantage of statistical sampling over nonstatistical sampling is that statistical sampling helps an auditor to minimize the failure to detect error and fraud. A. All above statements are correct B. First statement is correct, the second and third statement are not correct C. The third statement is correct, the first and second statements are not correct D. All above statements are not correct A1 PASSERS REVIEW CENTER///123 AW. 43. If an auditor is satisfied that there is only a remote likelihood of a loss resulting from the resolution of a matter involving an uncertainty, the auditor should express an unqualified opinion with a separate explanatory paragraph. AX. If an auditor is satisfied that there is only a remote likelihood of a loss resulting from the resolution of a matter involving an uncertainty, the auditor should express a qualified opinion or disclaimer of opinion, depending upon the materiality of the loss. AY. If an auditor is satisfied that there is only a remote likelihood of a loss resulting from the resolution of a matter involving an uncertainty, the auditor should express a qualified opinion or disclaimer of opinion, depending on whether the uncertainty is adequately disclosed. A. Second statement is correct; the first and third statements are not correct. B. All the above statements are not correct C. First statement is correct, the second and third statement are not correct D. The third statement is correct; the first and third second are not correct AZ. BA. BB. BC. BD. BE. BF. BG. BH. BI.44. With respect to the auditor’s consideration of an entity’s ability to continue as a going concern, which of the following statements is applicable’? A. If there is absence of reference to substantial doubt in the auditor’s report, this should be viewed as assurance as to an entity’s ability to continue as going concern. B. It is not necessary from the auditor to design audit procedure solely to identify conditions and events that indicate there could be substantial doubt about the entity’s ability to continue as going concern for a reasonable period of time. C. The auditor has a responsibility to evaluate whether there is substantial doubt about the entity’s ability to continue as going concern for reasonable period of time, not to exceed the date of financial statements being audited. D. The auditor’s working papers must include evidential matter which provides assurance that the entity will continue as a going concern. BJ. 45. Due to unusual circumstances, the financial statements contain a departure from generally accepted accounting principles otherwise the statement would be misleading. Under the situation, the auditor should explain the unusual circumstances in a separate paragraph and express an opinion that is A. Unqualified C. Qualified or adverse, depending on materiality B. Adverse D. Qualified E. 46. What would be the most likely use of analytical procedure when testing long term investment? A. To ascertain the reasonableness of the valuation of the marketable equity securities B. To ascertain reasonableness of the existence of unrealized gains or losses in the investment portfolio C. To ascertain the reasonableness of the completeness of recorded investment income D. To ascertain the reasonableness of the classification between current and noncurrent investment portfolio F.47. An auditor should design the written audit program in such a manner to ensure that A. All material transactions will be selected for substantive testing B. Each account balance will be tested under either tests of controls or test transactions C. The audit procedures selected will achieve specific audit objectives D. Substantive tests prior to the balance sheet date will be minimized G. 48. The procedure you most likely perform in auditing the statement of cash flows is to compare the amounts included in the statement of cash flows to similar amounts in the prior year statement of cash flows. H. I. The procedure you most likely perform in auditing the statement of cash flows is to compare the amounts included in the statement of cash flows to reconcile the cut off bank statements to verify the accuracy of the year end bank balances. J. K. The procedure you most likely perform in auditing the statement of cash flows is to compare the amounts included in the statement of cash flows is to vouch all bank transfer for the last week of the year and first week of the subsequent year.. A. All the above statements are not correct B. First statement is correct, the second and third statement are not correct C. The first statement is not correct; the second and third statement are correct D. All the above statements are correct L.49. analyses of capital stock ant other owners equity accounts are working papers that would most likely be included in an auditors permanent file. A1 PASSERS REVIEW CENTER///124 M. N. Documentation indicating that the audit work was adequately planned and supervised are working papers that would most likely be included in an auditors permanent file. A. True; True C. False; True B. False; False D. True; False E. 50. An auditor would most likely have substantial doubt an entity’s ability to continue as a going concern if research and development projects are postponed. F. G. An auditor would most likely have substantial doubt about an entity’s ability to continue as a going concern if significant related party transactions are pervasive. H. I. An auditor would most likely have substantial doubt about an entity’s ability to continue as a going concern if stock dividends replace annual cash dividends. A. All the above statements are not correct B. First statement is correct, the second and third statement are not correct C. Second statement is correct; the first and third statement are not correct D. Third statement is correct, the first and second statement are not correct J. K. L. M. N. O. P. Q. R. 51. Inherent risk is the risk that the auditor will not detect a material misstatement that exist in an assertion. S. T.Nonsampling risk includes only those aspects of audit risk that re not due to sampling U. V. Inherent risk is the susceptibility of an assertion to a material statement, assuming that there are no related internal control structures, policies or procedures. A. First statement is not correct, the second and third statement are correct B. All the above statements are not correct C. All above statement are correct D. First statement is correct, second and third statement are not correct W.52. When differences are discovered during the clients annual physical inventory count, this would most likely cause an auditor to consider whether material misstatements exist in an entity’s financial statements. X. Y.When reportable conditions previously communicated have not been corrected, this would most likely cause an auditor to consider whether material misstatements exist in an entity’s financial statements. Z. AA.When supporting records that should be readily available are frequently not produced when requested, this would most likely cause an auditor to consider whether material misstatements exist in an entity’s financial statements. A. Second statement is correct, the first and third statements are not correct B. First statement is correct, the second and third statement are not correct C. Third statement is correct, the first and second statement are not correct D. All above statements are correct AB.53. Operational auditing is primarily oriented toward future improvements to accomplish the goals of management AC. AD.Operational auditing is primarily oriented toward the accuracy of data reflected in management’s financialrecords. A. Second statement is not correct, the second statement is correct B. First statement is correct, the second statement are is not correct C. Both statements are correct D. Both statements are not correct AE.54. When evaluating an entity’s accounting estimates, what is one of auditor’s objectives? A. To determine whether the estimates are based an objective assumptions B. To determine whether the estimates are reasonable in the circumstances C. To determine whether the estimate are consistent with industry guidelines D. To determine whether the estimates are not subject to bias AF. A1 PASSERS REVIEW CENTER///125 AG. 55. To permit effective planning of the audit, the auditor must have an understanding of the internal control structure. As part of understanding the internal control structure, the auditor is required to consider factors that affect the risk of material statement AH. AI.To permit effective planning of the audit, the auditor must have an understanding of the internal control structure. As part of understanding the internal control structure, the auditor is required to ascertain whether internal control structure policies and procedures have been placed in operation. A. First statement is not correct, and the second statements is correct B. Both statements are correct C. Both statements are not correct D. First statement is correct, and the second statements is not correct AJ. 56. Identify which of the following an auditor would least likely verify when auditing inventories: A. That the financial statement presentation of inventories is appropriate B. That damaged goods and obsolete items have been properly accounted for C. That the client has used proper inventory pricing D. That all inventory owned by the client is on hand at the time of the count AK.57. What is an advantage of using statistical sampling on nonstatistical sampling methods? A. The statistical methods afford greater assurance than nonstatiscal sample of equal size B. The statistical methods can more easily convert the sample into a dual purpose test useful for substantive testing C. The statistical method provide an objective basis for quantitatively evaluating sample risk D. The statistical methods eliminate the need to use the judgement in determining appropriate sample sizes. AL. 58. The tolerable rate of deviations for a test of a control is generally (select the best answer) A. Identical to the expected rate of errors in related accounting records B. Lower than expected rate of errors in the related accounting records C. Unrelated to the expected rate of errors in the related account ting records D. Higher than the expected rate of errors in the related accounting records AM. AN. AO. AP. AQ. AR. AS. AT. 59. An auditor uses the knowledge provided by the understanding of internal control and the final assessed level of control risk primarily to determine the nature, timing and extent of the substantive test. AU. AV. An auditor uses the knowledge provided by the understanding of internal control and the final assessed level of control risk primarily to determine the nature, timing and extent of the tests of controls. AW. AX.An auditor uses the knowledge provided by the understanding of internal control and the final assessed level of control risk primarily to determine the nature, timing and extent of the compliance test A. Second statement is correct, the first and third statements are not correct B. All above statements are correct C. All above statements are not correct D. First statement is correct, the second and third statements are not correct AY. 60. A basic premise underlying the application of analytical procedures is that plausible relationships among data may reasonably be expected to exist and continue in the absence of known conditions to the contrary. AZ. BA.In the context of an audit of financial statements, substantive test are audit procedures that may be either test of transactions, direct tests of financial balances of analytical tests. BB. BC.A basic premise underlying the application of analytical procedure is that this procedure cannot replace tests of balances and transactions. A. Third statement is correct, but first and third second are not correct B. All above statements are correct C. First and second statements are correct, but the third statements is not correct D. Second statement is correct, but both first and third statements are not correct BD. BE. BF. ***END*** BG. BH. . BI. BJ. BK. A1 PASSERS REVIEW CENTER///126 BL. BM. BN. BO. BP. BQ. BR. BS. BT. BU. BV. BW. BX. BY. BZ. CA. CB. CC. CD. CE. CF. CG. CH. CI. CJ. CK. CL. CM. CN. CO. CP. CQ. CR. CS. CT. CU. DB. A1 PASSERS TRAINING, RESEARCH, REVIEW & DEVELOPMENT COMPANY CV. 2nd Floor Sommerset Bldg., Lopez Jaena St. Jaro, Iloilo City CW. Tel. No.: (033) 320-2728; 09106547262 CX. Email Address: [email protected] CY. CZ.BOARD OF CERTIFIED PUBLIC ACCOUNTANT DA. CERTIFIED PUBLIC ACCOUNTANT Licensure Examination SET A DC. DD. PRACTICAL ACCOUNITNG – PROBLEMS I DE. INSTRUCTION: Select the correct answer for each of the following questions. Mark only one answer for each item by shading the box corresponding to the letter of your choice on the answer sheet provided. STICTLY NO ERASURES ALLOWED. Use pencil no. 1 only. DF. DG. MULTIPLE CHOICE: DH. DI. 1. On January 1, 1996, a group of stockholders set up Toledo, Inc. they contributed cash of P1, 500, 000 and borrowed P220, 000. During the year, revenues from sales totalled P164, 000, while total cost and expenses were P128, 000. Toledo declared a cash dividend of P6, 000 on December 15, payable to the stockholders on January 15, 1997. There no additional activities affecting stockholders equity. By December 31, 1996, liabilities increased to P240, 000. Total assets should be reported on Toledo’s December 31, 1996, liabilities increased to P240, 000. Total assets should be reported on Toledo’s December 31, 1996 balance sheet at A. P 1,750,000 C. P1,770,000 B. P 1,764,000 D. P1,756,000 E. A1 PASSERS REVIEW CENTER///127 F. 2. Following are selected information in Carisbad Company: G. H. Cash balance, January 1, 1996 P130,000 I. Accounts receivable, January 1, 1996 P190,000 J. Collections from customers in 1996 P2,100,000 K. Stockholders’ equity, January 1, 1996 P380,000 L. Total assets, January 1, 1996 P750,000 M. Total assets, December 31, 1996 P880,000 N. Accounts receivable, December 31, 1996 P360,000 O. Total liabilities, December 31, 1996 P390,000 P. The pet income of Carisbad for 1996 is: A. P490,000 C. P150,000 B. P70,000 D. P110,000 E. F. 3. Stanford Co. had 300,000 shares of common stock issued and outstanding at December 31, 1995. G. No common stock was issued during 1996. On January 1, 1996, Stanford issued 200,000 shares of nonconvertible preferred stock,. During 1996, Stanford declared and paid P75, 000 cash dividends on the common stock and P60, 000 on the preferred stock. Not income for the year ended December 31, 1996 was P330, 000. H. I. What should be Stanford’s 1996 earnings per common share? A. P0.90 C. P1.10 B. P0.90 D. P0.65 E. F. 4. Ramagate.Co. Acquired a trademark for P10, 000,000 from Saint Albans Co. On January 2, 1996. The trademark is carried in the accounting records of Saint Albans at an unamortized cost of P7,600,00. Ramagates independent consultant has estimated that the remaining life of the trademark is 50 years. The trademark is being amortized by Ramagate over the maximum period of allowable. G. H. What amount should Ramagate report as accumulated amortization in its December 31, 1996 balance sheetr? A. P152,000 B. P200,000 C.P250,000 D.190,000 I. J. K. L. M. N. 5. Shown below are selected data from the balance sheet of Arizgoti Corp., a grocery store: O. Current liabilities P150,000,000 P. Noncurrent liabilities P120,000,000 Q. Cash P20,000,000 R. Accounts receivable P100,000,000 S. Inventory P195,000,000 T. Total assets P450,000,000 A. 0.8 to 1 B. 1.25 to 1 C. 2.1 to 1 D. 0.133 to 1 U. V. 6. The account balances for Villash Corp. As of December 31, 1997 W.Follow: X. Accounts payable P100,000 Y. Accounts receivable P120,000 Z. Building P400,000 AA.Capital stock P760,000 AB.Cash P60,000 AC. Equipment P160,000 AD. Land P50,000 AE.Notes payable P280,000 AF. Retained earnings P100,000 A1 PASSERS REVIEW CENTER///128 AG. AH. In a trial a balance prepared on December 31, 1997, the sum of the debit column is: A. P860,000 C. P790,000 B. P1,440,000 D. P1,240,000 E. 7. Pinetown Corp. Has current assets of P180, 000 and current liabilities of P360,000. Which of the following transactions would improve Pinetown’s current ratio? A. Refinancing a P60, 000 long term mortgage with with a short term note. B. Collecting P20,000 of short term accounts receivable C. Purchasing P100,000 of merchandise inventory with a short term accounts payable D. Paying P40,000 of short term Accounts payable F. 8. The net income for the year ended December 31 for Lambourne Corporation was P3, 520,000. Additional information are as follows: G. Purchases of plant assets P2,800,000 H. Depreciation of plant assets P1,480,000+ I. Dividends declared on plant assets 970,000 J. Net decrease in noncash current assets 290,000+ K. Loss on sale of equipment 130,000+ L. M. What should be the cash provided from operating activities in Lambourne Statement of cash flows for the year and ended December 31? A. P5,420,000 C. P7,250,000 B. P5, 130,000 D. P5,290,000 E. 9. On June 30, 1996, Louisiana Corp. Prepaid a P38, 000 premiums on an annual insurance policy. The premium payment was a tax deductible expense in Louisiana’s 1996 cash basis tax return. The accrual basis income statement will report a P19, 000 insurance expense in1996 and 1997. F. G. Louisiana’s income tax rate is 35% in 1996. Based on a new law, tax rate is 30% in subsequent years. H. I. In Louisiana’s December 31, 1996 balance sheet, what amount related to the insurance should be reported as deferred income tax liability? A. P11,400 C. P13,300 B. P6,650 D. P5,700 E. 10. During January 1997, Sydney Corp. won a litigation award for P15, 000 which was tripled to P45,000 to include punitive damages. The defendant, who is financially stable, has appealed only the P30, 000 punitive damages. The defendant, who is financially stable, has appealed only the P30, 000 punitive damages. F. G. In its 1997 financial statements, Sydney should report what amount of pretax gain? A. P15,000 C. P95,000 B. P50,000 D. P45,000 E. F. G. H. I. 11. A machinery was acquired by Clydeband Corp. In January 1994 at a cost of P1, 000,000. Estimated salvage value of the machinery is P200,000 at a estimated useful life of eight years, under the straight line method. In January 1996, clydeband has determined that there was a permanent impairment in the value of the machinery. Thus, the carrying amount should be P350,000 only, with remaining useful life of two years and a salvage value of P50,000 J. K. The machinery should be reported in Clydeband’s December 31, 1996 balance sheet at a carrying amount of: A. P300,000 C. P700,000 B. P350,000 D. P200,000 E. 12.on July 1, 1996, Clichy Corp. purchased Enry Corp. ‘s ten year , 8% bonds with a face amount P1,000,000 for P840,000. The bonds mature on June 30,2004 and pay interest semi-annually on June 30 and December 31. Using the interest method, Clichy recorded bond discount amortization of P3, 600 for the six month ended December 31, 1996 F. From this long term investment, Clichy should report 1996 revenue of: A. P40,000 C.P33,600 B. P36,400 D.P43,600 E. 13. Hearst Co. Sold selected merchandise on a consignment basis during 1996. Heart’s 1996 accounting records show the following: A1 PASSERS REVIEW CENTER///129 F. Information: G. Inventory January 1 P244,000+ H. Inventory on hand December 31 2890,000I. Inventory out on consignment December 31 40,000 J. Purchases 1,080,000 K. Freight-in 20,000 L. Freight-out 70,000 M. Freight out to consignees 10,000 N.What should amount Hearst report as cost of goods sold in its 1996 income statement? A. P1,054,000 C. P1,024,000 B. P1,094,000 D. P1,014,000 E. 14. During 1996, Melbourne, Inc. had the following activities related to its financial operations: F. Payment for the early retirement of G. Long term bonds payable (carrying value, P740, 000) P750, 000 H. Distribution in 1996 of cash dividend declared in 1995 I. To preferred shareholders 62,000 J. Carrying value of convertible preferred stock converted K. Into common shares 120,000 L. Proceeds from sale of treasury stock M. (Carrying value at cost, P86, 000) 95,000 N. O. In Melbourne’s 1996 statement of cash flows, net cash used in financing activities should be: A. P535,000 C. P597,000 B. P716,000 D. P717,000 E. 15. Cascade Range Inc. is determining the amount of its pretax financial income for 1997 by making adjustment to taxable from the Company’s 1997 income tax return. The tax return indicates taxable income of P380,000 on which a tax liability of P133,000 has been recognized (P380,000 x 35% = P 133,000) F. Following are list of items that may be required to determine pretax financial income from the amount of taxable income G. 1. Accelerated depreciation for income tax purposes was P134,000, straight line depreciation on these assets is P80,000 . H. 2. Goodwill of amortization of P45,000 was not included as a deduction in the tax return, but may be deducted in the income statement I. 3. Several expenses were included in the income tax return on an estimated basis. These items will be in the income statement at these same amounts but are the subject to change if new information in the future indicates that the original estimates were inaccurate. J. 4. Interest on treasury bills was not included in the tax return. During the year, P24, 700 was received on these investments. Cascade Ranges pretax financial income amounts to : A. P458, 700 B. P389,000 C. P359,700 D. P413,700 K. L. M. N. O. P. Q. R. 16.Doaktown Inc. had the following amounts of long term debt outstanding at December 31,1996 S. 14- ½% term note, due 1997 P6,000 T. 11-1/8% term note, due 2000 214,000 U. 8% note, due in 11 equal annual principal V. Payments, plus interest beginning W. December 31, 1997 220,000 X. 7% guaranteed debentures, due 2001 ___________ Y. P640, 000 Z. AA.Doaktown’s annual sinking fund requirement on the guaranteed debentures is P8,000 per year. AB. AC. What amount should Doaktown report as current maturities of long term debt in its December 31, 1996 balance sheet? A1 PASSERS REVIEW CENTER///130 A. P20,000 C. P14,000 B. P26,000 D. P8,000 E. 17. on October 1, 1995 Pontiac, Inc. Committed itself to a formal plan to sell its Hempstead division assets. On that date, Pontiac estimated that the loss from the disposal of assets in February 1996 would be P500,000. Pontiac also estimated that the loss from the disposal of assets in February 1996 would be P500, 000. Pontiac also estimated that Hempstead would incur operating losses of P2, 000,000 for the period of October 1, 1995 thought F. December 31, 1995 and P1, 000,000 for the period January 1, 1996 through February 28, 1996. These estimates were materially correct. G. H. Disregarding income taxes, what should Pontiac report as loss from discontinued operations in its comparative 1995b and 1996 income statements respectively? A. P2,500,000 and P1,000,000 C. P3,500,000 and P0 B. P0 and P3,500,000 D. P2,000,000 and P1,500,000 E. 18. Elmira Corp. Purchased bonds at a discount of P20,000. Subsequently, Elmira sold their bonds at a premium of P28,000. During the period that Elmira held this investment, amortization of the discount amounted to P4,000. F. What amount should Elmira report as a gain on the sale of bonds? A. P24,000 C. P52,000 B. P48,000 D. P44,000 E. 19. The working capital of Donostia Company is P600, 000 and its current ratio is 3 to 1. The amount of current assets is: A. P900,000 C. P600,000 B. P1,200,000 D. P1,800,000 E. 20. Salem Co. estimates its uncollectible accounts expense to be 2% of credits sales. Salem’s credit sales for 1997 were P1, 000,000. During 1997, Salem wrote off P18, 000 of uncollectible accounts. Salem’s allowance for uncollectible accounts had a P15,000 balance on January 1, 1997. F. In its December 31, 1997 income statement, what amount should Salem report as uncollectible accounts expense? A. P23,000 C. P20,000 B. P18,000 D. P17,000 E. 21. The bookkeeper of Dijon Co. recently prepared the following bank reconciliation: F. Dijon Co. G. Bank reconciliation H. December 31, 1997 I. Balance per bank statement P126,420 J. Add: K. Deposit in transit P8, 700 L. Checkbook printing charge 210 M. Error made in recording check no. 25 N. (issued in December) 1600 O. NSF check 5,000 15, 510 P. _________ Q. 141, 930 R. Deduct: S. Outstanding checks 4,480 T. Note collected by bank (includes U. P50 interest) 9,500 13,980 V. ___________ _____________ W.Balance per books P127, 950 X. --------------------Y. Z. AA. AB. AC. AD.Dijon has P9,100 cash on hand on December 31,1997. The amount Dijon should report as cash on the balance sheet as of December 31, 1997 should be: A. P120,260 C. P130,640 B. P139,740 D. P132,240 A1 PASSERS REVIEW CENTER///131 E. 22. The peso value LIFO inventory method was adopted by Peterborough Co. as of January 1, 1996. LIFO inventory layers are computed through a single inventory pool and internally computed price index. Following is Peterborough’s a peso value inventory at current year cost and at base year cost: F. G. H. I. J. At year current At base year K. Year cost cost L. _____________ ____________ M. January 1, 1995 P120,000 P120,000 N. 1995 layer 42,000 15,000 O. _____________ ____________ P. 162,000 135,000 Q. 1996 layer 78,000 45,000 R. _____________ _____________ S. December 31, 1996 P240,000 P180,000 T. --------------------------------------U. The peso value LIFO information also follows: V. January 1, 1995 P120,000 W.1995, layer 18,000 X. December 31, 1995 138,000 Y. The peso value LIFO inventory of Peterborough at December 31, 1996 was: A. P198,000 C. P180,000 B. P222,000 D. P240,000 E. 23. Roubaix failed to accrue warranty cost of P100,000 in its December 31, 1995 financial statements. In addition, a change from straight-line to accelerated depreciation made at the beginning of 1996 resulted in a cumulative effect of P60, 000 on Roubaix’s retained earnings. Both the P100,000 and the P60,000 are net of related income taxes. F. What amount should Roubaix report as prior-period adjustments in 1996? A. P0 C. P160,000 B. P60 D. P100,000 E. 24. Avon, Inc. accepted from a customer a P40, 000, P90-day, 12% interest-bearing note date August 31, 1996. On September 30, 1996, Avon discounted the note of city bank at 15%. However, the proceeds were not received until October 1, 1996. In Avon’s September 30, 1996 balance sheet, the amount receivable from the bank, based on a 360 day year, includes accrued interest revenue of: A. P200 C. P400 B. P300 D. P170 E. 25. Bressiure Corp. uses perpetual inventory system. At year end, Bressiure’s inventory account had a balance of P3,680,000, as against a complete physical inventory including goods on hand to cost only P3,610,000. F. Bressiure should: A. Record a P70,000 current liability B. Reduce the balance in its inventory controlling accounts and inventory subsidiary ledger by P70,000. C. Reduce the balance in its inventory controlling accounts and record a current liability, both in the amount of P70,000. D. Reduce its cost of goods sold by P70,000 G. 26. Brandon Co’s accounting records for the year ended December 31, 1996 included the following information: H. Work in process inventory increase P0 I. Finished goods inventory increase 70,000 J. Raw materials purchases 860,000 + K. Raw materials inventory decrease 30,000 + L. Freight-out 90,000 M. Direct labor 400,000 + N. Manufacturing overhead 600,000 + O. P. Cost of goods sold of Brandon for 1996 is: A. P1,910,000 C. P1,820,000 B. P1,900,000 D. P1,790,000 E. F. G. H. A1 PASSERS REVIEW CENTER///132 I. J. K. L. M. N. 27. Cambridge Co. purchased a machine costing P1, 250,000 for its manufacturing operations and paid shipping costs of P200, 000. Cambridge spent an additional P100, 000 testing and preparing the machine for use. O. What amount should Cambridge record as the cost of the machine? A. P1,350,000 C. P1,250,000 B. P1,450,000 D. P1,550,000 E. 28. Edmonton Corp. Owns 80% of Fargo Corp’s common stock. Fargo’s stock outstanding at December31, 1997 is as follows: F. 10% cumulative preferred stock P100,000 G. Common stock 700,000 H. I. Fargo reported net income of P60, 000 for the year ended December 31, 1997 J. What amount should Edmonton record as equity in earnings of Fargo for the year ended December 31, 1997? A. P56,000 C. P40,000 B. P48,000 D. P50,000 E. 29. On November 1, 1996, Cornwall Corp. Was awarded a judgement of P3,000,000 in connection with lawsuit. The decision is being appealed by the defendant, and it is expected that the appeal process will be completed by the end of 1997. Cornwall’s attorney feels that it is highly probable that an award will be upheld on appeal, but the judgement may be reduced by an estimated 40%. In addition to a footnote disclosure, what amount should be reported as a receivable in Cornwall’s balance sheet at December 31, 1996? A. P1,800,000 C. P0 B. P1,200,000 D. P3,000,000 E. 30. Drachten Company has the following information as of January 1, 1996 on its property, plant and equipment account: F. G. Historical cost: H. Land P25,000,000 I. Buildings and improvement 150,000,000 J. Machinery and equipment 200,000,000 K. L. Total P375,000,000 M. Accumulated depreciation: N. Buildings and improvements P18,750,000 O. Machinery and equipment 50,000,000 68,750,000 P. Q. Net book value P306,250,000 R. S. There were no additions or disposalsw during 1996. Depreciation expense is computed on the straight line method over 20 years for buildings and improvements; 10years for machinery and equipment were appraised as follows: T. U. Appraised values: V. Land P50,000,000 W. Buildings and improvements 250,000,000 X. Machinery and equipment 325,000,000 Y. P625,000,000 Z. Sound values: AA. Buildings and improvements P225, 000,000 AB. Machinery and equipment 225,000,000 AC. AD.Drachten booked the appraisal on December 31,1996. AE. AF. Drachten should report revaluation increment (appraisal increase) in property, plant and equipment under the stockholders equity of A. P143,750,000 C. P318,750,000 B. P207,500,000 D. P193,750,000 E. A1 PASSERS REVIEW CENTER///133 F. 31. Davenport Corp. Retired 50,000 shares of P5 par value common stock it held in treasury at an average cost of P26 per share on December 31, 1996. The balances in Davenport’s stockholders equity accounts before recording the retirement of the treasury stock are: G. H. Common stock- P1,080,000 additional paid in capital –P 1,500,000 treasury stock at cost – P1,300,000 and retained earnings P1,800,000 I. J. Davenport should report common stock outstanding in its December 31, 1996 balance sheet of: A. P1,080,000 C. P830,000 B. P500,000 D. P0 E. F. G. H. 32. The following balances were reported by Phoenix Co. at December 31, 1996 and 1995: I. J. 12/31/96 12/31/95 K. L. Inventory P2,600,000 P2,900,000 M. Accounts payable 750,000 500,000 N. Phoenix paid suppliers P4, 900,000 during the year ended December 31, 1996. O. P. What should phoenix report for cost of goods sold in 1996? A. P5,450,000 C. P4,950,000 B. P4,350,000 D. P4,850,000 E. 33. Royal Oak Corporation’s capital structure at December 31, 1995 was as follows: F. G. Shares issued H. And outstanding I. J. Common stock 200,000 K. Nonconvertible preferred stock 50,000 L. M. On October 1, 1996, Royal Oak issued 10% stock dividend on its common stock, and paid P200,000 cash dividends on the preferred stock. Net income for the year ended December 31, 1996 was P1, 920,000. N. O. Royal Oak’s 1996 earnings per common share should be: A. P8.20 C. P9.36 B. P8.70 D. P7.82 E. 34. Monterrey Corporation’s stockholders equity is compared of 1000 share P2 per common stock, and retained earnings of P6, 000. F. G. If a 40% stock dividend is declared when the stock is selling for P5 per share, what amount should be transferred from the retained earnings account to the paid in capital in excess of par account? A. P0 C. P2000 B. P800 D. P1,200 E. 35. At December 31, 1996, the following information was available from New Jersey Co.’s accounting records: F. G. COST RETAIL H. I. Inventory 1/1/96 P220,500 P304,500 J. Purchases 1,249,500 1,732,500 K. Additional mark-ups 63,000 L. Available for sale P1,470,000 P2,100,000 M. N. Sales for the year totalled P1,695,000. Markdown amounted to P221,000 O. Under the approximate lower average cost or market retail method, New Jersey’s inventory at December 31, 1996 was: A. P294,000 C. P323,400 B. P462 D. P420,000 A1 PASSERS REVIEW CENTER///134 E. 36. Purdue Corp. Maintains its accounting records on the cash basis but restates its financial statements to the accrual method of accounting. Purdue had P600,000 in cash basis pretax income for 1996. The following information pertain to Purdue’s operations for the years ended December 31, 1996 and 1995. F. G. 1996 1995 H. Accounts receivable P400,000 P200,000 I. Accounts payable 150,000 300,000 J. Under the accrual method, what amount of income before income tax should Purdue report in its December 31, 1996 income statement? A. P950,000 C. P550,000 B. P250,000 D. P650,000 E. 37. Cordoba, Inc. had the following activities during 1996: Acquired 2,000 shares of stock in Ventura Inc, for P26, 000. Sold an investment in Rosewood for P35, 000 when the carrying value was P33, 000. F. Acquired a P50, 000, 4-year certificate of deposit from a bank (during the year, interest of P3, 750 was paid to Cordoba). Collected dividends of P1, 200 on stock investments. G. In Cordoba’s 1996 statement of cash flows, net cash used in investing activities should be: A. P37,250 B. P41,000 C. P39,800 D. P38,050 H. I. J. K. 38. Bismarck’s Corp. Account balance during 1996 showed the following changes (all increases) L. M. Assets P178, 000 N. Liabilities 54,000 O. Capital stock 120,000 P.Additional paid-in Capital 12,000 Q. R. There were no changes in retained earnings for 1996 other than P26, 000 dividend payment and the years earning’s. S. T. Bismarck’s reported net income for 1996 is: A. P26,000 C. P18,000 B. P34,000 D. P8,000 E. F. 39. Carmel Inc. reported net income of P150,000 for 1996. Changes occurred in several balance sheet accounts during 1996 as follows: G. Investment in Crystal, Inc. stock H. Carried on the equity basis P5, 500 increase I. Accumulated depreciation, caused by a J. Major repair to machinery 2,100 decreases K. Premium on bonds payable 1,400 decrease L.Deferred income tax liability (long-term) 1,400 decrease M. N. In Carmel’s 1996 cash flows statement, the reported net cash provided by operating activities should be: A. P142,800 C. P148,300 B. P144,900 D. P150,400 E. F. 40. The December 31, 1995 financial of Hatfield Corp. Show appropriated retained earnings for construction of a new office building of P5, 250,000. The building cost P4, 500,000 when completed in 1996. An appropriation of retained earnings of P3, 600,000 for the construction of a new plant was made by Hatfield in 1996. In addition, cash of P6, 000,000 was restricted for the retirement of bonds due in 1997. G. H. What amount of appropriated retained earnings should Hatfield report in its 1996 balance sheet? A. P4,350,000 C. P9,600,000 B. P8,850,000 D. P3,600,000 E. F. 41. The balance in retained earnings of Brive Gailarde Company at the beginning of the year was P650,000. During the year, Brive la Gailarde Company earned revenue of P4, 500,000 and incurred expenses of P3, 800,000 dividends of P500,000 were declared and paid, and the balance of the cash account increased by P220,000. The Company’s net income and the yearend balance in the retained earnings account, respectively, are: A. P700,000 and P850,000 C. P200,000 and P850,000 B. P200,00 and P1,070,000 D. P700,000 and P1,070,000 A1 PASSERS REVIEW CENTER///135 E. F. 42. Chiavari Corporation had total assets of P4,000,000 and stockholders’ equity of P2,080,000 at the beginning of the year. During the year, assets increased by P520,000 and liabilities decreased by P820,000 G. H. Chiavari’s stockholders equity at the yearend totalled: A. P3,420,000 C. P3,380,000 B. 3,700,000 D. P1,780,000 E. F. 43. Twin falls Co. filed a patent infringement suit against Calgary, Inc. in May 1993 seeking P2, 850,000 for damages. Twin Falls was awarded P2,250,000 in damage by court verdict in November 1996;butb Calgary appealed the decision and its appeal is hot expected to be decided before 1998. The counsel of Twin Falls believes that it is probable that Twin Falls will be successful against Calgary and estimates an amount in the range between P1, 200,000 And P1, 650,000 with P1, 500,000 considered as the most likely amount. G. H. Twin Falls should record as income from lawsuit in the year ended December 31, 1996 the amount of: A. P0 C. P1,500,000 B. P2,250,000 D. P1,200,000 E. F. G. H. I. J. K. L. M. N. O. 44. Savanna corp. sells 1 and 2 year mail order subscriptions for its compact discs of the month business. Subscriptions are collected in advance and credited sales. Following is an analysis of the recorded sales activity: P. 1995 1996 Q. R. Sales P2,100,000 P2,500,000 S. Less cancellations 100,000 150,000 T. Net sales P2,000,000 P2,350,000 U. V. In Savannah’s December 31, 1996 balance sheet, the balance of unearned subscriptions revenue should be: A. P1,700,000 B. P2,475,000 C. P2,350,000 D. P2,325,000 W. X. 45. The following information wrev extracted from Giessen Co.’s Y. December 31, 1996 balance sheet Z. Noncurrent assets: AA. Noncurrent securities (carried at fair value) P482,250 AB. Shareholder’s equity AC.Net unrealized amount related to noncurrent securities (99,000) AD.Historical cost of the noncurrent securities was: AE. A. P482,250 B. P317,975 C. P383,250 D. P581,250 AF. AG. 46. On August 1, 1996, Basildon Corp.’s P2, 000,000 one yea, noninterest bearing note due July 31, 1997 was discounted at Alaska Bank at 10.08%. Basildon uses the straight line method of amortizing bond discount. AH. AI.What amount should Basildon report for notes payable in is December 31, 1996 balance sheet? A. P2,000,000 B. P1,910,000 C. P1,874,000 D. P1,784,000 AJ. AK. A1 PASSERS REVIEW CENTER///136 AL. AM. AN. AO. AP. AQ. AR. AS. AT. AU. AV. AW. AX. AY. AZ. BA. BB. BC. BD. BE. BF. BG. BH. BI. BJ. BK. BL. BM. BN. BQ. BO. BP. A1 PASSERS TRAINING, RESEARCH, REVIEW & DEVELOPMENT COMPANY BR. 2nd Floor Sommerset Bldg., Lopez Jaena St. Jaro, Iloilo City BS. Tel. No.: (033) 320-2728; 09106547262 BT.Email Address: [email protected] BU. BV.BOARD OF CERTIFIED PUBLIC ACCOUNTANT BW. BX.CERTIFIED PUBLIC ACCOUNTANT Licensure Examination SET A BY. BZ. PRACTICAL ACCOUNITNG – PROBLEMS I CA. INSTRUCTION: Select the correct answer for each of the following questions. Mark only one answer for each item by shading the box corresponding to the letter of your choice on the answer sheet provided. STICTLY NO ERASURES ALLOWED. Use pencil no. 1 only. CB. CC. MULTIPLE CHOICE: CD. CE. 1. Cygnus Inc. a new corporation formed and organized because of the recent consolidation of Pulsar Inc. and Quasar Inc. shall issue common shares with a par value of P100 for the total contributions of Pulsar and Quasar, including the allowances for goodwill equal to earnings in excess of 10% of net assets; capitalized at 20%. Relevant data on Pulsar and Quasar follows: CF. CG. PULSAR QUASAR CH. Total assets P9,000,000 P11,520,000 CI. Total liabilities 5,400,000 4,320,000 CJ. Annual earnings (average) 576,000 864,000 CK. CL.What is the goodwill to be recognized by Cygnus? A. P1,800,000 C. P360,000 B. P1,080,000 D. P720,000 A1 PASSERS REVIEW CENTER///137 E. 2. Mumar Company sells a franchise with initial franchise fee of P70, 000. A down payment of P20, 000 cash is required with the balance covered by issuance of a P50,000, 10% note payable in five equal annual instalments. F. G. If all material services have been substantially performed, collectability of note is reasonably assured but the refund period has not expired, what is the journal entry to record the transaction? A. Cash P20,000 H. Notes receivable 50,000 I. Franchise fees P70,000 B. Cash P20,000 J. Notes Receivable 50,000 K. Unearned Franchise fees P70,000 C. Cash P20,000 L. Notes Receivable 50,000 M. Franchise fees P50,000 N. Unearned Franchise fees P20,000 D. Cash P20,000 O. Notes Receivable P50,000 P. Franchise fees P20,000 Q. Unearned Franchise fees 50,000 R. S. 3. Quintanar Inc., a Bicol based agricultural corporation, opened a branch in Lucena City and ships merchandise there at 140% of cost. For 19X5, Lucena branch submitted the following data: T. U. Sales for 19X5 P325,000 V. Merchandise received from Home office 175,000 W.Merchandise purchased from other sources 140,000 X. Inventory, end ( of which P10,000 are from other source) 45,000 Y. No beginning inventory Z. What is Lucena branch’s ending inventory (at cost)? A. P45,000 C. P32,143 B. P35,000 D. P25,000 E. F. G. H. 4. Tadao Company, a wholly owned subsidiary of Hayashi Inc. issued 10% five year bonds at the face amount of P2, 675,000 on January 1, 19X5. Interest is due and payable every year on January 1, starting 19X5. On December 31, 19X6 Hayashi acquired in the open market Tadao’s outstanding bonds to the extent of 40% at yield of 12% , plus one year accrued interest. Present value of P1 in four years at 12% interest payable annually is 0.635518, and present value of P1 each year for four years at 12% is 3.037349. Ignoring tax impact, (1) the amount of cash paid by Hayashi for Tadao’s bonds on December 31,19X6, and (2) the gain (loss) on the extinguishment of bounds( rounded to nearest peso) are: A. (1) P1,112,000 (2) P65,000 B. (1) P1,070,000 (2) P42,000 C. (1) P1,047,000 (2) P42,000 D. (1) P1,005,000 (2) P107,000 I. 5. Windows Inc. shipped to its Laguna Branch merchandise billed at P165, 000. Additional shipments were made during the period and billed at P66, 000. Laguna Branch however returned some defective merchandise worth P4, 620 (at billed price). At the end of the period, the branch reported an ending inventory of P92, 400 and a net loss of P14, 300. If window’s shipments s to its Laguna Branch is billed at 20% of cost, what is the correct cost of the branch’s ending inventory? A. P77,000 C. P110,880 B. P73,000 D. P115,500 E. 6. Pinto Co. paid P900, 000 for a 90% interest in Swan Co. on January 2, 19X1 at a price P90, 000 in excess of the underlying book value. This excess was allocated P72,000 to undervalued equipment with a 3 year remaining life and P18,000 to goodwill with a 10 year write off period. F. G. How much goodwill will the 19X5 consolidated balance sheets of Pinto and Swan reflect? A. P18,000 C. P9,000 B. P15,000 D. P10,000 E. 7. Sabrina is admitted into the firm of Nina, Owen and Prudence. Then latter agreed to sell to Sabrina one fourth of their respective equities and profit shares. Sabrina paid a total price of P1, 000,000. Before A1 PASSERS REVIEW CENTER///138 A. P3,500,000 Sabrina’s admission, Nina, Owen and Prudence have capital C. P5,000,000 balances of P2,000,000, P1,000,000 and B. P4,000,000 P500,000, and they a share profits at the ratio of 6:3:1. Partnership D. P4,500,000 assets are fairly stated and implied E. 8.goodwill If the construction is to be recognized in progress prioraccount Sabrina’s hasadmission. balance ofCompute P1,000,000 the while new capital progress of the billings partnership. on construction contract has P800,000 how will these accounts be reflected on the balance sheet of the Ace Construction Company at the end of year 10? A. Progress Billings will be shown as current liability B. The difference between the two accounts will be shown as current liability C. Construction in progress will be shown as current asset. D. The difference between the two Accounts will be shown as current asset since Construction in progress is bigger F. 9. On May 1, 19X9, the business assets of Jose and Pedro appear below: G. H. JOSE PEDRO I. J. Cash ...................................................P 28,000 P 62,000 K. Accounts receivable ...........................200,000 600,000 L. Inventories ........................................120,000 200,000 M. Land...................................................600,000 N. Building ............................................. 500,000 O. Furniture and fixtures.........................50,000 35,000 P. Other assets .....................................2,000 3,000 Q. Accounts payable.............................180,000 250,000 R. Notes payable..................................200,000 350,000 S. T. Jose and Pedro agreed to form a partnership contributing their respective assets and equities subject to the following adjustments: U. *accounts receivable of P20, 000 in Jose’s books and P40, 000 in Pedro’s books are uncollectible. V. *inventories of P6, 000 and P7, 000 are worthless in Jose’ and Pedro’s respective books. W. *other assets of P2, 000 and P3, 000 in Jose’s and Pedro’s respective books are to be written off. X. The capital accounts of the partners after adjustments will be A. Jose, P592,000 and Pedro; P750,000 B. Jose, P600,000 and Pedro; P700,000 C. Jose, P592,000 and Pedro; P756,300 D. Jose, P600,000 and Pedro; P750,000 Y. Z. AA. AB. 10. On January 1, 19X5, Kuala Company purchased 90% equity of Lumpur Company .On January 3, 19X5, Lumpur sold equipment (with original cost of P750, 000 and carrying cost of P375, 000) to Kuala for P540, 000. The equipment has a remaining life of three years and was depreciated using the straight-line method by both companies. In Kuala’s consolidated balance sheet as of December 31, 19X5, the depreciation should be reported at: A. P100,000 C. P62,500 B. P90,000 D. P125,000 E. 11. Moore admits Nolan as a partner in the business. Balance sheet accounts of Moore on September 30, just before admission of Nolan, show: F. G. Cash P2600 H. Account receivable 12,000 I. Merchandise inventory 18,000 J. Accounts payable P6, 200 K. Moore, Capital 26,400 L. M. It is agreed that for purposes of establishing Moore’s interest, the following adjustments shall be made: N. O. (1) An allowance for doubtful accounts of 2%is to be established P. (2) Merchandise inventory is to be valued at P20, 200 Q. (3) Prepaid expenses of P350 and accrued liabilities of P400 are to be recognized. R. Nolan is to invest sufficient cash to obtain 1/3 interest in the partnership. How much is Nolan investment to the partnership? A. P14,155 C. P14,305 B. P17,600 D. P7,920 E. 12. Rubaiyat Inc.., a new corporation formed and organized because of the recent consolidation of Omar Inc. and Khayyam Inc., shall issue 10% participating preferred stocks with a par value of P100 of Omar’s and Khayyam’s net assets contributions, and common shares with a par value of P50 for the difference of total A1 PASSERS REVIEW CENTER///139 shares to be issued and the preferred shares to be issued. Total shares to be issued by Rubaiyat shall be equivalent to average annual earnings capitalized at 10%. Relevant data on Omar and Khayyam follows: F. G. OMAR KHAYYAM H. Total assets P1, 000,000 P1, 382,400 I. Total liabilities 648,000 518,400 J. Capital stock, par P100 450,000 750,000 K. Annual earnings (average) 69,120 103,680 L. M. Assuming fractional shares are not considered, Shariff, a holder of 500 Omar shares, will receive how many preferred and common stock from Rubaiyat as a result of the consolidation? A. Preferred : 960 Common: 288 B. Preferred : 576 Common: 230 C. Preferred: 1,152 Common: 115 D. Preferred: 480 Common :576 N. O. 13. When Mikki and Mylene, partners who share earnings equally, were incapacitated in an airplane accident, a liquidator was appointed to wind up their business. The accounts showed cash, P35,000; other assets, P110,000; Liabilities, P20,000; Mikki, capital, P71,000; and Myleene, capital, P564,000. Because of highly specialized nature of the noncash assets, the liquidator anticipated that considerable time would be required to dispose of them. The expenses of liquidating the business (advertisng, rent, travel, etc. ) are estimated at P10,000. P. Q. How much cash can be distributed safely to each partner at this point? A. P5,000 to Mikki; and P0 to Mylene B. P5,000 to Mikki; and P500 to Mylene C. P3,000 to Mikki; and P0 to Mylene D. P5,000 to Mikki; and P1,000 to Mylene R. S. 14. Shall Petula Company re4alize and, there upon, accordingly report a gain or loss when Clark Company, its partially owned subsidiary, issues with consent of minority and interests additional shares to (1) Petula and (2) general Public? A. (1) Yes (2) No B. (1) No (2) No C. (1) No (2) Yes D. (1) Yes (2) Yes T. U. V. W. X. Y. 15. Web. Inc., Xerxes Inc., and Yet.,. agree to consolidate. Data relative to their net assets and appraised values and average adjusted annual earnings follow: Z. Net Adjusted AA. Asset’s Earnings AB. Web P400,000 P60,000 AC.Xerxes 600,000 60,000 AD.Yeti 1,000,000 80,000 AE. AF. Parties collectively agreed that the new corporation, Vogue Inc., is to issue a single class of stock. If the distribution of new shares will be based on the earnings ratio, what is the distribution ratio to the shareholders of Web, Xerxes and Yeti, respectively? A. 30:20:50 C. 30:30:40 B. 30:40:30 D. 20:30:50 E. 16. Halo Co. purchases all of the issued and outstanding shares of Iglo Co. for P4, 000,000. Iglo’s capital balances prior to acquisition show: F. G. Capital stock P2, 500,000 H. Additional paid-in capital 500,000 I. Retained earnings 625,000 J. K. What is the amount of goodwill to be reflected in the consolidated balance sheet? A. P225,000 C. P100,000 B. P375,000 D. P425,000 A1 PASSERS REVIEW CENTER///140 E. 17. The Guam Company will issue share at P10 par value common stock for all the net assets of the Newark Co. Guam’s common has current market value of P40 per share. Newark’s balance sheet accounts are shown below F. Current assets..................................P320,000 G. Property and equipment..................880,000 H. Liabilities..........................................400,000 I. Common stock, P4 par....................80,000 J. Additional paid in capital................320,000 K. Retained earnings..........................400,000 L. M. The fair value of current assets is P400, 000 while that of property and equipment is P1, 600,000. All the liabilities are correctly started. Guam issued sufficient shares so that the fair market value of the stock equals the fair market value of Newark’s net assets. N. How many shares of stocks will Guam issue to recognize goodwill of P200,000? A. 40,000 C. 45,000 B. 50,000 D. 55,000 E. 18. Quasar Inc.. was formed and organized because of recent consolidation of Meteor Company and Nova Company. Quasar shall issue its common shares with a par value of P100 for the total contributions of Meteor and Nova, including allowances for goodwill equal to earnings in excess of 10% of net assets, capitalized at 20%. Relevant data on Meteor and Nova follows: F. METEOR NOVA G. Total assets P6,000,000 P7,680,000 H. Total liabilities 3,600,000 2,880,000 I. Annual earnings (average) 384,000 576,000 J. K. What is the total number of shares to be issued by Quasar? A. 54,000 C. 70,000 B. 86,000 D. 16,000 E. 19. The Angeles City branch of Rommel Enterprises Manila, was billed for merchandise shipments from home office at cost plus 25% in 19X4 and cost plus 20% in 19X5. Other pertinent data for 19X5 show: F. G. Angeles Branch Home Office H. Sales P63,000 P212,000 I. Inventory, beginning J. At cost 23,000 K. At billed price 8,900 L. Purchases 164,000 M. Inventory transfers N. To Angeles, at cost 42,000 O. From Manila, at billed price 50,400 P. Inventory, end Q. At cost 28,500 R. At billed price 11,700 S. Expenses 20,300 76,400 T. U. V. W. X. What will be the net income of Angeles Branch to be reported in Rommel’s combined income statement? A. P25,270 C. P22,430 B. P18,740 D. P19,100 E. 20. Fosforo Inc. manufactures matches per the design and specification of its clientele, and accordingly uses the job order cost system. In April19X6, it finished 50,000 pieces of Job at Shangri-la at the cost of P0.50 per unit for direct materials and P0.40 per unit for direct labor. Factory overhead is applied at 100% of direct labor. Job Shangri-la incurred 500 defective units (considered normal) that had to re worked at a cost of P0.10 per unit, in addition to the predetermined factory overhead rate. What is the total cost of work on the defective unit and to what account should this be charged? A. P50, work in process B. P100, factory overhead control C. P100, work in process D. P50, factory overhead control F. 21. The Angeles City Branch of Rommel enterprises, Manila, was billed for merchandise shipments from home office at cost plus 25% in 19X4 and cost plus 20% in 19X5. Other pertinent data for 19X5 show: G. H. ANGELES BRANCH HOME OFFICE I. Sales P63,000 P212,000 A1 PASSERS REVIEW CENTER///141 J. Inventory, beginning K. At cost 23,000 L. At billed price 8,900 M. Purchases 164,000 N. Inventory transfers O. To Angeles, at cost 42,000 P. From Manila, at billed price 50,400 Q. Inventory, end R. At cost 28,500 S. At billed price 11,700 T. Expenses 20,300 76,400 U. V. The net income (loss) of the Angeles City branch for 19X5 is: A. (P13,130) C. P3,330 B. P8,230 D. (P4,900) E. 22. The Avenida Rizal Branch of Bram Stoker Store is billed by home office for merchandise shipments of 40% over cost.. The following information relates tom Avenidas operations for calendar year 19X6: F. G. Sales P4,500,000 H. Local purchases 1,000,000 I. Shipments from home office 2,450,00 J. K. Avenida reports an ending inventory of P700,000 of which P175,000 is identified to come local purchases. Per home offices reckoning, how much is Avenida’s gross profit? A. P2,300,000 C. P1,750,000 B. P2,750,000 D. P2,200,000 E. 23. The Olongapo – SMBA branch of Freeport Corporation submitted the following trial balance of 30 June 19X6: F. G. Cash P28,600 H. Accounts receivable 173,800 I. Shipments from office 462,000 J. Home office- Current P324,500 K. Sales 369,600 L. Expenses 29,700 M. Total P694,100 N. O. Olongapo reported an ending inventory of P138, 600. Shipments are billed at a mark up of 40% on cost. What is the real net income of Olongapo branch? A. P92,400 C. P70,600 B. P138,600 D. P108,900 E. 24. Jane, a senior partner in an engineering firm, has a profit share of 30% in 19X6. During 19X6, Jane withdrew P130,00 against her capital but invested property with a fair value of P25,000. If Jane’s ending capital is P60, 000 lesser than her beginning capital, Compute Jane’s beginning capital for 19X6. A. P150,000 B. P175,000 C. P210,000 D. P240,000 F. G. H. I. J. K. 25. The annual appropriation (General fund) of Agency FFF for 19X6 of P5, 000,000 was released by DBM minus the 5% reserve on allotments. Agency FFF incurred total obligations during the year of P4, 000,000, of which P1, 000,000 relates to procurement of equipment. Records show that by year’s end P1, 250,000 of the total obligations have been liquidated. What is the appropriate entry in the books of Agency FFF to record the receipt of Advise of allotment from DBM? A. Current Surplus Unappropriated P5,000,000 Dr. L. Current Surplus Appropriated 5,000,000 Cr. B. No entry C.National Clearing Account M. Appropriation Allotted P4, 750,000 Dr. N. Appropriation Allotted 4,750,000 Cr. O. D. National Clearing Account P. Appropriation Allotted P5,000,000 Dr A1 PASSERS REVIEW CENTER///142 Q. Appropriation Allotted 5,000,000 Cr R. 26. Neneng’s Meat Corporation uses a process cost system and sells a variety of cooked meat, skin and cuttings. Four joint products [produced out of the process are as follows: S. T. Products Amount Produced U. LIT-TID 1,000 V. TAD-YANG 9,000 W. PANG-A 400 X. SOUP NO. 5 5,100 Y. Z. The split off point for this products occurs in division B and the cost incurred up to this point are: P20, 000 for direct materials, P15,000 for direct labor, and P7,000 for factory overhead . What are the joint cost allocated to each of the joint products by using the physical output method for Lit-tid and Tad-yang, respectively? A. P1,000 and P9,000 C. P20,000 and P15,000 B. P3,000 And P24,387 D. P2,710 and P24,387 E. 27. Jerez Inc. acquires 75% interest in KCB Inc. for P3, 120,000. The capital balance of KCB prior to acquisition show capital each stock; P2, 600,000; additional paid in capital, P1, 040,000; and deficit, P130, 000. What is the total minority interest to be shown in the consolidated balance sheet? A. P877,500 C. P650,000 B. P910,000 D. P950,375 E. 28. Dreams corporation opened its Paseo branch exactly a year ago and the latter submitted the following information on its operations: F. Sales P264,000 G. Expenses 137,500 H. Purchases 71,500 I. Ending inventory J. From purchases 16,500 K. From home office 66,000 L. M. Paseo remitted P151, 250 to home office . Shipments to branch are billed at cost-plus 20%. During the period, home office shipped merchandise to Paseo at a billed amount of P148, 500. What is Paseo’s closing inventory at cost? A. P55,000 C. P82,500 B. P71,500 D. P69,300 E. 29. Vandros Inc. sol on January 19X4 a reconditioned dump truck to We-haul Inc. for P1,200,000 which resulted in a gain to Vandros of P420,000, We-haul paid P310,000 and covered the balance with a chattel mortgage at P10% interest per annum. The mortgage is payable in five equal annual amortizations starting January 2, 19X5 is: A. P93,450 C. P62,300 B. P249,200 D. P311,500 E. 30. Purity Water Company bottles spring water. The spring water is filtered in Department A and then flows through the Department B where it is bottled. Party’s only direct materials cost occurs in the bottling process. The quantity schedules are follows: F. G. DEPARTMENT A H. Units started in process P110,000 I. Units transferred to Dept. B 80,000 J. Ending units in process 30,000 K. L. DEPARTMENT B M. Units received from Dept. A 80,000 N. Units transferred to finished O. Goods inventory 61,400 P. Ending units in process 18,600 Q. R. S. T. Ending units in process in both departments are 72% complete as to conversion costs. Department B would have how many equivalent gallons of production on the basis of conversion cost A.P74,792 C.61,400 B.80,000 D.68,929 E. F. 31. Capitalizing on alleged inside information, Dupe and Fluke formed a partnership to purchase, sell or otherwise trade in Bre –X mining shares. Bre- X recently made significant finding gold deposits in its property in Busang, Indonesia. They started cautiously by making initial but modest cash contribution of P137, 500,000 each. They agree to divide earnings equally and further agreed to settle and close the A1 PASSERS REVIEW CENTER///143 partnership after six months of furious but ferocious (insider) trading. Below is a synopsis of the transactions for six months: G. H.Purchases of shares: I. By Dupe P1,237,500,000 J. By Fluke 495,000,000 K. L. Sales of shares M. by dupe 1,339,250,00 N. by fluke 462,000,000 O. P. Interest Charges Q. paid by Dupe 3,200,000 R. paid by Fluke 1,375,000 S. T. Dividend income U. received by Dupe 1,100,000 V. received by Fluke 2,750,000 W. X.How much will Dupe receive (or pay) in final settlement of the partnership? A. (34,512,500) C. P2,885,500 B. P66,137,500 D. P31,625,000 E. F. 32. The Avenida Rizal Branch of Bram Stoker store is billed by home office for merchandise shipments at 40% above cost. The following information relates to Avenida’s operations for 19X6: G. H. Sales P4,500,000 I. Local purchases 1,000,000 J. Shipment from home office 2,450,000 K. L. Avenida reports an ending inventory of P700,000 of which P175,000 is identified to have come from local purchases. What is Avenida’s ending inventory, at cost? A. P375,000 C. P735,000 B. P550,000 D. P700,000 E. F. 33. Gochic Inc. purchases 80% interest in hals Inc. for P2,700,000 Hals assets have a fair value of P3,375,000. Hals capital balances before acquisition show capital stock of P2,700,000, additional paid in capital of P675,000 and retained earnings of P405,000. In the consolidated financial statements prepared after acquisition, how much will be reflected as minority interest? A.P675,000 C.P540,000 B.P856,000 D.P756,000 E. F. 34. Gilmore Inc. uses the instalment method of accounting for revenue. The following information are obtained from its books: G. 19X4 19X5 H.Instalment sales P375,000 P412,000 I. Cost of instalment sales 131,250 164,800 J. Instalment accounts receivables K. On 19X4 sales 225,000 125,000 L. On 19X5 sales 275,000 M. N.What is the total realized gross profit of Gilmore for 19X5? A. P35,000 C. P89,900 B. P87,950 D. P54,800 E. F. G. H. I. J. K. L. M. 35. Before the withdrawal of aliysa from Aliysa Benetta Cathyn, Lawyers, the partners collectively agreed to adjust the values . The respective asset accounts were debited fro increase and the corresponding credit(s) will be able to: A. Deferred credit B. Partner’s capital accounts A1 PASSERS REVIEW CENTER///144 C. Income and expense summary D. Appraisal increase-capital E. F. 36. Tondo Branch of Tyler Company was billed for merchandise shipments at 125% of cost. At the end of its first month of operation, Tondo branch submitted the following information: G. H. Merchandise shipments from home I. office(at billed price) P150,000 J. K. Merchandised purchased locally L. by Tondo branch 40,000 M. N. Closing inventory O. from home office 75,000 P. from local purchases 20,000 Q. R. Net sales 180,000 S. T. What is the correct closing branch inventory at cost? A. P75,000 C. P95,000 B. P80,000 D. P125,000 E. F. 37. Rhapsody Inc. is an 80% owned subsidiary of Sonata Company. On December 31, 19X5, working paper elimination for the consolidated final statements of Sonata and Rhapsody amounting to P3, 000,000. Rhapsody’s bonds were to mature on December 31, 2001. Ignoring tax impact, working paper eliminations entry for December 31, 19X6 includes a A. P3,000,000 credit to Retained earnings- Rhapsody B. P500,000 debit to minority Interest in Net assets of Rhapsody C. P2,400,000 credit to Retained earnings-Rhapsody D. P3,000,000 credit to Retained earnings – Sonata G. H. 38.Person Company acquired 80 percent of the outstanding common stock I. Of sonar, Inc, by issuing 20,0000 of its P 10 par value shares which are selling at 16, Fees paid to consultants and attorneys in connection with the exchange of shares amounted to P 30,000. Immediately prior to the acquisition ,. The trial balances of the two companies were as follows: J. K. Current assets PARSON BOOK VALUE SONAR L. Plant assets P1, 800,000 P 975,000 FAIR VALUE M. Investment in marketable 3,000, 000 1,300,000 P 1,025,000 N. Securities (1,100,000) 300,000 1,400,000 O. Current liabilities ( 600.000) (800,000) 280,000 P. Noncurrent liabilities ( 400,000) (400,000) 800,000 Q. Common stock ( 800,000) (200,000) 530,000 R. Additional paid in capital (1,900,000) (775,000) S. Retained earnings T. U. On the consolidated balance sheet after acquisition, how much will be the additional paid in capital of the controlling interest, assuming that all the requirements for pooling of interest, have been met? V. A. P 1,000,000 X. C. P 1, 160,000 W. B. P 1,200,000 Y. D. P 800,000 Z. 39. The following information pertains to a river- control project or Rathaus Konstrukt Inc. In Tabuk, Kalinga which was commented in 19X3 and completed the following year: AA. AB. Cost incurred to date AC. At June 30, 19X3 P 9, 750,000 AD. At June 30, 19X4 15, 750,000 AE. Estimated total cost AF. At completion AG. At June 30, 19X3 19,500,000 AH. At June 30, 19X4 20,250,000 AI. AJ. AK. AL. AM. AN.The project is a P22,500 fixed- price construction contract and Rathaus uses the percentage- of completion method of accounting. What is the income reported by Rathaus on its Kalinga project on June 30, 19X4? A1 PASSERS REVIEW CENTER///145 AO. A. P 750, 000 AQ. C. P 1, 750,000 AP. b. P 1, 500,000 AR. D. P 250, 000 AS. 40. Ovid company makes use of the two-way analysis of overhead variances. Ovid’s budgeted factory overhead for fixed costs is p 1,200,000 per month, plus a variable factory overhead rate of P96 for every direct labor hour. The standard direct labor hours allowed for May, 19X 6 productions are 432,000 hours. An analysis of the factory overhead showed that, in May, Ovid had an unfavourable budget (controllable) variance of P24,000 and a favorable volume balnce of P 12,000. What is the applied factory overhead in May? AT. A. P5, 371,200 AV. C. P5, 335, 200 AU. B. P5, 359, 200 AW. D. P5, 347, 200 AX. 41. Paradim Co. recently paid P5,000,000 cash for the net assets of Supreme Co. which had a book value of P 5,000,000 of P4, 000,000 . The book values of the assets and liabilities approximate their fair values except as noted below. AY. AZ. BOOK VALUE FAIR VALUE BA. Inventory............................. .P 400,000 P 450,000 BB. Land...................................... 300,000 500,000 BC.Building and equipment, net...............1,400,000 1,550,000 BD. BE. 68. Which of the two methods ( the present of the automated has higher income at the level of sales of 26,000 units? BF. a. Present, P 60,000 BH. c. Present, P 240,000 BG. b. Automated P60,000 BI. d. Automated, P 240,000 BJ. 69. A fira has the following capital structures : 70 debt and 320 equity interest rate on debt is 12 and on common stock is 14 if the weighted average cost of the capital is 9.66 then what is the marginal tax rate? BK. a. 350 BM. c 600 BL. b. 400 BN. d. 650 BO. 70. Samantha Manufacturing recently completed and sold an order of 50 units that had the following costs: BP. Direct materials P1, 500 BQ. Direct labor 8,500 BR. Variable overhead 4,000 BS. Fixed overhead 1,400 BT. TOTAL P 15,400 BU. BV. BW. Applied on the basis of direct labor hours. BX. Applied at the rate of 10 of variable cost. BY. BZ. The company has now been requested to prepare a bid order for another set of 50 units of the same product. CA. CB.If an 80 learning curve were applied, Samantha’s total cost on this order would be estimated at CC. a. P 6,400 CE. c. P 9,900 CD. b. P 8,600 CF. d. P 15,400 CG. 47. Timepieces Company manufactures and sells watches that carry a 2- year unconditional warranty against product failure. Based on a reliable statistical, timepieces knows that between the sale and the lapse of the product warranty; 10% of units sold will require an average cost of P 250 per unit for repairs. The following data reflect the recent experience of the company for the past 3 years. CH. CI. Number of units sold 2008 2009 2010 CJ. Number of units repaired: 6,000 8,500 10,000 CK.2008 80 150 200 CL. 2009 CM. 2010 250 350 CN. 400 CO. 48. Millennium Company provided the following information for 2010. CP. CQ. Net Income 2, 5000,000 CR. Total Assets 5,600,000 CS. Share Capital 1,000,000 CT. Share Premium 800,000 CU. Treasury shares at cost 200,000 CV. Dividends declared 1,500,000 CW. 400,000 CX. A1 PASSERS REVIEW CENTER///146 CY. CZ. DA. DB. DC. Prior period adjustment for overstatement of 2009 ending inventory DD. The debt to equity ratio 40% ( total liabilities divided by total shareholders’ equity) at December 31,2010 . What was the retained earnings balance on January 1, 2010? DE. a. 1, 000,000 DG. c. 1, 6000,000 DF. b. 1,800,000 DH. d. 1, 4000,000 DI. 49. On July 1, 2010, Dakota Corporation Issued office space for 5 years at a monthly rental of P 75,000. On that date , Dakota paid the lessor P 950,000 which included the following: DJ. DK. Rent deposit 300,000 DL. First months deposit 75,000 DM. Last month’s deposit 75,000 DN. Installation of new walls and offices 500,000 DO. DP. The entire amount of P 950,000 was charged to rent expense in 2010 . Monthly rentals were paid as scheduled from August to December 31, 2010 under the operating lease? DQ. a. 530,000 DS. c. 125,000 DR. b. 500,000 DT. d. 450,000 DU. 50. Brisbane company reported the following information for 2010 Sales revenue of P 2,800,000 cost of goods sold of P 1,000,000, distribution costs of P 400,000, administrative expenses of P 350,000, depreciation of P 250,000, interest expense of P 80,000 and income tax expense of P 280,000. All sales were made for cash and all expenses other than depreciation and bond premium amortization of P 20,000 were paid in cash. All current assets and current liabilities remained unchanged. What is the net cash provided by operating activities during 2010? DV. a. 440, 000 DX. c. 670,000 DW. b. 690,000 DY. d. 710. 000 DZ. EA. EB. EC. ED. EE. EF. EG. EH. EI. EJ. EK. EL. EM. EN. EO. EP. EQ. ER. ES. ET. EU. EV. EW. EX. EY. EZ. FA. FB. FC. FD. FE. FF. FG. FH. FI. FJ. A1 PASSERS REVIEW CENTER///147 FK. FL. FM. FN. FO. FP. FQ. ANSWERS FR. FS. PRACTICAL ACCOUNTING PROBLEMS FT. FU. 1. Understatement of 2009 warranty cost FV. FW. 2. Cumulative additional depreciation ( 1M/ 10X6) FX. Depreciation on carrying amount (2.5M)/5) FY. FZ. GA. ANSWERS 600,000 A 600,000 500,000 2010 Depreciation 3. GB. Unadjusted balances GC. Unrecorded sales GD. Goods sold FOB SP GE. Goods sold FOB destination GF. Goods held on consignment GG. Goods purchased FOB Dest (arrived) GH. Obsolete inventory GI. Adjusted inventory 1,100,000 B Perpetual Periodic 3,600,000 (200,000) (500,000) (150,000) 3,450,000 400,000 (150,000) (, 100,000) (500,000) (150,000) 300,000 3,360.000 3,300,000 D GJ. GK. GL. 4. Fair value of share alternative 60,000X35) 2,100,000 GM. Less. Fair value of cash alternative (30,000 X 50) GN. Grant date fair value of equity component GO. 2009 GP.FV of equity component (60,000//3) GQ. FV of liability component (30,000X 60/ 3) GR. Compensation expense GS. 2010 GT. FV equity component (60,000/3) GU. Fair value of liability component (30,000X75)/ 3X2) – 600,000 GV.Compensations expense GW. GX. GY.5. Lobstar ( 10,000)X 150) GZ.Shrimp (50,000X. 80) HA.Salmon( 30,000X 200) – 400,000 payment HB.Net unrealized gain HC. HD. 6. net investmen HE.Less: PV of GRV (500,000 x.58) HF. Net Investment to be recovered from rentals HG. Rentals (3,924,000/4.36) HH. Gross investment (900,000x6)+ 500,000 residual value HI. Less : Net Investment HJ. Financial revenue HK. HL. 7. Average earnings (4M-500,000)/5 HM. Capitalization rate HN. Net assets with goodwill HO. Less: Fair value without goodwill HP. Goodwill 1,500,000 600,000 200,000 600,000 800,000 200,000 900,000 1,100,000 C 1,5000,000 4,000,000 5,600,000 3,100,000 B 4,204,000 280,000 3,924,000 900,000 5,900,000 4,204,000 1,696,000 B 700,000 10% 7,000,000 5,500,000 1,500,000 C A1 PASSERS REVIEW CENTER///148 HQ. HR. HS.8. Total market value (except Stellar Company treasury share) HT. Less: Total cost HU. Unrealized gain on trading securities HV. HW. 9. Accrued pension cost 1/1 (6,500,000-5,000,000) HX.Pension expense HY. Contributions HZ. Accrued pension cost 12/31 IA. IB. IC. ID. IE. 10. Accounting Profit IF. Fines and penalties IG. Cash dividends received IH. Accounting profit subject to tax II. Unrealized gain on trading securities (taxable) IJ. Reversal of deductible temporary difference IK. Reversal of taxable temporary difference IL. Taxable profit IM. IN. 11. Gross profit from instalment accounts receivable(120,000/30%) IO. Instalment accounts receivable (400,000/40%) IP. IQ. 12. average general price index number (150 +300)/2 IR. Cash IS. AR IT. Inventory (1,200,000 x 300/225) IU. PPE (2,000,000 x 300/125) IV. Total assets IW. IX. Accounts payable 10,600,000 10,300,000 300,000 (1,500,000) (1,000,000) 1,200,000 (1,300,000) D 1,200,000 500,000 (800,000) 900,000 (200,000) (600,000) 350,000 450,000 B 400,000 1,000,000 C 225 300,000 800,000 1,600,000 4,800,000 7,500,000 400,000 IY. Bonds payable IZ. Total liabilities JA. JB. Contributed capital (1.2 M x 300/100) JC. RE (SQUEEZE) JD. Less ; Treasury shares (350,000 x 300/140) JE. SHE (7,500,000-1,100,000) JF. JG. 13. Replacement cost JH. Less:Accumolated depreciation (9M x 25%) JI. Sound Value JJ. Less: Carrying amount JK. Revaluation surplus-gross 6/30/10 JL. Less: Deferred tax lability (1.5 Mx 30%) JM. Revaluation surplus-net 6/30/10 JN. Less Realization (1,050,000/5x6/12) JO. Revaluation surplus-net 12/31/10 JP. Percentage of accumulated depreciation (1,750/7,000) JQ. JR. 14. Payment for operating expenses JS. (1,000-120-30+400-550+100-150) JT. JU. 15. cost JV. Less: Accumulated depreciation (3M x 10% x 3) JW.Carrying amount 1/1/10 JX. 2010 Depreciation (2.6 M-500)x 7/28) (7 years remaining) 700,000 1,100,000 JY. 16. PBO (5M +600+ (12% x 5M)-900-100) JZ. KA. 17. 10 dairy cattle- 3 years old KB. 2 hogs-4 years old 5,200,000 B 3,000,000 3,550,000 B 750,000 6,400,000 9,000,000 2,250,000 6,750,000 5,250,000 1,500,000 450,000 1,050,000 105,000 945,000 A 25% 650,000 B 3,500,000 900,000 2,600,000 525,000 D 580,000 600,000 A1 PASSERS REVIEW CENTER///149 KC.15 horses-2 year old KD.8 Carabao’s -3.5 years old KE. 4 dairy cattle-1.5 years old KF. 6 carabaos-1 yr old KG. Carrying amount on 12/31/10 KH. KI. 18.FV of land and building (lower than PV of minimum lease payments) KJ. 19. Net. Income to ordinary shareholders (900,000 x 15) KK. Dividends on preference shares KL. Total net income KM. Divide by : (900,000 plus (400,0000) KN.Diluted EPS KO. KP. 20. Deficit eliminated (1M +200+400+300) KQ. KR. KS. KT. KU.21. Total shareholders KV. Less: PSHE (1.25 M +250)+(1.25 M x 10% x 4 years) KW. Ordinary shareholders’ equity KX. Divide by (230,000 treasury) KY. Book value per ordinary share KZ. LA. 22. Increase in contributed capital (8.5 M -5M) LB. Stock dividend (4M x 25%) LC. Donated capital LD. Proceeds from share issuance LE. LF. Accumulated profits beg LG. Stock dividends (4M x 25%) LH. Cash dividends declared during the year (SQUEEZE) LI. Net income LJ. Accumulated profits end LK. Barrowing paid LL. Proceeds from shares issued LM.Dividends paid (800,000 + 500,000 beginning dividends payable) LN. Net cash used in financing activities LO. LP. 23. total cash dividends (500+300) LQ. Fair value of property dividend at date of payment on 5/1/2010 LR. Fair value of share dividend at date of declaration (10% x 200,000 x 15 ) LS. Decrease in RE LT. LU. 24. Royalty income for first half of the year LV. Royalty income for second half of the year (10M x 10%) LW.2011 Royalty Income LX. LY. 25. Account receivable LZ. Less: Required allowance MA. NRV. MB. MC. 26. Proceeds MD. Cash Surrender Value ME. Unexpired Insurance premium (90,000/ 12 x 2) MF.Gain MG. MH. 27. Accrued liability on Dec. 31 , 2010 MI. 28. profit (900,000/30%) MJ. 29. 2010 compensation expense –market value on 12/31/2010 MK. 30. balance per bank statement’ ML.Deposit in transit MM. Outstanding checks MN. Bank service charges MO. Interest income MP.Error in recording check (73,000-37,000) 1,350,000 290,000 200,000 140,000 3,160,000 A 5,950,000 B 13,500,000 500,000 14,000,000 1,200,000 11.67 C 1,900,000 A 10,400,000 2,000,000 8,400,000 210,000 4000 A 3,500,000 (1,000,000) (800,000) 1,700,000 1,500,000 (1,000,000) (800,000) 2,500,000 2,200,000 (1,500,000) 1,700,000 (1,300,000) (1,100,000) B 800,000 1,400,000 300,000 2,500,000C 800,000 1,000,000 1,800,000 A 4,000,000 300,000 3,700,000 A 3,000,000 (145,000) (15,000) 2,840,000 B 1,200,000 C 3,000,000 B 2,100,000 B 1,250,000 200,000 (80,000) 25,000 (75,000) (36,000) A1 PASSERS REVIEW CENTER///150 MQ. 31 Adjusted accounts payable (2.5 M +150T-300T) MR. MS. 32. Accumulated depreciation 12/31/09 MT.Impairment loss MU. Depreciation expense in 2010 (4M/4 years) MV.Total MW. MX. 33. option price (50,000x 100) MY.Share options outstanding MZ. Total NA.Less : Par value of shares issued (50,000 x 100) NB.Share premium NC. ND. 34. Payment for real property NE.Sale of AFS (500 + 100) NF. Purchase of H & E NG. Advances to other companies NH. Cost of patent NI. Net cash used NJ. NK. NL. 35. Cost NM. Less: 2006 and 2009 amortization (5M/ 5 x 2) ( Lease term) NN. Carrying amount 12/31/09 NO. Less: 2010 amortization (3M / 4 years) ( remaining life of asset) NP. CA 12/31/10 depreciation NQ. NR. 36. Adjusted RE 1/1/10 (3M-400,000x 70%) NS. NT. 37. Collections NU. Rent receivable and NV.White off NW. Total NX.Less: Rent receivable beginning NY. Rental revenue-accrual NZ. OA. 38. Accrued liability-12/31/2010 (25 x 50,000) OB. Less: Accrued liability or 12/31/2009 (17 x 50, 000) OC. 2010 compensation expense OD. OE. 39. B=552,250-10B OF. 1,108=550,000 OG. Bonus =(550.000/1.10) OH. OI. 40. Preference dividend in 2009 (10M x 12%) OJ. Preference dividend in 2010 (10M x 24%) OK. Ordinary dividends (130,000 -30,000)x 12 OL. Total dividend declaration OM. 2010 ordinary dividend rate (12.00 divided by 50.00) ON. OO. 41. BI OP.Net purchases OQ. TGAS OR. Less: Adjusted EI (600,000 + 80,000) OS. COS OT. OU. 42. 10% preference shares issued (4M-3M equals 1 M/100 par OV.Subscription price (300,000/25%) OW. Less: Par value OX. Increase in share premium from subscription OY. OZ.Increase in share premium-10% PS (1,500,000-300) PA. Increase in share premium from subscription PB.Share premium from issuance PC. PD.Proceeds from shares issued (1M par +500T share premium) 1,284,000 3,000,000 2,000,000 1,000,000 6,000,000 6,000,000 2,000,000 8,000,000 5,000,000 3,000,000 B 2,000,000 600,000 (800,000) (2,000,000) (150,000) 4,350,000 B 5,000,000 2,000,000 3,000,000 750,000 2,250,000 A 3,220,000 A 5,000,000 400,000 50,000 5,450,000 700,000 4,750,000B 1,250,000 850,000 400,000 B 500,000 B 1,200,000 2,400,000 1,200,000 2,800,000 A 24% 500,000 5,300,000 5,800,000 680,000 5,120,000 A 10,000 1,200,000 500,000 700,000 1,200,000 (700,000) 500,000 1,500,000 A1 PASSERS REVIEW CENTER///151 PE. Divide by shares issued PF. Average issue price PG. PH.43. Fair value on Dec. 31, 2010 PI. Less: Amortized cost on Dec 31,2010 PJ. Unrealized gain on available for sale securities PK. PL. 44. carrying amount of the liability extinguished (5M + 1.2M) PM. Carrying amount of inventory PN.Fair value of shares (40,000 x 60) PO. Gain on extinguishment PP. PQ. 45.Issued price (6M x 110%) PR.Less: FV of bonds ex-warrants PS. Equity component PT. Excess over par from issuance of shares (150-100) x ( 6,000 x 20) PU.Total share premium PV. PW. PX. PY. PZ. QA. QB. QC. 46. PCF (18,000 + 5,000) QD. 47. 2009 Warranty expense (850x250) QE. Less: 2009 Warranty services from 20009 sales (250 x 250) QF. 12/31/09 warranty liability QG. 2010 Warranty expense (1,000 x 250) QH. Total QI. Less : 2010 Warranty services from 2009 and 2010 sales (750 x 250) QJ. 12/31/10 Warranty liability QK. QL. QM. QN. 48. Total shareholders’ equity (5.6 M/140%) QO. Contributed capital (1M + 800,000) QP.Treasury shares QQ. Retained earnings 12/31 QR. QS. Retained earnings on Jan. 1 (SQUEEZE) QT. Net income QU. Dividend declared QV.Error QW. Retained earnings –Dec. 31 QX. QY.49. Rental expense from July 1 to December (75,000 x 8) QZ.Amortization of improvements (500,000/ 5x6/12) RA.Total expense related to operating lease RB. RC. 50. Net cash provided by operating activities RD. (2,800-1,000-400-350-100-280) RE.Interest paid (80,000+20,000) RF. RG. RH. RI. RJ. RK. RL. RM. RN. RO. RP. RQ. RR. 10,000 150 B 5,600,000 5,248,700 251,300 A 6,200,000 (2,500,000) (2,400,000) 1,300,000 B 6,600,000 5,600,000 1,000,000 6,000,000 7,000,000 A 23,000 C 212,500 62,500 150,000 250,000 400,000 167,500 212,500 D 4,000,000 (1,800,000 200,000 2,400,000 1,600,000 B 2,500,000 (1,500,000) (400,000) 2,400,000 450,000 50,000 500,000 B 670,000 C 100,000 B A1 PASSERS REVIEW CENTER///152 RS. RT. RU. RV. RW. RX. RY. RZ. SA. SB. SC. SD. SE. SF. SG. SH. SI. SJ. SK. SL. SM. SN. SO. SP. SQ. SR. SS. ST. SU. A1 PASSERS TRAINING, RESEARCH, REVIEW & DEVELOPMENT COMPANY SV. 2nd Floor Sommerset Bldg., Lopez Jaena St. Jaro, Iloilo City SW. Tel. No.: (033) 320-2728; 09106547262 SX. Email Address: [email protected] SY. SZ.BOARD OF CERTIFIED PUBLIC ACCOUNTANT TA. TB.CERTIFIED PUBLIC ACCOUNTANT Licensure Examination SET A TC. TD. AUDITING PROBLEMS TE. INSTRUCTION: Select the correct answer for each of the following questions. Mark only one answer for each item by shading the box corresponding to the letter of your choice on the answer sheet provided. STICTLY NO ERASURES ALLOWED. Use pencil no. 1 only. TF. TG. MULTIPLE CHOICE: TH. TI. PROBLEM I TJ. The following information was obtained in the audit of the cash account of CHELSEE COMPANY as of December 31, 2010. Assume that the CPA satisfied himself as to the propriety of the cash book the bank statements and the returned checks, except as noted: TK. 1. The bookkeepers bank recon dilation at November 30, 2010 TL. Balance per bank statement P 194,000 TM. Add: Deposit in transit 11,000 TN. Total P 205,000 TO. TP. Less: Outstanding checks TQ. No. 1434 P 1,400 TR. 1562 7,500 TS. 1571 5,800 TT. 1584 8,000 TU. 1591 300 23,000 TV. Balance per books P182,000 TW. TX. A1 PASSERS REVIEW CENTER///153 2. A summary of the bank statement for December 2010. TY. TZ. Balance brought forward P 194,000 UA.Deposits 1,487,000 UB.Total P 1,681,000 UC. Charges (1,325,000) UD. Balance, December 31, 2010 P356,000 UE. 3. Included with cancelled checks returned with the December bank statement were the checks listed below 4. The Chelsee Company discounted its. Own 60 day note for P 90,000 with the bank on December 1, 2010. The discount rate was 6 percent. The accountant recorded the proceeds as a cash receipt at the face value of the note. 5. The accountant records customers dishonoured checks as a reduction of cash receipts. When the dishonoured checks are red posited they are recorded as regular cash receipt. Two NSF checks for P 1,800 and P 2,200 were returned by the bank during December. Both checks were redeposit and were recorded by the accountant. 6. Cancellations of Chelsee Company checks are recorded by a reduction of cash disbursement 7. December bank charges were P200; in addition, a P100 service charge was made in December for the collection of a note receivable in November. These charges were not recorded on the books. 8. Check no. 1434 listed in the November outstanding checks was drawn in 2008. Since the payee cannot be located, the president of Chelsee Company agreed to the CPA’s suggestion that the check be written back into the accounts by a journal entry. 9. Outstanding checks at December 31,2010, totalled P 49,400 including checks 1432 and 1584 10. The cut-off bank statement disclosed that the bank had recorded a deposit of P 24,000 on January 2, 2011. The accountant had recorded this deposit on the books of December 31, 2010 and then mailed the deposit to the bank. UF. UG. UH. UI. UJ. UK. UL. Cancelled checks returned with the December Bank statement UM. UN. UO. UP. Comments Num Date Amo b of u e ch n r ec t k o f c h e c k UQ. UR. US.7 UT. This check was in payment of invoice for 57,500 and was recorded in 1562 11/28/ 5 the cash book as P 7,500 10 0 UU. UV.11/ UW. UX.This check was in the payment of an invoice for P 5, 800 and was 1571 28/ 5800 recorded in the cash book as P 5,800 10 UY. 1 UZ. 12/ VA. 1 VB. Examination of this check revealed that it was unsigned A discussion 5 04/ , with the client disclosed that it had been mailed inadvertently before it 8 10 5 was signed. The check was endorsed and deposited by the payee and 3 0 processes by the bank even though it was a legal nullity. The check 0 was recorded in the cash disbursements journal. VC.1 VD.12/ VE. 8 VF. This check replaces 1584. Which was returned by the payee because it 5 12/ , was mutilated? Check 1584 was not cancelled on the books 8 10 0 8 0 0 VG. VH.12/ VI. 2 VJ. This was counter check drawn at the bank by the president of the A1 PASSERS REVIEW CENTER///154 19/ 10 , company as a cash advance for travel expense. The president 0 overlook informing the bookkeeper about the check 0 0 VK. VL. 12/ VM. VN.The drawer of this check was the Celsea Company 20/ 3,00 10 0 VO. VP. 12/ VQ. VR.This check had been NSF and returned to the payee because the bank 1595 20/ 3,50 had erroneously believed that the check was drawn by the Chelseen 10 0 company. Subsequently, the payee was advised to redeposit the check VS. 1 VT. 01/ VU.1 VV. This check given to the payee on December 30, 2010 as a post dated 5 05/ 0 check with the understanding that it would not be deposited until 9 11 0 January 5. The check was not recorded on the books in December. 9 , 0 0 0 1. What is the correct amount of outstanding checks on December 31? a. P41,400 c. P48,000 b. P33,250 d. P40,000 2. What is amount of cash receipts per book in December? a. P1,496,900 c. P1,495,100 b. P1,504,900 d. P1,487,000 3. What is amount of cash disbursement per book in December? a. P1,254,850 c. P1,256,850 b. P1,252,850 d. P1,248,850 4. What is the cash in bank balance per book as of December 31? a. P426,050 c. P430,050 b. P428,250 d. P343,050 5. What is the adjusted cash balance as of December 31? a. P343.000 c. P347,000 b. P340,200 d. P344,200 e. f. PROBLEM NO. 2 g. CALACHUCHI CORP. h. ACCOUNT i. INVOICE BALANCE j. k. l. m. AMOU CU D D NT n. Ar o. P p. 1 r. 14,600 s. 21,180 q. 1 t. Na u. 2 v. 0 x. 2,000 y. 8,920 w. 0 z. Sy aa. 3 ab. 1 ad. 20,000 ae. 10,600 ac. 1 af. Tri ag. 4 ah. 1 aj. 23,140 ak. 22,000 A1 PASSERS REVIEW CENTER///155 ai. 1 al. Uy am. 3 an. 1 ap. 19,200 aq. 12,400 ao. 1 ar. Xa 6. 7. 8. 9. as. 1 at. 0 au. 17,400 av. aw. The estimated bad debt rates below are based on Calachuchi crop’s receivable collection experience. ax. Age of Accounts Rate ay. 0-30 days 1% az. 31-60 days 1.5% ba. 61-90 days 3% bb. 91-120 days 10% bc. Over 120 days 50% bd. be. bf. bg. The allowance for bad debts account had a debit balance of P5,500 on December 31, 2010 before adjustment. The company’s accounts receivable under 61-90 days category should be a. P 32,600 c. P 44,600 b. P44,320 d. P 42,000 The company’s accounts receivable under 91-120 category should be a. P 38,320 c. P 29,400 b. P 40,000 d. P 12,000 The allowance for bad debts to be reported on the statement on financial position at December 31, 2010 is a. P 9,699 c. P 4,199 b. P 15, 199 d. P5, 500 What entry should be made on December 31, 2010, to adjust the allowance for bad debts account? a. Bad debt expense 15,199 10. Allowance for bad debts 15,199 a. Bad debt expense 4,199 11. Allowance for bad debts 4,199 a. Allowance for bad debts 5,500 12. Bad debts expense 5,500 a. Bad debt expense 9,699 13. Allowance for bad debts 9,699 14. 15. PROBLEM NO. 3 16. The MALAWI COMPANY is an importer and wholesaler. Its merchandise is purchased from a number of suppliers and is warehoused until sold to consumers. 17. 18. In conducting his audit for the year ended December 31,2010, the company CPA determined that the system of internal control was good. Accordingly, he observed the physical. Inventory at an interim date, November 30, 2010 , instead of at year end. 19. 20. The following information was obtained from the general ledger. 21. Inventory, January 1, 2010 22. P 90,000 23. Inventory, November 30, 2010 24. 225,000 25. Sales for eleven months ended November 30, 2010 26. 800,000 27. Sales for year ended December 31, 2010 28. 950,000 29. Purchases for eleven months ended November 30,2010 (before audit adjustments) 31. Purchases for year ended December 31, 2010 (before audit adjustment) 33. 30. 720,000 32. 810,000 A1 PASSERS REVIEW CENTER///156 34. Additional Information is as follow: a. Goods received on November 28 but recorded as purchases in December 10,000 b. Deposits made in October 2010 for purchases to be made in 2011 but charge 35. To purchase 14,000 c. Defective merchandise returned to suppliers: 36. Total at November 30, 2010 5,000 37. Total at December 31, 2010, excluding November items 7,000 d. Goods shipped in November under FOB destination and received in 38. December were recorded as purchase in November 18,500 e. Through the carelessness of the client warehouseman, certain goods 39. Were damaged in December and sold in the same month at its cost. 20,000 f. Audit of the clients November inventory summary revealed the following: 40. Items duplicated 3,000 41. Purchases in transit 42. Under FOB shipping point 12,000 43. Under FOB destination 18,500 44. Items counted but not included in the inventory summary 7,000 45. Errors in extension that overvalued the items 4,000 46. 11. What is the correct amount of net purchases for the month of December 2010? a. P83,000 c. P82,500 b. P91,500 d. P101,500 12. The correct inventory on November 30, 2010 is a. P206,500 c. P237,000 b. P214,500 d. P218,500 13. What is the gross profit for eleven months ended November 30, 2010? a. P234,000 c. P224,000 b. P217,000 d. P237,500 14. 15. 16. 17. 18. What is the cost of sales ratio for eleven months ended November 30, 2010? a. 73% c. 28% b. 70% d. 72% 19. What is the estimated inventory on December 31, 2010? a. P183,100 c. P184,400 b. P175,900 d. P190,000 e. f. PROBLEM NO. 4 g. You have been asked by the proprietor of the SOMALIA CO. to verify the accountability of the cashier bookkeeper, who was allowed to take a vacation leave a few days ago h. i. A. The bank reconciliation statements prepared by the cashier bookkeeper are presented below: j. k. November 30, 2010 l. Balance per bank statement P 21,500 m. Cash on hand 500 n. Total 22,000 o. p. Outstanding checks: q. No. 2520 P2, 000 r. 2521 1,400 s. 2522 1,900 (3,300) t. Erroneous bank charge 2,000 u. Erroneous bank credit (500) v. Book balance P20,200 w. x. December 31, 2010 y. Balance per bank statement P 135,000 z. Cash on hand 6,300 aa. Total 141,300 ab. Outstanding checks ac. No. 2674 P31, 000 ad. 2675 10,300 ae. 2676 5,000 (41,300) A1 PASSERS REVIEW CENTER///157 a. b. c. 16. 17. 18. 19. 20. af. Erroneous bank charge 3,000 ag. Erroneous bank credit (600) ah. Book balance P 102,400 ai. aj. B. The cash in Bank account in the general ledger shows the following debits and credits during December ak. Cash in bank al. Dec. av. Dec. am.1balance aw. 1Checks Issued P 20,200 P2,000 an. 2 Received from ax. 5 Checks Issued customers 4,500 5,200 ao. 7 Received from ay. 14 Checks Issued customers 5,000 31,000 ap. 12 Received from az. 24 Checks Issued customers 20,000 46,000 aq. 17 Received from ba. 28 Checks Issued customers 30,000 7,600 ar. 23 Received from bb. customers 9,000 bc. as. 27 Received from bd. 31balaced customers 70,000 102,400 at. 31 Received from be. Total customers 48,500 P198,200 au. Total P198,200 bf. bg. C. the following summarized transactions were taken from the bank statements for the months of December 2010. bh. Balance, December 1, 2010 bi. Total deposits bj. The total deposits per bank statement include: Collection of notes receivable P 5,000 Correction of November erroneous bank charge 2,000 December 10 deposit of Lava, Inc. Credited in error to Somalia 600 bk. Total P3, 500 bl. bm.D. cash on hand per count in the morning of January 2, 2011 amounted to P 6, 300 bn. E. before leaving his company for a one week vacation, the proprietor had left several signed blank checks that the cashier bookkeeper had cashed for his personal use. bo. bp. bq. br. bs. bt. What is the adjusted cash balance on November 30, 2010? a.P16,500 c.P20,200 b.P13,200 d.P14,500 The amount of uncounted in December is a.P11,000 c.P9,000 b.P13,200 d.P15,100 The amount of unrecorded/unsupported disbursements in December is a.P15,100 c.P7,000 b.P10,900 d.P5,000 What is the total cash shortage as of December 31, 2010? a.P26,000 c.P33,000 b.P15,100 d.P7,000 What is the adjusted cash balance on December 31, 2010 a.P102,400 c.P87,400 b.P125,000 d.P111,400 e. A1 PASSERS REVIEW CENTER///158 f. g. h. i. j. k. l. Indicators that the assets of the unit were impaired. The carrying amounts of the assets and liabilities of the unit at December 31, 2010 were: Buildings Accumulated depreciation –buildings Factory machinery Accumulated depreciation –machinery Goodwill P4, 200,000 (1,800,000) 2,200,000 (400,000) 150,000 m. Inventory 800,000 n. Receivables 400,000 o. Allowance for doubtful accounts p. Cash (50,000) 200,000 q. Accounts payable 300,000 r. 200,000 Loans - 21. 22. 23. 24. 25. Depreciated at P 600,000 per annum Depreciated at P 450,000 per annum s. t. POTPOT: determined the value use of the unit to be P 5,350,000. The receivables were considered to be collectible, except those considered doubtful. u. v. During the year 2011, POTPOT increased the depreciation charge on building to P650, 00 per annum, and to P 500,000 per annum for factory machinery. The inventory on hand at January 1, 2011 was sold by the end of the year, at December 31, 2011, POTPOT Company, due to a retum in the market to use of the traditional barrels for wines and an increase in wine production, assessed the recoverable amount of the cash generating unit to be P 300,000 greater than the carrying amount of the unit. As a result, POTPOT recognized a reversal of the impairment loss. w. What amount of impairment loss on December 31, 2010 should be allocated to inventory? a.P56,000 c.P32,000 b.P 28, 829 d.P 0 What is the factory Machinery net carrying amount (after allocation of impairment kiss) on December 31,2010? a.P 1, 735, 135 c.P 1,800, 000 b.P 1, 674, 000 d.P 1, 728, 000 What amount or reversal of impairment loss should be recognized on December 31, 2011? a.P 300, 000 c.P 168, 000 b.P 268, 000 d.P 200, 000 Assume that the recoverable amount at December 31, 2011 was P 200,000 greater than the carrying amount of the cash generating unit. What is the net carrying amount of the buildings after recognition of the impairment recovery? a.P 1,313, 219 c.P 1, 800, 000 b.P 1, 768, 781 d.P 1,750, 000 Assume that the recoverable amount at December 31, 2011 was P 200,000 greater than the carrying amount of the cast generating unit and that the recoverable amount of the Buildings was P 1,750,000. What is the next carrying amount of the factory machinery after the recognition of the impairment recovery? a.P 1,332,000 b.P1,313,219 c.P1,228,000 d.P 1,324,000 26. 27. 28. 29. 30. Goding , Inc. preference 60,000 31. Sonata Co. ordinary 32. (30% ownership) investment in 1,200,000 600,000 1,290,000 34,200,000 33. 34. trading 33,900,000 Associate A1 PASSERS REVIEW CENTER///159 35. Jordan Co. ordinary 750,000 2,025,000 1,500,000 available for sale 36. 37. 38. BORDO COMPANY’S investment had the following market values at December 31, 2010 39. Omar Co. ordinary P 3,060,000 40. Goding, Inc. preference 1,290,000 41. Sonata Co. ordinary 33,450,000 42. Jordan Co. ordinary 1,700,000 43. 44. Assume that all of the above securities were acquired in 2009 and none of the indicated declines in market value are considered other than temporary. 45. 46. Which of the following should be reported in Bordo’s 2009 income statement? a.Unrealized loss of P 60,000 c.Unrealized loss of P 200, 000 b.Unrealized loss of P 585, 000 d.Unrealized loss of P 150, 000 47. What amount of loss in the above securities should be included in Bordo’s statement of comprehensive income for the year ended December 31, 2009 as component of other comprehensive income? a.P 675,000 c.P 585, 000 b.P 465, 000 d.P525,000 48. Assume that Jordan Co.s ordinary shares market decline is other than temporary. What valuation entry is required for this investment at December 31, 2009 under this change in assumption? a.Impairment loss 525,000 49. Investment in AFS securities 525,000 a.Unrealized loss on AFS securities 525,000 50. Investment in AFS securities 525,000 a.Impairment loss 525,000 51. Unrealized loss on AFS securities 525,000 a.No valuation entry is necessary 525,000 52. 53. Assume that the investment categories remain the same and that all declines in 2009 and 2010 are temporary except for the 2009 decline in Jordan Co.s ordinary shares 54. In its 2009 income statement, Bordo should report a.Unrealized gain of P 150, 000 c.Gain on impairment recovery of P 200, b.Unrealized gain of P 410, 000 000 d.Unrealized gain of P 210, 000 55. What amount of gain should be included in Bordo’s statement of comprehensive income for the year ended December 31, 2010 as component of other comprehensive income? a.P 0 c.P 325,000 b.P 200,000 d.P525,000 e. f. 2008 P 127,000 g. 2009 150,000 h. 2010 128,500 i. j. You are performing the audit for the year ended December 31,2010. During your examination you discover the following errors a. As a result of errors in the physical count, ending inventories were misstate as follows: k. December 31, 2009 P14, 000 overstated l. December 31, 2010 P23, 000 understated m. b. On December 29, 2010. Dominica recorded as a purchase, merchandise in transit which cost P 15,000. The merchandise was shipped FOB Destination and had not arrived by December 31. The merchandise was not included in the ending inventory. n. c. Dominica records sales on the accrual basis but failed to record sales on account made the end of each year as follows o. 2008 P4, 000 p. 2009 5,000 q. 2010 3,500 r. s. t. u. v. d. The company failed to record accrued office salaries as follows A1 PASSERS REVIEW CENTER///160 56. 57. 58. 59. 63. w. December 31, 2008 P 10,000 x. December 31, 2009 14,000 y. e. On March 1, 2009, a 10% stock dividend was declared and distributed. The par value of the shares amounted to 10,000 and market value was P 13,000. The stock dividend was recorded as follows z. Miscellaneous expense 13,000 aa. Ordinary share capital 10, 000 ab. Retained earnings 3,000 ac. f. On July 2009, Dominica acquired a three year insurance policy. The three year premium of P6,000 was paid on that date , and entire premium was recorded as insurance expense. ad. g. On January 1, 2010 Dominica retired bonds with a book value of P 120, 000 for P 106, 000 the gain was incorrectly deferred and is being amortized over 10 years as a reduction of interest expense on other outstanding obligations. ae. What is the adjusted net income for the year ended December 31, 2008? a.P 133,000 c.P121,000 b.P 117, 000 d.P 113, 000 What is the adjusted net income for the year ended December 31, 2009/ a.P 151, 000 c.P 104, 400 b.P 187, 000 d.P 203, 600 What is the adjusted net income for the year ended December 31, 2010? a.P129, 600 c.P 104, 400 b.P131,000 d.P 203, 600 What adjusting entry should be made on December 31, 2010, to correct the error describe in item B? a. Purchases 15,000 60. Accounts payable 15,000 a. Accounts payable 15,000 61. Purchases 15,000 a. Accounts payable 15,000 62. Cash 15,000 a. No adjusting journal entry is necessary The adjusting entry on December 31, 2009, to correct the error described in item E should include a debit to a. Retained earnings of P 16, 000 b. Ordinary share capital of P 10,000 c. Share premium of P 3,000 d. Miscellaneous expenses of P 3,000 64. 65. PROBLEM NO. 8 66. 67. In making the first adult of the Delivery Equipment account of DELTA CORPORATION as of December 31, 2010, you encounter the following facts. 68. DAT 69. PARTICULARS 70. DEBIT 71. CREDIT E 72. 1/1/0 73. Tracks 1,2,3, and 4 74. P3,200 75. 8 ,000 76. 3/15/ 77. Replacement of Truck 3 tires 78. 25,000 79. 09 80. 7/01/ 81. Truck 5 82. 800,00 83. 09 0 84. 7/10/ 85. Reconditioning of truck 4, which was damaged in a 86. 35,000 87. 09 collision 88. 9/1/0 89. Insurance recovery on truck 4 accident 90. 91. P33,000 9 92. 10/0 93. Sale of truck 2 94. 95. 600,000 1/09 96. 4/01/ 97. Truck 6 98. 1,000, 99. 150,000 10 000 100. 101. Repainting of truck 1 102. 103. 5/2/10 27,000 104. 105. Truck 7 106. 107. 6/30/10 720,000 108. 109. Cash received on lease of truck 7 110. 111. 2 A1 PASSERS REVIEW CENTER///161 12/1/10 2,000 112. 113. Accumulated Depreciation 114. 115. PARTICULARS 116. 117. C DATE DEBIT REDIT 118. 119. Depreciation expense 120. 121. P 12/31/08 300,000 122. 124. 125. 3 123. Depreciation expense 12/31/09 00,000 126. 128. 129. 3 127. Depreciation expense 12/31/10 00,000 130. 131. 132. 133. 134. Additional information: 1. On July 1,2009, Truck 3 was traded in for a new truck , truck 5 costing P 850,000 the selling party allowed a P 50, 000 trade in value for the old truck 2. On April 1, 2010, truck 6 was purchased for P 1,000,000; truck 1 and cash of p 850,000 being given for the new truck 3. You are instructed by the senior in charge of the audit to accept the depreciation rate of 20% by unit basis 4. Unit cost of trucks 1 to 4 is at P 800,000 each 135. 136. The net increase (decrease) in retained earnings should be a.(P1,102,000) c.P 80,000 b.(P545,000) d.(P1,022,000) 137. What is the loss on trade in of Truck No. 3? a.P430,000 c.P560,000 b.P510,000 d.P0 138. What is the gain on sale of Truck no. 2? a.P120,000 c.P80,000 b.P200,000 d.P 0 139. What is the loss on trade in of Truck No.1? a.P290,000 c.P150,000 b.P170,000 d.P 0 140. What is the correct cost of truck No. 5? a.P850,000 c.P900,000 b.P800,000 d.P560,000 141. What is the correct cost of truck No. 6? a.P440,000 c.P800,000 b.P305,000 d.P1,000,000 142. What is the correct depreciation expense for 2009? a.P725,000 c.P605,000 b.P305,000 d.P600,000 143. The entry correct depreciation expense for 2010? a.P552,000 c.P712,000 b.P592,000 d.P300,000 144. The entry correct the depreciatin charges for the years 2008/through 2010 should include a credit to accumulated depreciation of a.P645,000 c.P900,000 b.P937,000 d.P292,000 145. The balance of the Delivery Equipment accounts at December 31, 2010 should be. a.P5,770,000 c.P4,170,000 b.P3,320,000 d.P3,370,000 e. f. PROBLEM NO. 9 g. h. The GUNDING Co. is on calendar year basts. The following data were found during your audit. i. a. Goods in transit shipped FOB shipping point on December 28 by a supplier in the amount of P 100,000 had been excluded from the inventory and further testing revealed that the purchase had been recorded. b. Goods costing P 30, 000 had been received, included in inventory, and recorded as a purchase. However, upon your inspection the goods were found to be defective and would be immediately returned. A1 PASSERS REVIEW CENTER///162 c. Materials costing P170,000 and billed on December 30 at a selling price of P 264,000 had been segregated in the warehouse of shipment to a customer. The materials had been excluded from inventory as a signed purchase order had been received from the customer. Terms FOB destination. d. Goods costing P 70, 000 was out on consignment with Gundara Inc. since the monthly statement from Gundara listed those materials as on hand, the items had been excluded from the final inventory and invoiced on December 31 at P 80,000. e. The sale of P 150,000 worth of materials and costing p 120,000 had been shipped FOB point of shipment on December 31. However, this inventory was found to be included in the final inventory. f. Goods costing P 100,000 and selling for P140,000 had been segregated , but not shipped at December 31, and were not included in the inventory. A review of the customer’s purchase order set forth terms as FOB destination. The sale had not been recorded. g. Your client has an invoice from a supplier, terms FOB shipping point, but the goods had not arrived as yet. However, these materials costing P134,000 had been included in the inventory count, nut no entry had been made for their purchase. h. Merchandise costing p 200,000 had been recorded as a purchase but not included in Inventory. Terms of sale are FOB shipping point according the supplier warehouse which had arrived by December 31. j. k. l. m. n. o. Further Inspection of the clients records revealed the following December 31 balances: Inventory, P 1,350,000; Accounts receivable, P 630,000 ; accounts payable, P 690,000; sales P6,032,000; Purchases, P 3, 150,000; Net Income , P727,000. p. Based on the preceding information, determine the adjusted balances of the following: 46. Inventory a.P700,000 c.P1,900,000 b.P934,000 d.P1,840,000 47. Accounts receivable a.P286,000 c.P146,000 b.P380,000 d.P405,000 48. Purchases a.P3,354,000 c.P3,254,000 b.P3,150,000 d.P3,120,000 49. Sales a.P5,848,000 c.P5,683,000 b.P6,376,000 d.P5,768,000 50. Net Income a.P769,000 c.P829,000 b.P569,000 d.P709,000 e. PROBLEM 10: f. Presented below are COMBANTRIN COMPANY’S comparative statements of financial position and income statements g. COMBANTRIN COMPANY h. COMPARATIVE STATEMENTS OF FINANCIAL POSITION i. December 31, 2010 and 2009 j. Assets k. 2010 l. 2009 m. Current assets n. o. p. Cash q. P119,000 r. P98,000 s. Accounts receivable t. 312,000 u. 254,000 v. Inventory w. 278,000 x. 239,000 y. Prepaid expenses z. 35,000 aa. 21,000 ab. Total current assets ac. 744,000 ad. 612,000 ae. Available for sale securities af. 59,000 ag. ah. Property, plant and equipment ai. 536,000 aj. 409,000 ak. Accumulated depreciation al. (76,0000) am. (53,000) an. Total noncurrent assets ao. 519,000 ap. 356,000 A1 PASSERS REVIEW CENTER///163 aq. Total assets ar. at. au. Liabilities and shareholders’ equity av. aw. Current assets ax. Accounts payable ba. Accrued expenses bd. Dividends payable bg. Total current liabilities bj. Notes payable-due 2012 bm.Total liabilities bp. bq. Shareholder equity: br. Ordinary share capital bu. Retained earnings bx. Total shareholders’ equity ca. Total liabilities and shareholders’ equity cd. ce. cf. P1,263,000 ay. bb. be. bh. bk. bn. as. P968,000 P212,000 98,000 10,000 350,000 125,000 475,000 bs. bv. by. cb. az. bc. bf. bi. bl. bo. 600,000 188,000 788,000 P1,263,000 P198,000 76,000 _________ 274,000 __________ 274,000 bt. 550,000 bw. 144,000 bz. 694,000 cc. P968,000 cg. COMBANTRIN COMPANY ch. CONDENSED COMPARATIVE INCOME STATEMENTS ci. For the Years Ended December 31, 2010 and 2009 cj. ck. Net sales 2010 cl. Cost of goods sold P3, 561,000 cm. Gross income 2,789,000 cn. Expenses 772,000 co. Net income 521,000 cp. P251,000 cq. cr. Additional information for COMBANTRIN COMPANY a. All accounts receivable and accounts payable relate to trade merchandise b. The proceeds from the notes payable were used to finance plant expansion c. Ordinary shares were sold to provide additional working capital cs. ct. Compute the following for 2010. 146. Cash collected from accounts receivable (assume all sales are on account) a.P3,619,000 c.P3,561,000 b.P3,503,000 d.P3,249,000 147. Total purchases (assume all purchases of inventory are on account) a.P2,828,000 c.P2,550,000 b.P2,789,000 d.P2,750,000 148. Cash payments made on accounts payable to suppliers a.P2,630,000 c.P2,828,000 b.P2,842,000 d.P2,814,000 149. Net cash provided by operations a.P689,000 c.P222,000 b.P191,000 d.P199,000 150. Cash receipts not provided by operations a.P175,000 c.P50,000 b.P177,000 d.P125,000 e. f. g. h. i. j. k. l. m. n. 2009 P3, 254,000 2,568,000 686,000 486,000 P200,000 A1 PASSERS REVIEW CENTER///164 o. p. q. r. s. t. u. v. w. x. y. z. aa. ab. ac. ad. ae. af. ag. ah. ai. aj. ak. al. am. A1 PASSERS TRAINING, RESEARCH, REVIEW & DEVELOPMENT COMPANY nd an. 2 Floor Sommerset Bldg., Lopez Jaena St. Jaro, Iloilo City ao. Tel. No.: (033) 320-2728; 09106547262 ap. Email Address: [email protected] aq. ar. BOARD OF CERTIFIED PUBLIC ACCOUNTANT as. at. CERTIFIED PUBLIC ACCOUNTANT Licensure Examination SET A au. av. BUSINESS LAW AND TAXATION aw. INSTRUCTION: Select the correct answer for each of the following questions. Mark only one answer for each item by shading the box corresponding to the letter of your choice on the answer sheet provided. STICTLY NO ERASURES ALLOWED. Use pencil no. 1 only. ax. ay. MULTIPLE CHOICE: az. 1. A, B and C are partners in a universal partnership of profits engaged in operation of taxis. The partners contributed: ba. A- 100 Toyota Vios bb. B- 3,000 aqm land bc. C- Fuel and oils bd. Which of the following is correct? a. In case of loss due to fortuitous event, the partnership bears the loss b. The partnership becomes the owner of the things contributed to the partnership c. The partners retained the ownership over the things they contributed hence; they will bear the risk of loss. d. The partnership begins from the moment of the execution of the contract , except if stipulated otherwise 2. Which of the following statements is not correct? a. The creditor acquires real right to the fruits of the thing from the time they have been delivered b. An oral sale of land made by its owner in unenforceable c. The buyer acquires real right to the fruits of the thing from the perfection of the sale d. A sale of land made by an agent without a written authority from the owner is void 3. A 16 years old sold his property to B, who did not notice that A is a minor. One year later B sold the property to C who knew that A is a minor. Can A ask for annulment of the sale? a. Yes, because C is in bad faith b. No, because B is in good faith c. No, because B is not capable of returning the property A1 PASSERS REVIEW CENTER///165 d. Yes, whether C is in good faith or in bad faith because the title of B is voidable 4. A promissory note reads I Promise to pay A,B, C and D P24,000 Sgd. E, F and G. B can collect from E a. P 24,000 c. P 6,000 b. P 8,000 d. P 2,000 5. Today, S sold to B in S is name the land of O, without authority from O, B bought the land in good faith. The contract provides that the delivery will be 30 days from today. The contract is a. Valid if S owns the land on the day of delivery b. Void , because S was not the owner of the land when the contract was perfected c. Unenforceable, because S had no authority to sell the land d. Voidable, because S is guilty of fraud 6. A, B and C are solidary debtors of D for P90, 000 where the due date is on April 1, 2011. On January 1, 2011, A paid D P90, 000. Later A demanded reimbursement from B. if B effects reimbursement to A on July 1, 2011. Which of the following is correct? a. B pays A P60,000 plus interest from January 1,2011 to July 1, 2011 b. B pays A P30,000 plus interest from January 1,2011 to July 1, 2011 c. B pays A P30,000 plus interest from April 1,2011 to July 1, 2011 d. B pays A P45,000 plus interest from April 1,2011 to July 1, 2011 7. S sold his agricultural land to B for P1M .The FMV is P1.5 M. B. paid s in counterfeit peso bills. The sale is a. Void, because the cause is illegal b. Valid, because the cause is the selling price of PIM c. Rescissible, because of lesion which is more than ¼ of the value of the property d. Voidable, because of the fraud committed by B. 8. As regard cash dividend, which of the following is correct/ a. Declared by the board of majority vote and ratified by the stockholders by the majority vote. b. Declared by the board of majority vote and ratified by the stockholders by ½ vote of the outstanding capital stock c. Declared by the board of majority vote and need not be ratified by the stockholders d. Declared by the board by 3/2 vote and need not be ratified by the stockholders 9. 10. 11. 12. 13. A barrowed from B P2M and the secure its payment A orally agreed to deliver his 8 hectare agricultural land by way of antichresis and to pay 12% interest per annura. Which is correct? a.The contracts of loan and antichresis are both valid b.The contracts of loan is valid but antichresis is void c.The contracts of antichresis is valid but loan is void d.Both contracts of loan and antichresis are void 14. Elements of contracts of pledge and mortgage , except. a.Both are necessary contracts b.Both are real contracts c.Both cannot exist without a valid obligation d.In case of default, the creditors cannot appropriate the things pledged or mortgaged 15. Which of the following statements is not correct a.An agreement to constitute a commodatum is binding and is perfected from the moment there is meeting of minds b.Fixed, savings and current deposits of money in banks and similar institutions shall be governed by the provisions concerning muthium c.Precarium is a kind of commodatum where the bailor may demand the thing loaned at will d.In case of extraordinary expenses , the debtor in mutuum will shoulder 50% if he is in actual use of the thing barrowed 16. A contributed PIM, B contributed PIM and C contributed services. they agreed to divide the profits and losses equality . in case of loss of P3M for how much, if any, is C liable? a.Nothing because an industrial partner is exempt from losses b.PIM but with reimbursement from A and B equality c.A and B shall shoulder the loss equality at P1.5M each d.PIM, as stipulated 17. As regards the corporate by-laws , which of the following is true? a.To adopt, amend or repeal 3/2 of the outstanding capital stock is needed b.To delegate to the board of directors the power to amend, repeal or adopt new by-laws, majority of the outstanding capital stock c.To revoke the power delegated to the board to amend, repeal or adopt new by-laws 3/2 of the outstanding capital stock d.To amend, repeal or adopt new by-laws, majority of the outstanding capital stock. 18. A contract of antichresis is a.A contract of mortgage whereby possession of the mortgaged property is given to the creditor A1 PASSERS REVIEW CENTER///166 19. 20. 21. 22. b.A contract of loan secured by a real property, where possession is retained by the debtor c.A contract of loan without security d.A consensual contract as it is perfected by mere consent A subscribed to 1,000 voting shares of stock of X Corporation. She paid 25% of the said subscription during the stockholders meeting can A vote all her subscribed shares/ a.No, because the subscription has not yet been fully paid b.No, because A’s shares have become delinguent which cannot be voted c.Yes, as to the paid percentage of subscription d.Yes, because unpaid shares not delinquent can be voted To defraud his creditor , A sold his property to B (who was in good faith). Later B sold the property to C, who was in bad faith. May the creditor rescind the sale? a.Yes, because C, is bad faith b.No, because B is in good faith c.No, because B is not capable of returning the property d.Yes, because the contract is rescissible A was having his house repaired by B, who needed construction materials , so A orally told the seller C, give B the materials , I shall be responsible. I shall pay in 30 days. C delivered the materials As a result, a.The contract is unenforceable because A made an oral agreement to answer for the debt of B b.The contract is enforceable because A did not make a special promise to answer for the default of another person c.The contract shall be enforceable if there is ratification by A d.The contract shall be unenforceable if the value is at least P 500.00 Under the negotiable instruments law to be a holder in due course , the holder must have acquired the instrument before it is overdue . does this apply to the payee to whom the maker issued an overdue note? a.Yes, because the payee is still considered a holder b.Yes, because issuance to the payee is not considered a negotiation c.No, because the law does not distinguish between payee and subsequent holder d.No, because the payee is privy to the contract between himself and the maker 23. 24. 25. 26. 27. 28. 29. In which of the following is demand necessary to make the debtor in delay in the performance of his obligation? a.When the time of performance is of the essence b.When the time of performance has been stipulated c.When the law so provides d.When demand would be useless 30. With written authority from his principal, the agent sold orally the land of the principal. The sale is a.Rescissible c.Uncoforceable b.Voidable d.Void 31. Without authority from B, A orally sold B’s house in B’s name. The sale is a.Rescissible c.Uncoforceable b.Voidable d.Void 32. S sold B his land on September 19, 2010. On September 25, an absolute deed of sale was executed and notarized on September 30, the sale was registered with the Registry of Deeds , On October 5, B took actual possession of the land by building a fence thereon. When did B acquire ownership of the land? a.On September 19 c.On September 30 b.On September 25 d.On October 5 33. S sold to B his car on September 19, 2010. On September 25, an absolute deed of sale was executed and notarized . on September 30, the sale was registered with the LTO. On October 5, B took actual possession of the car. When did B acquire title to the car? a.On September 19 c.On September 30 b.On September 25 d.On October 5 34. A and B are co owners of a percel of land A donated his share to C can B redeem the said share from C? a.Yes, because the law looks with disfavour at co-ownership b.No, because legal redemption applies only in case of onerous transfers c.No, unless he enforce his right through count action d.Yes, but in proportion to his interest in the land as co-owner 35. D’owes C a sum of money with M as mortgagor of his land to secure the loan. Is the mortgage valid even if the mortgagor is not the debtor? a.Yes, provided M is the absolute owner of the property mortgaged b.Yes, provided it is in writing and registered c.No, the mortgagor must be the debtor himself A1 PASSERS REVIEW CENTER///167 d.No, unless the mortgagor is a co-debtor 36. In three of the following case s, the agency cannot be revoked without a just cause except: a.It is means of fulfilling an obligation already existing b.A bilateral contract depends upon it c.It is coupled with interest common to the great and principal d.Partner is appointed manager in the articles of partnership 37. P appoints A as guarantee commission agent to sell his products A sold them to B who failed to pay despite the diligent efforts of A in collecting is A still liable for the purchase price? a.Yes, because of the guarantee commission given to him b.No, because he was not at fault in the collection c.No, if her returns to guarantee commission d.Yes, but he is not entitled anymore to the commissions 38. P appointed A1 and A2 as his agents for a common transaction and they agreed that they shall be solidarity liable for P for damages in case of violations of their obligations as such. What is the agent at fault the only one liable for damages even if solidarity has been stipulated? a.If the other agent is not at fault b.Both of them shall be liable always since solidarity was agreed upon c.If the one at fault shall answer for all the damages d.If the one at fault acted in excess of authority 39. A, B, and C entered into an oral contract of partnership each contributing P2M each of the common fund but failed to register the partnership with the SEC. In the partnership valid/ a.No, because every contract of partnership having a capital of three thousand pesos or more in money or property must be in public instrument and registered with the SEC b.Yes, because public instrument is necessary is case of contributions of immovable’s c.Yes, because a partnership contract does not require a specific form to be valid d.No, because registration with the SEC is essential for a partnership to be valid and acquire judicial personality 40. A and B are co-owners of a parcel of land from where they share the profits equally as co-heirs in inheritance. Is there a partnership a.There is a prima facie presumption of partnership because of the equal sharing of profits b.There is no partnership because co-ownership by itself does not establish a partnership despite the sharing of profits c.There is no partnership since in partnership, division of profits is not always necessary among partners d.There is a prima facie presumption of partnership they being co-owners and co-possessors 41. 42. 43. 44. 45. A and B are partners in AB partnership. While A was performing his duties as a partner in the course of business , he negligently cause damage to X and third person. Who shall be liable to X and up to what extent? a.Only the partnership is still liable it being a juridical person separate and distinct from the partners b.Only A shall be liable for he is the only one at fault c.Both A and B shall be liable solidarity to X d.A,B and the partnership are all liable solidarity to X 46. A, B, and C as partners in a partnership stipulated that A shall not share in the profits and losses. Is the agreement valid? a.Yes, among the partners but not valid to third persons b.Yes, valid as long as third persons are not prejudiced c.Not valid stipulations exempting a partner from sharing in the profits and losses d.Not valid as against third persons or partnership creditors 47. In the matter of management of the corporation this is supreme a.President c.Stockholders b.Chairman of the Board of Directors d.Board of Directors e. f. 34-37 A promissory note reads g. “I promise to pay P or order Php 100,000. Sgd. M, The promissory note was issued by M to P in payment of merchandise purchased from P. However, P delivered only Php 60,000. Later P. made a qualified endorsement to A, who was not aware of the partial absence of consideration. Thereafter, A made a special endorsement to B, then B made a blank endorsement to C both B and C are aware of the partial absence of consideration. 48. In the hands of A, A can collect from M a.Php 100,000 c.Php 40,000 b.Php 60,000 d.P 0 49. In the hands of B, B can collect from M a.Php 100,000 b.Php 60,000 A1 PASSERS REVIEW CENTER///168 c.Php 40,000 d.P 0 50. Using the preceding number, if M dishonours the nota, B can collect from P a.Php 100,000 c.Php 40,000 b.Php 60,000 d.P 0 51. In the hands of C, C can collect from M a.Php 100,000 c.Php 40,000 b.Php 60,000 d.P 0 52. Which of the following is not negotiable? a.I promise to pay P or order Php 100,000 , 30 days after date Sgd M b.I promise to pay P or bearer Php 100,000 Sgd M c.Pay to P or order Php 100,000 on Dec. 25, 2010 Sgd M d. Pay to P or bearer Php 100,000 and charge the name against my deposit with you Sgd M to Y 53. 54. 39-40 55. 56. M male a negotiable promissory note payable to the order of P. later made a special endorsement to A. Then B stole the note from A and indorsed it to C who received the note in good faith. Later C indorsed the note to D. which of the following is correct? a.D can collect from M but not from A c.C can collect from endorsers P and B b.If M will not pay , D can collect from P 57. 58. Using the preceding number, if M pays D in good faith, which of the following is incorrect? a.The obligation of M is extinguished c.The obligation of P is extinguished b.A can collect from M d.A should collect from B, the thief 59. 60. When a property is transferred during lifetime to relieve the transferor from the burden of the management of such property, and the transfer of its subject to tax, the basis of the tax is a.The FMV at the time of donation b.The FMV at the time of death of the donor c.The FMV at the time the donor required the property d.The cost of the property in the hands of the donor 61. 62. Which of the following taxes had been replaced by the VAT? a.Percentage tax on international air and shipping carriers doing business in the Philippines b.Tax on life insurance premiums c.Caterer a tax on operators of restaurant and eating places d.Tax on agents of foreign insurance companies 63. 64. 65. 66. 67. The DOC RESTAURANT, a VAT registered taxpayer , issued the following official receipt dated September 18, 2010. 68. 2 pieces of crispy chicken P200.00 69. Siomai special 45. 00 70. Sago gulaman 85.00 71. Mushroom and chicken soup 100.00 72. Braised beef with cauliflower 240.00 73. Coke light in can (2xP40) 80.00 P750.00 74. Locat tax 15.00 75. Service charge 60.00 76. Total P825.00 77. 78. The output vat on the sale is a.P99.00 c.P80.36 b.P 88.39 d.P90.00 79. Mr. And Mrs. A donated a conjugal agricultural land to their son who is getting married with ten (10) months after the date of donation. The fair market value at the time of marriage was P 800,000, but its fair market value at the time of donation was P 600,000. The donated property was subject to P100,000 mortgage which was assumed by the donors. 80. The taxable net gift of Mr. A is a.P240,000 c.P390,000 b.P290,000 d.P 340,000 81. An individual taxpayer is qualified to substituted filing of income tax return if he meets the following requisite except: a.Pure compensation income only b.One employer during the calendar year A1 PASSERS REVIEW CENTER///169 c.Creditable withholding tax is correct d.Should be a minimum wage earner 82. A taxpayer received as assessment notice on February 14, 2010. He filed a motion for consideration with the BIR on February 22, 2010. The submission of necessary documents to support his motion for consideration is a.March 14, 2010 c.April 15, 2010 b.April 23, 2010 d.March 22, 2010 83. Under absolute community of property, which of the following is a community property? a.Property inherited by the surviving spouse during the marriage b.Property bought during the marriage using the money inherited of the decedent before marriage c.Personal belongings (e.g. clothes , shoes, etc.) bought during the marriage for the exclusive use of the decedent d.Jewelry inherited by the decedent during the marriage 84. Which of the following properties owned by a non resident alien decedent is not subject to the rule of reciprocity? a.Shares of stock in a Philippine domestic corporation b.Investment in stock in a US corporation 80% of the business of which is in the Philippines c.Investment in bonds in a US corporation that have acquired business suits in the Philippines d.40% share in a Philippines partnership 85. Which of the following shall not be exempt from value added tax? a.Sales by the agricultural cooperatives duly registered with the Cooperative Development Authority to their members as well as sale of their products, whether in its original state or processes form to non members b.Gross receipts from the lending activities by credit of multipurpose cooperatives duly registered with the Cooperative Development Authority whose lending operation is limited to their members c.Sales of non agricultural, non-electric and non credit cooperatives duly registered with Cooperative Development authority if the capital contribution of each member does not exceeds P 15,000 d.Sales of electric cooperatives duly registered with Cooperative Development Authority relative to the generation and distribution of electricity 86. A common carrier by land is engaged in the transport of passenger , goods and cargoes . A’s gross receipts amounted to P3M, 50% of which came from transport of passengers. He is not VAT registered, what business taxes will A pay? a. 12% value added tax b. 3% common carrier tax c. 3% tax on VAT exempt persons on gross receipts from transport of goods and cargoes and 3 ½ common carrier tax on gross receipts from transport of passengers d. 12% VAT on gross receipts from transport of goods and cargoes and 3% common carriers tax on gross receipts from transport of passengers. 87. A keeper of garage whose gross receipts for the year exceed P 1,500,000 is subject to: a. Value added tax c. Excise tax b. Common carrier tax d. Franchise tax 88. 89. 90. 91. 92. The following are related to the tax assessment of a tax payer 93. Assessment received January 1,2010 94. Petition for reconsideration filed with BIR February 1,2010 95. Documents supporting the petition filed with BIR February 7, 2010 96. Decision of BIR denying the petition received by the taxpayer March 22, 2010 97. Second request for reconsideration filed with BIR March 30, 2010 98. Decision of denial of second request for reconsideration was received April 12, 2010 99. 100. Last day to appeal to the Court of Tax Appeals a.May 12, 2010 c.May 4, 2010 b.July 4, 2010 d.June 4, 2010 101. Which of the inherent powers may be exercised even by public service corporations and public utilities? a.Power of taxation c.Police power b.Power of eminent domain d.A and B 102. A law was passed by Congress which granted tax amnesty to those who have not paid income taxes for a certain year without at the same time providing for the refund of taxes to those who have already paid them . the law is: a.Valid because Congress provided for a valid classification b.Not valid because those who did not pay their taxes are favoured over those have paid their taxes c.Valid because Congress has the sole discretion of determining whom to tax d.Not valid because only the grant amnesty is the sole prerogative of the precedent A1 PASSERS REVIEW CENTER///170 103. How long must the books of accounts be kept? a.For a period beginning to preceding taxable year until the last day of the current taxable year b.For a period beginning the current taxable until the last day of the following taxable year c.For a period beginning from the last entry in each book until the last day prescribed within which the Commissioner is allowed to make an assessment d.For a period of 3 years beginning from the current taxable year until the last day of the third taxable year 104. Due to financial difficulties , ABC Corp requested that its 2009 income tax liability of P400,000 be paid in four (4) equal monthly instalments , starting April 15, 2010. Its request for instalment payment was approved on April 8, 2010. The tax due on May 15, 2010 is. a.P 100,000 c.P 105, 000 b.P 101, 667 d.P106, 667 105. Which of the following is not correct? a.A tax credit certificate validity issued by the BIR may be used as credit against internal revenue taxes except against withholding tax b.All taxes imposed and collected by the BIR are internal revenue taxes c.Public Officials who acquire information on violation of internal revenue laws in the performance of their duties during their incumbency are entitled to informer reward. d.The proceedings on the protest in the BIR is an administrative proceedings 106. On March 31, 2010, A received 100 shares of stock with a fair market value P 120/share as of March 31, 2010. The shares of stock were intended to pay the services rendered by A on November 20, 2009, at which time the fair market value was P 100/share. The per value per share if P50. The income A is a.P12,000 c.P5,000 b.P 10,000 d.P7,000 107. Which of the following statements is not correct? a.An article subject to excise tax may be exempt from vat b.An article subject to excise tax may be subject to vat c.A tax payer may be have two business where one as subject to vat and the other is subject to percentage tax d.Manufacturers and importers of goods are subject to excise tax 108. Mr. Araki a non resident alien stockholders, received a dividend income of P300,000 in 2010 from a domestic corporation whose gross income from within and without the Philippines for three years preceding 2010 show: 109. SOURCE OF INCOME 110. 2007 111. 2008 112. 2009 113. From within the Philippines 114. P 115. P12,00 116. P24,000 16,000,000 0,000 ,000 117. From without the Philippines 118. 18,000,00 119. 14,000, 120. 16,000, 0 000 000 121. 122. How much of the individual income received by Mr. Araki considered income from sources within the Philippines? a.Zero b.P156,000 c.P300,000 d.P144,000 123. 124. 125. 126. 127. Which of the following is not a condition imposed by the constitution for the exercise of the power of eminent domain? a.The consent of the owner of the private property to sell the same to the government b.The payment of just compensation c.The observance of due process in making of private property d.The existence of public use of the taking of a private property 128. Mr. Bagongbahay acquired his old principal residence in 1990 at a cost of P4M. he sold the said property on January 1, 2010 for P16M, although the FMV on that date was P20M. within the reglementary period, Bagongbahay acquired his new principal residence for P 12M. 129. Which of the following is incorrect? a.Bagongbahay has to pay capital gains tax of P 300,000 b.The cost basis for the new principal residence is P3.2 M c.The utilized amount of the gross selling price is P 12M d.If the proceeds were not used to buy a new principal residence, the capital gains 130. 131. 63-64- For the month of August and September 2010, ABC Bank has the following income/loss A1 PASSERS REVIEW CENTER///171 132. INTEREST INCOME WITH MATURITY - Of less than 5 years - Of more than 5 years 139. rentals 142. Net trading gain (Loss) 133. ST 135. 00 137. 0 140. 0 143. 00) AUGU P800,0 134. SEPTE MBER 136. P1M 800,00 138. 1M 400,00 141. 800,000 (100,0 144. 200,000 145. 146. The gross receipts tax of ABC Bank for August is a.P76,000 c.P69,000 b.P68, 000 d.P63,000 147. The gross receipts tax of ABC Bank for September is a.P125,000 c.P116,000 b.P130,000 d.P123,000 148. Which of the following is not subject to vat? a.Non life insurance premiums c.Non life insurance commissions b.Non life reinsurance d.Non life reinsurance commissions 149. It is aspect of taxation that administrative in character and the power to exercise it is vested on the Department of Finance a.Levying c.Imposition b.Collection d.Legislation 150. For donor tax purposes this is a stranger to the donor a.Daughters of a brother c.Grandson of sons granddaughter b.Son of an uncle d.Granddaughter of mother’s sister 151. 152. 68-70 153. A , caregiver , is under three year employment contract with a hospital in USA . His wire , B and a minor child live in the Philippines. He earns P70,000 (converted) a month for the annual income of P840,000, A shall pay income tax a.For the entire amount it being taxable in the Philippines b.Of nothing because as OFW, he is considered a non resident citizen c.For half the full amount because the other half pertains to his wife d.For the ½ share of his wife because the wife is a resident citizen 154. Using the above data, if A puts 30% of his earnings in a dollar deposit account with a local , interest income therein a.Is taxable for the full amount at 7.5% final withholding tax b.Of P 126,000 shall be taxed at 7.5% final withholding tax c.Maybe exempt from income tax provided he presents proof of non residency to the bank d.Of P840, 000 shall be taxed at 7.5% final withholding tax 155. If the dollar deposits account above is joint ( with B) with the same percentage of earning invested , interest income in that account a.Is capable for the full amount at 7.5 % final withholding tax b.Of P126,000 shall be taxed at 7.5% final withholding tax c.May be exempt from the income tax provided he presents proof of non residency to the bank d.Of P840,000 shall be taxed at 7.5% final withholding tax 156. A filed his income tax return for the calendar year 2007 on March 20, 2008. When is the last day for the BIR to collect assuming A did not pay the tax due upon filing? a.March 20, 2011 b.April 15, 2011 c.April 15, 2013 d.March 20, 2013 157. 158. 159. 160. 161. Using the above data, assuming A filed his income tax return on April 20, 2008 and the return is not fraudulent, when is the last day for the BIR to make an assessment ? a.April 20, 2011 c.April 20, 2013 b.April 15, 2011 d.April 15, 2013 162. Using the above data , if the BIR issued a deficiency income tax assessment on March 10, 2010, when is the last day for the BIR to collect? a.March 10, 2013 b.April 15, 2013 A1 PASSERS REVIEW CENTER///172 c.March 10, 2015 d.April 15, 2015 163. A filed his ITR for 2008 on April 15, 2009 and will pay his tax liability in instalment. The return shown an income tax due of P100,000 and income tax withheld of P60,000. If A pays the 2 nd instalment only on October 15, 2009, the amount to be paid (excluding compromise penalty)is a.P26,000 c.P55,000 b.P35,000 d.P37,500 164. On April 15, 2010 , A filed his income tax return for his 2009 income and paid the tax due thereon. In 2011. A discovered that he made an overpayment in his income tax return. He filed a written claim for refund of tax erroneously collected with the CIR on October 1, 2011. On April 14, 2012, without receiving a decision on his claim for refund, A filed in the CTA a petition for review on his claim for refund of tax erroneously collected. Which of the following is incorrect? a.The CTA did not acquire jurisdiction over the petition for review because the appeal is premature, there being no decision yet on said claim for refund b.The CTA jurisdiction is to review by appeal decisions of the BIR Commissioner so if there is no decision there is nothing to review c.The CTA acquired jurisdiction over the petition since it is the right of a taxpayer to go to the CTA with or without decision rendered by the BIR Commissioner d.The CTA acquired jurisdiction over the petition as the peremptory period of 2 years within which a claim for refund is about to expire and the failure of the CIR to act on the claim is tantamount to denial of the claim 165. One of the following is not exempt from VAT a.Lease of the residential units with a monthly rental not exceeding P10,000 b.Services subject to percentage tax c.Services rendered by individuals pursuant to employer –employee relationship d.Export sales by persons who are vat-registered 166. A sold a personal property held as capital asset be acquired 9 years ago with an acquisition cost P50,000 for P 270,000. The payment shall be made of the following terms. Cash down payment of P40,000 and additional P 10,000 at the end of the year sale. The property sold has been mortgaged for P 160,000 which assumed by the buyer issued a note payable for the balance which is to be paid equally in two years following the year of sale. The contract price is. a.P 270,000 c.P 120,000 b.P110,000 d.P130,000 167. The taxable profit for the first year is a.P120,000 c.P30,000 b.P60,000 d.P15,000 168. XYZ filed its income tax return and paid the tax due for calendar year 2008 showing a tax liability of P175,000. However, upon audit, it was discovered that its income tax return was false or fraudulent because it did not report other taxable income. Per investigation the correct income tax due is P 350,000. The corporation was duly informed of this finding through a preliminary assessment notice. Failing to protect on time against the preliminary assessment notice, a formal letter of demand and assessment notice was issued on May 31, 2010 calling for payment of deficiency income tax on or before July 15, 2010. The amount due on July 15, 2010 is a.P 306,250 b.P 437, 500 c.P240, 625 d.P612, 500 169. Where no notice or audit or investigation of tax return, statement or declaration has been actually served upon the taxpayer, the tax return may be modified, changed or amended by the taxpayer , from the date of such filing, within. a.30 days b.180 days c.3 years d.5 years 170. 171. 172. 173. 174. 175. 176. 177. 178. 179. A1 PASSERS TRAINING, RESEARCH, REVIEW & DEVELOPMENT COMPANY 180. 2nd Floor Sommerset Bldg., Lopez Jaena St. Jaro, Iloilo City 181. Tel. No.: (033) 320-2728; 09106547262 A1 PASSERS REVIEW CENTER///173 182. Email Address: [email protected] 183. BOARD OF CERTIFIED PUBLIC ACCOUNTANT 185. CERTIFIED PUBLIC ACCOUNTANT Licensure Examination SET A 187. 184. 186. 188. MANAGEMENT ADVISORY SERVICES 189. INSTRUCTION: Select the correct answer for each of the following questions. Mark only one answer for each item by shading the box corresponding to the letter of your choice on the answer sheet provided. STICTLY NO ERASURES ALLOWED. Use pencil no. 1 only. 190. 191. MULTIPLE CHOICE: 192. 1. In comparing management accounting, which of the following statements is true? a.Both are historical cost as their primary unit of measurement b.Both depend on the double entry system of accounting c.Both require adherence to PFRS d.Financial accounting reports are more objectives , whereas management accounting reports are more subjective 2. Management accounting and financial accounting are similar in which of the following respects? a.Both use the sense unit of measurement b.Both rely heavily on the double entry systems c.Both produce almost all of their respective informational reports on a routine monthly basis d.Both provide relevant and useful information to management 3. Firms with a high degree of operating leverage a.Will have a more significant shift in income as sales volume changes b.Have lower fixed costs c.Have low contribution margin ratios d.Are less dependent volume to add profits 4. The treasurer function includes a.Tax administration c.Investor relations b.Evaluating and consulting d.Economic appraisal 5. The use of information systems facilities on focus on the collection, organization, integration and long term storage of entity wide data is known as a.Data independence c.Data redundancy b.Data mining d.Data warehousing 6. Which of the following technologies would an organization use to prevent access to its private network? a.Digital signature c.Web assurance service b.Firewall d.Encryption 7. Which of the following systems is one where the transactions are posted and the master and open files are updated, immediately? a.Microcomputer system c.Batch system b.Mainframe computer system d.Real times system 8. Which of the following controls most likely would assure that an organization can reconstruct its files? a.Hardware controls that are built into the computer by the computer manufacturer b.Back up copies of the data and program files that are stored away from originals c.Personnel, who are independent of data input , perform parallel simulations d.System flowcharts that provide accurate descriptions of input and output operations 9. Which of the following statements about the visual fir method is (are) true? 10. I. The method results in the creation of a scatter diagram 11. II. The method is not totally objective because of the manner in which the cost line is determined 12. III. The method is especially helpful in the determination of outliers a.II only c.I and III b.I and II d.I, II and III 13. A flatter slope is the variable cost line indicates a a.High variable cost per unit b.High influence of activity on total variable cost c.Low influence of activity on total variable costs d.Large amount of fixed costs 14. 15. 16. 17. A1 PASSERS REVIEW CENTER///174 18. Cielo’s Hotdog Stand sells hotdogs for P25 each. The variable costs per hotdog are P10. Cielo’s fixed costs are currently P 8,000 per month. Cielo is considering expending her business to three hotdog stands which will increase fixed costs per month by P 12,000. 19. If Cielo’s does expand her business to three stands, how many additional hotdogs will need to be sold per year in order to break even a.800 c.9600 b.600 d.7200 20. Do Dante Corporation is contemplating to method a new product that requires an increases in fixed costs by P 1,400,000 for production of 50,000 units or less and P1,800,000 for more than 50,000 units. The variable costs ratio is 60 percent of sales. The product is expected to sell for P80 per unit. How any units must Don Dante sell just breakeven? a.43,750 c.50,000 b.56,250 d.Either 43,750 or 56,250 21. In most companies , machines break down occasionally and employees are often less than perfect which type of standard acknowledges these characteristics when determining the standard cost of a product? a.Efficiency standard c.Practical standard b.Ideal standard d.Budgeted standard e. f. Question number 14 and 15 are based on the following g. Moon Products applies fixed overhead at a rate of P3 per direct labor hour. Each unit produced is expected to take 2 direct labor hours. Moon expected production in the current year to be 10,000 units but 9,000 units were actually produced. Actual direct labor hours were 19,000and actual fixed overhead costs were P 62,000 22. Moons fixed overhead spending variance is: a.P 8,00 F c. P 2,000 F b. P 8,000U d.P 2,000U 23. Moons fixed overhead volume variance is: a.P 2,000 c.P 8,000 b.P 6,000 d.P0 24. Which of the following statements is correct regarding budgeting? a.Is a primary focused on past performance b.Is primarily bookkeeping task c.It should be build from the ground up each year d.It involves input from a broad range of managers 25. It is most budget in the master budget is likely the a.Cash budget c.Personnel budget b.Capital budget d.Purchases budget 26. Which of the following is not a typical cash outflow associated with a capital investment? a.Repairs and maintenance needed for purchased equipment b.Additional operating costs resulting from the capital investment c.Salvage value received when the newly purchased equipment is sold d.Purchase price of a new equipment 27. Depreciation charges indirectly affect the after tax flow because the company a.Can deduct depreciation expenses on their financial statements, reducing reported income before tax b.Can deduct depreciation expenses on their financial statements , increasing cash inflows c.Can deduct depreciation expenses on their income tax returns, reducing cash outflow for taxes d.Cannot deduct depreciation expenses on their income tax returns 28. The calculation of the profitability index (PI) is most helpful for which type of decisions? a.Screening c.Qualitative b.Preference d.Short term 29. O Mai Mai Company purchased an asset costing P 90,000. Annual operating cash inflows are expected to be P 20,000 each year for six years. No salvage value is expected at the end of the asset life. The company applies a 16 percent minimum acceptable rate of return for this kind of investment . the details of the present values at 16% sex periods and 30. Present value of ordinary annuity of 1 3.6847 31. Present value of annuity due of 1 4.2743 32. 33. Assuming O’maimai cost of capital is 16 percent , what is the asset net present value? ( ignore income taxes) a. P 16,306 b.P30,000 c.P4,514 d.P4,800 34. 35. 36. A1 PASSERS REVIEW CENTER///175 37. 38. 39. 40. 41. Pacam, Inc. Requires all its capital investment to generate an internal rate of return of 14 percent the company is considering an investment costing P80,000 that is expected to generate equal annual cash inflows for 5 years. The present value of 1 . end of 5 years is 0.51937 and the present value inequity of 1 is 3.4331 based on 14 percent required rate of return. 42. To meet the 14% minimum acceptable rate return, the estimated annual cash inflow (ignoring income taxes) is : a.P23,803 c.P274,648 b.P51,550 d.P154,033 e. f. Question Nos. 23 through 25 are based on the following g.Lakewood Company is considering the purchases of a purpose bottling machine for P 280,000. It is expected to have a useful life of 7 years with a zero terminal disposal price. The plant manager estimates the following savings in cash operating costs h. i. j. ANNUAL CASH SAVINGS k. m. o. q. s. u. w. l. P140,000 n.110,000 p.80,000 r. 60,000 t. 40,000 v. 30,000 x.30,000 y. z.Lakewood uses required rate of return of 12% in its capital budgeting decisions. remittal tax rate of 40%. The company uses straight-line depreciation. The present value of annuity of 1 at 12% for years 4,56376. The details of the present value aa. ab. ANNUAL CASH SAVINGS ac. ae. ag. ai. ak. am. ao. ad. 0.89286 af. 0.79719 ah. 0.71178 aj. 0.63552 al. 0.56743 an. 0.50663 ap. 0.45235 aq. 43. The number of years to recover the amount investment is a.4.83 years c.2.38 years b.3.63 years d.4.00 years 44. What is the net present value of this investment? a.P4,723 c.P(2.159) b.P8,559 d.P0 45. What is the accounting rate of return based on initial investment a.6.43% c.20.71% b.12.86% d.41.42% 46. Escape Company regularly pays its accounts payable on the tenth day and enjoys the 2 percent cash discount , term: net 30. Because of an oversight, one supplier invoice is not paid within the discount period but paid 10 days after. What is the annual cost of that incident of paying an invoice on the 20 th day instead of the tenth day? Use 360 days a year a.73.44% c.30.02% b.36.73% d.2.04% 47. P18,591,650 at 8 persons on a one year discounted loan, what is the effective interest rate? a.8.00 percent c.9.07 percent b.8.70 percent d.7.41 percent A1 PASSERS REVIEW CENTER///176 48. Toledo, Inc. Has recently calculated the inventory turnover for current year to be 30 . in prior years , the same ratio was always lower. Which of the following statements would be the best interpretation for the reason for the ratios change? a.The company had less sales in the current year than in prior years b.The company purchased less inventory in the current year than in prior years c.The company took fewer days to sell its inventory in the current year than in prior years d.The company took more days to sell its inventory in the current year than in prior years 49. 50. 51. 52. 53. 54. 55. Which of the following statements is true regarding a company’s segment margin? a.It is primarily used to make short term decisions such as cost volume profit (CVP) analysis and special order decisions b.It ignores all fixed costs c.It is primarily a measure of long term profitability d.It should not be used to make decisions on whether or not to drop a product line 56. Which of the following statements about the balanced scorecard approach is false? a.It requires managers to focus on financial measures more than nonfinancial measures b.It looks at performance from the following perspectives, financial, customer, internal business and learning and growth c.It helps balance short term operating performance with long term strategies d.It recognizes that traditional measures of performance are often not adequate to fully assess company’s performance 57. Determining the market share for a company’s product would be a measure of performance , which perspective of the balanced scorecard? a.Internal business c.Financial b.Customer d.Learning and growth 58. Which of the following statement about quality costs is true? a.Even in theory , external failure costs can never be reduced to zero b.The less of company’s spreads on prevention costs, the less they will have to spend on internal failure costs in the future c.If a company produces and sells a defective product, external failure cost are likely to exceed all the other types of quality costs d. Low level of internal failure costs usually indicators that more attention needs to be paid to prevention and appraisal costs 59. HNW Ltd. Manufactures and sells food processors. A popular consumer magazine has recently evaluated food processors and has ranked HNWs processors as being of “poor quality”. As a result HNWs management team has begun to analyze all costs associated with their food processors in more detail and the following data has been compiled: 60. Scrap costs 61. P 80,000 62. Quality training 63. 75,000 64. Warranty 65. 100,000 claims 66. Rework costs 67. 50,000 68. Inspection of 69. 30,000 incoming materials 70. Product quality 71. 60,000 audits 72. Statistical 73. 40,000 process control 74. 75. What are HNW’s total internal failure costs? a.P 50,000 c.P230,000 b.P130,000 d.P170,000 76. Which of the following items used to measure customer response time would not be used to measure manufacturing cycle time? a.Time it takes to process raw materials into a completed product b.Time spent waiting for a machines availability c.Time spent inspecting the materials for quality control d.Time spent delivering the product to the customer 77. 78. Question Nos. 35 and 36 are based on the following information A1 PASSERS REVIEW CENTER///177 79. Polka Devices normally produces and sells 62,000 units CK-6 each month. CK -6 is a small electrical relay used in the automotive industry as a component part in various products. The selling price is P46 per unit, variable costs are P42 per unit, fixed manufacturing overhead expenses total P280,000 per month, and fixed selling expenses total P88,000 per month. 80. Strikes in the companies that purchase the bulk of the CK-6 units have caused Polks Devices sales to temporarily drop to only 26,000 units per month. Polka devices estimates that the strikes will last for about two months, after which sales of CK-6 should return to normal. Due to the current low level of sales, however, Polica Devices is thinking about closing down its own plant during the strikes. If Polks Devices does close down its plant, it is estimated that fixed manufacturing overhead expenses can be reduced to P210,000 per month and that fixed selling expenses can be reduced by 10% . start up expenses at the end of two months shutdown period would total P4,000. Since Polica Devices uses just in time production methods, no inventories are on hand. 81. Assuming that the strikes continue for two months as estimated, how much would the advantage or disadvantage of closing the plant? a.P 54,400 disadvantage c.P 50,400 disadvantage b.P 58,400 disadvantage d.P 29,200 disadvantage 82. 83. 84. 85. 86. At what level of sales (in units) for two month period should Polica Devices be indifferent between closing the plant or keeping it open? a.38,400 units c.39,000 units b.39,400 units d.38,000 units 87. Vera Cruz Company has projected cost of goods sold P 2 million including fixed costs of P100,000 and variable costs are expected to be 75 percent of net sales. What will be the projected net sales/ a.P 2,133,333 c.P2,666,667 b.P2,400,000 d.P3,200,000 88. Gibo Zamora sells helium balloons in Luneta Park on Saturdays as a means on making some extra spending money. He buys balloons from a party store at P12 each and sells them at a price of P30.00 each. Demand of the balloons ranges between 35 and 45 per day. Gibo normally purchases 40 balloons. The helium losca its resilence in a couple of days, so balloons not sold cannot be saved for the following Saturday. Gibo usually just releases the balloons that do not sell. As he watched the most recent batch of the five balloons floating into space, he thought surely, there must be some way to void this waste. 89. What is the estimated on Gibo profit if the helium canister is rented assuming there are 5 balloons being unsold? a.No effect c.Increase the profit by P90 b.Increase the profit by P150 d.Decrease the profit by P80 e.Use the following data for Ulap Company in answering numbers 39 and 40 f. g. h. 2010 i. 2009 j. Net sales k.P5,520,000 l. P4,000,000 m. Cost of n.3,795,000 o.3,000,000 goods sold p.Gross profit q.P1,725,000 r. P1,000,000 s. 90. Assume that the selling price increased by 20 percent effective January 1, 2010. What is the amount of increase in sales due to change in selling price and units sold, respectively? a.P920,000 and P 600,000 c.P720,000 and P800,000 b.P 1,104,000 and P416,000 d.P (1,380,000) and P2,900,000 91. Independent of question number 39 , assuming that the cost price per unit decreased by 4 percent effective January 1, 2010. How much were the change in sales due to change in quantity sold and change in cost of goods sold due to change in unit cost? 92. Change in sales change to cost of Goods Sold Due 93. Change in Quantity to change in unit cost a.P1,270,800 increase P158,125 decrease b.P249,200 increase P953,125 decrease c.P1,262,400 increase P151,800 decrease d.P1,262,400 increase P953,125 decrease 94. The primary purpose of management advisory service is a.To conduct special studies, preparation of recommendations , development of plans and provision of advice and assistance in their implementation b.To provide services or to fulfil some social need c.To improve the clients use of its capabilities and resources to achieve the objectives of the organization A1 PASSERS REVIEW CENTER///178 d.To earn the best rate of return on resources entrusted to its care with safety of investment being taken into account and consistent with the firms social and legal responsibilities 95. The fixed overhead application rate is a function of a predetermines normal activity level. If standard hours allowed for good output equal predetermined activity level for a given period the volume variance will be a.Zero c.Unfavourable b.Favorable d.All of the above 96. The AMVians is planning its annual Blast Extravaganza. The extravaganza committee has assembled the following expected costs for the event: 97. Dinner per person P70 98. Programs and souvenier per person 30 99. Orchestra 15,000 100. Tickets and advertising 7,000 101. Auditorium rental 48,000 102. Floor show and strolling entertainment 10,000 103. 104. The committee members would like to charge P300 per person for the evening activities 105. Assuming that only 250 persons are expected to attend the extravaganza, what ticket price must be charged to breakeven? a.P420 c.P320 b.P350 d.P390 106. 107. 108. 109. 110. 111. Gabgalang Company uses a monthly cost formula for overhead of P50,000 + 30,000 for each direct labor hour worked. For the coming year, gabgalang plans to manufacture 200,000 units. Each unit requires five minutes of direct labor. Gabgalang total budgeted overhead for the coming year is a.P550,000 c.P1,100,000 b.P1,200,000 d.P650,000 112. How will a favorable volume variance affect net income under each following methods? 113. ABSORPTION VARIABLE a.Reduce no effect b.Reduce increase c.Increase no effect d.Increase reduce 114. The following information has been extracted from Bautista & Magsombol Company’s financial records for its first year of operations? 115. 116. Units produced 10,000 117. Units sold 7,000 118. Variable costs per unit 119. Direct materials P8 120. Direct labor 9 121. Factory overhead 3 122. Selling 4 123. Fixed costs: 124. Manufacturing overhead P70, 000 125. Selling 30,000 126. 127. Based on absorption costing, Bautista & Magsombol’s income in its first year of operations will be a.P21,000 higher than it would be under variable costing b.P70,000 higher than it would be under variable costing c.P30,000 higher than it would be under variable costing d.Higher than it would be under variable costing, but he exact difference cannot be determinable from this information 128. Balara Company has intra company service transfers from Internal Division , a cost center, to World Division , a profit center. Under stable economic conditions, which of the following transfer prices is likely to be most conductive to evaluating whether both divisions have met their responsibilities? a.Actual cost c.Standard variable cost b.Market price d.Negotiated price 129. De Chavez Company is considering an investment in a new casting machine to replace its existing one. Information on the existing machine and the replacement machine follows: 130. 131. Cost of new machine P40,000 132. Net annual savings in operating costs 9,000 A1 PASSERS REVIEW CENTER///179 133. 134. 135. 136. 137. 138. Salvage value now of the old machine Salvage value of the old machine in 8 years Salvage value of the new machine in 8 years Estimated 6,000 0 5,000 8 years What is the expected payback period for the new machine? a.4.44 years c.4.00 years b.3.22 years d.3.78 years 139. Kiel Dimayaga Company has a total assets turnover of a 0.30 and a profit margin of 10 percent Patrick. The President , is unhappy with the current return on assets and be thinks it could be doubled. This could be accomplished (1) by increasing the profit margin to 12 percent , and (2) by increasing the total assets turnover. What new asset turnover ratio, along with the 12 percent profit margin, is required to double the return on assets? a.25% c.36% b.50% d.60% 140. The examples of activating at the product level include a.Scheduling , setting up and moving c.Beating, lighting and security b.Designing changing and advertising d.Cutting, painting and packaging 141. A systematized approach knows as acro-based budgeting a.Presents the plan for only one level of activity and does not adjust to changes in the level of activity b.Presents a statement of expectations for a period of time but does not present a firm commitment c.Divides the activities of individual responsibility centers into a series of packages which are ranked ordinally d.Classifies budget requests by activity and estimates the benefits arising from each activity 142. The CEO of a rapidly growing high technology firm has exercised centralized authority over all corporate functions. Because the company now operates in four geographically dispersed locations. The CEO is considering the advisability of decentralizing operation control over production and sales. Which of the following conditions probably would result from and be a valid reason for decentralizing? a.Greater local control over compliance with government regulations b.More efficient use of headquarters staff officials and specialists c.Quicker and better operating decisions d.Greater economies in purchasing 143. The following are details of the monthly unit cost to manufacture and sell a particular product for Grace Company: 144. Manufacturing cost: 145. Direct materials P3.00 146. Direct labor 4.00 147. Variable indirect 2.00 148. Fixed indirect 1.50 149. 150. Marketing costs: 151. Variable 2.00 152. Fixed 1.00 153. 154. Grace must decide to continue making the product or buy it from outside supplier. The supplier has offered to make the product at the same level of quality that the company can make it. Fixed marketing costs would be unaffected, but variable marketing costs would be reduced by 25% if the company were to accept the proposal. What is the minimum amount per unit that Grace can pay the suppliers without decreasing its operating income? a.P9.50 c.P11.50 b.P10.50 d.P12.50 155. The Salinas Division of Soriano Company expects the following result for 2011. 156. Unit sales 70,000 157. Unit selling price P10 158. Unit variable cost P4 159. Total fixed cost 160. Total fixed costs P300, 000 161. Total investment P500, 000 162. 163. The minimum required RO1 is 15percent, and divisions are evaluated on residual income. A foreign customer has approached Salinas manager with an offer to buy 10,000 units at P7 each. If Salinas accepts the order, it would not lose any of the 70,000 units at the regular price. Accepting the order would increase fixed costs by P10,000 and investment by P40,000 164. 165. What is the minimum price that Salinas could for the order and still maintain its expected residual income? A1 PASSERS REVIEW CENTER///180 a.P5.00 c.P5.60 b.P4.75 d.P9.00 166. Morr Co. has a total annual cash requirement of P 9,075,000 which are be paid uniformly. Morr has the opportunity to invest the money at 24% per annum. The company spends, on the average P40 for every cash conversion to markeable securities. 167. What is the optimum average cash balance? a.P60,000 c.P55,000 b.P45,000 d.P27,500 168. The dividends and stock price of Feudo Company are expected to grow at 7 percent per year by the end of this year. Feudos common stock sells for P25 per share, it last dividend was P2.50 and the company will pay P2,675 at the end of the current year. Feudo should pay P 2.50 flotation cost. 169. What are the expected returns on retained earnings for Feudo Company? a.17.7 percent c.18.89 percent b.18.45 percent d.19.72 percent 170. Baguio Company’s accounts receivable were P600, 000 at the beginning of the year and P 800,000 the end of the year. Cash sales for the year were P300, 000. The accounts receivable turnover for the year was 5 times. Baguio Company’s total sales for the year were a.P 800,000 b.P 3,300,000 c.P1,300,000 d.P3,800,000 171. 172. 173. 174. 175. The Baraguio Corporation purchases 60,000 headbands per year. The average purchase lead time is 20 working days. Maximum lead time is 27 working days. The corporation works 240 days per year. 176. The appropriate safety stock level and the recorder point for the company are 177. 178. Safety Stock Recorder Point a. 1,750 6,750 b.1,750 5,250 c.1,167 6,750 d.1,167 5,250 179. Almires Building company uses the critical path method to monitor construction jobs. The company’s is currently 2 weeks behind schedule on Job WW. Which subject to a P10,500 per week completion penalty. Path A-B-C-F-G-H-I has a normal completion time of 20 weeks and critical path A-D-E-F-G-H-I- has a normal completion time of 22 weeks. The following activities can be crashed. 180. Activities cost to crash 1 week cost to crash 2 weeks 181. B-C P8,000 P15,000 182. D-E 10,000 19,600 183. E-F 8,800 19,500 184. 185. Almires desires to reduce the normal completion time of job WW and at the same time, report the highest possible income for this year. Almires should crash a.Activity B-C 1 week and Activity EF 1 week b.Activity B-C 2 weeks c.Activity D-E 1 week and activity B-C 1 week d.Activity D-E 1 week and activity E-F 1 week 186. The capital asset pricing model (CAPM) computes the expected return on a security by adding the risk rate return to the incremental yield of the expected market return which is adjusted by the company’s beta. What is MNO’s expected rate of return if the equity market is expected to earn 12 percent; the treasury bonds are currently yielding 5 percent. The beta coefficient for MNO is estimated to be 0.60. MNO is subject to an effective corporate income tax rate of 40%. a.12.00 percent c.12.20 percent b.9.20 percent d.7.20 percent 187. Trading on the equity refers to the a.Amount or working capital b.Amount of capital provided by owners c.Use of barrowed money to increase the return to owners d.Number of times interest in caned 188. Demand for a product tends to be price inelastic if a.The product is considered a hourly item b.Few goods complements for the product are available c.The population in the market area is large A1 PASSERS REVIEW CENTER///181 d.People spend a large share of their income on the product Because of the existence of economies of scale , business fines may find that a.Each additional unit of labor is less efficient than the previous unit b.As more labor is added to a factory, increases in output will diminish in the short run c.Increasing the size of a factory will result in lower average cost d.Increasing the size of a factory will result in lower total costs 190. For a given level of tax collections , prices and interest rates, a decrease in governmental purchases will result in a (an) a.Increase aggregate demand c.Increase in aggregate supply b.Decrease in aggregate demand d.Decrease in aggregate supply 191. The most effective fiscal polity program to help reduce demand pull inflation would be to a.Decrease the rate of growth of the money supply b.Increase both taxes and government spending c.Decrease taxes and increase government spending d.Increase taxes and decrease government spending 192. 193. CABRAL industries received an order for a piece of special machinery from Tigok Company. Just as CABRAL completed the machine, Tigok declared bankruptcy, defaulted on the order , and for feted the 10 percent deposit paid on these selling price P72,500 194. 195. 196. 197. 198. 199. 200. 201. CABRAL’s manufacturing manager identified the cost already incurred in the production of the special machinery for Tigok as follows 202. Direct material P16,600 203. Direct labor 21,400 204. Manufacturing overhead 205. Variable P10,700 206. Fixed 5,350 16,050 207. Fixed selling and administrative costs 5,405 208. Total P59, 455 209. 210. Another company , kay Corporation , will buy the special machinery if it is reworked to Kay’s specification. CABRAL offered to sell the reworked machinery to Kay as a special order for P68, 400 Kay agreed to pay the price when it takes delivery in two months. The additional identifiable costs to rework the machinery to Kay specifications are as follows. 211. Direct materials P6,200 212. Direct labor 4,200_____ 213. Total P10,400 214. 215. A second alternative available to CABRAL is to convert the special machinery to the standard model . which sells for P62,500. The additional identifiable costs for this conversion are as follows. 216. Direct materials P2,850 217. Direct labor 3,300 218. Total P6,150 219. 220. The third alternative for CABRAL is to sell the machine as is for a price P52,000, However , the potential buyer of the unmodified machine does not want it for 60 days. This buyer has offered a P7,000 down payment , with the reminder upon delivery. 221. 222. The following additional information is available regarding CABRAL’s operations: 1. The sales commission rate on sales of standard models is 2 percent, while the rate on special orders is 3 percent 2. Normal credit terms for sales of standard models are 2/10, net /30. This means that the customer receives a 2 percent discount if payment is made within 10 days, and payment is due no later than 30 days after billing. Most customers take the 2 percent discount. Credit terms for a special order are negotiated with the customer. 3. The allocation rates for manufacturing overhead and fixed selling and administrative costs are as follows 223. Manufacturing costs: 224. Variable 50% of direct labor costs 225. Fixed 25% direct labor costs 226. Fixed selling and administrative costs 10% of the total manufacturing costs 189. A1 PASSERS REVIEW CENTER///182 227. 4. Normal time required for rework is one month 66. How much peso contribution would the sale to Kay Corporation add to CABRAL before tax profit? a.P53,848 c.P55,900 b.P55,948 d.P9,300 67. How much peso contribution would the alternative of converting the special machinery to standard model add to CABRAL’s before tax profit? a.P52,200 c.P52,825 b.P54,475 d.P7,650 68. If Kay makes CABRAL a counteroffer, what is the lowest price CABRAL should accept fot the reworked machinery from Kay? a.P10,400 c.P10,722 b.P12,500 d.P12,887 69. How much would the alternative of selling unmodified machinery to the potential buyer contribute to CABRAL’s before tax profit? a.P50,440 c.P49,920 b.P1,740 d.P49,400 70. A construction company has just completed two bridges over the Visayan area. The first bridge the company ever built required 100 weeks to complete. Now having hired a bridge construction crow with some experience, the company would like to continue building bridges. Because of the investment in heavy machinery needed continuously by this crew, the company believes it wound have to bring the average construction time less than a year (52 weeks) per bridge to earn a sufficient return on investment. The average construction time will follow an 80% learning curve. To bring the average construction time (over all bridges constructed) below one year per bridge, the crew would have to build approximately a.6 additional bridges c.3 additional bridges b.7 additional bridges d.8 additional bridges e. f. A1 PASSERS TRAINING, RESEARCH, REVIEW & DEVELOPMENT COMPANY g. 2nd Floor Sommerset Bldg., Lopez Jaena St. Jaro, Iloilo City h. Tel. No.: (033) 320-2728; 09106547262 i. Email Address: [email protected] j. k. BOARD OF CERTIFIED PUBLIC ACCOUNTANT l. m. CERTIFIED PUBLIC ACCOUNTANT Licensure Examination SET B n. o. MANAGEMENT SERVICES p. q. INSTRUCTION: Select the correct answer for each of the following questions. Mark only one answer for each item by shading the box corresponding to the letter of your choice on the answer sheet provided. STICTLY NO ERASURES ALLOWED. Use pencil no. 2 only. r. s. Items 1 to 3 are items based on the following information t. Rafa enterprises is contemplating a major investment in a new flexible manufacturing systems (EMS). The company’s minim desired rate of return is 16%. The total investment is P2.5 million with a 10 percent residual value at the end of its 8 year expected life. The company uses straight line depreciation. The FMS should produce P1.4 million of additional revenue additional revenue annually. Annual cash operating costs area expected to be P776,375 u. v. 1. What is expected payback period for the proposed FMS purchase? A. 4.009 years C. 3.625 years B. 3.954 years D. 3.125 years E. 2. What is the expected net present value of the proposed FMS? (Round off factors to three decimal places). A. Positive net present value of P4,051,450 C.Positive net present value of P285,277 B. Negative net present value of P12,455 D.Negative net present value of P62,205 E. 3. Using the accounting rate of return method, what is the expected rate of return from the proposed FMS purchases? A. 37.7 percent C. 24.9 percent B. 26.6 percent D. 19.2 percent E. 4. Dividend payout ÷ dividend yield= A. Price earnings ratio C. Plowback ratio B. Retention ratio D. No meaningful amount A1 PASSERS REVIEW CENTER///183 E. 5. Nadal Corporation produces custom designed safety gear for police vehicles. During September 20,2010, the company incurred the following cost of quality: F. Produce testing costs P11,200 G. Product warranty claims 13,000 H. Scrap and rework costs 10,500 I. Product engineering design costs 18,900 J. Employee training costs 10,100 K. Product simulation costs 16,400 L. M. Total quality costs of conformance for September were A. P80,100 C. P42,400 B. P56,600 D. P40,200 E. 6. Roger Company expects to incur P4, 000 per month of five costs during the first three months of the year, and P10 per unit variable overhead costs. Expected [production for January, February and March is 4,000, 5,000 and 3,000 respectively. Based on this information, the predetermined overhead rate for the first three months of the year is A. P11.00 per unit C.P0.80 per unit B. P10.00 per unit D.P0.33 per unit E. 7. a general rule in choosing among alternative investments is the greater the risk taken, the A. Greater the return required B. Lower the profits expected C. Greater the price of the investment D. Greater the price of the investment F. G. H.8. The following financial ratios relate to Novak Company:  Debt ratio  Return on sales  Assets turnover: 2 times  Times interest earned: 4 times I. What is the Novak’s return on equity? A. 6.25% C. 25.00% B. 9.38% D. 37.50% E. 9. What type of responsibility center are accounting personnel and legal departments? A. Cost center C. Revenue center B. Profit center D. Investment center E. 10. Federer Company operates two segments. Income statement for both segments are shown below F. G. Segments Segment “Áussie Open” Segment ÜS Open” H. Sales P500,000 P750,000 I. Unit Level Variable Costs (400,000) (550,000) J. Contribution Margin 100,000 200,000 K. Facility –level Fixed Costs (135,000) (120,000) L. Income (loss) (P35,000) P80,000 M. N. Management is considering discontinuing the operation of Segment “Aussie Open” is eliminated? A. P35,000 increase C. P100,000 decrease B. P35,000 decrease D. P135,000 decrease E. 11. Which of the following would be the nonvalue-added activity for a pizerria? A. Home delivery C. Recording daily cash receipts B. Buying fresh pepperoni D. Preparing thick pizza dough E. 12. Complete the following table assuming the cost is a fixed cost F. Units of products sold 15 25 G. Total expected sold P7,500 “X” H. Average per unit P500 “Y” I. J. The amounts in the cells labelled “X” and “Y” is , respectively A. P12,500 and P500 C. P12,500 and P625 B. P7,500 and P500 D. P7,500 and P300 E. 13. Which of these events will cause an increase in both current ratio and quick ratio assuming both ratios are greater than 1 at present? A. Payment of long term loan B. Sale of merchandises on account A1 PASSERS REVIEW CENTER///184 C. Purchase of inventories on account D. Approval of short term loan application E. 14. Consider the following events that occurred at Andy Company during 2010:  January 1: issued bonds for P100,000  January 20: used cash from the sale of bonds to purchase equipment for P80,000  October 1: sold used equipment for P15, 000. This equipment had been acquired in 2003, and was sold at a P15,000. This equipment had been acquired in 2003, and was sold a P5,000 glass  December 31: paid interest of P8,000 on the bonds F. G. Based on the factor above, what would be Andy’s net cash flow from investing activities for 2010? A. P95,000 decrease C. P10,000 increase B. P65,000 decrease D. P5,000 increase E. 15. Which of the following activities is not a batch level activity? A. Production line set up C. Installation of vehicle engine B. Production line inspection D. Scheduling E. 16. The Soderling Company is in the process of preparing a purchases budget for the second quarter of 201. Forecast of sales for the second quarter follow: F. April 14,900 units G. May 13,500 units H. June 16,200 units I. The March sales were P12, 500 units. Cost of goods sold is expected to be P8 per unit. Soderling would like to have ending inventory each month equal to 151 of the following month’s predicted sales. J. What is the total cost of purchases in April? A. P119,200 C. P108,000 B. P117,600 D. P104,560 E. F. 17. the most effective fiscal program to help reduce demand pull inflation would be to A. Decrease the rate growth of the money supply B. Increase both taxes and government spending C. Decrease taxes and increase government spending D. Increase taxes and decrease government spending G. 18. Murray Legal Services Company has fixed costs of P56,000 and variable cost of P120 per hour of service rendered. Clients are charged an hourly rate of P120 per hour . According to Company budget, the company expect to provide clients with 1,200 hours of service H. I. What is the margin of sotety as a percentage of budgeted hours? A. 80% C. 20% B. 40% D. 10% E. 19. Which ratio is preferred to be lower for a large retail company? A. Price earnings ratio C. Inventory turnover ratio B. Defensive interval ratio D. Times interest earned ratio E. F. Items 20 to 22 are based on the following information G. Robin Company produces two automotive parts, carburettors and air filters. Both products are made in the same manufacturing facilities but are produced under processes. To accomplish an accurate allocation of production costs, the company uses the activity based costing. The cost accountant for the company provided information about the activity used to produce the company’s products. The activities were organized into the following overhead cost categories. The most appropriate cost driver for each category is also provided. H. I. Category Estimate cost Cost Driver Carburettors Air Filters J. Unit- level P60,000 labor hours 900 700 K. Batch- level P22,000 Set ups 20 30 L. Product- level P45,900 Storage space 2,000 sq. Ft 4,000 sq. Ft. M. Facility –level P100,000 Machine hours 7,500 12,500 N. O. 20. The amount of batch level cost that should be allocated to the carburetor product line would be A. P14,250 C. P9,625 B. P13,200 D. P8,800 E. 21. The amount of facility level coat that should be assigned to air filters is A. P37,500 C. P62,500 B. P50,000 D. P100,000 A1 PASSERS REVIEW CENTER///185 E. 22. If carburettors and air filters require the same amount of direct labor, what will be effect if labor hours are used as the allocation base for product level cost? A. Air filters will be over costed B. Carburettors will be over costed C. Air filters and carburetors will be over costed D. Air filters and carburetors will be under costed F. 23. Gonzales Company has annual demand of 5,000 units for its lone product. If the unit carrying cost is P0.40 and optimal order quantity is 500 units, what would be the annual ordering costs? A. P20 C. P100 B. P40 D. P200 E. 24. Which of the following statements is true? A. As an advantage of the payback period is that it consider the time value of money B. The internal ratio of return of a project will be equal to its unadjusted rate of return. C. If an investment opportunity is acceptable based on its net present value, it will also be acceptable based on its internal rate of return D. All methods of capital planning are based on cash flows only F. G. Items 25 and 26 are based on the following information H. Williams Company makes a white liquid substance that is used to correct errors made on printed documents. Williams expect to use 4 ounces of a chemical known as Fatal per bottle of the correction fluid. Fatal is expected to cost P0.40 per ounce. Actual materials cost amounted to P0.46 per ounce. Williams expected to make and sell 1,000,000 bottles of correction fluid during the accounting period. Actual production amounted to 900,000 bottles and 4,095,000 ounces. I. J. 25. The materials price variance for Fatal is A. P245,700 favorable C. P40,000 favorable B. P245,700 unfavorable D. P40,000 unfavorable E. F. 26. The materials usage variance for Fatal is A. P198,000 favorable C. P245,700 favorable B. P198,000 unfavorable D. P245,700 unfavorable E. 27. Debt is generally the least expensive source of capital. This is primarily due to A. Fixed interest payments B. Its position in the priority of claims on assets and earnings in the event of liquidation C. The tax deductibility of interest payments D. The secured nature of a debt obligation F. 28. The following information is for Serena Company: G. Product A-Revenue P8.00 H. Variable cost P2.00 I. Product B- Revenue P4.00 J. Variable cost P1.00 K. L. Total fixed costs are P400, 000. Actual sales are 300,000 units (sales mix is one unit of product A and two units of product B). What is the degree of operating leverage? A. 3 times C. 2 times B. 2.5 times D. 1.5 times E. 29. The difference between variable costing and absorption costing net income centers on how to account for A. Direct material cost C. Variable indirect costs B. Fixed indirect costs D. Variable and fixed indirect cost E. 30. Roddick Company reports the following December 31, 2010 balance sheet data: F. Current liabilities P280, 000 G. Bonds payable, 16% 120,000 H. Preferred stock, P100 par 200,000 I. Common stock, P 25 per value, 16,800 shares 420,000 J. Premium on common stock 240,000 K. Retained earnings (adjusted) 180,000 L. Total liabilities and Stockholders equity P1, 440,000 M.  Market price per share of common stock is P35  Income before tax for 2010 is P160,000  The tax rate is P40%  No additional contribution from stockholders was made during 2010 A1 PASSERS REVIEW CENTER///186  Common stockholders equity in the previous year was P860,000 N. O. If 2010 earnings per share is P3.50, then what should be the preferred dividend rate? A. 18.6% C. 48.0% B. 29.4% D. 50.6% E. 31. Which is a term not associated with working capital management? A. Baumol model C. Du Point technique B. Lockbox system D. Materials requirement planning system E. 32 Kim laboratory is considering acquiring new equipment that management estimates will reduce its cash operating expenses by P 50,000 each year for the next five years. After five years, the company believes the equipment will be technologically obsolete and will have no salvage vale. The equipment will cost P180, 000 and the company will have to spend P20, 000 immediately to train its staff to use the new equipment F. G. What is the closest internal rate of return of this investment project? A. 25% C. 8% B. 12% D. 5% E. Items 33 and 34 are based on the following information F. Caroline Company planned to make 50,000 units of product. Caroline actually produced 51,000 units of product. The fixed cost predetermined overhead rate was P1. 50 per unit. Actual fixed overhead costs amounted to P210, 000. G. 33. The fixed cost volume variance for Caroline Company is A. P19,500 favorable C. P4,500 favorable B. P19,500 unfavorable D. P4,500 unafavorable E. 34. The fixed cost spending variance for Caroline Company is A. P15,000 favorable C. P19,500 favorable B. P15,000 unfavorable D. P19,500 unfavorable E. 35. The opportunity cost of funds invested receivable refers to A. The firms cost of short term borrowing B. The cost of credit analysis and bad debt expense C. The rate of return on the best alternative use of funds D. The lost sales involved with a tight credit policy F. 36. Maria is going to sell ad-hoc disks for P40 a box; on; box is planning to rent a booth at the upcoming Area Computer show. She has three options for attending the show: G. H. Option 1: paying a fixed fee of P3, 000 I. Option 2: paying a P1, 000 fee plus 10% of hr revenue made at the show J. ,or K. Option3: paying 25%of her revenue made at the convention L. M. What would be the indifference point between option 1 and option 2? A. 5000 units C. 300 units B. 4000 units D. 200 units E. 37. The basic objective of financial management is to A. Maximize corporate profits B. Maximize the wealth of shareholders in the long run C. Maximize sales revenue D. Maximize the wealth of the society in the long run F. 38. Ana computer expects to pay a P2 dividend in the coming year. Security analysis expects Ana’s dividends and earnings to grow at a rate of 9% for the indefinite future. G. H. If the market’s required rate of return on Ana stock is 25% what is the current stock price? A. P 4.00 C. P8.00 B. P7.50 D. P12.50 E. 39. A highly automated plant would generally have A. More variable than fixed cost C. All fixed costs B. More fixed than variable costs D. All variable costs E. 40. Justine Corp. Plans to replace a [production machine that was acquired several years ago. The old machines acquisition cost was P450, 000 with salvage value of P50, 000. The machine being considered is worth P800, 000 and the supplier is willing to accept the old machine at a trade in value of P60, 000. Should the company decide or not to acquire the new machine, it needs to repair the old one at a cost of P200, 000. Tax-wise, the trade in transaction will not have any implication but the cost to repair is tax-deductible. The effective corporate tax rate is 35% of net income subject to tax. F. G. For purpose of capital budgeting, the net investment in the new machine is A. P540,000 C. P660,000 B. P610,000 D. P800,000 A1 PASSERS REVIEW CENTER///187 E. 41.Mauresmo Company presented to you its budgeted data for the year 2010. It is estimated that the company will sell P40 units for the year 2010. The estimated costs of these sales are follows: F. Amount Per Unit G. Materials P240,000 P1,000 H. Direct labor 40,800 170 I. Fixed overhead 98,400 410 J. Administrative expenses 100,800 420 K. L. Selling expenses are expected to be 20% of sales. Pre-tax profit amounts to P500 per unit. The selling price per unit must be set as at: A. P2,000 C. P2,625 B. P2,500 D. P3,125 E. 42. An IT auditor is least likely to use computer software to A. Construct parallel simulations C. Prepare spreadsheets B. Access client data files D. Assess computer control risk E. 43. Using 360 day year, what is the annual cost of trade credit if a supplier gives a credit term of 2/10 n/60 and the customer delays payment as a means of short term credit financing? A. 14.69% C. 12.24% B. 14.40% D. 12.00% E. 44. Host firm choose accelerated depreciation method because A. Reported net income is higher B. Tax payments are made sooner, resulting in lower deferred tax liability C. Operating expenses are correspondingly reduced D. Income taxes are deferred, providing the firm with a significant present value benefit. F. 45. Elena Company’s budgeted sales for the coming year are P40, 500,000 of which are 80 to be expected to be credit sales at terms of n/30. Elena estimates that a proposed relaxation of credit standards would increase credit sales 200 and increase in the average collection period from 30 days to 40 days. Cost of money is 15%. Based on a 360 day year, how much opportunity cost is involved with the proposed relaxation of credit standard? A. P243,000 C. P900,000 B. P540,000 D. P1,620,000 E. 46. Simple regression analysis provides the means to evaluate a line of regression which is fitted to a plot of data and represents A. The way cost change in respect to independent variable B. The way cost change in respect to dependent variable C. The variability of expense with peso of operation D. The variability of expense with peso of production F. 47. If the investment turnover decreased by 20% and profit margin decreased by 30%, then ROI would A. Increase by 30% C. Decrease by 44% B. Decrease by 20% D. Not be determined without more information E. 48. Each unit of product Ivanovic taxes five direct labor hours to make. Quality standards are high and 8% of units produced are normally rejected due to substandard quality. Next month’s budgets are as follows: F. Beginning inventory of finished goods 3,000 units G. Planned ending inventory of finished goods 7,600 units H. Budgeted sales of Ivanovic 36,800 units I. J. All stocks of finished goods must have successfully passes the quality control check. What is the direct labor budget for the month? A. 198,720 hours C. 223,500 hours B. 200,000 hours D. 225,.500 hours E. 49. Which of the following statements pertains to a batch processing system? A. Data are immediately available to users upon entry B. It permits both immediate posting and inquiry of master files as transactions occur C. It accumulate transaction records into groups for processing against the master file D. It is in direct communication with computer, giving it the capability to handle transactions as they are entered. F. 50. Sharapova Company has total debt of P360,000 and shareholders equity of P500,000sharapova is seeking capital to fund an expansion . sharapova is planning to issue an additionalP300,000 in common stock and is negotiating with a bank to borrow additional funds. The bank requires a debt to equity ratio of 0.75. G. What is the maximum additional amount Sharapova will be able to borrow? A. P240,000 C. P440,000 B. P330,000 D. P600,000 E. 51. Which is not considered in determining the cash conversion cycle? A. Age of receivable C. Age of marketable securities B. Inventory conversion period D. Payable deferral period A1 PASSERS REVIEW CENTER///188 E. 52. Stosur Company is a producer and distributor of various motorized recreational scooter, bike and motorcycle products. The first division handles scooters and would like to earn a long run rate of return 0f 20%. The first division will change its unit selling price as necessary to provide this return. The following data are available on the division and its product: F. Variable cost per scooter P200 G. Total annual fixed cost P1,220,00 H. Long run normal demand 10,000 units to each year I. Average operating assets P1,400,00 J. K. Compute the per unit selling price that will provide the desired rate of return. A. P2.82 C. P3.22 B. P2.94 D. P3.50 E. 53. The following data was taken from the records of Wilfred Corporation: F. 2009 2010 G. Inventory, 12/31 P120, 000 P100,000 H. Receivable 12/31 88,000 62,000 I. Cash sales 800,000 1,000,000 J. Average age, inventory 30 days 45 days K. Average age, receivable 24 days 36 days L. What is the 2010 gross profit? (Assume 360 day year) A. P870,000 B. P820,000 C. P200,000 ` D. P120,000 M. 54. Electronic fund transfer (EET) is a service provided by financial institutions worldwide that is based on electronic data interchange (EDI) technology. Which of the following risk is not greater in an EET environment than in a manual system using paper transactions? A. Unauthorized access and activity B. Duplicate transaction processing C. Highest per transaction D. Inadequate backup and recovery capabilities N. 55. At the end of 2010, ABC Plastic Products had P240, 000,000 total capital. ABC Management desires to increase its production machinery during 2010 by P40, 000,000 in order to provide for new product lines. Bond financing will be O. 13% and wills ell at par. Preferred will be sold at a par value of P100 and will have a 12 percent dividend payment. Common stock, which currently is selling at P30 per share, can be sold at P28, net of related flotation costs. Internal funding available from retained earnings is estimated to be P5, 000,000. The firm has a 9% dividend yield and a growth rate is 40%. ABC’s present capital structure listed below is considered optimal. P. Q. Debt (8% coupon bonds) P20,000,000 R. Debt (10% coupon bonds) __40,000,000__ S. Total Debt P60,000,000 T. U. Preferred stock P36,000,000 V. Common stock (P10 par) 60,000,000 W. Retained earnings _84,000,000__ X. Total equity _P180,000,000 Y. TOTAL _P240,000,000_ Z. AA. In order to maintain the present capital structure , how much the new capital assets must be financed by equity capital? A. P24,000,000 C. P25,000,000 B. P24,500,000 D. P25,500,000 E. 56. The income statement for Martin Company for 2010 shows cost of sales of P7, 420. The following information is also available. F. Inventory, `1/1 P520 G. Inventory, 12/31 610 H. Accounts payable, 1/1 280 I. Accounts payable, 12/31 480 J. K. Cash paid in 2010 for merchandise was A. P 7,130 C. P7,710 B. P7,310 D. P7,790 E. F. Items 57 to 61 are based on the following information A1 PASSERS REVIEW CENTER///189 G. Boston Company’s income statement for last month is given below H. Sales(15,000 units @ P30) 450,000 I. Less variable expenses 315,000 J. Contribution margin 135,000 K. Less fixed expenses 90,000 L. Net income 45,000 M. N. The industry in which Boston Company operates is quite sensitive to cyclical movements in the economy. Thus, profits very considerably from year to year according to general economic conditions.. the company has a large amount of unused capacity and is studying ways of improving profits. O. P. 57. New equipment has come onto the market that would Boston Company to automate a portion of its operations. Variable costs would be reduced by P9 per unit. However, fixed costs would increase to a total of P225,000 each month. Q. R. How much income for the month would the company earn if the new equipment is purchased? A. P45,000 C. P60,000 B. P30,000 D. P75,000 E. 58. How many units as increase or decrease in breakeven point if the equipment is purchased? A. Zero C. 3,200 units B. 2,500 units D. 4,000 units E. 59. The degree of operating leverage during the month where the new equipment is used is: A. 3.0 times C. 6.0 times B. 4.5 times D. 5.0 times E. 60. Refer to the original data. Rather than purchase new equipment, the president is thinking about changing the company’s marketing method. Under the new method, sales would increase by 20% each month and net income would increase by one third. Fixed costs could be slashed to only P48, 000 per month. Compute the break even point for the company after the change in marketing method. A. 8,000 units B. 9,000 units C. 10,000 units D. 12,500 units F. 61. Assuming that during the month following the month that the new equipment has been started in use, the unit sales increased by 4,500 units. The variable expenses per unit and the monthly fixed costs as affected by the acquisition of the new equipment are expected to remain constant.. G. H. What is the expected profit of the company for that month? A. P126,000 C. P81,000 B. P85,500 D. P45,000 E. F. Items 62 to 68 are based on the following information G. Due to erratic sales of its sole product – a high capacity for laptop computers, New York Company has been experiencing difficulty for sometime. The company’s income statement for the most recent month is given below: H. Sales (19,500 units @P300) P5, 850,000 I. Less variable expenses 4,095,000 J. Contribution margin 1,755,000 K. Less fixed expenses 1,800,000 L. Net loss P(45,000) M. N. 62. The break in peso sales for New York Company is: A. P6,000,000 C. P7,500,000 B. P2,571,429, D. 63. The president believes that a P160, 000 increases in the monthly advertising budget, combined with an intensified effort by the sales staff, will result in P800, 000 increases in monthly sales. If the president is right, what will be the effect on the company’s monthly net income or loss? A. P120,000 increase C. P120,000 decrease B. P80,000 increase D. P80,000 decrease E. 64. Refer to the original data. The sales manager is convinced that a 10% reduction in the selling price, combined with P600, 000 increases in the monthly advertising budget, will cause unit sales to double. What will the new profit or loss if these changes are adopted? A. P60,000 C. P45,000 B. P(60,00) D. P(45,000) E. 65. Refer to the original data. The marketing department thinks that a fancy new package for the laptop computer battery would help sales. The new package could increase packaging costs by P7.50 per unit. A1 PASSERS REVIEW CENTER///190 A. B. E. A. B. E. Assuming no other changes, how many units would have to be sold each month to earn a profit of P97, 500? 21,818 C. 25,450 23,000 D. 28,000 66. Refer to the original data. By automating certain operations, the company could reduce variable costs by P3 per unit.. However, fixed costs would increase by P72, 000 each month. How would be the break even point in units change if the company automated the operations? I,000 units increase C. 3,000 units increase 1,000 units decrease D. 3,000 units decrease 67. At what level of production the automation of the production process be indifferent to the present process? 18,000 C. 24,000 21,000 D. 28,000 A. B. E. F. Patents, net................................. 100,000 60,000 G. Long term debt.......................... (500,000) (400,000) H. I. At how much will the building and equipment and land be recorded, respectively? A. P1,400,000 and P500,000 C. P1,550,000 and P500,000 B. P1,400,000 and P300,000 D. P1,550,000 and P300,000 E. 42. The condensed balance sheet accounts of Lani Inc. and magi Inc, are shown below: F. Lani Magi G. H. Assets P3, 125,000 P875,000 I. Liabilities 1,625,000 250,000 J. Capital stock (par P100) K. Lani 1,250,000 L. Mani 500,000 M. Retained earnings 250,000 125,000 N. O. Lani buys all of the issued and outstanding shares of Magi at P140 per share. In the consolidated balance sheet, the total asset is: A. P4,000,000 B. P3,150,000 C. P4,025,000 D. P4,075,000 P. 43. Triad Corporation’s shipments to and from Olongapo City branch are billed at 120% of cost. On December 31, Olongapo branch reported the following data, at billed prices: inventory, January 1, of P33,600; shipments received from home office of P840,000: shipments returned of P48,000; and inventory, December 31, of P36,000. What is the balance of the allowance for over valuation of branch inventory on December 31 before adjustments? A. P5,600 C. P6,000 B. P137,600 D. P145,600 E. 44. Paro Company acquires all the shares of Subido Co. for a consideration valued at P5, 000,000. Costs of registering and issuing securities used in the business combination were P60, 000, and direct costs amounted to P40,000. Subido’s stockholders equity is P4, 000,000. F. G. After the business combination, how much is the goodwill under the purchase method? A. P1,060,000 C. P1,040,000 B. P1,100,000 D. P1,000,000 E. 45. Pre-consolidation data on the balances sheet accounts of Mulligan Corp; and Northrop Corp. Show: F. Mulligan Northrop G.Assets P212,500 P150,000 H. I. Liabilities 62,500 50,000 J. Capital stock (par P100) 100,000 62,500 K. Additional paid in capital 12,500 25,000 L. Retained earnings 37,500 12,500 M. Liabilities and Equity P212, 500 P150, 000 N. O.The shareholders of Mulligan and Northrop agree to consolidate whereby Obet Inc. a new corporation, it p acquire the net assets of the constituent corporation. Obets share are to be issued in exchange for shares of Mulligan and Northrop on 5-for -1 basis. The consolidation is appropriately reckoned as pooling of A1 PASSERS REVIEW CENTER///191 interests. if the share of Obet have a par value of P25, what is the additional paid in capital to be recorded in its books? A. P0 C. P78,125 B. P37,500 D. P46,875 E. 46. Nona Inc. acquired on January 1, 19X6 all the issued and outstanding common share of Olga Inc. for P310, 000 in a transaction properly treated as a purchase. On this day, the assets and liabilities of Olga. Inc. show F. G. Cash P30, 000 H. Merchandise inventory 90,000 I. Plant and equipment 160,000 J. Goodwill, net 50,000 K. Liabilities (60,000) L. M. Net Assets P270, 000 N. O. The additional paid in capital to be recorded in its books? A. P0 C. P78,125 B. P37,500 D. P46,875 E. 46. Nona Inc. acquired on January 1, 19X6 all the issued and outstanding common shares of Olga Inc. for P310, 000 in a transaction properly treated as purchased. On this day, the assets and liabilities of Olga Inc. show: F. G. Cash P30,000 H. Merchandise inventory 90,000 I. Plants and equipment 160,000 J. Goodwill, net 50,000 K. Liabilities (60,000) L. M. Net assets P270,000 N. O. Per appraisal, plan t and equipment and merchandise inventory were valued at P190, 000 and P75,000, respectively. What is the amount of goodwill resulting from this transaction? A. P125,000 C. P75,000 B. P40,000 D. P0 E. F. G. H. I. J. K. 47.Rado owns a 70% interest in a partnership, and has an capital balance of P80,0000. Seto owns a 30% interest and has a capital balance of P60,0000. Tato invest P50, 000 for a 20 percent L. M. Under the goodwill method, how much goodwill will be recorded and to whom will it be given? A. P12,000 goodwill to radio and Seto C. P15,000 goodwill to Rado and Seto B. P15,000 goodwill to Tato D. P60,000 goodwill to Rado and Seto E. F. 48. Ingram Julian and Keno form a partnership and agree to maintain average investment of P100, 000, P50, 000 and P 50,000, respectively. Interest on an excess or deficiency (as the case may be) in capital contributions to be computed at 6%. After interest allowances, residual sums shall be shred by Ingram, Julian and Keno at 5:3:2. A net loss of P2, 500 is incurred by the partnership for the first six months. Average amounts invested by the partners during these six months. Average amounts invested by the partners during these six-months are: G. H. Ingram P120,000 I. Julian 55,000 J. Keno 40,000 K. L. How much is Ingram’s share in the net loss? A. P875 B. P1,475 C. P1,250 D. P1,750 A1 PASSERS REVIEW CENTER///192 M. 49. Debbie Jones a senior partner in law firm, has a 30% participation in the firm’s profits and losses during the year 19X5. Jones withdrew P130, 000 against her capital but contributed property with a fair value to the firm of P25,000. What was the net income of the law firm for 19X5? A. P150,000 B. P550,000 C. P233,333 D. P350,000 N. 50. Bureau MNO’s appropriation for the year amounted to P1, 000,000. MNO received from the Department of Budget and Management (DBM) a General Allotment Release Order (GARO) in the amount of P900,000. The receipt of the Advice of Allotment from the DBM will be recorded in the books of: A. Bureau of Treasury B. Department of Finance C. Bureau MNO and Commission on Audit D. Bureau MNO only O. P. ***END*** Q. R. S. T. U. V. W. X. Y. Z. AA. AB. AC. AD. AE. AF. AG. AH. AI. AJ. AK. AL. AM. AN. AO. AP. AQ. A1 PASSERS TRAINING, RESEARCH, REVIEW & DEVELOPMENT COMPANY AR. 2nd Floor Sommerset Bldg., Lopez Jaena St. Jaro, Iloilo City AS. Tel. No.: (033) 320-2728; 09106547262 AT.Email Address: [email protected] AU. AV. BOARD OF CERTIFIED PUBLIC ACCOUNTANT AW. AX.CERTIFIED PUBLIC ACCOUNTANT Licensure Examination AY. AZ. AUDITING PROBLEMS BA. BB.PCF voucher dated Dec. 31,19X7 BC. For Miscellaneous expenses 100 BD. Cash in the bank balance per books 48,200 BE. A bank debit memo for P10, 200 BF. representing returned customer check BG. marked “DAUD” including a service charge of P200 has not been recorded BH. on December 31, 19X7. BI. BJ. 3. Marketable securities at cost declined by P50, 000 in 19X7. In 19X7, a sale was made at a gain of P10, 000 and P60, 000 was credited to Sales. A1 PASSERS REVIEW CENTER///193 BK. BL. 4. One percent of total sales has to be set up for uncollectible accounts, the allowance of which is maintained by the home office. BM. BN. 5. Accumulated depreciation balance for 19X5 and 19X6 were P175, 000 and P350, 000, respectively. BO. BP.6. The bank loan carries a 20% interest per annum which was secured on July 1, 19X6. The amount of P250, 000 has been paid in 19X7 together with the accrued interest thereon. Interest on this loan was paid up to July 1, 19X7. A second loan worth P1, 000,000 was secured on July 1, 19X7 to buy additional equipment. The interest for one year at 18% was deducted in advance and charged to operating expenses. BQ. BR. 7. Patent amortization was charged to operations. BS. BT. 8. In January, 19X7, the company established a branch in Baguio which reported the following data on December 31, 19X7: BU. Debit (Credit) BV.Cash.....................................................P 50,000 BW. Receivables..........................................60,000 BX.Inventories...........................................110,000 BY.Furniture and equipment (net).............400,000 BZ.Payables............................................ (120,000) CA. Sales.................................................. (1,000,000) CB. Cost of goods sales..............................800,000 CC. Operating expense, including CD. Depreciation..............................100,000 CE. CF.9. The inventories excluded the CG. * unsold goods with consignees at cost 50,000 and CH. *freight cost corresponding to the CI. unsold consigned goods 2,000 CJ. not yet recorded , but included CK. * goods received from consignor............50,000 CL. CM. 10. The payables include the accrued interest for 19X7 on the bank loan, which amount was debited to operating expenses. CN. Based on the foregoing information and any additional assumption which may be given in each number, you have to answer the following 40 questions. Your choice must be the one nearest to the correct answer. CO. CP. CQ. CR. CS. 1. The journal entry to correct the recording of stock premium will credit. A. Subscribed common stock CT. CU. INSTRUCTION: Select the correct answer for each of the following questions. Mark only one answer for each item by shading the box corresponding to the letter of your choice on the answer sheet provided. STRICTLY NO ERASURES ALLOWED. Use the pencil no. 1 only. CV. CW. SITUATIONAL CX. CY.Situation 1. You were requested by the board of the directors of the WW Industries to audit its books for the year ending December 31, 19X7. The condensed comparative balance sheet items of the company for December 31, 19X6 and 19X7 are shown below. CZ. 19X7 19X6 DA. Cash P50, 000 P100, 000 DB. Marketable securities 150,000 200,000 DC. Receivables 500,000 350,000 DD. Inventories DE. Property and equipment (net) DF.Patents 750,000 550,000 4,750,000 3,000,000 250,000 300,000 A1 PASSERS REVIEW CENTER///194 DG. Branch 500,000 0 DH. DI. Payables 900,000 160,000 DJ. Accrued expenses 50,000 40,000 DK. Bank loan 3,250,000 2,500,000 DL.Common stock 2.000,000 1,500,000 DM. Subscribed common stock (net DN. of Subscriptions rec’ble of P125,000)225,000 0 DO. Retained earnings 525,000 300,000 DP. DQ. Income operating and other data are DR. Sales 7,500,000 5,000,000 DS. Cost of goods sold 4,500,000 3,200,000 DT. Operating expenses, including 2,535,000 1,500,000 DU. Depreciation 250,000 175,000 DV. Bad debts 20,000 7,500 DW. Amortization of patents 50,000 25,000 DX. Taxes 400,000 150,000 DY. Net profit 465,000 300,000 DZ. Dividends: 400,000 EA. 19X7, cash, P200, 000, stock, P200, 000 EB. EC. Your examination revealed the following additional information: ED. 1.The company was organized on January 3, 19X6 with an authorized common shares worth P6,000,000, par value per share , P1,500,000. The shares issued in 19X7 were at a premium amounting to P325, 000 which was credited to retained earnings. a 10% stock dividend was declared on December 31, 19X6 but recorded on January 10, 19X7 and issued February 1, 19X7. The company shares were selling at 110. 1. The cash balance in 19X7 consists of the following:  Imprest petty cash fund showed the following as per count: EE. Currency and coin P2,700 EF. Post dated check of an officer 2,000 EG. A. Common stock EH. B. Retained earnings EI.C. Additional paid in capital EJ. 2. The 19X6 10% stock divedends need approval of all the following except one A. Bangko Sentral ng Pilipnas C. Stockholders B. Board of Directors D. Securities and Exchange commission E. 3. The 19X6 dividends will be recorded using per share A. Market value C. Par value B. Book value D. Liquidating value E. 4. The journal entry to take up the issuance of the stock dividends will be debit A. Paid in capital C. Stock dividends payable B. Retained earnings increase D. Common stock E. F. 5. The 19X7 cash dividends will result to A. Retained earnings decrease C. Common stock decrease B. Paid in capital decrease D. Retained earnings increase E. 6. The 19X7 subscribed common stock will be at A. P300,000 C. P250,000 B. P225,000 D. P350,000 E. 7. The 19X6 stock dividend in number of shares will be A. 1,650 C. 2,000 B. 1,500 D. 1,815 E. 8. The 19X7 stock dividend will be based on the number of shares issued and outstanding? A. 20,000 C. 18,150 B. 16,500 D. 15,000 E. 9. If treasury shares were in the books, they will be A. Shown as current asset C. Deducted from common stock B. Deducted from paid capital D. Deducted from total stockholders equity E. 10. Treasury stocks are recorded at A. Cost C. Book value B. Market value D. Par value A1 PASSERS REVIEW CENTER///195 E. 11. The petty cash funds will be shown in the 19X7 balance sheet at A. P4,800 C. P5.400 B. P4,700 D. P5,900 E. 12. Replenishment of patty cash fund will mean a debit to A. Miscellaneous expense C. Receivable- officer and Miscellaneous B. Petty cash fund expense D. Receivable- officer E. 13. The customer’s returned check will be recorded debiting A. Accounts receivable P10,000 & Miscellaneous expense P200 B. Accounts receivable P10,200 C. Cash in bank P10,200 D. Accounts receivable P10,000 F. 14. Subscription receivable, common will be shown in the 19X7 balance sheet as A. Deduction form subscribed common stock C. Deduction from common stock B. Deduction from paid in capital D. Current asset E. 15. The cash storage amounts to A. P0 C. P200 B. P100 D. P300 E. 16. The marketable securities will be shown in the 19X7 balance sheet at A. P110,000 C. P90,000 B. P150,000 D. P100,000 E. 17. Allowance for doubtful accounts will have a 19X7 balance of A. P85,000 B. P75,000 C. P74,400 D. P84,400 A1 PASSERS REVIEW CENTER///196 E. 18. The property and equipment original cost was A. P2,075,000 C. P4,825,000 B. P3,675,000 D. P3,350,000 E. 19. Patent amortization in 19X7 is A. P60,000 C. P50,000 B. P30,000 D. P40,000 E. 20. Property and equipment acquired in 19X7 amounted to A. P4, 825,000 C. P2,200,000 B. P2,400,000 D. P3,675,000 E. 21. The accrued interest on the bank loan on December 31, 19X6 is A. P250,000 C. P225,000 B. P450,000 D. P325,000 E. 22. The 19X6 patent amortization was A. P40,000 C. P25,000 B. P35,000 D. P50,000 E. 23. The branch reported profit / loss in 19X7 is A. P110,000 profit C. P90,000 profit B. P80,000 profit D. P100,000 profit E. 24. The home office will record the branch profit by debiting and crediting respectively A. Profit and loss debit; Branch credit C. Branch debit; retained earnings credit B. Branch debit; branch profit and loss credit D. Branch debit; profit and loss credit E. 25. The 19X7 combined the net income of the WW Industries Company will be A. P255,600 C. P427,500 B. P365,600 D. P315,600 E. 26. The 19X6 retained earnings and as adjusted was A. P135,000 C. P150,000 B. P200,000 D. P300,000 E. 27. The 19X7 balance sheet will show combined cash account of A. P100,000 C. 93,000 B. P103,000 D. P89,700 E. 28. The combined 19X7 book value property and equipment amounts to A. P5,330,000 C. P5,000,000 B. P5,150,000 D. P5,550,000 E. 29. the corrected 19X7 combined Sales will be A. P8,500,000 C. P8,440,000 B. P8,400,000 D. P7,440,000 E. 30. The 19X7 combined net realizable value of receivable amounts to E. 33. Inventories received from consignor will be A. Not recorded but included in the inventories total B. Either recorded or not recorded C. Not recorded, but included in the notes to the balance sheet D. Recorded with a debit to inventories F. 34. The 19X7 combined operating expenses will be A. P2,770,000 C. P2,660,000 B. P2,635,000 D. P2,812,500 E. 35. The 19X7 combined accounts payable will be A. P1,020,000 C.P902,000 B. P572,000 D.P452,000 E. 36. The reciprocal accounts will be eliminated at A. P600,000 B. P690,000 C. P500,000 D. P590,000 F. 37. The 19X7 combined inventories will be A. P612,000 A1 PASSERS REVIEW CENTER///197 B. P852,000 C. P602,000 D. P862,000 G.38. The 19X7 combined accrued expenses will be A. P300,000 B. P400,000 C. P450,000 D. P500,000 H.39. The accrued interest on December 31, 19X7 will be A. P250,000 B. P225,000 C. P450,000 D. P375,000 I. 40. The 19X7 stock dividend was issued at A. Market value B. Book value C. Par value D. Liquidation value J. K. L. ***END*** M. A1 PASSERS TRAINING, RESEARCH, REVIEW & DEVELOPMENT COMPANY N. 2nd Floor Sommerset Bldg., Lopez Jaena St. Jaro, Iloilo City O. Tel. No.: (033) 320-2728; 09106547262 P. Email Address: [email protected] Q. R. BOARD OF CERTIFIED PUBLIC ACCOUNTANT S. T. CERTIFIED PUBLIC ACCOUNTANT Licensure Examination U. V. BUSINESS LAW AND TAXATION W. X. INSTRUCTION: Select the correct answer for each of the following questions. Mark only one answer for each item by shading the box corresponding to the letter of your choice on the answer sheet provided. STICTLY NO ERASURES ALLOWED. Use pencil no. 2 only. Y. Z. MULTIPLE CHOICES: AA. AB. 1. Takes place when two persons in their own right, are creditor and debtor of each other A. Remission C. Novation B. Confusion D. Compensation E. 2. Whenever in an obligation a period is designated, it is presumed to have been established for the benefit of: A. Both creditor and the debtor C. The debtor B. The creditor D. The third party E. 3. An act of liberality whereby a creditor condones the obligation of the debtor: A. Confusion C. Remission B. Compensation D. Novation E. 4. When the thing deteriorates pending the fulfilment of the suspensive condition without the fault of the debtor, the impairment is A. To be borne by the party who is caused the deterioration B. To be borne partly by the debtor and partly by the creditor C. To be borne by the debtor D. To be borne by the creditor F. 5. A owes P10, 000. With consent of both, C pays B P5, 000. Now B and C and the creditors of A to the amount of P5,000 each. Suppose A bus only P5, 000. Which is correct? A. B and C should divide the P5,000 equally C. A may choose whom to pay B. C should be preferred D. B should be preferred E. 7. An obligation where only the prestation has been agreed upon but the debtor may render another in substitution is: A. Conjoint obligation C. Simple obligation B. Facultative obligation D. Alternative obligation A1 PASSERS REVIEW CENTER///198 E. 8. XYZ are solidarily liable to A for P30, 000 which mature on July 1, 1993. On May 1, 1993, X paid A fort he whole amount of the debt. If on December 1, 1993, X will be reimbursed by Y; the latter will be liable for: A. P10,000 with interest from July 1993 to December 1, 1993 B. P10,000 without interest C. P10,000 with interest from May 1, 1993 to July 1, 1993 D. P10,000 with interest from May 1, 1993 to December 1, 1993 F. 9. A thing is not deemed lost when it: A. Perishes B. Disappear in such a way its existence is unknown or it can not be recovered C. Goes out of commerce D. Deteriorates G. 10. Which of the following is not a fundamental obligation of the agent? A. To subordinate his interest in favor of his principal if there is a conflict of interest B. To borrow if he is authorized to lend C. To render an accounting of the transactions D. Not to carry out the agency even if that is the instruction if he knew it would result in a loss or damage to his principal H. 11. a form of delivery which takes place after the seller of the property continues possession of said property no longer as owner but as a mere possessor: A. Traditio constitutum possessorium B. Ttraditio calvis C. Tradito brevi manu D. Quasi-tradition I. 12. When goods are delivered to the buyer on sale or return for a period of seven days, ownership of the goods passes to the buyer, A. Upon delivery of the goods C. Upon acceptance by the buyer of the offer of B. Upon expiration of seven days the seller D. Upon perfection of the contract E. 13. A sold to B a parcel of land for P30,000. The sale is evidenced by memorandum of agreement of sale written in Cebuano dialect. One week later, A sold the same parcel of land to C for 40,000 which was evidenced by a formal deed of sale. upon buying the property C who is aware of the first sale, immediately took possession of the land and registered the deed of sale on her favor with the register of deeds. When informed of the second sale, B subsequently registered an adverse claim to the property. The parcel of land shall belong to: A. B, because sha has got an older title B. C, because the sale was made on a formal deed of sale C. C, because she is the first to register the deed of sale D. C, because she is the first to take possession of the land F. 14. 1st statement: if the agent contracts in the name of the principal and the principal does not ratify the contract, the contract shall be void if the party with whom agency contracted is aware of the limits of the powers granted by the principal. G. 2nd statement: a third person cannot set up the fact that the agent has exceeded his powers if the principal has ratified or has signified his willingness to ratify the agents acts. A. Both statements are false C. 1st statement is false, 2nd statement is true B. Both statements are true D. 1st statement is false, 2nd statement is false E. 15. In case redemption is made, which of the following will not be paid by the seller to the buyer? A. Expenses incidental to the sale paid by the buyer B. All necessary expenses on the thing sold and to be redeemed C. The consideration of the sale paid by this buyer D. Interest on the price paid by the buyer F. 16. Dacion en pago as distinguished from sale: A. The object is always existing and specific C. There is no pre existing obligation B. There is a greater degree of freedom in fixing D. The cause is the price the price E. 17. Ben pledged his watch to V.Y.. Domingo Agencia for P5, 000, On due date, Ben failed to redeem his watch. The pawnshop sold the watch at a public auction to the highest bidder at P4, 000 A. The pawnshop can recover the deficiency of P1,000 from Ben B. The pawnshop cannot recover the deficiency of P1,000 unless there is stipulation C. The pawnshop cannot recover the deficiency of P1,000 even if there is a stipulation D. The pawnshop can recover the deficiency of P1,000 even without a stipulation F. 18. The following are instances of pledged created by operation of law, except: A. Hotel keeper retains the thing brought into the hotel by guest who cannot pay his hotel bills B. An agent retains in pledge the thing which is the object of the agency C. A mechanic retains the car her repaired until he is paid D. A depository retains the thing in pledge until full payment of what is due him A1 PASSERS REVIEW CENTER///199 G. 19. Basco pledged his Omega watch to Cruz pawnshop for P10,000. Basco failed to pay the pawnshop the P10,000 on due date.. Cruz sold the Omega at a public auction to the highest bidder P8,000. A. Cruz can recover the deficiency of P2,000 from Basco B. Cruz cannot recover the deficiency of P2,000 even if there is a stipulation C. Cruz can recover the deficiency of P2,000 even without a stipulation D. Cruz cannot recover the deficiency of P2,000 unless there is stipulation H. 20. A distribution by a corporation of shares held by it in another corporation is A. Stock dividend C. Sale of treasury stock B. Sale of capital assets D. Property dividend E. 21.if the proceeds of the sale of the thing pledged sold at public auction is less than the principal obligation , can the creditor recover the deficiency from the debtor? F. G. 1st answer: No, the creditor is not entitled to recover the deficiency from the debtor. H. I. 2nd answer: Yes, if there is a stipulation to that effect in the contract of pledge. A. 1st answer correct, 2nd answer wrong C. Both answers are correct B. Both answers are wrong D. 1st answer wrong, 2nd answer correct E. 22. Partner who does not participate in the management though he shares in the profits or losses: A. Nominal C. Silent B. Ostensible D. Liquidating E. 23. Which of the following liabilities of the partnership shall rank first in the order of payment? A. Those owing to partners in respect to capital B. Those owing to creditors other than partners C. Those owing to partners other than capital and profits D. Those owing to partners in respect to profits F. 24. Alan, Ben and Cris are partners with capital contribution of P15, 000, P10, 000 and P5, 000 respectively. Suppose on dissolution, the assets of their partnership amounts to only P46,000 and it owes Dan the amount of P50,000. Cris owes Ely on his personal account P6, 000. The partners have no separate personal property except Cris separate property amounts to P7, 000. Which is correct? A. Dan and Ely shall divide Cris separate property equally B. Dan and Ely shall divide Cris separate property pro- rata C. Dan shall be preferred as regards Cris separate property D. Ely shall be preferred as regards Cris separate property G. 25. X,Y and Z, capitalist partners, each contributed P10,000 and A, the industrial partner contributed his industry. The partnership is indebted to B in the amount of P90,000. The remaining assets of the partnership amounted to only P30,000. Suppose B got the P30, 000, how can he recover the deficiency of his credit? A. B can recover P15,000 each from X,Y,Z and A B. B can recover P60,000 from either X,Y, and Z C. B can recover P15,000 each from X,Y and Z, but A is exempt because he is an industrial partner D. B can recover P20,000 each from X,Y and Z only H. 26. The change in the relation of the partners caused by any partner ceasing to be associated in the carrying on of the business is: A. Winding up C. Liquidation B. Dissolution D. Termination E. 27. Which of the following is not correct? In a limited partnership composed of A, B and C, the contributed may be as follows: A. A- property (limited partner); B-services (general partner); C- cash ( general partner) B. A- cash ( limited partner); B- property (general partner); C- services (general partner) C. A- services (limited partner); B- cash ( general partner); C- services ( general partner) D. A- cash (limited partner); B- cash ( general partner) C- services ( general partner) F. 28. Partner who is both a secret and silent partners: A. Limited C. Dormant B. Nominal D. Ostensible E. 29. A person who directly represents himself or consents or allows him to be represented as a partner in an existing partnership or with one or more persons who are not actually partners is: A. Partner by estoppels C. Ostensible partner B. Silent partner D. Nominal partner E. 30. They are issued to those who helped in incorporating the company or for services rendered in launching the welfare of the company: A. Preferred stock C. Founders stock B. Stock in escrow D. Promotion stocks E. 31. Any direction of a corporation may be removed from office by a vote of the stockholders holding or representing: A. Majority of stockholders present C. 2/3 of the outstanding capital stock B. ¾ of the outstanding capital stock D. Majority of the outstanding capital stock E. 32. Three (3) of the following are attributes of a corporation. Which is the exception? A1 PASSERS REVIEW CENTER///200 A. Created by agreement of the incorporators B. An artificial being C. Has right of succession D. Has powers, attributes and properties expressively authorized by the law incident to its existence F. 33. As regards to corporate by-laws, which of the following rules is not correct? A. To adopt by-laws, majority of the outstanding capital stock is needed B. To delegate to the Board of Directors or trustees the power to amend or repeal the bylaws, 2/3 of the outstanding capital stock is needed C. To revoke the power delegated to the Board of Directors to adopt new by-laws, 2/3 of the outstanding capital stock or of its member is needed. D. To amend or repeal the by-laws, majority of the Board of Directors and of outstanding capital stock is needed. G. 34. ABC partnership becomes insolvent due to mismanagement and cannot pay its liability to XYZ corp. With the approval of its directors and stockholders, XYZ Corp.., entered into a management contract with ABC Partnership to manage their partnership business with the agreement that 50% of the net profits will be applied to the payment of the partnership debt to the corporation. H. Question 1. Can XYZ. Validly enter into the above management contract? I. Question 2. Is XYZ Corp. Considered as partner of ABC partnership by the fact that is receiving a share of the net profits of the partnership? J. ANSWER I: Yes, a corporation has the power to enter into such management contract provided by the contract is approved by the Board of Directors and majority of outstanding capital stock. K. ANSWER 2: No, XYZ Corp. Cannot be considered a partner because it is receiving a share of the net profits not as actual share of the profits but merely as payment for the partnership’s obligation to corporation. A. 1st answer correct; 2nd answer wrong B. 1st answer wrong; 2nd answer correct C. Both answers are correct D. Both answers are wrong L. 35. The power to deny pre-emptive rights as corporate power is classified as: A. Incidental power C. Implied power B. Express power D. Discretional power E. 36. As regards treasury stocks, which is not correct? A. They have no voting rights as long as they remain in the treasury B. They may be distributed as property dividend if there are retained earnings from operations C. They are not entitled to dividends D. They are considered part of earned or surplus profits and therefore distributable as dividends F. 37 Issued to those who in some way worked for incorporating the company or for services rendered i launching the welfare of the corporation is called: A. Common stock C. Founders stock B. Promotion stock D. Shares in escrow E. 38. The drawee bank is not primarily liable: A. Cashier’s check C. Manager’s check B. Certified check D. Traveller’s check E. 39. If the instrument is payable to the order of the third person A. He is liable to all parties subsequent to the payee B. He is not liable to any party C. He is liable to the payee and to all subsequent parties D. He is liable to all parties subsequent to the maker or drawer F. 40. Which of the following is not negotiable? A. Pay to D or order P10,000 , on or before December 31, 1997. (sgd B) B. Pay to A or order P10,000, notice of dishonor waived. (sgd B) C. Pay to B or order P5,000 , or deliver two horses at the option of the holder. (sgd. C) D. Pay to C or order P10,000 and to deliver 10 sacks or rice. (sgd. D) G. 41. A bill of exchange drawn on a bank and payable on demand is a’, A. Check C. Domestic bill B. Treasury bill D. Bill of lading E. 42. Which is not correct? The acceptor by accepting the instrument A. Admits the existence of the drawer , the genuineness of his signature and his capacity and authority to draw the instrument B. Admits the existence of the payee and his capacity to indorse C. Engages that he will pay it according to the tenor of his acceptance D. Admits the existence of the endorser, the genuineness of his signature and his capacity and authority to draw the instrument F. 43. CASE 1: angel buys a diamond ring for P50, 000 for which he issued a check. Later Angel found out the diamond to be an ordinary glass. G. CASE 2: Ben obtains the signature of Cris for autograph purpose. Ben writes a promissory note above Cris signature and endorses the note to Dan, a holder in due course. H. What kind of defences may be availed of by the marker? A1 PASSERS REVIEW CENTER///201 A. Real defense in 1st case , personal defense in 2nd case B. Personal defense in 1st case, real defense in 2nd case C. Real defense in both cases D. Personal defense in both cases I. 44. Maturity of undated negotiable instrument issued payable 30days after sight is computed from: A. Date of first endorsement C. Date of first presentation for acceptance B. Date of last endorsement D. Date of issue E. 45. When is negotiable instrument payable to order? A. When drawn payable to the order of a specified person to him or his order B. When payable to the order of a fictitious or non-existing person, and such fact was known to the person making it. C. When the name of payee does not purport to be the name of any person D. When the only or last endorsement is an endorsement in blank F. 46. Which of the following is not the characteristic of a bill of exchange? A. Original parties are the drawer, drawee, payee C. Drawer is primarily liable and acceptor D. Contains unconditional order` B. Acceptance is generally required E. 47. a taxed payer is assessed for taxes for filing false tax return and for failure to pay taxes. The taxpayer files an administrative protest with the BIR contesting said assessment. May the BIR during the pendency of the protest file a criminal action against the taxpayer for filing a false return? Which of the following answers is not correct? A. Yes, for what is involved in this case is not the collection of taxes but a criminal prosecution for collection for violation of the Tax Code B. The protest of taxpayer against the assessment cannot stop the prosecution for violation of the Tax Code C. Yes, because there is no requirement for the precise computation and assessment of the tax before there can be a criminal prosecution D. No, the filing of the criminal action is premature and precipitate since the protest has not yet been resolved F. 48. Which of the following cases constitute objectionable double taxation? A. A license fee and a tax imposed on the same business or occupation for selling the same articles B. A tax imposed both on the occupation of fishing and on fishpond operation C. Person engaged in leasing or selling real property are subject to real estate dealers tax and their sales are also subject to 10% VAT D. A tax of 1% is imposed for bank reserve deficiency while a penalty of 1/10 of 1% a day is also imposed as a consequence of such reserve deficiency. G. 49 Police power as distinguished from Eminent Domain Power: A. Just compensation is received by owner of the property B. Maybe exercised by private individuals C. Superior to the impairment clause of the constitution D. Property is taken by the government for public purpose H. 50. A tax where in both the incidence of the liability for the payment of the tax as well as the burden of the tax falls on the same person: A. Direct tax C. Indirect tax B. Personal tax D. Property tax E. 51. The running of prescriptive period of assessment and collection of taxes is suspended unde3r the following circumstances, except one: A. When the tax payer is out of the country B. When the BIR commissioner is prohibited from assessing or collecting the tax C. When the taxpayer requests for reinvestigation D. When the warrant of distrait and levy is duly served F. 52. The seizure by the government of personal property to enforce the payment of taxes to be followed by its public sale if the taxes are not voluntarily paid: A. Confiscation C. Forfeiture B. Distraint D. Levy E. 53. The following constitute double the taxation, except when A. Both taxes are imposed in the same year B. Both taxes are levied for the same purpose C. Both taxes are imposed by the same taxing authority D. Both taxes are imposed upon the same person F. 54. The BIR Commissioner may compromise payment of internal revenue taxes when: G. 1st ground: a reasonable doubt as to the validity of the claim against the tax payer exists H. 2nd ground: when the tax or any portion thereof appears to be unjustly or excessively assessed. A. Both grounds are wrong C. Both grounds are correct B. 1st ground wrong, 2nd ground correct D. 1st ground correct, 2nd wrong E. 55.under the following cases, except one, a working wife may claim the additional exemptions for dependent children. Which is the exception? A. The husband has no income of his own A1 PASSERS REVIEW CENTER///202 B. The husband has no compensation income, his income comes only from profession C. The husband waived the additional exemptions in favor of the wife D. The husband is non- resident citizen and his income is purely from sources outside the Philippines F. 56. CASE 1: Mr. Ramos inherited from his parents large parcels of undeveloped land acquired by them years ago with total cost of P500, 000. Mr. Ramos now sells all of these parcels for P2 million. How much of the gains should Mr. Ramos report for income tax purposes? G. CASE 2: Suppose that when Mr Ramos inherited these parcels of land, they ere already developed real estate subdivisions with small lots being sold on instalment basis. He now sells all these parcels for P4 million. How much of the gain should be reported by Mr. Ramos for income tax purposes? H. ANSWER TO CASE 1: NONE, because the properties are capital assets and the transaction is subject to capital gain tax and not income tax. I. ANSWER TO CASE 2: Mr. Ramos should report gains from the Sale amounting to P3.5 million because the properties sold are ordinary assets. The transaction is subject to income tax. A. Both answers to both cases are correct B. Both answers to both cases are wrong C. Answer to case 1 wrong, answer to case 2 correct D. Answer to case 1 correct, answer to case 2 wrong J. 57. Mr. Lacson filed an income tax return for the calendar year 1989 on March 10, 1990. The BIR assessed a deficiency income tax on April 10, 1992. When is the last day for the BIR to make assessment? A. April 15, 1993 C. April 15, 1992 B. April10,1992 D. March 10, 1993 E. 58. The following tax payers can claim tax credit, except one: A. Domestic corporation B. Members or beneficiaries of partnership or trust C. Filipino citizens D. Non-resident alien F. 59. 1st statement: if the tax is not billed separately in the inventory, the VAT shall be determined by multiplying the gross selling price including the amount intended to cover the tax by the factor 1/11. G. 2nd statement: if the tax is billed erroneously, the VAT shall be determined by multiplying the gross selling price including the tax billed erroneously by the factor 1/11. A. Both statements are wrong C. Only 1st statement is correct nd B. Only 2 statement is correct D. Both statements are correct E. 60. 1ST STATEMENT: In case tax exempt products are sold domestically to a VAT registered person, the VAT otherwise due on such product shall be considered as input tax creditable against his output tax payable: F.2Nd STATEMENT: Export sales by a VAT registered person are subject to zero rating and so he can claim and enjoy a credit for the tax invoiced to him on his purchases. If he is not VAT registered, his export sales are exempt, but he is not entitled to tax credit for inputs. A. Both statements are true C. Both statements are false B. 1st statements false; 2nd statements true D. 1st statement true, 2nd statement false E. 61. Which one of the following is not considered a percentage tax? A. ½ of 1 % stock transaction tax on shares traded through local stock exchange B. 10-20% stock transaction tax on shares not traded through a local stock exchange C. 10% overseas communication stock D. 3% tax on keepers of garage F. 62. The following are some instances where a VAT registered person may apply for cancellation of registration, except one: A. A change of ownership in the case of single proprietorship B. A person who has registered prior to planned business operation, fails to actually start his business C. A person has retired from business D. A person who voluntarily registered under the VAT system can only revert to exempt status after the lapse of 12months from the date of registration G. 63. Matubo Corp. Donated P100,000 for the purpose of cementing a barangay road where its factory is located. H. 1st STATEMENT: The donation is exempt from donor’s tax I. 2ND STATEMENT: The Corporation may claim a full deduction for income tax purposes A. Both statements are incorrect C. Both statements is correct B. Only 1st statement is correct D. Only 2nd statement is correct E. 64. Which percentage of vanishing deduction shall be applied if the prior decedent died with in more than three years but not more than four years prior to the death of the present decedent or if the property was transferred to the present decedent by gift within the same period prior to his death. A. 20% C. 80% B. 60% D. 40% E. 65. Mr. Soltero, a resident citizen, died on August 2, 1996. When will notice of death be filed as required by the Tax Code? A. April 15, 1997 C. November 2, 1996 B. October 2, 1996 D. February 2, 1997 A1 PASSERS REVIEW CENTER///203 E. 66. The following are motives of tax payers that preclude transfer in contemplation of death except one: A. To relive the tax payer of the burden of C. To avoid payment of estate tax management D. To make dependent financially independent B. To save income and property taxes E. 67. Which of the f following statements is not true and incorrect? A. A tax assessment is necessary to a criminal [prosecution for wilful attempt to defeat and evade payment of taxes. B. A conviction for tax evasion is not a bar for collection of unpaid taxes C. Criminal proceedings under the Tax Code is now a mode of collection of internal revenue taxes, fees and charges D. If the tax payer is acquitted in a criminal violation of the tax code, this acquittal does not exonerate him from his civil liability to pay the taxes. F. 68. Where a return was filed, as a general rule, the prescriptive period for assessment after the date the return was due or was filed, whichever is later, is within: A. Three years C. 10 years B. Two years D. Five years E. 69. The seizure by the government of personal property to enforce the payment of taxes to be followed by its public sale if the taxes are not voluntarily paid is called: A. Forfeiture C. Distraint B. Levy D. Garnishment E. 70. Mr. Geolingo files his 1996 income tax return and paid the tax shown there on April 10, 1997. Upon investigation, The BIR determined the deficiency income tax of P20,000. The tax payer agreed with the findings of the BIR examiner and on July 15, 1997 to pay the amount shown thereon. Mr. Geolingo failed to pay on due date of the assessment but agreed to pay the deficiency income tax together with interest and penalties imposed by law on December 15, 1997. How much is the total correct amount to be paid by the tax payer? A. P35,208.33 B. P28,437.50 C. P34,666.67 D. P28,000.00 F. G. A1 PASSERS TRAINING, RESEARCH, REVIEW & DEVELOPMENT COMPANY H. 2nd Floor Sommerset Bldg., Lopez Jaena St. Jaro, Iloilo City I. Tel. No.: (033) 320-2728; 09106547262 J. Email Address: [email protected] K. L. BOARD OF CERTIFIED PUBLIC ACCOUNTANT M. N. CERTIFIED PUBLIC ACCOUNTANT Licensure Examination O. P.MANAGEMENT SERVICES Q. R. INSTRUCTION: Select the correct answer for each of the following questions. Mark only one answer for each item by shading the box corresponding to the letter of your choice on the answer sheet provided. STICTLY NO ERASURES ALLOWED. Use pencil no. 2 only. S. T. MULTIPLE CHOICE: U. V. 1. In deciding whether to manufacture a part or buy it from an outside vendor, a cost that is irrelevant to the short- run decision is A. Variable overhead B. Fixed overhead that will be avoided if the part is bought from an outside vendor C. Direct labor D. Fixed overhead that will continue even if the part is bought from an outside vendor W.2. The working capital of RED Company at December 31,1996n was P10, 000.000. Selected information for the year 1996 for Red is as follows X. Y. Working capital provided from operations P1,700,000 Z. Capital expenditures 3,000,000 AA. Proceeds from shot terms borrowings 1,000,000 AB. Proceeds from long term borrowings 2,000,000 AC. Payments on short term borrowings 500,000 AD. Payments on long term borrowings 600,000 AE. Proceeds from issuance of common stock 1,400,000 AF. Dividends paid in common stock 800,000 A1 PASSERS REVIEW CENTER///204 AG. AH.What is RED’s working capital AT December 31, 1997? A. P11,200,000 C. P10,700,000 B. P11,500,000 D. P12,000,000 E. 3. At the end of Kiko Company’s first year of operations, 1,000 units of inventory remained on hand. Variable and fixed manufacturing costs per unit were P90 and P20, respectively. If Kiko uses absorption costing rather than direct (variable) costing, the result would be a higher pre-tax income of A. P20,000 B. P70,000 C. P0 D. P90,000 F. G.4. JERRY Company budgeted sales of P400, 000 plastic gums of P40 per unit for 1997. Variable manufacturing costs were budgeted at P16 per unit and fixed manufacturing costs T p10 per unit. A special order offering buy 40,000 plastic gums for P23 each was received by JERRY in March 1997. JERRY has sufficient plant capacity to manufacture the additional quantity: however the production would have to be done on an overtime basis at an estimated additional cost of P3 per plastic gums. Acceptance of special order would not affect JERRY’s normal sales and no selling expenses would be incurred. What would be the effect of the operating profit if the special order were accepted? A. P240,000 decrease C. P160,000 increase B. P120,000 decrease D. P280,000 increase H. 5. Breakeven analysis assumes that over the relevant range. A. Selling price are unchanged B. Variable cost are nonlinear C. Total costs are unchanged D. Fixed costs are nonlinear I. J. K. L. 6. In preparing its cash budget for July 1997, ART Company made the following projections M. N. Sales P1, 500,000 O.Gross profit (based on sales) 25% P. Decrease in inventories P70, 000 Q.Decrease in accounts payable for inventories 120,000 R. S. For July 1997 what where the estimated cash disbursements for inventories? A. P1,050,000 C. P1,175,000 B. P1,055,000 D. P935,000 E. 7. KAREN Company is considering replacing an old machine with a new machine. Which of the following items is economically relevant to KAREN’s decisions? Ignore income tax consideration A. Carrying amount of old machine-Yes F. Disposal value of new machine- Yes B. Carrying amount of old machine-Yes G. Disposal value of new machine- No C. Carrying amount of old machine-No H. Disposal value of new machine- Yes D. Carrying amount of old machine-No I. Disposal value of new machine- No J. K. 8. DIGNA Company had the following transactions in 1997, their first year of operations L. M. Sales (90% collected in 1997) P1, 500,000 N. Bad debt written-offs 60,000 O. Disbursements for costs and expenses 1,200,000 P. Disbursements for income taxes 90,000 Q. Purchases of fixed assets 400,000 R. Depreciation of fixed assets 80,000 S. Proceeds from issuance of common stock 500,000 T. Proceeds from short term borrowings 100,000 U. Payments on short term borrowing 50,000 V. W.What is the cash balance at December 31, 1997? A. P210,000 C. P280,000 B. P150,000 D. P170,000 A1 PASSERS REVIEW CENTER///205 E. 9. WHITE Company plans to invest P2, 000 at the end of the next ten years. Assume that WHITE will earn interest at an annual rate of 6% is 13.181. The present value of P1 for ten periods at 6% is 0.558. The present value of an ordinary annuity of P1 for ten periods at 6% is 7.360. The investment after the end of ten years would be A. P26, 362 C. P14,720 B. P21,200 D. P27, 478 E. 10.PINOY Company temporarily has excess production capacity. The idle plant facilities can be used to manufacture a low margin item. The low-margin item should be produced if it can be sold for more than it’s. A. Variable costs plus any opportunity cost of idle facilities B. Indirect costs plus any opportunity cost of the idle facilities C. Fixed costs D. Variable costs F. 11. All of the following are included in the systems implementing process EXCEPT A. Training C. Testing B. Documentation D. System design E. 12. An auditor used data to verify the existence of controls in a certain computer program. Even though the program performed well on the test, the auditor may still have a concern that A. Generalized audit software may have been a better tool to use B. Data entry procedures may change and render the test useless C. The test data will not b relevant in subsequent audit periods D. The program tested is the same one used in regular production runs F. 13. A quantitative technique used for selecting the combination of resources that maximizes profits or minimizes costs is: A. Curvilinear analysis B. Queuing theory C. Linear programming D. Dynamic programming G. H. I. J. 14. The following computations were made from BIRD Company’s 19979 books K. L. Number of days sales in inventory 61 M. Number of days sales in trade accounts receivable 33 N. O. What was the number of days in BIRD’s 1997 operating cycle? A. 33 C. 61 B. 94 D. 47 E. 15. The following information pertains to NPRY Company F. G. Cost volume profit relationship: H. Breakeven point in units sold 1,000 I. Variable costs per unit P500 J. Total fixed costs P150,000 K. L. How much will be contributed to profit before income taxes by the 1,001 st unit sold? A. P500 C. P650 B. P150 D. P0 E. 16. JOY Company is preparing a flexible budget for 1997 and requires a breakdown of the cost of steam used it its factory into the fixed and variable elements. The following data on the cost of steam used and direct labor hours worked are available for the last six months of 1997: F. G. Months Cost of Steam Direct Labor Hours H. July P15, 850 3,000 I. August 13,400 2,050 J. September 16,370 2,900 K. October 19,800 3,650 L. November 17,600 2,670 M. N. December 18,500... 2,650 O. P. Total P101,520 16,920 Q. R. Assuming that JOY uses the high –low point’s method of analysis, the estimated variable cost of steam per direct-labor hour should be: A. P6.00 B. P4.00 A1 PASSERS REVIEW CENTER///206 C. P5.82 D. P5.42 E. 17. On December 31, 1997, ALLAN Company collected a receivable due from a major customer. Which of the following ratios would be increased by this transaction? A. Inventory turnover ratio C. Receivable turnover ratio B. Quick ratio D. Current ratio E. 18. A system that permits suppliers and buyers to have direct access to portions of each other’s databases, including inventory date, to enhance service and deliveries is A. Cooperative purchasing C. Electronic data interchange B. Interactive processing D. Electronic mail E. 19. WILLIAM’s Company is budgeting sales to 42,000 units of product Y for March 1997. To make one unit of finished product, three kilos of raw materials A are required. Actual beginning and desired ending inventory of raw materials A and product Y as follows: F. G. March 1, 1997 March 31, 1997 H. Raw materials A 100,000 kilos 110,000 kilos I. Product Y 22,000 units 24,000 units J. K. There is no work on process inventory for product Y at the beginning and end of March. For the month of March, how many kilos of raw material A is WILLIAM planning to purchase? A. 132,000 C. 126,000 B. 136,000 D. 142,000 E. 20. The method of project selection which considers the time value in a capital – budgeting decision is accomplished by computing the A. Accounting rate of return on initial investment B. Payback period C. Accounting rate of return on average investment D. Discounted cash flow F. G. H. 21. MARY Company recently acquired machine at a cost P64, 000. It will be depreciated on a straight line basis over eight years with no estimated salvage value. MARY estimates that this machine will produce an annual net cash inflow (before income taxes) of P18, 000. Assuming an income tax rate of 35%, what is the approximate payback period on this investment? A. 4.4 years C. 7.1 years B. 12.8 years D. 3.6 years E. 22. In a decentralized company in which division may buy goods from one another, the transfer pricing system should be designed primarily to A. Aid in the appraisal and motivation of managerial performance B. Increase the consolidated value of inventory C. Allow division managers to buy from outsiders D. Minimize the degree of autonomy or division managers. F. 23. In preparing its budget for July 1997, JOY Company has the following accounts receivable information available: G. H. Accounts receivable at June 30, 1997 P350,000 I. Estimated credited sales for July P400,000 J. Estimated collections in July for credit K. sales in July and prior months P320,000 L. Estimated write-offs in July for M. uncollectible credit sales P16,000 N. Estimated provision for doubtful O. accounts for credit sales in July P12,000 P. Q. 24. Which of the following ratios should be used in evaluating the effectiveness with which the company uses its assets? R. A. Receivables turnover= No Divident payout ratio=No B. Receivables turnover= Yes Divident payout ratio=No C. Receivables turnover=Yes Divident payout ratio=Yes D. Receivables turnover= No Divident payout ratio=Yes S. 25. The following information pertains to AL Corporation as of and for the year ended December 31, 1997. T. Liabilities 60,000 U. Stockholders equity 500,000 V. ‘Shares of common stock issued and outstanding 10,000 W.Net income P30, 000 X. A1 PASSERS REVIEW CENTER///207 Y. During 1997, AL officers exercised stock options for 1,000 shares of stock at an option price of P8 per share. What was the effect of exercising the stock option? A. No ratios were affected C. Debt to equity ratio decreased to 12% B. Asset turnover increased to 5.4% D. Earnings per share increased by P0.33 E. 26. The statement below about project feasibility studies are true, EXCEPT: A. Since data gathering is a basic step in its preparation, all the necessary and required information will always be available B. It also covers the social desirability aspects of a proposal undertaking C. Any change which can materially alter the assumptions used in the preparation of the forecast will render it useless. D. It is important for government agencies in order to determine entitlement to government incentives. F. 27. In considering special order situation that will enable a company to make use of presently idle capacity, which of the following costs would be irrelevant? A. Depreciation C. Materials B. Variable overhead D. Direct labor E. 28.If a company wishes to establish a factory overhead budget system in which estimated costs can be derived directly from estimated activity levels, it should prepare a: A. Flexible budget B. Discretionary budget C. Fixed budget D. Capital budget F. 29. A CPA should reject management advisory services engagement if A. It would require him to make management decisions for an audit client B. The proposed engagement is not accounting related C. His recommendation are to subjected to a review by the client D. He audits the financial statements of a subsidiary of the prospective client G. H. I. 30. CARE Company’s 1997 fixed manufacturing overhead cost totalled P100, 000 and variable selling costs totalled P80, 000. Under direct costing, how should these costs be classified? J. K. Period costs Product costs A. P0 P180,000 B. P80,000 P100,000 C. P100,000 P80,000 D. P180,000 P0 L. M. 31. The capital budgeting technique known as internal rate of return uses A. Cash flow over entire life of project – No N. Time value of money- Yes B. Cash flow over entire life of project – Yes O. Time value of money- Yes C. Cash flow over entire life of project – Yes P. Time value of money- No D. Cash flow over entire life of project – No Q. Time value of money- No R. 32. Selected information from the accounting record of BEST Company is as follows S. T. Net sales for 1997 P1, 800,000 U. Cost of Goods sold for 1997 1,200,000 V. Inventory at December 31, 1996 360,000 W.Inventory at December 31, 1997 312,000 X. BEST inventory turnover for 1997 is A. 5.77 times C. 5.36 times B. 3.57 times D. 3.85 times E. 33. during 1996,. RPS Corporation supplied hospitals with a comprehensive diagnostic kit for P120. At a volume of 80,000 kits, RPS had fixed cost of P1, 000,000 and a profit before income taxes of P200, 000. Due to an adverse legal decision , RPS ‘s 1997 liability insurance increased by P1,200,000 over 1996. Assuming the volume and other costs are unchanged, what should be the 1997 price be if RPS is to make the same P200, 000 profit before income taxes? A. P135 C. P120 B. P150 D. P240 E. 34. What type of EDP system is characterized by data that are assembled from more than one location and records that are updated immediately? A. On-line real time system C. Batch processing system B. Microcomputer system D. Minicomputer system A1 PASSERS REVIEW CENTER///208 E. 35. After preliminary audit arrangements have been made, engagement confirmation letter should be sent to the client. The letter usually would not include: A. An estimate of the time to be spent on the audit work by audit staff and management B. A reference to the auditor’s responsibility for the detection of errors or irregularities. C. A statement that management advisory services would be made available upon request D. A statement that management letter will be issued outlining comments and suggestions as to any procedure requiring the client’s attention. F. 36. The invested capital employed turnover rate would include A. Invested capital in the denominator C. Invested capital in the numerator B. Net income in the numerator D. Sales in the denominator E. 37. If net earnings were higher using standard direct costing than using standard absorption costing, what can be said about sales during the period if inventory is priced using the LIFO method? A. Sales increased’ C. Sales decreased B. Sales exceed production D. Sales were less than production E. 38. The SIGMA Company wants more information on the demand for its products. The following data are relevant F. G. Units demanded Probability of unit demand total cost of unit demand H. 0 .10 P0.00 I. 1 .15 1.00 J. 2 .20 2.00 K. 3 .40 3.00 L. 4 .10 4.00 M. 5 .05 5.00 N. O. What is the total expected value or pay off with perfect information? A. P2.40 C. P9.00 B. P9.15 D. P7.40 E. F. 39. Non financial performance measures are important to engineering and operations manager in assessing the quality levels of their products. Which of the following indicators can be used to measure product quality? I. Returns and allowances III. Production cycle time II. Number and types of customer complaints A. I,II, and III C. I and II only B. II and III only D. I and III only E. 40. DENNY Company is planning to purchase a new machine for P600, 000. The new machine will be depreciated on the straight line basis over, six year period with no salvage, and a full year’s depreciation will be taken in the year of acquisition. The new machine is expected to produce cash flow from operation, net of income taxes of P150, 000 a year in each of the next six years. The accounting (book value) rate of return on the initial investment is expected to be A. 16.7% C. 8.3% B. 12.0% D. 25% E. 41. The economic order quantity formula can be used to determine the optimum size of a F. PRODUCTION RUN PURCHASE ORDER A. Yes No B. Yes Yes C. No No D. No Yes G. 42. The use of a healer label in conjunction with magnetic tape is most likely to prevent errors by the: A. Maintenance technician C. Computer operator B. Computer programmer D. Key[punch operator E. 43. RON Company is considering a proposal to replace existing machinery used for the manufacture of product A. the new machine are expected to cause increased annual fixed costs of P120,000; however, variable costs should decrease by 20% due to a reduction in direct labor hours and more efficient usage of direct materials. Before this change was under consideration, RON had budgeted product as sales and costs for 1997 as follows: F. Sales P2, 000,000 G. Variable costs 70% of sales H. Fixed costs P400, 000 I. Assuming that RON implemented the above proposal by January 1, 1997. What would be the increase in budgeted operating profit for product for 1997? A. P360,000 C. P480,000 B. P280,000 D. P160,000 E. 44. A quantitative technique used to discover and evaluate possible cause and effect relationship is: A. Correlation analysis B. Linear programming A1 PASSERS REVIEW CENTER///209 C. Program evaluation review technique (PERT) D. Poison distribution models E. 45. In a quality control program, which of the following is (are) categorized as internal failure costs? I. Rework II. Responding to customer III. Statistical quality control complaints procedures A. I,II and III C. III only B. I only D. II only E. 46. Which of the following items coast of least likely to appear in performance report based on responsibility accounting techniques for supervisor of an assembly line in a large manufacturing situation? A. Materials C. Repairs and maintenance B. Supervisor’s salary D. Direct labor E. 47. A management advisory services engagement generally involves the following activities in what order? I. Post engagement follow up V. Preparing for and sand starting the II. Implementing the recommendation engagement III. Conducting the engagement VI. Evaluating the engagement IV. Negotiating the engagement VII. Preparing the and presenting the report and recommendations A. VII, VI, V, IV, III, II, I C. IV, III, V, VII , II, VI and I B. III. IV, V, VI, VII, I and II D. IV, V, III, VII, II, VI and I E. 48. When using one of the discounted one cash flow methods to evaluate the desirability of a capital budgeting project, which of the following factors generally is not important? A. The method of financing the project under consideration B. The impact of the project on income taxes to be paid C. The timing of cash flows relating to the projects D. The amount of cash flows relating to the projects F. 49. Which of the following cost allocation methods would be used to determine the lowest price that could be quoted for a special order that would utilize idle capacity within a production area? A. Job order C. Variable B. Standard D. Process E. 50. JET Company sells 20,000 radius evenly throughout the year. The cost of carrying one unit in the inventory for one year is P8, and the purchase order cost per order is P32. What is the economic order quantity? A. 200 C. 283 B. 400 D. 625 E. F. G. A1 PASSERS TRAINING, RESEARCH, REVIEW & DEVELOPMENT COMPANY H. 2nd Floor Sommerset Bldg., Lopez Jaena St. Jaro, Iloilo City I. Tel. No.: (033) 320-2728; 09106547262 J. Email Address: [email protected] K. L. BOARD OF CERTIFIED PUBLIC ACCOUNTANT M. N. CERTIFIED PUBLIC ACCOUNTANT Licensure Examination O. P. PRACTICAL ACCOUNTING PROBLEMS Q. R. S. MULTIPLE CHIOCE: MARK FULLY with PENCIL No. 2 the letter of your choice on the answer sheet provided. Make the mark DARK but do not use too much pressure. ERASURES ARE STRICTLY NOT ALLOWED T. U. 1. Flynn Company failed to accrue warranty to costs of P600, 000 in its December 31, 2009 financial statements. In addition, a change from straight-line to accelerated depreciation made at the beginning of 2010 resulted in a cumulative effect of P2, 000,000. Both the P600, 000 and P2, 000,000 are before taxes. What amount before tax should Flynn report as prior period error in the 2010 statement of changes equity? A. 600,000 C. 2,000,000 B. 2,600,000 D. 1,400,000 E. 2. Tiger Company received a government grant related to depreciable assets five years ago on January 1, 2005 in the amount of P1, 000,000. This grant as deducted from the capital cost of the asset purchased at a total amount P6,000,000 on the same date with a useful life of 10 years and no residual value. On January 1, 2010, the entire P1, 000,000 because repayable due to the lack of compliance with the conditions attached to the grant by the government. What is the total amount to be recognized as depreciation expense for 2010 in relation to the asset and repayment of the grant? A. 600,000 B. 1,100,000 A1 PASSERS REVIEW CENTER///210 C. 700,000 D. 1,000,000 E. 3. The inventory control account balance of Ray-Ray Company at December 31, 2010 was P1, 600,000 using the perpetual inventory system. A physical count conducted on that day found inventory on hand worth P3, 450,000. Net realizable value for each inventory item held for sale exceeded cost. An investigation of the discrepancy revealed the following: a. On December 31, 2010, Ray-Ray sold goods costing P200, 000 FOB shipping terms to Daimler Company for P300, 000. The goods were dispatched on December 31, 2010 but the sales invoice had not been recorded until January 5, 2011. b. Goods costing P500, 000 sold to Lincoln Company, FOB shipping point, were shipped on December 30, 2010 but not recorded on such date and inadvertently included in the physical count due to a clerical error. c. Goods costing P150, 000 were sold on credit to Chrysler Company for P220, 000 on December 27, 2010 on FOB destination terms. The goods were still in transit on December 30, 2010. The sale invoice was raised and processed on December 31, 2010. d. Goods worth P100,000 held on consignment for Michelin Company had been included in the physical count but not recorded as a purchase, e. Goods costing P300, 000 were purchased on credit from Porsche Company on December 24, 2010 FOB shipping point. The goods were shipped on December 28, 2010 but, as they had not arrived by December 31, 2010, were not included in the physical account. The purchase invoice was received and processed on December 31, 2010. f. Goods costing P400, 000 were purchased on credit (FOB destination) from Corvette Company on December 29, 2010. The goods were received on December 31. 2010 and included in the physical count. The purchase invoice was received on January 2, 2011. g. Obsolete inventory items valued P150, 000 were discovered during the physical count. These items were still recorded on December 31, 2010 but were omitted from the physical count records pending their write off. F. G. What is Ray-Ray Company’s adjusted inventory amount? A. 3,800,000 B. 3.400.000 C. 3,150,000 D. 3,300,000 H. I. J. K. L. 4. On January 1, 2009, Hope Company grants to 300 employees a share based on payment with cash and share alternative. The provisions include the right to cash payment to the value of 100 phantom shares for each employee or 200 ordinary shares for each employee with a par value of P40. The grant is conditional upon the completion of three years vesting data. At grant date, the share price is P50. At the end of 2009, 2010 and 2011 the share prices are P60, P75 and P78, respectively. Hope does not expect to pay dividends in the next three years. After taking into account the effect of post vesting transfer restrictions,. Hope Company estimates that the grant dates fair value of the share alternative P35. On January 1, 2012, the employee selected the share alternative. What is the amount of compensation expense to be recognized for the year ended December 31, 2010? A.2,340,000 C.1,100,000 B.2,400,000 D.1,700,000 E.5. Sandy Seafood Corporation regularly hedges its purchase requirements for its restaurants for the following month using the variety of hedging instruments such as forwards, furniture and options. Sandy uses the information to enter into these contracts based on its prediction on which way prices on heading. On December 1, 2010, Sandy Company entered into the following three contracts for its January 1, 2011 purchases: F. G. Type of contract Quantity Strike price Market price pre kilo H. Per kilo December 31, 2010 I. Lobster forwards 10,000kg P2, 000 P2, 150 J. Shrimp future 50,000kg 800 720 K.Salmon call option 30,000kg 1,500 1,700 L. M. All three contracts to be settled on January 1, 2011. Sandy paid P400, 000 to purchase the call option. All derivative contracts are designated as cash flow hedges. What is the net unrealized gain to be recognized as other comprehensive income in 2010? A. 3,600,000 C. 3,500,000 B. 3,100,000 D. 0 E. 6. Kearns Company is in the business financing the acquisition of tractor trucks and trailers. As a lessor, Kearns expects a 10% return on its net investment with insignificant amount of direct cost. All leases are classified as direct financing leases. At the end of the lease term, the tractor track and trailer’s ownership is not transferred to the lessee and will revert back to Kearns. On January 1, 2010 a tractor truck is leased to A1 PASSERS REVIEW CENTER///211 A. 1,196,000 another company for 6 years with payments to b made onC.December 1,976,00031 of each year beginning December B. 1,696,000 31, 2010. The cost of the tractor truck is P4, 204,000 and D. the1,581,000 guaranteed residual value is P500, 000. What E. 7.isGentle the amount Company of financial is planning revenue to acquire that Kearns Mable willCompany earn overon theDecember 6 year lease 31, term? 2010. (Round The cumulative off present income value factors of Mable to for 2 decimal past 5 years places)was P4, 400,000 that includes expropriation gain of P500, 000 in 2008. Mable’s Company net assets averaged at a fair value of P6,000,000 for the same period, while the current fair value of assets on December 31, 2010 F. Was P5, 500,000. The rate of return in Mable’ line of business on average net assets was determined to be 8% and goodwill is determined by capitalizing average earnings at 10%. What is the amount of goodwill from the purchase by Gentle of Mable? A. 1,000,000 C. 1,500,000 B. 2,500,000 D. 2,200,000 E. 8. Stellar Company has the following investments on December 31, 2010: F. Cost Market G. Trading equity securities H. Millar Corporation 2,000,000 2,200,000 I. Villar Corporation 1,300,000 1,500,000 J. Stellar Company 850,000 1,000,000 K. Gellar Company 1,500,000 1,000,000 L. M. Trading debt securities N. Pilar Company 3,000,000 3,600,000 O. Viber Corporation 2,500,000 2,300,000 P. Q. How much unrealized gain related to these investments should be reported in Stellar Company’s statement of comprehensive income for the year ended December 31, 2010? A. 450,000 B. 50,000 C. 400,000 D. 300,000 R. S. T. U. V. 9. The following information relates to the defined benefit plan of Yvette Company: W. X. Assets and obligations on 1/1.10 Y. Plan assets at fair value 5,000,000 Z. Projected benefit obligation 6,500,000 AA. AB. Amounts to be recognized in 2010 AC. Pension expense 1,000,000 AD. Contribution 1,200,000 AE. Unrecognized gains (net) 300,000 AF. Unrecognized past service costs 800,000 AG. Benefits paid 750,000 AH. AI.The actual return was equal in the expected return in 2010 and unrecognized actual gains are due to decrease AJ. In the projected benefit obligation as a result of changes in actual assumptions. What is the accrued pension cost on December 31, 2010? A. 1,500,000 C. 1,050,000 B. 1,950,000 D. 1,300,000 E. F. 10. Keiths Company is determining the amount of taxable profit from the 2010 statement of comprehensive income that indicates an accounting profit of P1, 200,000. The following is the list of items that may be required to determine taxable profit. G. H. Payments for government fines and other penalties 500,000 I. Cash dividends received from domestic corporation but not J. included in the 2010 tax returns 800,000 K. Reversal of deductible temporary difference 600,000 L. Unrealized gain on investment classified as trading M. securities not taxable until sold 200,000 N. Reversal of taxable temporary difference 350,000 O. P. What was Keiths Company’s taxable profit? A1 PASSERS REVIEW CENTER///212 A. 900,000 C. 950,000 B. 450,000 D. 750,000 E. F. 11. Halina Company sells household furniture. Customers who purchase furniture on this instalment basis make payments in equal monthly instalments over a two year period, with no down payment required. Halima’s gross profit on instalment sales is 40%. G. H. For financial accounting purposes, sales revenue is recognized at the time sale. For income tax purposes, however the instalment method is used. There are no other book and income tax accounting differences. Halina’s income tax rate in 2010 and in future years is 30%. If there is a deferred tax liability of P120, 000 arising from the difference between book and tax treatment of the instalment sales, what should be reported as instalment accounts receivable on December 31, 2010? A. 900,000 C. 1,000,000 B. 400,000 D. 1,400,000 E. F. 12. Merriman Company operates in a hyperinflationary economy and provides the following assets, liabilities and equity items on December 31, 2010: G. H. Property, plant and equipment 2,000,000 I. Inventory 1,200,000 J. Cash 300,000 K. Accounts receivable-net 800,000 L. Share capital issued 500,000 M. Share premium 700,000 N. Accounts payable 400,000 O. Bonds payable 700,000 P. Retained earnings 1,500,000 Q. Revaluation surplus 750,000 R. Treasury shares acquired on December 31, 2008 350,000 S. T. U. V. W. X. Y. The general price index had moved on December 31 of each year as follows 2006-100, 2007- 125, 2008140, 2009-150, and 2010-300. The share capital was issued on December 31, 2006 while the property, plant and equipment were acquired on December 31, 2007. The bonds payable was issued on December 31, 2008. Inventories during 2010 were acquired evenly throughout the year and the revaluation surplus was recognized on December 31, 2010. What is the balance of Merriman’s retained earnings on December 31, 2010adjusted for hyperinflation? A. 2,800,000 C. 3,000,000 B. 3,550,000 D. 4,350,000 E. 13. On June 30, 2010, the statement of financial position of Brenda Company reported equipment at a cost of P7, 000,000 and an accumulated depreciation of P1, 750,000. The equipment was measured using the cost model and depreciated on a straight line basis over a ten year period. On the sane date, the directors of Brenda decided to change the basis of measuring the equipment from the cost model to the revolution model. The equipment was revalued at a replacement cost of P9, 000,000 and an expected remaining useful life of five years with no residual value from the date of revaluation. If the income tax rate is 30%, what amount should Brenda report as revaluation surplus on December 31, 2010? A. 945,000 C. 840,000 B. 1,400,000 D. 1,050,000 E. 14. Affleck Company provided the following data for the year 2010: F. G. Sales 5,500,000 H. Cost of gold sold 2,300,000 I. Rent income 150,000 J. Operating expenses 1,000,000 K. Casually loss 200,000 L. Tax expense 650,000 M. N. December 31 January 1 O. Prepaid operating expenses 400,000 550,000 P. Accounts payable 300,000 900,000 Q. Inventory 800,000 500,000 R. Accounts receivable 500,000 150,000 A1 PASSERS REVIEW CENTER///213 S. Salaries payable 150,000 100,000 T. Unearned rent income 30,000 90,000 U. Income tax payable 80,000 140,000 V. W.Operating expense include depreciation and doubtful account expense of P120, 000 and P30, 000 respectively. Using the direct what is the cash payment for operating expenses in 2010? A. 850,000 C. 800,000 B. 650,000 D. 750,000 E. 15. Cameron Company acquired delivery equipment on January 1, 2007 at a cost of P3,500,000 and a residual value of P500,000 and depressed it at 10% per annum, using the straight line method. On January 1, 2010, the entity decided to use the sum of year’s digits method to depreciate its delivery equipment with no change in residual value and remaining useful life. What is depreciation expense for the year ended December 31, 2010? A. 300,000 C. 260,000 B. 650,000 D. 525,000 E. 16. The following information relates to the defined benefit plan of Brandon Company on January 1, 2010: F. G. Projected benefit obligation 5,000,000 H. Fair value of plan assets 3,500,000 I. Past service cost 1,000,000 J. Unrecognized actuarial gain 700,000 K. L. The actuary provided the following data for the year ended December 31, 2010: M. N. Current service cost 600,000 O. Benefit payments to retire 900,000 P. Contribution to the fund 1,000,000 Q. Actual return on plan assets 500,000 R. Decrease in projected benefit obligation due to S. changes in actuarial assumptions 100,000 T. Expected return on plan assets 10% U. Settlement discount rate 12% V. W. X. The vesting period for the employees covered by the past service cost is 10years while the average remaining service life of the employees is 5 years. What is the proposal benefit obligation on December 31, 2010? A. 5,300,000 C. 5,400,000 B. 5,200,000 D. 5,800,000 E. 17. The Aniston Company has different kinds of farm animals on January 1, 2010. During 2010, several acquisitions occurred related to these farm anima’s. A detailed summary of these transactions is as follows: F. G. Carrying amounts on January 1, 2010: H. 10 Dairy cattle (2 years old) I. 20 Hogs (3years old) J. 15 Horses (1 year old) K. 8 Carabaos (2.5 years old) L. M. Purchase on June 30, 2010 N. 4 Dairy Cattle (1 year old) O. 6 Carabaos (6 months old) P. Q. Fair value less cost to sell on December 31, 2010: R. 10 Dairy cattle (2 years old) S. 20 Hogs (3 years old) T. 15 Horses ( 1 year old) U. 8Carabaos (2.5 years old) V. 4 Dairy Cattle (1 year old) W. 6 Carabaos (6 months old) X. 10 Dairy cattle (3 years old) Y. 20 Hogs (4 years old) Z. 15 Horses ( 2 year old) AA. 8 Carabaos (3.5 years old) AB. 4 Dairy Cattle (1.5 year old) AC. 6 Carabaos (1 year old) AD. A1 PASSERS REVIEW CENTER///214 AE. There were no farm animals sold during the year and neither were there any newborns nor deaths. What is the carrying amount of the biological asset s on December 31, 2010? A. 3,160,000 C. 2,800,000 B. 2,350,000 D. 2,380,000 E. 18. On January 1, 2010, Hogan Company leased land and building for 40 years and after that time, title to the land and building will pass to Hogan. The useful life of the building is also 40 years. On January 1, 2010, the present value of the 40 animal lease payments of P500, 000 at the end of each year is P5, 962,500 at a rate of 8%. It was ascertained that land’s fair value is P2, 380,000 while the building’s fair value is P3, 570,000. What total amount of asset should be recognized initially in relation to the lease? A. 5,962,500 C. 3,570,000 B. 5,950,000 D. 2,380,000 E. 19. Tory Company is a listed entity. Tory’s financial statements for the year ended December 31, 2010 reported basic earnings per share of P15 on each of its 900,000 ordinary shares. On April 1, 2010, Tory issued 100,000 preference shares that can be converted to 400,000 ordinary shares. During the year, Tory paid dividends of P600,000 to ordinary shareholders and P500,000 to shareholders of the convertible preference shares. What is the amount of diluted earnings per share for 2010? A. 10.76 C. 11.67 B. 11.25 D. 10.38 E. 20. Mathew Company has been incurring net losses for the several years due tot the negative impact of globalization that has affected Mathew’s industry. On the recommendation of its president, the board of directors voted to quasi- recognized the company by reducing the per value of its shares subject to approval of the Securities and Exchange Commission. Immediately prior to the restatement, the shareholders equity of Mathew on December 31, 2010 showed the following balances: F. G. Ordinary shares, P40 par, 100,000 shares 4,000,000 H. Share premium 500,000 I. Accumulated losses (1,000,000) J. K. The SEC approved the quasi-reorganization and Mathew Company determined that some assets and liabilities amounted to P300, 000. After quasi-reorganization, what is the amount of profit to be accumulated before Mathew can declare dividends to its shareholders? A. 1,900,000 B. 1,000,000 C. 1,400,000 D. 0 L. 21. The shareholders equity of Wilma Company on December 31, 2010 included the following capital balances: M. N. Preference share capital, 105 cumulative, P50 par 25,000 shares O. Ordinary share capital, P20 par, 23,000 shares P. Ordinary treasury shares, 20,000 shares at cost Q. Share premium R. Accumulated profits S. T. Preference share dividends are in arrears for 3 years as of January 1, 2010 and will be paid par and a liquidation premium of P250, 000 upon liquidation. What is the book value per ordinary share on December 31, 2010? A. 40.00 C. 40.60 B. 39.00 D. 36.50 E. 22. The following information has been extracted from the accounting recorded of Armani Company at the end of each year: F. G. 2010 2009 H. Borrowings 1,500,000 3,000,000 I. Dividends 0 500,000 J. Share capital 6,000,000 4,000,000 K. Share premium 2,500,000 1,000,000 L. Accumulated profits 2,200,000 1,500,000 M. N. During the year Armani paid half of its borrowing. Also on January 1, 2010, the company declared and issued a 25% stock dividend and on June 1, 2010, one of its shareholders donated real property with a fair value of P800, 000. All other movements in contributed capital came from the issuance of shares for cash. Net income for the year ended December 31, 2010 was P2, 500,000 and all dividends declared were paid. What is the net cash used in financing activities in the 2010 statement of cash flows? A. 300,000 C. 600,000 B. 1,100,000 D. 2,100,000 A1 PASSERS REVIEW CENTER///215 E. 23. De Niro Company declared cash dividends twice during 2010. The first was on June 30, 2010 to shareholders of record on July 31, 2010 and paid on August 10, 2010 amounting to P500,000. The second one was declared December 20, 2010 to shareholders of record on January 10, 2011 to be paid on February 1, 2011 amounting toP300, 000. Property dividends were also declared and paid in the form of inventories with a carrying amount of P1, 000,000. The fair value of inventories on March 1, 2010, the date of declaration was P1, 200,000 and the fair value of May 1, 2010, the date of payment was P1, 400,000. In addition, De Niro declared a 10% stock dividend on its P10 par value ordinary shares on December 30, 2010, to be issued on January 31, 2011, at which time De Niro had 200,000 ordinary shares issued and outstanding that had a fair value of P15 per share. At the time the shares were issued in 2011, the fair value was P20 per share. What was the decrease in retained earnings during 2010 as a result of dividends during the year? A. 1,500,000 C. 2,500,000 B. 2,300,000 D. 2,100,000 E. 24. During 2010, an erotic novel written by Dr. Dante Dela Cruz was sold to Ace Publishing Corporation, for royalties of 10% on sales. Royalties are received semi-annually on March 31; for sales in July through December of the prior year, and on September 30, for sales in January through June of the same year. F.  Royalty income of P650,000 was received on March 31, 2011 and P800,000 on September 30, 2011  Dr. Dante learned from Ace that sales subject to royalty were estimated at P10, 000,000 for the last half of 2011. G. H. What amount of royalty income should Dr. Dante Dela Cruz recognize for the year ended December 31, 2011? A. 1,800,000 C. 2,450,000 B. 1,450,000 D. 1,650,000 E. 25. At the close of its first year of operations, December 31, 2010, Archibald Company had accounts receivable of P4, 000,000 after it had written off uncollectible accounts of P100,000 that it had charged to expense under direct write-off method. But before preparing its 2010 financial statements, Archibald decided to use the allowance method and determined that the required allowance is P300, 000. In a treated matter, P30, 000 of receivables write-off were collected after the end of the reporting period. What is the carrying amount of accounts receivable on December 31, 2010? A. 3,700,000 B. 3,770,000 C. 3,870,000 D. 3,830,000 F. G. H. I. 26. During 2010, Apolinario Company pays an insurance premium of P90,000 on a P3,000,000 life insurance policy covering the president, Mr. Casket. Apolinario is the beneficiary under the life policy. The cash surrender value of the policy will increase from P120, 000 to P150, 000 by the end of 2010. Dividends of P10, 000 were received fro, the insurance company in 2010. Mr Casket died at the end of October 2010 and the policy indicates that the cash surrender value is P145, 000 at that date. What is the gain on life insurance? A. 2,835,000 C. 2,855,000 B. 2,840,000 D. 3,000,000 E. 27. During 2010, Atty. Jack De Vera became involved in a tax dispute with the BIR resuming the non-payment of taxes with regard to his undisclosed income. At December 31, 2010. Atty. De Vera tax adviser believed that an unfavourable outcome was probable and a reasonable estimate of additional taxes was P1, 500,000 but could be as much a P2, 100,000. However, before the 2010 financial statements were issued, Atty. De Vera through several connections received and accepted a BIR settlement offer of P12, 200,000. What amount t of accrued liability should be reported by atty. De Vera on December 31, 2010? A. 1,500,000 C. 1,200,000 B. 1,800,000 D. 2,100,000 E. 28. The following information was taken from Angelina Corporation 2010 statement of comprehensive income: F. G. Income before taxes 2,000,000 H. Income tax expense: I. Current 900,000 J. Decrease in deferred tax liability (300,000) 600,000 K. Net income 1,400,000 L. M. The company has 30 percent tax rate. Angelina reported a deferred tax liability of P500, 000 on January 1, 2010 and P200, 000 on December 31, 2010 while there were no deferred tax assets at the beginning and of A1 PASSERS REVIEW CENTER///216 A. B. E. 2010. Assuming no other differences existed between accounting profit and taxable profit, what was the amount of taxable profit for 2010? 2,000,000 C. 1,000,000 3,000,000 D. 1,400,000 29. Nicholas Company issued fully paid shares to 300 employees on December 31, 2010. Normally, shares issued to employees vest over a three year period but these shares have been given as bonus to employees because of their above average performance during the year. The shares have a market value of P2, 100,000 on December 31, 2010 and an average fair value of P1, 800,000 for the year. What amount would be expensed for this share based payment transaction? 600,000’ C. 700,000 2,100,000 D. 1,800,000 30. Falcon Company’s bank statement for the month of December included the following information: A. B. E. F. G. Ending balance, December 31 H. Bank service charge for December I. Interest by bank to Falcon for December net of withholding tax J. K. In comparing the bank statement to its own cash records, Falcon found the following L. Deposits made but nit yet recorded by bank M. Checks written and ailed but not yet recorded by the bank N. O. In addition, Falcon discovered that it had drawn erroneously recorded a check for P73, 000 that should have been recorded for P37, 000. What is Falcon’s cash balance per ledger December 31, 2010? A. 1,456,000 C. 1,356,000 B. 1,284,000 D. 31. Arguille Company is preparing its financial statements fro the year ended December 31, 2010. Accounts payable amounted to P2, 500,000, before any necessary year and adjusting entry related to the following items E. :  At December 31, 2010. Arguille has a P150,000 debt balance in its accounts payable to a supplier resulting from P150,000 advance payment for goods to be delivered in 2011  A check in the amount of P300, 000 was written to vendors and recorded on December 31, 2100. This withdrawal caused Arguille’s account in BDO to be overdrawn by P100, 000. The check was mailed on January 2, 2011 F. G.What amount should Arguille report as accounts payable on December 31, 2010? A. 2,950,000 B. 2,800,000 C. 3,050,000 D. 2,650,000 H. I. 32. Keaton Company determined that due to obsolescence, equipment with an original cost of P9,000,000 and accumulated depreciation on December 31, 2009 of P3,000,000 had suffered permanent impairment and as a result should be written down to its recoverable amount of P4,000,000 as of the end of the year. In addition, the remaining useful life of the equipment was reduced from 6 years to 4 years. In its December 31, 2010 statement of financial position, what amount should Keaton report as accumulated depreciation? A. 6,000,000 C. 7,000,000 B. 4,000,000 D. 4,500,000 E. 33. On January 1, 2010, Cervelli Corporation, granted share options for P50, 000 of its P100 par value ordinary shares to its key employees. The market price of its ordinary shares on that date was P180 per share and the option price was P120. “The Monte Carlo “option pricing determine total compensation expense to P2, 000,000. The options are exercisable beginning January 1, 2013, provided those key employees are still in the employ of Cervelli at the time the options are exercised. The share options expire on January 1, 2014. On January 1, 2013, when the market price of the share was P200, all 50,000 share options were exercised. What was the amount of share premium recorded by Cervelli from the issuance of ordinary shares? A. 1,000,000 C. 5,000,000 B. 3,000,000 D. 4,000,000 E. 34. Donovan Corporation provided the following transactions for the year ended December 31, 2010:  Borrowed P2, 000,000 from a local bank and disbursed the entire amount for the purchase of real property.  Sold available for sale securities costing 500,000 at a gain of P100,000  Paid dividends of P700,000  Issued 50,000 ordinary shares for P3,000,000 for cash  Purchased machinery and equipment for P800,000 cash A1 PASSERS REVIEW CENTER///217  Made advances tom other companies amounting to P2,000,000 and collected P100,000 as interest at year end  Filing and registration of a patent amounted to P150,000 F. G. What was the net cash used in investing activities for 2010? A. 2,350,000 C. 3,350,000 B. 4,350,000 D. 4,200,000 E. 35. Ondoy Company completed least hold improvements costing P5, 000,000 in December 31, 20017. The improvements had an estimated useful life of 6 years and were depreciated using the straight line method. The related lease which would have terminated on December 31, 2012, was renewable for an additional for five year term and exercisable until December 3, 2010. On December 31, 2010. Ondoy exercised the renewal option. What is the carrying amount of the leasehold improvement on December 31, 2010? A. 2, 250,000 C. 2,000,000 B. 2,625,000 D. 2,500,000 E. 36. Piedra Company reported retained earnings balance of P3, 500,000 at January 1, 2010. In preparing the 2010 financial statement, Piedra determined that rentals received in advance of P600, 000 in March 2009 for a three year period beginning January 1, 2009 had been received and recognized as income in full in 2009. Piedra has a tax rate of 30%. What is the corrected balance retained earnings on January 1, 2010? A. 3,220,000 C. 3,900,000 B. 4,100,000 D. 3,780,000 E. 37. Donald Company reported revenue of P5, 000,000 in its rash basis income tax return for the year ended December 31, 2010. Additional information as follows: F. G. Rent receivable on December 31, 2010 400,000 H. Rent receivable ob December 31, 2009 700,000 I. Uncollectible rent written off during year 50,000 J. K. Under the accrual basis, what is the rental revenue for 2010? A. 4,700,000 C. 4,650,000 B. 4,750,000 D. 5,350,000 A1 PASSERS REVIEW CENTER///218 E. 38. On January 1, 2008, Maldives Corporation established a share appreciation rights plan for its executive. It entitled them to receive cash at any time during the next four years or until December 31, 2011 for the difference between the market price of its ordinary shares and the pre established price of P100 on 50,000 shares. Current market price of the shares is as follows: F. G. January 1, 2008 100 per share H. December 31, 2008 120 per share I. December 31, 2009 117 per share J. December 31, 2010 125 per share K. December 31, 2011 130 per share L. M. N. On December 31, 2010, Maldives executive exercised 30,000 share appreciation rights. What amount of compensation expense should Maldives recognize for the year ended December 31, 2010? A. 512,500 B. 400,000,.kl C. 312,500 D. 375,000 O. P. 39. Miami C heats Corporation has an incentive compensation plan under which the president receives a bonus equal to 10 percent of the company’s income after deduction for bonus but before income tax. Income before bonus and income tax is P5, 500,000. Miami Cheat has an income tax rate of 30%. What is the bonus for the year? A. 550,000 B. 500,000 C. 400,000 D. 450,000 Q. R. 40 The stockholders equity of Garrison Corporation at December 31, 2010 is as follows: S. T. 12% fully participating cumulative preference U. shares, P100 par authorized 150,000 shares issued 10,000,000 V. Ordinary shares, P50 par authorizes 500,000, 130,000 issued 6, 500,000 W.Share premium 5,000,000 X. Retained earnings 6,000,000 Y. Treasury ordinary shares- 30,000 shares (2,000,000) Z. Total shareholders’ equity 25,500,000 AA. AB.Dividends on the preference shares are arrears for two years. On December 31, 2010, Garisson wants to pay cash dividends of P12.00 per share to its ordinary shareholders. What total amount of dividends should be declared on December 31, 2010? A. 5,120,000 B. 4,900,000 C. 5,200,000 D. 4,980,000 AC. AD.42. The shareholders equity of Mc Nutty Corporation at the of 2010 and 2009 is as follows: AE. AF. 2010 2009 AG. 10% preference share capital P100 par 4,000,000 3,000,000 AH.12% cumulative preference share capital P50 par 5,000,000 2,500,000 AI.Ordinary share capital P20 par 10,000,000 7,000,000 AJ. Share premium-10% preference shares 1,500,000 300,000 AK. Share premium-12% preference shares 1,500,000 800,000 AL. Share premium- ordinary shares 5,000,000 2,000,000 AM. Subscribed 10% preference share capital 500,000 AN.Subscribed ordinary share capital 1,000,000 AO. Subscription receivable-preference share (300,000) AP. Subscription receivable- ordinary share (800,000) AQ. Retained earnings 3,000,000 2,000,000 AR.Treasury shares at cost (500,000) (500,000) AS. AT. Mc Nutty has 100,00 authorized 10% preference shares, 200,000 authorized 12% preference shares and 1,000,000 authorized ordinary shares. During 2010, subscribers of the ordinary share capital from 2009 fully paid their subscription that represents 50% of the total subscription price and Mc Nutty issued the corresponding shares, while the subscribers of 10% preference share capital paid 75% of the subscription price. All other increases in share capital and share premium represent shares issued for cash. What is the average issue price per share of the 105 preference shares issued in 2010? A. 220 B. 150 C. 200 D. 180 AU. AV. AW. AX. AY. AZ.43. On January 1, 2010, Hermanos Company purchased P5, 000,000race value boxes of Ocho’s Company to be held as “available for sale securities” for P5,317,000 that includes direct transaction cost of P117,000 at an effective rate of 10%. These bonds have a nominal rate of 12% and pay interest annually every December 31. Hermanos Company uses the effective interest method of amortization. The entire issue will be redeemed at its maturity on December 31, 2013. Ocho’s debt securities are actively traded in bond market and market price December 31, 2010 is P5, 500,000. What is the unrealized gain in Hermanos statement of changes in equity on December 31, 2010? A. 251,300 B. 114,700 C. 500,000 D. 183,000 BA. BB.44. Moreland Company owes owed its lenders P5, 000,000 in the form of 12% four year are dated January 1, 2008. Because of Moreland’s financial difficulties in 2011 as a result of a major- major problem, unpaid interest had accumulated on this note for 2 years or a total of P1, 200,000 as of December 31, 2011. To pay of this debt, Moreland offered equity interest in the company in the form of 40,000 P50 par value ordinary shares that are currently trading at P60 per share and inventory currently carried in Moreland’s books at P2, 500,000, but with a fair value of P2, 800,000. The lenders accepted the offer to release Moreland from the debt and accrued interest. How much is the gain on extinguishment of debt that will be recognized by Moreland in the 2011 statement of comprehensive income? A. 1,700,000 B. 1,000,000 C. 1,400,000 D. 1,300,000 BC. BD. 45. On January 1, 2010, Willis Company issued P6, 000 of 12% bonds payable maturing in five years. The bonds pay interest semi-annually on July 1 and January 1. Each P1, 000 bonds includes nondetachable share warrant that gives the bond holder the right to purchase twenty, P100 per value ordinary share Willis at a price of P150 per share within the next three years. The bonds and warrants were issued at 110. The value of the warrants at the time of issuance was P800, 000. However it was ascertained that Willis Company‘s bonds would have sold for only P5, 600,000 without the share warrants. On December 31, 2010, the bondholders exercised all the share warrants wand Willis issued the corresponding shares. What is the credit to share premium in 2010 from the issuance of the shares through the exercise of the share warrants? A. 7,000,000 B. 6,000,000 C. 6,800,000 D. 6,825,000 BE. BF. 46. The petty cash fund of Brian Company showed the following details: BG. BH. Coins and currency 18,000 BI.Paid vouchers BJ. Transportation 6,000 BK. Gasoline 4,000 BL. Office supplies 1,500 BM. Postage stamps 1,300 BN. Due from employees 8,200 21,000 BO. Employee’s check returned by bank marked “DAIF” 6,000 BP.Check drawn by the company to the order of the petty cash custodian 5,000 BQ. BR. What is the correct amount of petty cash fund? A. 29,000 B. 18,000 C. 23,000 D. 50,000 BS. BT. BU. BV. BW. BX. ` BY. BZ. CA. CB. CC. CD. CE. CF. CG. CH. CI. CJ. CK. CL. CM. CN. CO. CP. CQ. CR. CS. CT. CU. CV. CW. CX. CY. CZ. DA. DB. DC. DD. DE. DF. DG. DH. DI. DJ. DK. DL. DM. DN. DO.